Kaplan Cumulative: Cardio

Ace your homework & exams now with Quizwiz!

___________- is also important to recognize, as it is a neurologic emergency. It is caused by compression of the lower lumbar and sacral nerve roots, however, and produces sensory loss in a *+saddle distribution, decreased reflexes, urinary incontinence, and flaccid and weak legs*

cauda equina syndrome

_________- gives off the common hepatic, splenic, and left gastric arteries. This artery primarily supplies blood to the colon

celiac trunk

A 2-month-old baby boy is brought to the clinic by his mother for a routine checkup. On physical examination, the health care provider hears a continuous machinery murmur near the left sternal border at the second interspace. What is the most likely diagnosis?

A continuous machinery murmur is characteristic of patent ductus arteriosus (PDA). PDA is also characterized by bounding pulses with wide pulse pressure. The ductus ateriosus is a structure that is necessary during gestation but becomes unnecessary after birth when the newborn takes a deep breath and the lungs begin to function. It normally closes within a few hours to days after birth. It most commonly fails to close in premature infants or in infants who are born to mothers living in high altitudes. If the ductus allows a large volume of blood to flow through, there is a wide pulse pressure that is identified along with bounding pulses. A continuous, machinery-sounding murmur is classically associated with this disorder. The murmur is continuous because the aortic pressure is higher than the pulmonary pressure during both systole and diastole. EKG may be normal or may show signs of ventricular hypertrophy. Indomethacin can be used to close the ductus and surgical ligation can be used if indomethacin fails. Diuretics may be used to treat extra volume states until more definitive closure can be performed.

A 15-year-old boy is concerned about multiple stretch marks that have developed on his chest, shoulders, and thighs over the previous 3 to 4 years. He plays center on his high school basketball team and is annoyed by the appearance of the skin lesions. His medical history is remarkable for myopia, upward lens displacement, and mitral valve prolapse. He is quite tall for his age and has long, thin extremities and fingers. He is accompanied by his father, who has the same tall stature and thin extremities. The skin lesions this young man is worried about are multiple striae in the pectoral, deltoid, and thigh regions. His chest is flattened on either side with forward projection of the sternum, and he has long, tapering fingers and toes. Which of the following is the most likely cause of his condition?

A genetic defect in the fibrillin gene. This patient has Marfan syndrome, an autosomal dominant inherited genetic defect of fibrillin. Abnormalities appear in 3 organ systems: the eye, the skeletal system, and the cardiovascular system. Skin manifestations may consist of striae (stretch marks). The skeletal features include long, narrow extremities and tapering fingers and toes. The patients are typically unusually tall. They may also have pectus excavatum, pectus carinatum, flat feet, and hyperextensibility of the knees and other joints. The ocular findings include long orbits causing myopia, and ectopia lentis (with upward displacement in 70%) sometimes causing acute glaucoma because of drainage obstruction secondary to lens displacement. Cardiovascular defects consist of weakness of the aortal intima leading to aneurysms and mitral valve prolapse. Acute dissection of the aorta is a common cause of death in these patients.

A 40-year-old woman is brought to the emergency department following a suicide attempt with imipramine (Tofranil). Her physical examination is otherwise unremarkable. On an electrocardiogram, which of the following abnormalities would most likely reflect possible cardiac toxicity?

A prolongation of the QT interval is highly predictive of both cardiac and CNS toxicities from tricyclic antidepressant ingestion. This medication has high lethality associated with its overdose because of its cardiac effects.

A 67-year-old man comes to the clinic complaining of steady, dull back pain over the past 3 weeks. He states that he has recently moved after retiring from a career in banking and is searching for a new health care provider. His past medical history is significant for diverticulosis, prior smoking, and hypertension. He says that he has run out of his blood pressure medication. He denies trauma to his back and otherwise feels well. On physical examination, his blood pressure is 170/93 mm Hg with a pulse of 88/min. He has no tenderness over the spinal processes or paraspinal areas. His abdomen is obese, but there is a suggestion of a non-tender, pulsatile mass in the epigastric region. The remainder of the physical examination is normal. Which of the following diagnoses should be considered at this time?

AAA (Abdominal Aortic Aneurysm- The combination of the history of hypertension and smoking, the new back pain, and a pulsatile mass on examination is highly suggestive for abdominal aneurysm. Additionally, he is a male older than 65 years of age, which also puts him at higher risk for AAA development.

___________- should be considered in older males, smokers, and patients who have atherosclerosis who have a palpable mass in the abdomen. It is often found incidentally when the patient has an imaging procedure on the abdomen as part of evaluation for another abdominal condition. Risk for rupture is highly dependent on the diameter of the aorta. Elective repair is recommended for males who have a >_____-cm and females who have a >____-cm diameter. Elective surgery has a much better prognosis than when Rupture Occurs

AAA, 5.5, 5 (Abdominal aortic aneurysm)

Aortic regurgitation may be helped by vasodilator therapy, which will act as afterload reducers. In addition to _________, pure vasodilators such as hydralazine and nifedipine can also be used. For patients who have ongoing symptoms who are surgical candidates, aortic valve replacement is performed when the disease decompensates

ACE (ACE inhibitor)

This patient has diabetes, hypertension, and proteinuria. Her blood pressure must be well controlled to prevent progression of nephropathy. _________ have been shown to prevent progression of diabetic nephropathy in type 1 diabetics and likely have similar preventive effects for type 2 diabetics. It should be first-line therapy for diabetic patients who have hypertension. This patient's potassium is not elevated, so this class of medications is not contraindicated.

ACE (ACE inhibitors)

_________ Meds- are useful in delaying the progression of renal disease in patients who have diabetes and in treating patients who have congestive heart failure, especially if they have impaired left ventricular function.

ACE (ACE inhibitors)

A 66-year-old woman with newly diagnosed type 2 diabetes comes to the health care provider for a blood pressure check. At a previous visit 3 months ago, her blood pressure was 140/95 mm Hg and she was advised to attempt a moderate weight loss with a low-impact exercise program and diet modification. Unfortunately her weight has increased 2 kg (4.4 lb) since her last visit. Her blood pressure is now 144/95 mm Hg. She is mildly resistant to starting medication at this time. Start on ___________

ACE (Lisinopril)

In a patient with CHF what medication will you place them on if you need to BOTH: -Decrease Preload and Afterload

ACE (captopril, enalapril, lisinopril, benazepril, ramipril)

__________- work by blocking the conversion of angiotensin I to angiotensin II in the lungs. Angiotensin II is both a potent vasoconstrictor and a stimulator of aldosterone production. Aldosterone acts by promoting sodium (and thus water) reabsorption by the kidney. An ACE inhibitor will therefore promote vasodilatation (reducing afterload), as well as reduce intravascular volume (decreasing aldosterone, as less renin is being released).

ACE (captopril, enalapril, lisinopril, benazepril, ramipril)

A 64-year-old man comes to the office for a routine checkup. He has been taking metformin (Glucophage) for his diabetes. The patient's vital signs show an elevated blood pressure, which is confirmed by his daily blood-pressure measurements at home. Urine studies show evidence of mild proteinuria. The health care provider decides to add a new medication to his regimen to treat his hypertension. What is the best class of medication to add at this time?

ACE (drug class of choice for treating hypertension in patients who have diabetic nephropathy, because they improve renal function and delay end-stage renal disease. ACE inhibitors block the angiotensin-converting enzyme in the renin-angiotensin system, causing vasodilation and increasing blood flow to the kidneys)

_________- are first-line agents for patients who have heart failure because they act as vasodilators. They decrease afterload, which allows the heart to have improvement in its ejection fraction. They do NOT have an inotropic or chronotropic effect on the heart. There is a mortality benefit demonstrated with the use of these agents.

ACE inhibitors (Beta-blockers and the aldosterone antagonist, spironolactone, also have been shown to have an improvement in mortality in patients who have chronic heart failure)

Treatment of Diastolic Heart Failure include Decreasing Risk Factors and Medications of _______ and ________ & _________

ACE, Beta Blockers and Diuretics

Patients who have stable chronic heart failure should be optimally medically managed before elective surgery being undertaken. Medical management of chronic heart failure includes (4x) to decrease secondary coronary vascular events

ACE, Beta Blockers, Diuretics and Statins

Acute onset of _________ with a rapid ventricular response is associated with hypotension. The patient may have acute deterioration that would necessitate immediate cardioversion/defibrillation. Patients need to be given volume and airway support, and management of patients in an emergency department setting is preferred. Because of the patient rapidly deteriorating, patients should be transferred to the emergency setting via emergency medical support personnel, who can intervene in case the patient becomes unstable.

AF (atrial fibrillation)

A 65-year-old patient has experienced several transient ischemic attacks over the past few months. Because his general health is poor, he is not considered an appropriate candidate for carotid endarterectomy. Examination reveals a regular rate on heart examination with a blood pressure of 130/86 mm Hg. The decision is made to treat him medically. Which of the following agents would be most appropriate for his therapy?

ASA (Daily aspirin therapy has been shown in prospective, randomized studies to reduce the incidence of stroke and death in patients who have transient ischemic attacks. If alternative therapy is needed, either because the patient cannot tolerate aspirin or because aspirin therapy has failed, the antiplatelet agent clopidogrel -Plavix- can be used)

A 75-year-old man with angina pectoris has recurrent episodes of atrial tachycardia (240/min). A rapid sequence of normal QRS waves is seen on EKG. The episodes are controllable by the patient's performance of vagal maneuvers. Which of the following is the most likely etiology of this arrhythmia?

AV Nodal Reentry (paroxysmal supraventricular tachycardia -PSVT-, which is a regular, rapid -150-250/min- arrhythmia originating in the atria or AV node. AV nodal reentry is the most common cause of this arrhythmia -about 70% of patients. In this condition, the AV node is pathologically divided into two functional pathways. The electrical impulse usually proceeds anterograde down the slow pathway and retrograde up the fast pathway. The P waves are recorded nearly simultaneously with the QRS complexes-which occur in rapid sequence- and are therefore obscured on EKG)

Paroxysmal supraventricular tachycardia (PSVT) abruptly starts and abruptly terminates. This condition can occur in the setting of coronary disease, heart failure, increased catecholamine state, digoxin toxicity, and alcohol use. Patients who have _____________ will have abrupt cessation of the tachycardia with vagal maneuvers or with the administration of adenosine (Adenocard), which temporarily stops conduction through the AV node. EKG may show P waves with RP interval caused by retrograde activation of the atria. It is the Most Common Cause of PSVT.

AV nodal re-entry tachycardia (patients who do not respond to vagal maneuvers or adenosine can be referred for an electrophysiologic test and ablation of the abnormal accessory pathway. Patients who have this abnormal accessory pathway should not have verapamil (Calan, Isoptin, Verelan) used to treat supraventricular tachycardia because of this agent's tendency to block conduction through the normal pathway, which leads to conduction only going through the abnormal re-entry pathway)

A 75-year-old man with angina pectoris has recurrent episodes of atrial tachycardia (240/min). A rapid sequence of normal QRS waves is seen on EKG. The episodes are controllable by the patient's performance of vagal maneuvers. Which of the following is the most likely etiology of this arrhythmia?

AV nodal reentry. This patient has paroxysmal supraventricular tachycardia (PSVT), which is a regular, rapid (150-250/min) arrhythmia originating in the atria or AV node. AV nodal reentry is the most common cause of this arrhythmia (about 70% of patients). In this condition, the AV node is pathologically divided into two functional pathways. The electrical impulse usually proceeds anterograde down the slow pathway and retrograde up the fast pathway. The P waves are recorded nearly simultaneously with the QRS complexes (which occur in rapid sequence) and are therefore obscured on EKG. This arrhythmia is commonly seen in older patients, about half of whom have underlying heart disease. Reentry PSVTs can be reverted to normal sinus rhythm by interrupting the reentry pathway. For example, the performance of vagal maneuvers often improves the condition by increasing AV nodal refractoriness

An arteriogram is performed on a patient who has atherosclerosis. Luminal narrowing of which of the following vessels would compromise blood flow through the renal arteries?

Abdominal Aorta (The renal arteries emerge from the abdominal aorta at about the level of the L1/L2 intervertebral disk and travel at nearly right angles to it-on the right, passing posterior to the inferior vena cava- to enter the hilum of the kidney)

A 68-year-old man is brought to the emergency department with excruciating back pain that began suddenly 45 minutes ago. The pain is constant and is not exacerbated by sneezing or coughing. He is diaphoretic and has a systolic blood pressure of 90 mm Hg. There is an 8-cm pulsatile mass deep in his epigastrium, above the umbilicus. A chest radiograph is unremarkable. Two years ago, he was diagnosed with prostatic cancer and was treated with orchiectomy and radiation. At that time his blood pressure was high, for which he declined treatment. Which of the following is the most likely diagnosis?

Abdominal aortic aneurysms have a high incidence of rupture once they reach or exceed a size of 5.5 cm in males and 5.0 cm in females. Often the first manifestation is excruciating back pain, as the blood leaks into the retroperitoneal space before the aneurysm blows out into the peritoneal cavity. The combination of a big pulsatile mass and sudden severe back pain should always lead to this presumptive diagnosis. Looking for orthopedic or neurologic explanations can be a deadly mistake. In any patient who has a rupturing aneurysm or who is hemodynamically unstable, immediate surgical intervention is required.

A 63-year-old woman comes to the emergency department complaining of severe mid-abdominal pain. The patient reports that the pain has increased in intensity over the past few days. There has been no associated nausea or vomiting, no change in bowel habits, and no relief afforded by position changes. The woman is postmenopausal and does not take hormone replacement therapy. She has a 30-year history of hypertension and has been noncompliant with her calcium channel blocker and thiazide diuretic therapy. On examination, her abdomen is obese, but there is a suggestion of a nontender, pulsatile mass in the epigastric region. The remainder of the physical examination is normal. Imaging studies are done, and immediate surgical evaluation is indicated. Which of the following results were most likely obtained from the imaging studies?

Abdominal aortic aneurysms larger than 5 cm have greater than a 30% chance of rupturing in 3 years. As most ruptures result in death, and as the mortality of surgical repair has fallen sharply, surgical repair of the aneurysm is indicated. Aneurysms that are not repaired expand on average at about 0.4 cm per year. Repair of these aneurysms can be accomplished via either traditional open approaches or new endovascular approaches.

A 73-year-old man comes to the emergency department complaining of abdominal pain. He describes a dull, aching, constant pain in his mid-umbilical region. The pain has persisted over the past few days with increasing intensity, and it is not relieved by changes in position or eating. The patient has a past medical history significant for hypertension and coronary artery disease. He had a myocardial infarction 3 years ago. The man has moderate peripheral vascular disease with a prior femoral-popliteal bypass graft on the left. On physical examination, his blood pressure is 180/100 mm Hg and his pulse is 86/min. He has a loud S4 on chest examination. Abdominal examination shows a pulsatile mass in his abdomen and venous stasis changes bilaterally on his lower extremities. Which of the following is the most appropriate diagnostic test at this time?

Abdominal ultrasound (U/S) is the most cost-effective screening test for a suspected abdominal aortic aneurysm (AAA). Ultrasound of the abdomen is considered to be the definitive test to perform when AAA is suspected, as it has a sensitivity and specificity of almost 100%.

A 71-year-old man comes to the hospital with an episode of bright red blood per rectum. The patient reports that, a few hours ago, he passed a grossly bloody bowel movement. The passage was associated with some cramping lower abdominal pain. The patient's past medical history is significant for coronary artery disease and a myocardial infarction 3 years ago. He is poorly compliant with his beta-blocker and diuretic therapy, and his blood pressures have run around 140/85 mm Hg. While the patient is in the hospital, he has another episode of large volume, bright red blood per rectum. His blood pressure is 100/60 mm Hg while supine, and his pulse is 120/min. His hematocrit is 28%. He then begins to complain of substernal chest tightness radiating to his left shoulder. An electrocardiogram shows new T-wave inversions in the anterior leads. Which of the following is the most appropriate intervention at this juncture?

According to the available data, the patient is presenting with myocardial ischemia (as manifested by his angina) in the presence of anemia, which is caused by his acute bleeding secondary to diverticulosis. The appropriate treatment, therefore, is a blood transfusion, because he is not tolerating the acute blood loss.

A 64-year-old woman comes to her primary health care provider for management of her hypertension, which has been treated unsuccessfully for several years. She was recently hospitalized for pulmonary edema, and an echocardiogram at that time showed a moderately depressed ejection fraction. She was diagnosed with congestive heart failure. Her medications include a thiazide diuretic and a calcium channel blocker. She has an allergy to furosemide. Her review of systems is positive for two-pillow orthopnea and occasional paroxysmal nocturnal dyspnea. On physical examination, her blood pressure is 150/80 mm Hg and her pulse is 80/min and regular. Her lungs are clear, and there are no extra heart sounds. Her extremities are without edema. Which of the following is the most appropriate management at this time?

Add ACE-I. This patient has both hypertension and congestive heart failure (CHF). An important concept to recognize in the treatment of medical conditions is that certain medications overlap syndromes and are efficacious in many areas. This "co-treatment" option maximizes each drug in a regimen and often addresses two or more issues simultaneously. In this case, ACE inhibitors have been shown to be very beneficial in prolonging the survival of CHF patients.

A 37-year-old African American man with a history of mild hypertension comes in for an annual examination. He has no complaints. He reports compliance with his low-salt diet. His only medication is hydrochlorothiazide, 25 mg each day. A thorough review of systems is negative. His temperature is 37.0°C (98.6°F), blood pressure 160/90 mm Hg, pulse 83/min, and respirations 10/min. Physical examination is within normal limits.

Add a second antihypertensive medication. This patient most likely has essential hypertension. Secondary forms of hypertension should only be suspected in those patients who develop hypertension at extremes of age (less than 25 or more than 55 years of age), who have suspicious physical examination findings, or who are resistant to multiple blood pressure medications. This patient is a healthy man who is only on one medication and does not have any laboratory or physical examination stigmata of secondary hypertension. Simply adding another medication to his current regimen is appropriate at this time.

37-year-old African American man with a history of mild hypertension comes in for an annual examination. He has no complaints. He reports compliance with his low-salt diet. His only medication is hydrochlorothiazide, 25 mg each day. A thorough review of systems is negative. His temperature is 37.0°C (98.6°F), blood pressure 160/90 mm Hg, pulse 83/min, and respirations 10/min. Physical examination is within normal limits. What should be done at this time?

Add second HTN medication (Pt most likely has Essential HTN. This patient is a healthy man who is only on one medication and does not have any laboratory or physical examination. He needs a second medication at this time)

A 72-year-old woman with no prior medical history comes to the emergency department because of a 3-hour episode of crushing substernal chest pain. The pain radiates to her arm and neck. An electrocardiogram reveals ST-segment elevation in leads II, III, and aVF. The patient has no obvious contraindication to anticoagulation. Which of the following is the treatment that should be currently pursued?

Administer Thrombolytic Therapy, Heparin and ASA (Pt Having STEMI- Lysis with a thrombolytic agent has been shown to decrease mortality from early post-myocardial infarction. Aspirin prevents both platelet aggregation and reocclusion of the reperfused vessels. When aspirin and lytic therapy are given in the setting of ST-segment elevation MI, heparin is added to the regimen to stop new clots from forming. Thrombolytics given up to 75 yo)

A 72-year-old woman with no prior medical history comes to the emergency department because of a 3-hour episode of crushing substernal chest pain. The pain radiates to her arm and neck. An electrocardiogram reveals ST-segment elevation in leads II, III, and aVF. The patient has no obvious contraindication to anticoagulation. Which of the following is the treatment that should be currently pursued?

Administration of thrombolytic therapy, heparin, and aspirin. The patient is having an acute ST-segment elevation myocardial infarction. The infarct occurs as a result of an atherosclerotic plaque with thrombus formation, leading to coronary artery obstruction. Lysis with a thrombolytic agent has been shown to decrease mortality from early post-myocardial infarction. Aspirin prevents both platelet aggregation and reocclusion of the reperfused vessels. When aspirin and lytic therapy are given in the setting of ST-segment elevation MI, heparin is added to the regimen to stop new clots from forming.

A 16-year-old boy is brought to the emergency department by his parents because of an episode of severe chest pain several hours earlier. Review of symptoms reveals that the patient is recovering from a flu-like illness that started a week earlier. He had been complaining of fever, chills, abdominal discomfort, and feeling tired during the prior week. He has had a low-grade fever for 5 to 6 days and a sore throat, but pharyngeal swab cultures done at the primary care physician's office had been negative and he was taking over-the-counter cold medication only. Physical examination reveals a well developed and well nourished young man in moderate distress. His temperature is 37.8°C (100.0°F), pulse is 120/min, and respirations are 28/min. A differential white blood cell count shows a normal number of neutrophils and marginally elevated lymphocytes. An electrocardiogram shows low voltage QRS complexes throughout the limb leads. Chest radiography is remarkable for increased pulmonary markings and an enlarged heart silhouette. Which of the following is the most appropriate next step in the management?

Admit (signs of myocarditis and should be admitted to a monitored hospital bed for further evaluation and management. Etiology is MC Viral- Adenovirus and Coxcackie. Or Bacteria- Diptheria, rickettsia, Fungi or Parasites. Chest radiography shows an enlarged heart and pulmonary edema. Electrocardiography shows sinus tachycardia, reduced QRS complex, and abnormal S and ST waves. Echocardiography shows poor ventricular function and possible pericardial effusions, and absence of congenital heart disease and coronary artery involvement. The diagnosis is confirmed by endomyocardial biopsy. Tx- Heart Failure & Arrhythmia)

A man is an unrestrained front-seat passenger in a car when it crashes; he sustains closed comminuted fractures of both femoral shafts. Shortly after admission, he develops a blood pressure of 80/50 mm Hg, pulse 110/min, and venous pressure of 0. He becomes pale, cold, and clammy. The rest of the physical examination and x-rays of the chest and pelvis are unremarkable. Sonogram of the abdomen performed in the emergency department is likewise negative. Which of the following most likely explains the patient's low blood pressure?

After extensive trauma to certain areas of the body, enough blood may accumulate to send the patient into hypovolemic shock. One of these areas is the femur (as seen in the present patient); the others are the thorax, abdomen, and pelvis. These patients should be treated with massive resuscitation and transfusion to overcome their hypovolemia until reduction and fixation can be performed. After extensive trauma, the thorax, abdomen, pelvis, or femurs have the potential to accumulate enough blood to produce hypovolemic shock. Intracranial bleeding does not lead to hypovolemic shock; there is not enough space inside the head for sizable blood loss to accumulate.

____________-decrease peripheral vascular resistance and are used in the treatment of hypertension and benign prostatic hyperplasia.

Alpha 1 Blockers

A 65-year-old patient has experienced several transient ischemic attacks over the past few months. Because his general health is poor, he is not considered an appropriate candidate for carotid endarterectomy. Examination reveals a regular rate on heart examination with a blood pressure of 130/86 mm Hg. The decision is made to treat him medically. Which of the following agents would be most appropriate for his therapy?

Although treatment of transient ischemic attacks (actually prophylaxis against stroke) remains a controversial area, you should be aware of current recommendations. Daily aspirin therapy has been shown in prospective, randomized studies to reduce the incidence of stroke and death in patients who have transient ischemic attacks. If alternative therapy is needed, either because the patient cannot tolerate aspirin or because aspirin therapy has failed, the antiplatelet agent clopidogrel (Plavix) can be used. Anticoagulation with coumadin does not decrease the risk for stroke and death any better than aspirin for patients who do not have atrial fibrillation, but it does increase the risk for intracerebral hemorrhage. When a patient develops a TIA, he or she is at extremely high risk for acute stroke situation. These patients need to have evaluation as to why they developed the TIA to address the underlying cause so that stroke can be prevented. If significant carotid stenosis is found, carotid endarterectomy is recommended. For patients who are not surgical candidates, stenting or angioplasty is recommended. Antiplatelet agents are preferred first-line agents in patients who have TIAs who do not have underlying atrial fibrillation or who do not have significant stenosis (<60%) post-TIA. For patients who have atrial fibrillation, warfarin or dabigatran (Pradaxa) are preferred nonsurgical therapies.

_____________- is a commonly used antiarrhythmic agent. It has efficacy in treating both ventricular and atrial arrhythmias. It is a class III antiarrhythmic under the Vaughan Williams Classification system, and has prolongation of the QT interval as a complication. Prolonged QT intervals put a patient at risk for torsades des pointes or R on T phenomena leading to ventricular tachycardia.

Amiodarone (Cordarone)

A 15-year-old girl is brought to a pediatric cardiology clinic with a complaint of chest pain. She states the pain has come and gone over the past year but has increased in frequency over the past few weeks. She describes it as a sharp pain over her left chest. Physical examination reveals a healthy appearing 15-year-old girl. Her temperature is 37.2°C (99°F), pulse 90/min, and respirations 20/min. Lung examination is normal. Cardiac examination reveals a late systolic murmur preceded by a click at the apex. No heave or rub is present. Electrocardiogram and chest radiograph are unremarkable. Which of the following is the most likely diagnosis?

An apical click followed by a late systolic murmur is classic for mitral valve prolapse. It can be a source of subjective chest pain in children. Mitral valve prolapse is more common in females. Antibiotic prophylaxis is no longer routinely recommended by the AHA prior to dental procedures unless prolapse is accompanied by severe regurgitation.

Which of the following factors is recognized as contributing to atherosclerotic risk?

Decreased fibrinolytic activity or an elevated plasminogen activator inhibitor helps to modulate the balance between fibrinogen factors. Fibrinolytic activity helps to clear areas of plaque, which contributes to atherosclerosis.

An elderly homeless man who regularly resides outside the emergency department is acting strangely. He is often seen drinking alcohol and asking for money, but today he seems disoriented, confused, and weak. It is bitterly cold and there is snow on the ground, so one of the health care providers is concerned that the combination of cold weather and alcohol would predispose the patient to hypothermia. Two emergency medical technicians help bring the man inside so he can be fully evaluated. His temperature is 32.0°C (89.6°F), blood pressure 80/40 mm Hg, pulse 53/min, and respirations 10/min. Electrocardiogram would most likely show which of the following findings?

An upward deflection following the R wave (classically in lead II) describes an Osborn wave (or J wave of Osborn), which is pathognomonic of hypothermia. The Osborn wave appears on the EKG following ST-segment elevation and is seen when the patient's body temperature is 32-33°C (89-91°F). Although these findings are pathognomonic for hypothermia, their absence does not rule out heart involvement in a patient who has hypothermia.

A young man sustains a gunshot wound to the base of his neck. He was shot point blank with a .38-caliber revolver. The entrance wound is above the left clavicle, below the level of the cricoid cartilage, and just lateral to the sternomastoid muscle. The exit wound is just above the spinous process of the right scapula. He has normal breath sounds on both sides, is awake and alert, is talking with a normal tone of voice, is neurologically intact, and is hemodynamically stable. Portable radiographs of the neck and chest taken in the emergency department show some air in the tissues of the lower neck but are otherwise non-diagnostic. Which of the following is the most appropriate next step in management?

Angiogram, esophagogram, esophagoscopy, and bronchoscopy prior to surgical exploration. Gunshot wounds to the base of the neck need exploratory surgery, but the exact approach and incision are determined by a more accurate knowledge of the location and extent of the injuries. If time permits, diagnostic studies thus should precede surgical intervention. The major vessels, the tracheobronchial tree, and the esophagus are the potential targets that have to be investigated. The esophagram needs to be performed with a water-based solution in case the patient has damage to the mediastinum, which would result in leakage from the esophagus and potential mediastinitis.

The hallmark of ____________ is sudden onset, severe chest and abdominal pain that often radiates from an anterior to posterior direction. The pain is classically described as tearing in quality. Hypertension and trauma are common precipitants for this condition.

Aortic Dissection (a common aortic catastrophe requiring admission to the hospital. This condition results from an extension of an intimal tear in the wall of the artery.)

The EKG of a 60-year-old man reveals widened QRS intervals of 0.14 seconds with distinctly abnormal configurations. Physical examination is significant for paradoxic splitting of the second heart sound. Which of the following valvular defects is likely in this patient?

Aortic Stenosis (bundle branch block, as implied by the QRS interval greater than 0.12 seconds and by paradoxic splitting of the second heart sound. The typical sequence of valve closure is mitral, tricuspid, aortic, and pulmonic. If there is a paradoxic splitting of the second heart sound, the aortic valve closes after the pulmonic valve.)

A 28-year-old man seeks help for progressive edema affecting his right lower extremity. It first started about 2 years ago but has become much worse in the last 6 months. Physical examination shows pitting edema from the groin to the toes, as well as very prominent varicose veins and an area of chronic cellulitis above the medial malleolus. These findings are confined to the right side and are not present at all on the left. He also is noted to have a palpable thrill and an audible bruit over the right groin. His resting pulse is 115/min. He gives a history of having suffered a gunshot wound to his right groin 3 years ago, for which he had no specific therapy. Which of the following is the most likely diagnosis?

Arteriovenous fistula. The bruit and the resting pulse rate are virtually pathognomonic for an arteriovenous fistula, particularly in view of the history of untreated penetrating trauma to the groin. Significant venous hypertension can develop in that setting, leading to the other presenting symptoms.

A 55-year-old man with hypertension and a past medical history of myocardial infarction is prescribed atenolol (Tenormin). This medication will lower his blood pressure by

Atenolol is a beta-adrenergic receptor blocking agent used in the treatment of hypertension. Medications in this drug class lower blood pressure by reducing both cardiac output and decreasing renin release from the kidney (to a lesser extent). Beta-blockers work as antihypertensive agents by blocking the sympathetic effect of the beta-adrenergic system. Blocking the beta-adrenergic system causes a lowering of the heart rate. The force of contraction of the ventricles is impeded and this results in a lower cardiac output. Because the heart is not contracting as strongly, there is less myocardial oxygen demand in the heart, which is one of the reasons that these agents are cardioprotective in patients who have heart disease. Because the heart is not contracting as strongly, cardiac output is lessened. Beta-blockers also cause an increase in smooth muscle contraction, which is why some patients who have reactive airways disease do not tolerate these agents because of an increased tendency for bronchospasm. Beta-blockers should not be used by patients who have peripheral vascular disease, as the impaired cardiac output will further decrease the delivery of blood into the peripheral circulation.

On a routine physical examination, a midsystolic ejection murmur is detected in the pulmonic area of a 35-year-old woman. The cardiac examination also reveals a prominent right ventricular cardiac impulse and wide and fixed splitting of the second heart sound. EKG shows right axis deviation and chest radiograph shows enlargement of the right ventricle and atrium. Which of the following is the most likely diagnosis?

Atrial Septal Defect (+prominent right ventricular cardiac impulse, a systolic ejection murmur heard in the pulmonic area and along the left sternal border, and fixed splitting of the second heart sound. These findings are caused by an abnormal left-to-right shunt through the defect, creating a volume overload on the right side. The increase in volume on the right side creates the flow murmur, the dilatation of the right-sided chambers, and the delayed closure of the pulmonic valve, all of which are present in this case. The delayed closure of the pulmonic valve occurs because of the increased volume of blood that is delivered to the right ventricle.)

Widening of the P waves in lead II is associated with ____________. -This condition is often referred to as "P mitrale," as it is often associated with mitral stenosis and subsequent increased force of pressure that the left atrium has to exert to overcome this resistance.

left atrial enlargement

14-year-old boy is brought to the health care provider with decreased exercise tolerance. He is up to date on all of his childhood immunizations and has been generally healthy until now. He is noted to have a grade III/VI systolic ejection murmur best heard at the left upper sternal border and a grade II/VI mid-diastolic murmur at the lower left sternal border. The first heart sound is normal. The second heart sound is widely split and fixed. A right ventricular impulse is palpated. On a chest roentgenogram, the pulmonary artery segment is enlarged, and pulmonary vascular markings are increased. An electrocardiogram shows right axis deviation. Which of the following congenital heart diseases does this patient most likely have?

Atrial Septal Defect (One of the most common types of structural congenital heart disease to present in adolescence is atrial septal defect -ASD-, and the most common presentation is a heart murmur. Some patients, however, present with arrhythmias, decreased exercise tolerance, or a paradoxic embolus)

___________-It may not cause symptoms or it can typically present in the adolescent period. If untreated, the patient may go on to develop right atrial enlargement, cardiac arrhythmias, and heart failure. Principal features include: 1. Fixed splitting of the second heart sound 2. A midsystolic pulmonary flow or ejection murmur 3. Enlargement of the right atrium and right ventricle on imaging studies 4. On palpation, a right ventricular impulse is present

Atrial septal defect

___________- are appropriate agents in the management of angina pectoris because they decrease both heart rate and blood pressure and thus the heart's oxygen demand. They also reduce mortality in patients who have coronary artery disease and post-myocardial infarction. These agents do not prevent vasospasm, so they are NOT clinically indicated in the setting of variant or Prinzmetal angina

Beta Blockers

A 28-year-old man presents to his primary care provider for his yearly physical examination. He currently smokes approximately one pack of cigarettes a day and is interested in quitting. He has no significant past medical history and has no allergies. He is interested in smoking cessation classes but would like additional help. Which of the following medications would be the most appropriate agent for treating nicotine dependence in this patient?

Bupropion was approved by the U.S. Food and Drug Administration in 1996 as a treatment for nicotine dependence. The specific mechanism of action is unclear but is thought to relate to reducing craving for nicotine as well as reducing withdrawal symptoms after cessation. When used as a smoking cessation therapy, the medication is known as Zyban. This agent is an antidepressant that inhibits neuronal uptake of norepinephrine and dopamine.

______________- supplies the lateral surface of the left ventricle. Damage to this artery results in a lateral wall myocardial infarction, which has some mortality associated with it but is not as life threatening as left main blockage

left circumflex artery

A 45-year-old woman comes to the emergency department complaining of recurring episodes of chest pain that each last a few minutes since she awoke this morning. She denies shortness of breath, radiation of the pain, or chest pain on exertion. She reports that she had similar episodes within the last month. The patient states that she has been healthy otherwise, takes no medications, and has no family history of coronary artery disease. Her vital signs and physical examination are within normal limits. Electrocardiogram shows transient ST-segment elevation. Serial cardiac enzymes reveal no abnormalities over a 24-hour period. Cardiac catheterization shows no significant areas of plaque or stenosis. Which of the following classes of drugs may be most effective in this patient?

CCB (Dx- Prinzmetal angina, or variant angina, is classically characterized by angina without the associated precipitating factors of angina pectoris. It is caused by a transient coronary artery spasm and displays a transient ST elevation on electrocardiogram. It usually has no other diagnostic findings. *Calcium channel blockers* have been shown to treat and prevent episodes of coronary vasospasm by decreasing contractility of cardiac muscle and producing vasodilation)

__________- may lead to pulmonary hypertension and cause elevated right-sided pressures, which may cause shortness of breath. The wedge pressure will be decreased, and the right ventricle will be dilated. The left ventricle would not be affected.

COPD

A 54-year-old man comes to his health care provider complaining of intermittent palpitations. The patient reports that a few times over the past few months he has had episodes of "pounding in his chest" that are associated with shortness of breath and occasional chest pain. He is forced to sit down if he is standing, because of weakness and vertigo. The patient has a history of hypertension and mitral valve prolapse. He takes nifedipine (Procardia, Adalat) and hydrochlorothiazide daily. While sitting in the office, the patient begins to complain of increasing shortness of breath and palpitations. His blood pressure is 85/50 mm Hg and his pulse is 110-130/min and irregularly irregular. Which of the following is the most appropriate management at this time?

Call 911 (patient has acute atrial fibrillation-AF- with rapid ventricular response-RVR- and is consequently hypotensive. This is a *medical emergency*. Even the PCP Cannot Deal with a potentially life-threatening episode of AF with RVR. Activating the emergency medical response system is always appropriate! The patient will need--> IV access, IVF, and medication or cardioversion as part of the treatment regimen. According to the Advanced Cardiac Life Support protocol, a patient who has AF with RVR who is HYPOtensive should be *cardioverted*, something that emergency medical personnel will have available to them.)

A 23-year-old man sustains multiple stab wounds to his left chest. At the time of emergency department admission, his blood pressure is 70/45 mm Hg, pulse 110/min, and central venous pressure 4 mm Hg. He is diaphoretic and anxious, and he is notably pale. It is quickly determined that he has a left hemopneumothorax and so a chest tube is inserted. A total of 380 mL of blood is recovered initially, and another 120 mL is suctioned during the next hour. He is given 2 liters of Ringer's lactate, followed by 2 units of blood. Reassessment at the end of the first hour shows that his lung is expanded and his central venous pressure has gone up to 22 mm Hg, but his blood pressure is only 85/70 mm Hg, his pulse has gone up to 115/min, and he remains diaphoretic. Which of the following is the most likely diagnosis at this time?

Cardiac/pericardial tamponade is caused by the accumulation of blood within the pericardial space, creating an increase in the intrapericardial pressure. This in turn results in restricted cardiac filling with a resultant decrease in cardiac output. Shock with high central venous pressure in a chest trauma victim suggests either pericardial tamponade or tension pneumothorax. The latter has been ruled out by the re-expanded lung. The problem was not initially obvious because of concomitant bleeding but became clear once blood volume was restored. Patients who have this condition will have hypotension, jugular venous distention, and distant heart sounds (known as Beck triad).

___________- of the baroreceptors will cause vagus nerve stimulation of the heart to increase, resulting in an increased parasympathetic action on the heart. This stimulation will slow the heart's conduction through the AV node. When the speed of the impulse is slowed through the antegrade pathway, it will break the reentry cycle of PSVT by slowing the antegrade pathway to a speed that approaches the speed of the retrograde pathway, thus terminating the essential requirement that the reentry dysrhythmia needs to work.

Carotid Massage

Which of the following best describes why carotid sinus massage will result in the slowing of PSVT along with return of the patient to normal sinus rhythm?

Carotid massage of the baroreceptors will cause vagus nerve stimulation of the heart to increase, resulting in an increased parasympathetic action on the heart. This stimulation will slow the heart's conduction through the AV node. When the speed of the impulse is slowed through the antegrade pathway, it will break the reentry cycle of PSVT by slowing the antegrade pathway to a speed that approaches the speed of the retrograde pathway, thus terminating the essential requirement that the reentry dysrhythmia needs to work.

A 77-year-old man comes to the health care provider because of decreasing exercise tolerance. Just 1 year earlier he was able to play doubles tennis for 2 hours. Over the past few months, however, he has had progressive dyspnea on exertion and now can walk only 2 blocks on level ground before becoming short-winded. He has also been awaking from sleep with shortness of breath and requires 3 pillows to sleep comfortably. He has a history of rheumatic fever as a teenager. On physical examination his blood pressure is 168/60 mm Hg, pulse 92/min, and respirations 18/min. He is afebrile. He has jugulovenous distention lying supine. He has bibasilar rales extending 1/4 up both posterior lung fields. He has a regular S1 and S2, with a blowing diastolic murmur heard at the aortic area, which is grade II/VI. An S3 is audible. The liver edge is mildly tender, and there is moderate lower extremity edema extending to both knees. Which of the following medications will most likely be effective in the management of his cardiac disorder?

Catopril (ACE- physical findings of aortic insufficiency and the development of congestive heart failure with biventricular failure. Left ventricular symptoms described here are the findings of pulmonary congestion, and right-sided heart failure is demonstrated by the jugulovenous distention, congested liver, and peripheral edema. Because the "backward" failure symptoms are caused by regurgitant flow across the incompetent aortic valve, the most useful therapy would be afterload reduction with an ACE inhibitor such as captopril, which will also help to prevent cardiac remodeling as the left ventricle becomes overloaded from the increased amount volume of blood that occurs because of aortic regurgitation.)

A 59-year-old woman comes to the health care provider because of recurrent episodes of lightheadedness upon getting up in the morning and occasionally upon standing up from a chair. She reports that on two occasions she has "passed out" soon after getting up from bed. Her temperature is 37°C (98.6°F), blood pressure is 130/80 mm Hg, pulse is 70/min and regular, and respirations are 14/min. She takes a beta-blocker and a thiazide diuretic for moderate hypertension diagnosed 6 months ago. She had rheumatic fever as a child. A thorough physical examination, including chest auscultation, is unremarkable. Which of the following is the most likely cause of this patient's symptoms?

Clinical history is crucial in the diagnostic approach to syncope. Furthermore, measurement of orthostatic blood pressures and pulses should be performed first with the patient in a supine position, and then checked again sitting and standing. This patient's episodes of lightheadedness and syncope can be best explained as an effect of antihypertensive therapy. Orthostatic hypotension is one of the most frequent side effects of antihypertensive drugs and should always be considered in the differential diagnosis of syncope of unexplained origin.

A patient admitted to the emergency department with chest pain is diagnosed with myocardial infarction. On discharge, the patient is prescribed aspirin but develops an allergic hypersensitivity reaction. Clopidogel (Plavix) is then prescribed. Which of the following is the mechanism of action of this drug?

Clopidogrel (Plavix) is an antiplatelet drug that inhibits adenosine diphosphate-induced platelet aggregation and interacts with platelet glycoprotein IIb/IIIa, a fibrinogen receptor that links platelets together and leads to platelet plug formation. The drug thus prevents fibrinogen from binding to platelets. The drug is useful both for patients who have coronary disease and for those who have cerebrovascular disease such as stroke. Of course, hemorrhagic stroke should first be ruled out before it is deemed safe to use any anticoagulant. Clopidogrel is used in both acute coronary syndrome (for patients not having an open bypass) and for thrombotic event prevention. For stroke prevention, it is used when aspirin or warfarin (Coumadin) cannot be used. Antiplatelet agents have been shown to be effective in the setting of acute coronary syndrome and for prevention of thromboembolic events. Aspirin is commonly used in these settings, but for patients who have a contraindication for aspirin use, clopidogrel (Plavix) is a useful alternative. The primary side effect with these antiplatelet agents is bleeding. There is a Black Box warning for clopidogrel for patients who are poor metabolizers of this medication; if they cannot metabolize clopidogrel appropriately, they wil have diminished efficacy with its use. Genotype testing is available to identify these patients.

On physical examination of a patient, the health care provider hears a rumbling mid-diastolic murmur with an opening snap heard best at the apex of the heart. What is the most likely explanation for this finding?

mitral stenosis

A 43-year-old woman comes to the health care provider with a several-month history of dyspnea on exertion. She denies chest pain or a family history of coronary artery disease. She has a history of rheumatoid arthritis. Chest radiograph is remarkable for calcification of the heart border on the lateral film. She appears to have constrictive pericarditis. Which of the following physical examination findings would most likely be expected in this patient? A. Central cyanosis B. Opening snap and diastolic rumble C. Pericardial knock D. Systolic ejection murmur E. Widely split S2

Constrictive pericarditis impedes normal diastolic filling and can be a complication of acute pericarditis. This leads to a tense effusion in the pericardial space and constriction by the thickened pericardium. Electrocardiogram is the most useful test in the assessment of this condition; it demonstrates concave ST-segment elevation globally along with PR-segment depression. Treatment involves treating the underlying cause. NSAIDs can be used to decrease inflammation. A pericardial knock, caused by early cessation of diastolic ventricular filling, is indicative of constrictive pericarditis.

A 22-year-old construction worker is brought to the emergency department after being pinned by a steel beam that fell on his legs and mid-torso. The patient was brought to the hospital by the EMS team who had extricated him. At the scene, his Glasgow score was 15/15, and he was alert and oriented. On arrival the patient appears pale and bloody and in a significant amount of pain, but still alert. Examination reveals bilaterally crushed lower extremities with a visibly pulsating bleed.

Crush injuries such as the one sustained by this patient often result in a massive release of muscle contents. Creatine kinase (CK) is an enzyme released by dead or damaged muscle into the blood. By itself it is harmless, but it is a marker for myoglobin, which is directly nephrotoxic. Much data exist showing early intervention with copious alkalinized IV crystalloid can prevent renal damage.

A 71-year-old retired physician complains of increasing calf pain when walking uphill. The symptoms have gradually increased over the past 2 months. His past medical history is significant for a transient ischemic attack 6 months ago. Over the past month, his blood pressure has worsened despite previous control with diltiazem, hydrochlorothiazide, and propranolol. His other medications are isosorbide dinitrate and aspirin. On physical examination, his blood pressure is 152/90 mm Hg and his pulse is 66/min. There is a right carotid bruit. His lower extremities are mildly cool and have diminished pulses at the dorsalis pedis. An ACE inhibitor is added to help control his blood pressure. Which of the following is the most appropriate in the management of his leg pain?

Decrease propranolol (Inderal) dose. Propranolol, a beta-blocker, is considered to be relatively contraindicated in patients who have peripheral claudication. Although beta-blockers are excellent medications in patients who have hypertension, the beta blockade may predispose to arterial vasoconstriction peripherally. In addition, beta-blockers are negative inotropic agents that decrease the force of contraction of the heart. Because the heart is not contracting as strongly, there is less blood being propelled into the peripheral vascular system. The dose of beta-blockers in this patient should therefore be reduced or gradually eliminated.

A 57-year-old woman with a history of rheumatic fever as a child comes to the her health care provider complaining of a 6-month history of slowly progressive dyspnea on exertion and orthopnea. Her temperature is 37°C (98.6°F), blood pressure is 110/60 mm Hg, pulse is 93/min and irregular, and respirations are 18/min. Cardiac examination reveals a localized mid-diastolic murmur near the apex. There is a loud opening snap heard after S2. The rhythm appears irregular. Which of the following additional findings will most likely be present on physical examination?

Decreased S1 Intensity (patient has mitral stenosis. Most adults who have mitral stenosis have had rheumatic fever as a child, although not all patients are aware of having had this infection. Mitral stenosis decreases left ventricular filling and elevates left-sided atrial pressures. This causes pulmonary congestion and results in symptoms of left-sided heart failure, such as shortness of breath and dyspnea on exertion. Hemoptysis sometimes occurs as a result of rupture of small pulmonary blood vessels.)

After an accident at work resulting in severe hemorrhage, a machinist is rushed to the emergency department. Which of the following sets of autonomic responses would be predicted in this patient? -_______ BP -_______ Pulse

Decreased, Increased (After a decrease in blood pressure e.g., after a hemorrhage, one would expect an increase in sympathetic outflow and decrease in parasympathetic outflow. As a result of the hemorrhage, there would be less blood in the body to circulate, with resultant increase in heart rate, decrease in both blood pressure and gastrointestinal motility, and dilation of the pupils)

A 55-year-old man with hypertension and a past medical history of myocardial infarction is prescribed atenolol (Tenormin). This medication will lower his blood pressure by

Decreasing Cardiac Output (Beta Blockers- Atenolol- lower BP by both dec CO and Renin Release from Kidneys)

A 60-year-old woman with a long history of hypertension comes to the emergency department with a complaint of increasing and recurrent shortness of breath with minimal exertion. She is noted to have a prominent precordial impulse, and a chest radiograph reveals a prominent left ventricular shadow. A stress test is negative for ischemia. She is found to have left ventricular hypertrophy and a normal ventricular ejection fraction on echocardiogram. An S3 is heard on auscultation of the heart. Which of the following is the most likely underlying diagnosis causing this presentation?

Diastolic Dysfunction (Increased resistance to filling of one or more cardiac ventricles has been termed diastolic heart failure and can produce increased pulmonary capillary wedge pressures and respiratory complaints. In myocardial hypertrophy, impaired diastolic relaxation occurs. The key to differentiating systolic from diastolic types of heart failure is in the assessment of ejection fraction; it is normal and preserved with diastolic heart failure and impaired with systolic)

___________- is the clinical syndrome of heart failure, with symptoms of pulmonary and peripheral congestion in the setting of normal left ventricular systolic function (as verified with finding a normal ejection fraction). Risk factors include advancing age, female sex, hypertension, obesity, chronic kidney disease, diabetes, and coronary artery disease. Risk factor modification is the treatment of choice along with ACE inhibitors or ARBs + beta-blockers and diuretics.

Diastolic heart failure

Although _________ is a mainstay in the therapy of CHF, it has no effect on either preload or afterload. Instead it works by having a positive inotropic and negative chronotropic effect on the heart. The mechanism involves inhibition of Na+/K+ ATPase and a consequent increase in intracellular calcium, which in turn increases the heart's contractile force.

Digoxin

An elderly man presents with complaints of dizziness, headaches, diarrhea, nausea and vomiting, weakness, palpitations, and a change in vision with a yellowish to blue tint to his vision. He is taking multiple medications. He has a history of chronic heart failure and hypertension. His wife states that he has had a few episodes of confused, delirious behavior over the past few weeks. Which of the following agents might be responsible for this man's symptoms?

Digoxin (Lanoxin- )

A patient who is being treated for hypertension related to a myocardial infarction that occurred 2 hours ago is medicated with IV nitroprusside (Nipride). Which of the following is the expected action of this drug?

Dilatation of Arterioles and Venules (Nitro IV has rapid onset and used in ER & ICU. It can be titrated to effect on Drip-to-Drip basis. Nitro is on ACLS Protocol as well as Labetolol for HTN Emergency/Urgency.

__________- recommended first-line treatment for mild to moderate essential hypertension. This class of drugs decreases blood pressure by decreasing sodium, blood volume, and afterload. These drugs also provide symptomatic relief for patients who have congestive heart failure

Diuretics

A 59-year-old man with a history of myocardial infarction presents to his health care provider complaining of shortness of breath. On examination, his pulse is 110/min and his respiratory rate is 22/min. He has rales in both lung fields, a normal sinus rhythm with an S3 gallop, and 2+ pitting ankle edema. A chest radiograph reveals cardiomegaly, and his ejection fraction on echocardiogram is calculated at 37%. Which of the following medications would alleviate this patient's symptoms by significantly reducing both the preload and afterload on the heart without affecting its inotropic state?

Enalapril (Everything in this case points to congestive heart failure -CHF- dyspnea, elevated heart rate, S3, peripheral edema, and reduced ejection fraction. A drug that will alleviate the symptoms by: *decreasing both preload and afterload is necessary*)

Sx of ___________- +Roth spots are oval, pale, retinal lesions that are surrounded by hemorrhages. +Janeway lesions, which are hemorrhagic, painless, macular plaques typically located on the palms and soles, +Osler nodes, which are small, painful nodular lesions typically found on the pads of the fingers or toes.

Endocarditis

_______________- is an infection that involves the endocardial surface of the heart and includes the valves. If the patient is symptomatic, presentation includes +fever, tachycardia, and fatigue. The patient may have a new-onset heart murmur or a change in the previously heard heart murmur. Echo should be performed in all patients suspected of having this. Three sets of blood cultures should be performed to identify the pathogen. Injection drug users are at high risk for _________ infection leading to infective endocarditis.

Endocarditis, Staph Aureus (Infective endocarditis)

A 59-year-old man with a history of myocardial infarction presents to his health care provider complaining of shortness of breath. On examination, his pulse is 110/min and his respiratory rate is 22/min. He has rales in both lung fields, a normal sinus rhythm with an S3 gallop, and 2+ pitting ankle edema. A chest radiograph reveals cardiomegaly, and his ejection fraction on echocardiogram is calculated at 37%. Which of the following medications would alleviate this patient's symptoms by significantly reducing both the preload and afterload on the heart without affecting its inotropic state? Digoxin, DIltiazem, Enalapril, Furosemide, Propranolol

Everything in this case points to congestive heart failure (CHF): dyspnea, elevated heart rate, S3, peripheral edema, and reduced ejection fraction. A drug that will alleviate the symptoms by decreasing both preload and afterload is necessary. Enalapril is an angiotensin-converting enzyme (ACE) inhibitor. ACE inhibitors (captopril, enalapril, lisinopril, benazepril, ramipril) work by blocking the conversion of angiotensin I to angiotensin II in the lungs. Angiotensin II is both a potent vasoconstrictor and a stimulator of aldosterone production. Aldosterone acts by promoting sodium (and thus water) reabsorption by the kidney. An ACE inhibitor will therefore promote vasodilatation (reducing afterload), as well as reduce intravascular volume (decreasing aldosterone, as less renin is being released). ACE inhibitors are first-line agents for patients who have heart failure because they act as vasodilators. ACE inhibitors decrease afterload, which allows the heart to have improvement in its ejection fraction. They do not have an inotropic or chronotropic effect on the heart. There is a mortality benefit demonstrated with the use of these agents. Beta-blockers and the aldosterone antagonist, spironolactone, also have been shown to have an improvement in mortality in patients who have chronic heart failure.

First-line management of *shock* is _______--> If the patient does not respond --> ________ and colloids such as albumin can be used along with vasopressors, which can help to maintain the vascular integrity and can be used to continue to stimulate the heart.

Fluids, Blood Transfusion (fluid resuscitation with crystalloids, Blood Transfusion-primarily to allow better release of oxygen to the tissues)

A 59-year-old woman comes to the health care provider because of recurrent episodes of lightheadedness upon getting up in the morning and occasionally upon standing up from a chair. She reports that on two occasions she has "passed out" soon after getting up from bed. Her temperature is 37°C (98.6°F), blood pressure is 130/80 mm Hg, pulse is 70/min and regular, and respirations are 14/min. She takes a beta-blocker and a thiazide diuretic for moderate hypertension diagnosed 6 months ago. She had rheumatic fever as a child. A thorough physical examination, including chest auscultation, is unremarkable. Which of the following is the most likely cause of this patient's symptoms?

HTN Tx ( Furthermore, measurement of orthostatic blood pressures and pulses should be performed first with the patient in a supine position, and then checked again sitting and standing. This patient's episodes of lightheadedness and syncope can be best explained as an effect of antihypertensive therapy. Orthostatic hypotension is one of the most frequent side effects of antihypertensive drugs and should always be considered in the differential diagnosis of syncope of unexplained origin)

A 29-year-old man is brought to the emergency department in a comatose state a few hours after complaining of sudden onset of excruciating headache. His friend does not know if the patient has any underlying medical conditions. Neurologic examination reveals dilated pupils poorly responsive to light. A CT scan of the head without contrast demonstrates hyperdensity within the suprasellar cistern, whereas MRI scan is unremarkable. Lumbar puncture shows hemorrhagic cerebrospinal fluid. Which of the following is the most likely diagnosis?

Headache of sudden onset ("thunderclap" headache), rapid deterioration of mental status, and blood in the CSF are virtually diagnostic of ruptured berry aneurysms. Note the characteristic hyperdensity on CT scan of the suprasellar cistern, indicating blood in the subarachnoid space. Rupture of a berry aneurysm is the most common cause of subarachnoid bleeding. Berry aneurysms develop as a result of congenital weakness at branching points of the arteries in the circle of Willis. These outpouchings tend to expand progressively, but in most cases they remain asymptomatic. Hypertension facilitates development and rupture of berry aneurysm. One third of patients recover, one third die, and one third develop re-bleeding. Rapid onset of coma is an ominous sign.

After an accident at work resulting in severe hemorrhage, a machinist is rushed to the emergency department. Which of the following sets of autonomic responses would be predicted in this patient? (BP & HR)

Hypotension & Tachycardia.After a decrease in blood pressure (e.g., after a hemorrhage), one would expect an increase in sympathetic outflow and decrease in parasympathetic outflow. As a result of the hemorrhage, there would be less blood in the body to circulate, with resultant increase in heart rate, decrease in both blood pressure and gastrointestinal motility, and dilation of the pupils.

Crush injuries such as the one sustained by this patient often result in a massive release of muscle contents. Creatine kinase (CK) is an enzyme released by dead or damaged muscle into the blood. By itself it is harmless, but it is a marker for myoglobin, which is directly nephrotoxic. Myoglobin, which is not measured by conventional assay, is released after damage to muscle. Aldolase levels can also be used to assess the degree of rhabdomyolysis. Much data exist showing early intervention with ____________ can prevent renal damage.

IV Crystalloid and Bicarbonate

A 63-year-old woman comes to the emergency department complaining of chest pain. The patient states that the pain began during her morning walk. It started as a dull pressure over her breastbone and then radiated to her left arm. Over the next few minutes, it escalated in intensity and was not relieved by rest. She called 911 and was brought to the emergency department. Her past medical history is significant for hypertension and hyperlipidemia. Her medications include atenolol and simvastatin daily. On physical examination, her blood pressure is 190/100 mm Hg, and her pulse is 60/min. Which of the following is the most appropriate agent to lower her blood pressure?

IV Nitro (Nitroglycerin is a potent vasodilator that acts predominantly on venous compliance to reduce preload to the heart. When administered intravenously, it can be rapidly titrated to produce optimized blood pressure control. IV nitroglycerin is effective when given in the setting of unstable angina)

The management of myoglobinuria includes: 1. _______ challenge to maintain urine output 1-2 mL/kg/h 2. Alkalinization of the urine: add __________ 3. Forced diuresis: add ________

IVF, Sodium Bicarb, Mannitol

A 59-year-old man who is scheduled for an abdominal aortic aneurysm (AAA) repair in 3 weeks presents to the health care provider's office. The patient's AAA was diagnosed last week via ultrasound. Imaging at that time revealed a 5.5-cm aneurysm of his abdominal aorta extending bilaterally into his iliac arteries. The patient also has moderate hypertension, with a mean daily blood pressure of 150/95 mm Hg. On physical examination, the patient appears in no distress. He weighs 274 pounds and is 5 feet 9 inches tall (BMI is 40.5). His lungs are clear, and he has a loud S4. His AAA is palpable as a pulsatile mass in his abdomen. He complains of very mild back pain. Which of the following is the most appropriate intervention to prepare this patient for surgery?

Improve blood pressure control. The most important interventions involve lessening the chance for rupture of an abdominal aortic aneurysm (AAA). Other interventions include counseling for smoking cessation and improving blood pressure control. Abdominal aortic aneurysm rupture typically has devastating consequences, which is why elective repair is indicated when the aneurysm size is 5.5 cm in males or 5.0 cm in females. Once an AAA is identified, patients are assessed for factors that are known to increase the risk for rupture, such as uncontrolled hypertension, smoking, and trauma to the abdomen. Beta-blockers are typically given, as they are known to be cardioprotective and because they will lower blood pressure. Beta-blockers have been shown to reduce morbidity and mortality for high-risk patients undergoing major noncardiac surgery, so these should be given to patients who have AAA unless there is a contraindication for their use.

A 52-year-old man is discharged from the hospital after an uncomplicated myocardial infarction. Several weeks later, he visits his primary care provider complaining of insomnia, anorexia, and depressed mood. He appears to be clinically depressed. He denies any current chest pain or shortness of breath. Which of the following would be the most appropriate medication to initiate for this patient?

In a patient who has cardiac complications, a selective serotonin reuptake inhibitor (SSRI) such as sertraline has been demonstrated to be the safest and most effective medication for the treatment of clinical depression. It has overtaken tricyclics in the management of depression in patients who have pre-existing heart disease, because it is not associated with prolongation of the QT interval that the tricyclics are known to cause. When compared to tricyclics, SSRIs also have a safer profile if a patient overdoses.

A 43-year-old woman comes to the health care provider with a several-month history of dyspnea on exertion. She denies chest pain or a family history of coronary artery disease. She has a history of rheumatoid arthritis. Chest radiograph is remarkable for calcification of the heart border on the lateral film. She appears to have constrictive pericarditis. Which of the following physical examination findings would most likely be expected in this patient?

In constrictive pericarditis, the lateral view of the chest may demonstrate calcification of the anterior pericardium. This may be seen in 50% of patients who have longstanding constriction. The pericardial thickening may be seen on an echocardiogram. A pericardial knock is heard after the aortic valve closes, which is caused by the sudden cessation of ventricular filling.

A 60-year-old woman with a long history of hypertension comes to the emergency department with a complaint of increasing and recurrent shortness of breath with minimal exertion. She is noted to have a prominent precordial impulse, and a chest radiograph reveals a prominent left ventricular shadow. A stress test is negative for ischemia. She is found to have left ventricular hypertrophy and a normal ventricular ejection fraction on echocardiogram. An S3 is heard on auscultation of the heart. Which of the following is the most likely underlying diagnosis causing this presentation?

Increased resistance to filling of one or more cardiac ventricles has been termed diastolic heart failure and can produce increased pulmonary capillary wedge pressures and respiratory complaints. In myocardial hypertrophy, impaired diastolic relaxation occurs. The key to differentiating systolic from diastolic types of heart failure is in the assessment of ejection fraction; it is normal and preserved with diastolic heart failure and impaired with systolic.

A 71-year-old man comes to his health care provider for follow-up of a recent emergency department visit. The patient has a 2-year history of mild congestive heart failure in the setting of longstanding hypertension. He reports that yesterday he sought care at the local emergency department for palpitations and shortness of breath. He was told that his heart was "fibrillating," but later the fibrillation had "stopped on its own." His medications include a thiazide diuretic and an ACE inhibitor. On physical examination he appears well and in no distress. His blood pressure is 130/80 mm Hg and pulse 100/min and regular. His lungs have scant bibasilar rales and no gallops are appreciated. He has a grade 2 holosystolic murmur heard best at the apex. His jugular venous pressure (JVP) is 10 cm at 30 degrees. An electrocardiogram taken in the office reveals atrial fibrillation at a rate of 94/min with normal ST segments. Which of the following is the most appropriate next step in management?

Initiate Beta Blocker Tx

A 67-year-old man comes to the clinic complaining of steady, dull back pain over the past 3 weeks. He states that he has recently moved after retiring from a career in banking and is searching for a new health care provider. His past medical history is significant for diverticulosis, prior smoking, and hypertension. He says that he has run out of his blood pressure medication. He denies trauma to his back and otherwise feels well. On physical examination, his blood pressure is 170/93 mm Hg with a pulse of 88/min. He has no tenderness over the spinal processes or paraspinal areas. His abdomen is obese, but there is a suggestion of a non-tender, pulsatile mass in the epigastric region. The remainder of the physical examination is normal. Which of the following diagnoses should be considered at this time?

It is imperative to recognize the potential presence of an abdominal aortic aneurysm (AAA). The combination of the history of hypertension and smoking, the new back pain, and a pulsatile mass on examination is highly suggestive for abdominal aneurysm. Additionally, he is a male older than 65 years of age, which also puts him at higher risk for AAA development. The back pain occurs as the expanding mass compresses structures in the retroperitoneum. It is particularly important to make the diagnosis, because large aneurysms (greater than 5 cm in diameter) are associated with a very high risk for rupture and subsequent mortality.

A 58-year-old man with history of atrial fibrillation is prescribed warfarin to prevent clot and embolism formation. His prothrombin time (PT) is regularly monitored. Administration of which of the following drugs would result in an increase in his PT and require readjustment of his warfarin dosage? Rifampin, Carbemazepine, Ketoconazole, Phenytoin

Ketoconazole. This question is testing a favorite exam concept—the relationship between the hepatic cytochrome P-450 system and the metabolism of drugs. Certain drugs can affect the metabolism of other drugs by either inducing or inhibiting hepatic microsomal enzyme activity. This means that the amount of circulating warfarin increased and that you are looking for a drug that inhibits the P-450 system (thereby decreasing warfarin metabolism). Of the drugs listed, only ketoconazole inhibits the P-450 system. Other important inhibitors include cimetidine (Tagamet) and isoniazid (INH).

Widening of the QRS complexes in leads V5 and V6 is associated with __________ and _________. -This increased force of contraction is in sharp contrast to a patient who has impaired ventricular dysfunction caused by cardiomyopathy.

LBBB and left ventricular hypertrophy

____________- this artery has primary arterial supply to the left ventricle. Blockage of this artery leads to anterior wall and lateral wall myocardial infarction. Mortality for patients who have left main coronary artery disease is more than 10 times greater than in patients who have one- or two-vessel disease involving the other coronaries.

Left Main Artery

The mortality rate per year is greatest with disease of which coronary anatomy?

Left Main Coronary (primary arterial supply to the left ventricle. Blockage of this artery leads to anterior wall and lateral wall myocardial infarction. Mortality for patients who have left main coronary artery disease is more than 10 times greater than in patients who have one- or two-vessel disease involving the other coronaries_

_________- They act by inhibiting electrolyte reabsorption in the thick ascending loop of Henle.

Loop Diuretics (Furosemide and Ethacrynic Acid)

________-it decreases intravascular volume (and hence preload), but at typical doses it has no significant effect on either afterload or inotropic state. They are often used in the treatment of CHF, and it is used for the immediate relief of pulmonary congestion.

Loop Diuretics (Furosemide)

12-year-old African American boy is brought to the office for a well child examination. He has been in good health and only complains of an occasional headache. He has been doing reasonably well in school but has some social problems that his mother attributes to his physical appearance. She has tried persuading her son to eat healthier, but he seems to enjoy eating fast food much more than home cooked meals. His past medical history is unremarkable, and the family history is significant for adult-onset hypertension in his father's family. He takes no medication. On physical examination, the patient is in no acute distress. He is 145 centimeters tall and weighs 92 kilograms. His vital signs are within normal limits, but his blood pressure is 145/90 mm Hg. The health care provider checks that the cuff size is appropriate and remeasures his blood pressure in all four extremities, only to confirm that it is above the ninety-fifth percentile for his age. After discussing the finding with his mother, he is scheduled for several more blood pressure evaluations over the following 6 weeks. All readings yield results mildly above the ninety-fifth percentile for his age. Which of the following recommendations is most appropriate at this time? -1st _________, _______ and _______

Lose Weight, Decrease Sodium and Exercise (Dx- Systemic HTN is BP > than 95% percentile for Age on repeated measurements over 6weeks. Therapy of hypertension in children is the same as in adults: diet, exercise, and medication. Pharmacologic management includes angiotensin-converting enzyme inhibitors, calcium-channel blockers, and diuretics.)

Patients having dilated cardiomyopathy will have which of the following changes on the EKG?

Low Amplitude QRS Complex

Patients having dilated cardiomyopathy will have which of the following changes on the EKG?

Low amplitude of the QRS complexes. In dilated cardiomyopathy the EKG often shows sinus tachycardia, or atrial fibrillation, ventricular arrhythmias, left atrial abnormality, diffuse nonspecific ST-wave abnormalities, intravascular conduction defects, and low voltage. The low voltage is caused by the impaired cardiac function of the heart because of the cardiomyopathy. Because the EKG measures the electrical strength of contraction, dilated cardiomyopathy will show impairment of the strength of contraction as the cardiac myocytes become less functional.

A patient with chronic, well-compensated congestive heart failure presents with increased dyspnea and peripheral edema. Which of the following precipitating factors least likely contributes to this clinical outcome? -Start __________

Low sodium diet (is indicated in patients who have mild heart failure and may provide symptomatic improvement, especially if accompanied by periods of physical rest. Indiscriminate use of salt is one of the major precipitants for deterioration in the clinical status of a patient who has chronic heart failure. Increased sodium in the diet leads to increased water retention with an increase in the amount of intravascular volume that the patient must circulate.)

A 25-year-old man comes to the health care provider with chest pain at rest that is not always related to exercise. He reports two prior episodes of fainting during exercise but has otherwise been healthy. He reports a similar history in other family members and notes that his father suddenly collapsed and died at age 50 after playing tennis. He denies use of cocaine or other recreational drugs and does not take any medications. Physical examination reveals a systolic ejection murmur that is loudest along the left sternal border. The rest of the physical examination is unremarkable. Echocardiography shows asymmetric septal hypertrophy without obstruction. Which of the following interventions would be most likely to decrease this patient's systolic murmur?

Lying Down (pt has Hypertrophic Obstructive Cardiomyopathy- The systolic ejection murmur is diminished when the patient lies down, because this maneuver increases cardiac size by increasing venous return and tends to diminish the intensity of the murmur. Afterload is increased, and venous return is increased to the heart. This increases the ventricular size and diminishes the murmur.)

A 55-year-old man with no known cardiac history comes to the emergency department complaining of crushing substernal chest pressure that began 20 minutes prior to his arrival. He took sildenafil earlier in the evening prior to sexual intercourse. His past medical history is significant for bilateral inguinal hernia repairs 6 weeks ago, remote peptic ulcer disease, and prior cocaine and heroin addiction. On examination, he is diaphoretic and appears anxious. His blood pressure is 150/75 mm Hg with a pulse of 100/min. An electrocardiogram (ECG) obtained while he had severe chest pain revealed tall positive T waves and 1-mm elevation of the ST segments in leads V2 through V5. A serum creatine kinase (CK) drawn on presentation returned at 85 U/L (normal <250 U/L). Which of the following is the most likely diagnosis?

MI. The combination of substernal chest pressure with ECG findings of hyperacute T waves and ST elevation in this clinical setting suggests early anterior myocardial infarction. The myocardial damage causes time-dependent effects on the electrical properties of the myocardial cells. The earliest electrical evidence of myocardial injury is often development of hyperacute T waves, followed by elevation of the ST segments, inversion of the T wave, return of the ST segments to normal, and finally, the development of Q waves. It does not matter that the CK, a marker of myocardial injury, is not elevated, as it takes several hours for the CK to appreciably rise after myocardial damage has occurred. Myocardial infarction is diagnosed by having positive findings in at least two out of the following three criteria: typical symptoms of chest pain, EKG changes (>1 mm ST-segment elevation or depression in two or more contiguous leads, T-wave inversions), and elevations in cardiac markers. There is no need to wait for the cardiac markers to be elevated to diagnose myocardial infarction. The markers take at least 4 hours to become elevated. Treatment can be based on the classic EKG findings and patient history alone when deciding whether an ST-segment elevation MI is occurring. There are definite time limitations when considering thrombolytic therapy, although that window continues to increase in the setting of continued chest pain on the part of the patient.

During the performance of a supraclavicular node biopsy under local anesthesia, a hissing sound is suddenly heard, and the patient suddenly dies. At the time of the catastrophic event, the target node was under traction, and the final cut was being made blindly behind it to free it up completely. The patient, an otherwise healthy 24-year-old man, was inhaling at that moment. Which of the following most likely caused this patient's death?

Major vein injury with air embolism . Major veins at the base of the neck have negative pressure during inspiration and, if injured at that moment, will suck air rather than bleed. The air embolism then leads to sudden death.

A 63-year-old white man who has recently retired from work as a plumber for over 30 years returns to his health care provider saying that he has been feeling very down lately, and has been having decreased appetite and a loss of interest in activities that used to give him pleasure. He is a smoker, drinks no alcohol, and is being treated by his health care provider for moderate essential hypertension. Physical examination is unchanged from his previous visits. Which of the following is the most appropriate next step in management?

Many medications used to control hypertension, such as propranolol, and in the past, reserpine, are known to occasionally lead to depressive symptoms. By evaluation of the patient's medication record, the physician can evaluate which medications were started at what time and can make adjustments to dosage or switch to alternative medications to control hypertension. Changing the antihypertensive medication will possibly improve the depressive symptoms without the need to start an antidepressant.

Which of the following medications is the most appropriate choice for the treatment of hypertension during pregnancy?

Methyldopa (alpha 2 agonist, decrease BP by Decreasing Peripheral Vascular Resistance. *Drug of Choice for Tx HTN during Pregnancy that may lead to Pre-Eclampsia.)

Which of the following medications is the most appropriate choice for the treatment of hypertension during pregnancy? Lisinopril, Atenolol, Verapamil, Furosemide, Methyldopa

Methyldopa, an alpha-2 adrenergic agonist, decreases blood pressure by decreasing peripheral vascular resistance. It is the drug of choice for treating hypertension during pregnancy that may lead to pre-eclampsia, which is the most common cause of maternal and fetal morbidity and mortality. It acts on the central circulation as an alpha-agonist. It is listed as category B for use in pregnancy, which maintains that it has been proven safe for use in the first trimester of pregnancy.

A 57-year-old man comes to his health care provider for a preoperative evaluation. He has been a long-time patient in this office and has been treated for hypertension and gastritis. He has been scheduled for an elective cholecystectomy in 2 days because of ongoing gallbladder symptoms. He currently takes omeprazole (Prilosec) for his gastritis and thiazide for his hypertension daily. He smokes two packs of cigarettes per day. His home blood pressure log shows that his systolic pressures range from 150 to 190 mm Hg, and his diastolic pressures range from 80 to 105 mm Hg, indicating that his blood pressure may be not adequately controlled for the surgical procedure. Which of the following medications is most appropriate in the perioperative period for added blood pressure control?

Metoprolol (extensive body of literature indicating that beta-blockers given to non-cardiac surgical patients who are at risk for cardiac events are associated with a more favorable outcome in terms of postoperative cardiovascular morbidity and mortality. This patient has somewhat poorly controlled hypertension, as well as at least three cardiovascular risk factors -hypertension, tobacco, age. Ideally one would like to have better control of the blood pressure and to reduce any risk for adverse perioperative events before the patient has elective surgery. Beta-blockers can achieve both of these endpoints. Beta Blockers may have utility in the perioperative setting because of their cardiac morbidity benefit)

On physical examination of a patient, the health care provider hears a rumbling mid-diastolic murmur with an opening snap heard best at the apex of the heart. What is the most likely explanation for this finding?

Mitral Stenosis

A 57-year-old man comes to his health care provider for a preoperative evaluation. He has been a long-time patient in this office and has been treated for hypertension and gastritis. He has been scheduled for an elective cholecystectomy in 2 days because of ongoing gallbladder symptoms. He currently takes omeprazole (Prilosec) for his gastritis and thiazide for his hypertension daily. He smokes two packs of cigarettes per day. His home blood pressure log shows that his systolic pressures range from 150 to 190 mm Hg, and his diastolic pressures range from 80 to 105 mm Hg, indicating that his blood pressure may be not adequately controlled for the surgical procedure. Which of the following medications is most appropriate in the perioperative period for added blood pressure control?

Metoprolol. There is an extensive body of literature indicating that beta-blockers given to non-cardiac surgical patients who are at risk for cardiac events are associated with a more favorable outcome in terms of postoperative cardiovascular morbidity and mortality. This patient has somewhat poorly controlled hypertension, as well as at least three cardiovascular risk factors (hypertension, tobacco, age). Ideally one would like to have better control of the blood pressure and to reduce any risk for adverse perioperative events before the patient has elective surgery. Beta-blockers can achieve both of these endpoints. These agents are available parenterally and can be given on a dose titration basis. Although the most recent blood pressure guidelines do not list beta-blockers as first-line agents for blood pressure control, these agents may have utility in the perioperative setting because of their cardiac morbidity benefit.

A 21-year-old professional dancer complains of several episodes of near loss of consciousness during a performance. She has been in excellent health and is a principal dancer in the New York City Ballet Corps. She has no family history of coronary artery disease. She does not smoke, and a recent cholesterol profile was normal. On physical examination, her blood pressure is 142/88 mm Hg and her pulse is 84/min and regular. She has a brisk carotid upstroke with a double impulse palpable. She has a loud S4 and a harsh systolic murmur heard along the left sternal border. The murmur is accentuated during the Valsalva maneuver and when she stands from a squatting position. An electrocardiogram reveals severe left ventricular hypertrophy. Which of the following is the most appropriate medication in the management of this patient?

Metoprolol. This patient has the presentation of hypertrophic cardiomyopathy, a frequent cause of syncope or near syncope in young patients. She has a systolic ejection murmur that is heard at the lower left edge of the sternum, which is exacerbated by exercise and standing and lessened by lying supine or squatting. This murmur can be distinguished from other systolic murmurs by its increase with the Valsalva maneuver or standing from a squatting position. Any maneuver that acts to decrease venous return of blood to the heart and left ventricular size will increase the murmur of hypertrophic cardiomyopathy, because the obstructive component increases as the left ventricular cavity shrinks. Beta-blockers such as metoprolol help to slow ventricular rate, thereby increasing ventricular filling time. When there is increased time for ventricular filling, there will be more blood in the left ventricle and the patient's obstruction will be lessened. This patient is at risk for sudden cardiac death and should be kept from exercising, which may decrease blood volume and which can have accentuated venous blood pooling when the patient rests from exercise. An implantable cardiac defibrillator may also be used to treat the patient's sudden cardiac death should it occur. All affected patients need to refrain from high intensity exercise.

A 7-year-old girl is brought to the office because of fever, headache, and malaise that has been increasing over the previous 7 days. She had been away at her grandparents' house in the countryside for vacation and had come down with a sore throat and fever. Her grandmother had given her some over-the-counter cold medication and kept her in bed for a few days, but when she had not been getting better, they had brought her back to the city. Her mother noted a faint red rash that has been waxing and waning on her chest, back, and shoulders, that would become very red after a hot bath. The girl's past medical history is unremarkable and she has not been on any medications recently. On physical examination, the patient is in moderate distress with a temperature of 39.4°C (102.9°F), pulse of 120/min, and respirations of 30/min. She complains of chest discomfort and repeatedly coughs during the examination. Deep breaths cause moderate chest pain, so her breathing is shallow. She also has difficulty swallowing. The tonsils are edematous, erythematous, and covered with purulent exudate. Her ankles are swollen, erythematous, and with limited range of motion. The rash her mother described is present only on the right shoulder in the form of an annular erythematous plaque, 5 cm in diameter. A rapid strep test done in the office is positive. Which of valvular disease is most likely to develop in this patient?

Mitral Valve. This patient has acute rheumatic fever with signs of evolving carditis. Mitral valve disease is the most common complication, occurring in up to 70% of cases. Acute rheumatic fever is caused by infection with group A β-hemolytic streptococci. It is most commonly seen in children aged 5-15 years because they are most susceptible to infection with group A β-hemolytic streptococci. Skin infections with group A β-hemolytic streptococci do not predispose to rheumatic fever, but upper respiratory infections do. Acute rheumatic fever usually presents 1-5 weeks after a preceding streptococcal pharyngitis. Diagnosis is based on the Jones criteria that include clinical signs and symptoms. Acute rheumatic fever is diagnosed if there are two major criteria, one major and two minor criteria plus preceding streptococcal infection, and chorea without any other underlying cause. If arthritis is one of the presenting major criteria, then arthralgia cannot be a presenting minor criterion. Polyarthritis is very common and may be the earliest manifestation of rheumatic fever. It commonly involves the large joints of the lower extremities (knees, ankles) and is migratory in character. Aspirin readily suppresses the signs and symptoms and prevents further migration. Erythema marginatum occurs in up to 13% of patients as 1- to 3-cm pink macules and papules and annular plaques that wax and wane within hours and are greatly accentuated by heat. Subcutaneous nodules are infrequently found on the extensor surface of the extremities. Management of acute rheumatic fever begins with treatment of the streptococcal infection and monthly penicillin prophylaxis. Salicylates help control the arthritis and also treat carditis without failure. Steroids are used when there is carditis with heart failure, and heart failure is treated in the conventional manner. Valvular disease is the most common complication of rheumatic fever. In order of frequency, the mitral, aortic, tricuspid, and pulmonary valves may be involved. Mitral insufficiency presents as a new murmur that is apical, pansystolic, high-pitched, and of a blowing character. It radiates to the left axilla and is grade 2/6 or greater. Pancarditis is the second most common and the most serious complication of acute rheumatic fever, presenting as dyspnea, mild to moderate chest discomfort, pleuritic chest pain, edema, cough, or orthopnea. Rheumatic fever can be prevented by administering penicillin within 9 days of becoming infected with group A β-hemolytic streptococci. Rheumatic fever is most common in developing countries, especially in the tropics.

A 55-year-old woman has been known for years to have mitral valve prolapse. She has now developed exertional dyspnea, orthopnea, and atrial fibrillation. She has an apical, high-pitched, holosystolic heart murmur that radiates to the axilla and back. Because of her deterioration, surgery has been recommended. Which of the following is the most appropriate procedure?

Mitral valve annuloplasty. The physical findings are classic for mitral valve insufficiency. Whenever possible, repair of the native mitral valve is preferable to replacement. The way to repair an insufficient valve is to tighten the annulus, bringing the leaflets closer to one another, allowing better coaptation of the valve. Repair of the mitral valve, rather than replacement, allows better preservation of the left ventricle and no need for long-term anticoagulation. Mitral insufficiency (regurgitation) is associated with a holosytolic blowing murmur at the apex of the heart that radiates into the axilla. Mild to moderate disease can be asymptomatic for years, but as it progresses there is eccentric cardiac hypertrophy with increased left ventricular end diastolic volume. This volume overload leads to left ventricular dysfunction. Mitral valve prolapse is a risk factor for the development of mitral regurgitation. Treatment of mitral regurgitation will depend on whether the condition is acute or chronic. For chronic cases, treatment will depend on the left ventricular function; with an adequate left ventricular ejection fraction, acceptable forms of treatment are ACE inhibitors, beta-blockers, diuretics, and surgery, which would include valvuloplasty or annuloplasty (preferentially) or mechanical valve replacement with or without anticoagulation (depending on whether it is a mechanical valve or bioprosthetic valve). Mitral valve repair is preferred over replacement because it is able to better support left ventricular function.

_________- second-degree heart block: progressive lengthening of the PR interval with successive heartbeats, culminating in a nonconducted atrial depolarization. This particular type of heart block is benign and does not require therapy.

Mobitz type 1 (Wenckebach phenomenon)

___________- second-degree heart block (intermittent failure of atrial depolarization to conduct to the ventricles), which often progresses to complete (third-degree) heart block. When the patient is symptomatic, management typically requires the placement of a permanent pacemaker.

Mobitz type 2

_____________- is inflammation of the myocardium without acute or chronic ischemia. It is a clinical syndrome of nonischemic myocardial inflammation resulting from a heterogeneous group of infectious, immune, or nonimmune diseases. Histologically, it is characterized by an inflammatory cellular infiltrate *without evidence of myocyte injury*. Systemic corticosteroids may be helpful with inflammatory and autoimmune myocarditis. Treatment of the underlying cause is helpful if a cause can be found.

Myocarditis

A 55-year-old man comes to the emergency department complaining of chest pain that radiates to his left arm and shortness of breath for the past hour. Which of the following cardiac enzymes is the most sensitive for acute muscle damage? CK-MB, LDH, Myoglobin, AST, Troponin-T

Myoglobin identifies any muscle damage but is so sensitive that it is not routinely used in the evaluation of myocardial infarction. It is heme protein that is mainly found in muscle. It rises in 2-3 hours and is rapidly cleared by the kidneys. It may be used as a "rule out" for patients presenting urgently to a health care facility because of its sensitivity; if myoglobin is normal, there is little chance that any significant muscle damage is present. Any muscle damage in the body will cause myoglobin levels to increase. Patients suspected of having myocardial infarction typically will have only the CK-MB and troponin levels drawn. Myoglobin test could be used but rarely is in the setting of suspected cardiac damage because it is too sensitive at identifying muscle damage. Troponin I is more specific than troponin T because it is not affected by chronic kidney diseaseMost providers will primarily rely on troponin I to help risk-stratify patients who present to an urgent care setting with complaints related to the chest.

A 32-year-old woman comes to the hospital for an elective repeat cesarean delivery. Four years ago she had a primary cesarean delivery for a nonreassuring fetal heart rate tracing. Two years ago she chose to have an elective repeat cesarean delivery rather than attempt a vaginal birth after cesarean (VBAC). Her prenatal course was uncomplicated except that she has mitral valve prolapse. An echocardiograph demonstrated the mitral valve prolapse but no other structural cardiac disease. Which of the following is the correct management of this patient specifically as related to her mitral valve disease? Are ABX Needed?

NO (Consequently the issue of mitral valve prolapse and the need for antibiotics is frequently debated.)

_________ and ________- are parenteral agents indicated for treating hypertensive urgency and hypertensive emergency. They may also be used to treat hypertension that is related to sympathomimetic drug use (such as cocaine, amphetamines, phenylpropanolamine, MAO-related hypertensive crisis). As a potent arterial and venous dilator, it is able to reduce both preload and afterload. It is typically administered with a beta-blocker, because its potent nitroprusside-induced vasodilation induces a strong compensatory tachycardia that might increase stress to the system.

Nitro and Labetolol (Sodium nitroprusside and Labetolol)

A 61-year-old man is brought to the emergency department for chest pain. The patient has a long history of coronary artery disease and is status post-coronary bypass procedure 6 years ago. The patient has chronic stable angina that is usually precipitated by activity and relieved by rest. About 3 weeks ago, his health care provider prescribed sildenafil (Viagra), and he has been using the drug with success. This morning he developed acute onset of substernal chest pain radiating to his left arm. This pain is not relieved by rest. The patient last took a sildenafil (Viagra) the night before. Which of the following treatments is absolutely contraindicated in this situation?

Nitroglycerin (CONTRAINDICATED to take Nitro within 24hrs of Sildenafil/Viagra!The vasodilatory effects of nitrates are profoundly amplified when administered in the presence of sildenafil, which can lead to refractory and life-threatening hypotension and cardiovascular collapse because of a synergistic effect. Patients using sildenafil therefore should be instructed to report their use on presentation to any emergency department and to never take nitrates while using the drug.)

A 63-year-old woman comes to the emergency department complaining of chest pain. The patient states that the pain began during her morning walk. It started as a dull pressure over her breastbone and then radiated to her left arm. Over the next few minutes, it escalated in intensity and was not relieved by rest. She called 911 and was brought to the emergency department. Her past medical history is significant for hypertension and hyperlipidemia. Her medications include atenolol and simvastatin daily. On physical examination, her blood pressure is 190/100 mm Hg, and her pulse is 60/min. Which of the following is the most appropriate agent to lower her blood pressure? IV Norepinephrine, IV Nitroglycerine, PO HCTZ, PO Metoprolol, PO Furosemide

Nitroglycerin is a potent vasodilator that acts predominantly on venous compliance to reduce preload to the heart. When administered intravenously, it can be rapidly titrated to produce optimized blood pressure control. IV nitroglycerin is effective when given in the setting of unstable angina. Oral forms of hydrochlorothiazide and metoprolol are excellent first-line antihypertensives with proven mortality benefit. The onset of their antihypertensive activity is relatively slow and therefore not appropriate for this case, in which blood pressure control must be achieved quickly. IV beta-blockers may be used to lower the blood pressure in an expeditious manner. The combined alpha/beta-blocker labetolol (Trandate) is a first-line agent for lowering blood pressure when given intravenously. Patients who have abrupt hypertension and angina symptoms need to have blood pressure lowered in an expeditious way. Nitroglycerin is the preferred agent in the setting of hypertension and angina because of its ability to drop the preload, vasodilate, and dilate the coronary arteries. Parenteral agents are preferred because of their rapid onset of activity and ability to be titrated. Various parenteral agents are approved in this setting, including sodium nitroprusside (Nipride) and labetolol (Trandate).

___________- is a regular, rapid (150-250/min) arrhythmia originating in the atria or AV node. *AV nodal reentry* is the most common cause of this arrhythmia (about 70% of patients). In this condition, the AV node is pathologically divided into two functional pathways. The electrical impulse usually proceeds anterograde down the slow pathway and retrograde up the fast pathway. The P waves are recorded nearly simultaneously with the QRS complexes (which occur in rapid sequence) and are therefore obscured on EKG. This arrhythmia is commonly seen in older patients, about half of whom have underlying heart disease. Tx- Vagal Maneuvers

PSVT (paroxysmal supraventricular tachycardia )

__________- can improve perfusion of vital organs and reduce the workload of the heart because it is both an arteriolar and venodilator. Problems sometimes encountered with this drug include hypotension (best avoided by starting with a low dose and continuously monitoring systemic arterial and pulmonary capillary wedge pressures) and accumulation of toxic metabolite (thiocyanate) of cyanide in patients who have liver or renal failure.

Nitroprusside

A patient who is being treated for hypertension related to a myocardial infarction that occurred 2 hours ago is medicated with IV nitroprusside (Nipride). Which of the following is the expected action of this drug?

Nitroprusside is a very useful IV agent that causes dilatation of both arterioles and venules. It has a very rapid onset of action and is typically used in an emergency department or intensive care unit situation. It can be titrated to effect on a drip-to-drip basis. It is used in settings in which the blood pressure needs to be carefully controlled. It is listed on ACLS protocols as one of the treatments (along with the combined alpha- and beta-blocker labetolol) for hypertensive emergency/hypertensive urgency. Nitroprusside can improve perfusion of vital organs and reduce the workload of the heart because it is both an arteriolar and venodilator. Problems sometimes encountered with this drug include hypotension (best avoided by starting with a low dose and continuously monitoring systemic arterial and pulmonary capillary wedge pressures) and accumulation of toxic metabolite (thiocyanate) of cyanide in patients who have liver or renal failure. Many physicians prefer to use IV nitrate rather than nitroprusside because of its lesser toxicity.

A 1-day-old infant girl appears dusky in the newborn nursery during feeding. Oxygen is immediately administered by nasal cannula. Shortly afterward, she develops tachypnea. On physical examination her blood pressure from the right upper arm is 50/30 mm Hg, pulse 180/min, and respirations 60/min. Echocardiogram is consistent with hypoplastic left heart syndrome. Which of the following would likely be found on auscultation?

No murmur, precordial hyperactivity, loud second heart sound. Hypoplastic left heart (HLH) syndrome is a group of closely related cardiac anomalies characterized by underdevelopment of the left cardiac chambers, atresia or stenosis of the aortic and/or the mitral orifices, and hypoplasia of the aorta. These anomalies are an especially common cause of heart failure in the first week of life. The left atrium and ventricle often exhibit endocardial fibroelastosis. Pulmonary venous blood traverses a patent foramen ovale, and a dilated and hypertrophied right ventricle acts as the systemic as well as pulmonary ventricle; the systemic circulation receives blood by way of a patent ductus arteriosus. Infants who have HLH syndrome develop poor perfusion, and metabolic acidosis when systemic blood flow decreases. When the ductus closes, inadequate blood flow to the body occurs because the ductus is the only path for blood to flow from the right ventricle to the body. Even if the ductus remains open, when the infant is given oxygen, the oxygen will dilate the vasculature of the pulmonary circulation, and blood will preferentially flow to the lower pressure pulmonary system, depriving the systemic circulation of adequate perfusion. ECG usually shows right axis deviation, right atrial and ventricular enlargement, and nonspecific ST- and T-wave abnormalities in the left precordial leads. Chest radiography may show only slight enlargement shortly after birth, but with clinical deterioration there is marked cardiomegaly with increased pulmonary vascular markings. Echocardiography is diagnostic and will show a diminutive aortic root and left ventricular cavity and absence or poor visualization of aortic and mitral valves. An infant who has HLH syndrome has a hyperdynamic precordium because the enlarged right ventricle is contracting against systemic pressure. The infant also has a loud second heart sound (S2) because the pulmonary artery acts as the aorta by pumping blood to the systemic circulation through the ductus arteriosus. The high end-systolic pressure markedly enhances S2. The flow from the right ventricle to the pulmonary artery is not turbulent; therefore, there is usually no significant murmur heard on auscultation. When the ductus closes, or when the pulmonary vasculature resistance falls, the flow to the systemic circulation will decrease, causing greatly diminished peripheral pulses. Management of the hypoplastic left heart syndrome patient includes infusion of prostaglandin E1 and administration of room air while on a ventilator (instead of oxygen, which will dilate the vasculature of the pulmonary circulation, causing blood to flow into the pulmonary system rather than the systemic circulation). Prostaglandin E1 may open the ductus arteriosus and restore systemic blood flow. Administration of room air or even hypobaric oxygen (FiO2 less than 21%) and the use of muscle relaxants can prevent hyperventilation and subsequent pulmonary vasodilation, thus reversing systemic hypoperfusion and metabolic acidosis.

Which of the following drugs antagonizes both the vascular and cardiac actions of norepinephrine (NE)? Atenolol, Esmolol, Labetalol, Metaproterenol, Prazosin

Norepinephrine (NE) is an agonist at alpha-1, alpha-2, and beta-1 receptors. NE exerts its vascular actions via alpha (predominantly alpha-1) receptors and its cardiac actions via beta-1 receptors. Labetalol is a selective antagonist at alpha receptors and nonselective antagonist at beta receptors and therefore could prevent all actions of NE.

Which of the following best describes why carotid sinus massage will result in the slowing of PSVT along with return of the patient to normal sinus rhythm?

PSVT is caused by a reentry circuit that involves the AV node, and vagal stimulation from this maneuver will stimulate the vagus nerve to break the reentry cycle.

A 14-year-old boy is brought to the health care provider with decreased exercise tolerance. He is up to date on all of his childhood immunizations and has been generally healthy until now. He is noted to have a grade III/VI systolic ejection murmur best heard at the left upper sternal border and a grade II/VI mid-diastolic murmur at the lower left sternal border. The first heart sound is normal. The second heart sound is widely split and fixed. A right ventricular impulse is palpated. On a chest roentgenogram, the pulmonary artery segment is enlarged, and pulmonary vascular markings are increased. An electrocardiogram shows right axis deviation. Which of the following congenital heart diseases does this patient most likely have?

One of the most common types of structural congenital heart disease to present in adolescence is atrial septal defect (ASD), and the most common presentation is a heart murmur. Some patients, however, present with arrhythmias, decreased exercise tolerance, or a paradoxic embolus. The physical examination can show classic findings of an ASD; in some cases, however, the findings may be extremely subtle. The murmur associated with the ASD is not caused by blood flow traversing the actual defect but rather by the increased volume of blood flow across the pulmonary valve and, to a lesser extent, across the tricuspid valve. The murmurs of an ASD thus are a systolic ejection murmur at the upper left sternal border and a mid-diastolic murmur at the lower left sternal border. The second heart sound is widely split and fixed with regard to respiration. The S2 split is caused by the delay of the pulmonic valve to close because of the increase in blood volume on the right side of the heart. On palpation, a right ventricular impulse is present. The chest radiograph shows evidence of an enlarged pulmonary artery segment in the posteroanterior projection. The superior vena cava shadow may not be visible because of the rotation of the heart secondary to right ventricular volume overload. Pulmonary vascularity is increased, and the heart may be somewhat enlarged. The lateral projection shows the right ventricular enlargement with filling of the retrosternal airspace. The ECG has a normal to rightward axis and a right ventricular volume overload pattern in the precordial leads.

_________-It is freely filtered at the glomerulus and is not reabsorbed. Its primary action occurs at the proximal tubule.

Osmotic Diuretic (Mannitol)

A 16-year-old boy is brought to the urgent care clinic with a temperature of 38.4°C (101°F) and low back, wrist, and knee pain. He had a sore throat 1 month earlier. His arthritis is diffuse. Pea-sized swellings are noted over the skin on his knees. He has a serpiginous erythematous area on his anterior trunk. His blood and throat cultures are negative, and his CBC is unremarkable. His antistreptolysin-O (ASO) titer is high. Which of the following is the most appropriate therapy?

PCN and ASA. This patient has acute rheumatic fever from group A streptococci. He has migratory polyarthritis, erythema marginatum, and subcutaneous nodules. Other features absent in this patient are chorea and carditis. His ASO titer indicates recent infection with Streptococcus. It is advisable to administer penicillin for the infection. The arthritis can be managed with salicylates. The patient will need long-term treatment with penicillin as a result of his rheumatic fever.

A 25-year-old woman is discovered on physical examination to have a midsystolic click and a high-pitched heart murmur. Which of the following additional cardiovascular findings is she most likely to exhibit?

PVC. A midsystolic click and a high-pitched heart murmur in a young woman is the classic presentation of mitral valve prolapse. Patients are usually asymptomatic but may have dyspnea, tachycardia, chest pain, syncope, eventual congestive heart failure, or, rarely, sudden death. Prolapse may coincide with tricuspid or pulmonary valve disease or with psychiatric conditions such as anxiety or depression. Complications may include atrial thrombosis, calcification, infective endocarditis, emboli to the brain, rupture of chordae, mitral regurgitation, arrhythmias, and premature ventricular contractions (PVCs). Atrial fibrillation (choice A) may result from chronic mitral stenosis, not mitral valve prolapse. This valvular disease is associated with an early diastolic opening snap. Decreased peripheral pulse pressure (choice B) and slowed carotid upstroke (choice D) are seen in aortic valve stenosis. A systolic ejection click is associated with this valvular disease. Wide pulse pressure (choice E) is seen clinically as bounding pulses. It is associated with aortic valve insufficiency. Mitral valve prolapse is the most common valvular heart condition in the United States. Most patients are asymptomatic and rarely have cardiac-related symptoms. A conclusion from the Framingham Heart Study found that the only association found with mitral valve prolapse was low body weight. If a patient does have symptoms, palpitations are the most common manifestation.

A 2-month-old baby boy is brought to the clinic by his mother for a routine checkup. On physical examination, the health care provider hears a continuous machinery murmur near the left sternal border at the second interspace. What is the most likely diagnosis?

Patent Ductus Arteriosus

Which of the following EKG abnormalities is treated as an equivalent to an ST-segment elevation acute myocardial infarction in a patient who has acute onset of chest pain?

Patients who have acute signs and symptoms consistent with myocardial infarction pattern who display a new-onset left bundle branch block pattern are treated equivalently to patients who have ST-segment elevation myocardial infarction until cardiac enzyme patterns are completed. Under appropriate circumstances, these patients are eligible for percutaneous cardiac intervention and thrombolytic therapy. Patients who have acute myocardial infarction often have such damage to the left ventricle that the repolarization pattern is affected, which results in a left bundle branch block pattern on the EKG. Other causes of left bundle branch block include heart failure, aortic stenosis, and left ventricular aneurysm. Conversely, Right bundle branch block pattern, primarily occurs in a patient who has increased pulmonary pressures or blockage of the right bundle of His. This is a commonly encountered EKG abnormality that is not typically pathologic in nature. It often occurs in the setting of left axis deviation (left anterior hemiblock), making this biphasicular block pattern very common.

A 16-year-old boy is brought to the urgent care clinic with a temperature of 38.4°C (101°F) and low back, wrist, and knee pain. He had a sore throat 1 month earlier. His arthritis is diffuse. Pea-sized swellings are noted over the skin on his knees. He has a serpiginous erythematous area on his anterior trunk. His blood and throat cultures are negative, and his CBC is unremarkable. His antistreptolysin-O (ASO) titer is high. Which of the following is the most appropriate therapy?

Penicillin and ASA (Pt has Acute Rheumatic Fever from Group A Strep + other sx- PCN for Infection and ASA for Pain relief)

Rheumatic fever is treated with _______ (for the bacterial infection) and _______ (for the arthralgias). Remember, ________ is NOT indicated in viral infections because of the potential to *cause Reye syndrome*. Penicillin that is given within 9 days of infection is most likely to prevent rheumatic fever.

Penicillin and ASA, ASA

A 43-year-old woman comes to the health care provider with a several-month history of dyspnea on exertion. She denies chest pain or a family history of coronary artery disease. She has a history of rheumatoid arthritis. Chest radiograph is remarkable for calcification of the heart border on the lateral film. She appears to have constrictive pericarditis. Which of the following physical examination findings would most likely be expected in this patient?

Pericardial Knock (constrictive pericarditis, the lateral view of the chest may demonstrate calcification of the anterior pericardium. This may be seen in 50% of patients who have longstanding constriction. The pericardial thickening may be seen on an echocardiogram. A pericardial knock is heard after the aortic valve closes, which is caused by the sudden cessation of ventricular filling)

A 1-week-old baby girl has a coarctation of the aorta just distal to the subclavian arteries. The blood pressure distal to the constriction is 50% lower than normal. Which of the following is increased in this infant?

Plasma Levels of Renin (aorta is constricted at a point beyond the arterial branches to the head and arms but proximal to the kidneys. Collateral vessels in the body wall carry much of the blood flow to the lower body, and the arterial pressure in the lower body is about 50% lower compared with the pressure in the upper body. The lower than normal pressure at the level of the kidneys causes renin to be secreted with the eventual conversion to angiotensin I and angiotensin II. The renin-aldosterone system results in salt and water retention, so that within a few days to weeks the arterial pressure in the lower body (at the level of the kidneys) increases to normal, but in doing so, the blood pressure in the upper body has increased to hypertensive levels)

____________-It acts on the collecting tubule to inhibit the reabsorption of Na+ and the secretion of K+.

Potassium Sparing Diuretic (Triamterene and Amiloride, but Spironolactone is an aldosterone antagonist that binds to its receptor.)

Which of the following diuretic agents is most likely to cause hyperkalemia?

Potassium Sparing Diuretics (Triamterene, Spironolactone, amiloride)

A 60-year-old alcoholic female who appears malnourished presents to her health care provider complaining of shortness of breath and gasping for air on awakening. Cardiac examination reveals an S3 heart sound, a diastolic murmur, and jugular venous distention. Pulmonary rales and peripheral edema are evident. An echocardiogram would be expected to reveal which of the following?

Primary acquired cardiomyopathy can occur in the peripartum period or from tachycardia-induced dilated cardiomyopathy. Secondary, toxicity-related cardiomyopathy can occur as a result of alcohol, doxorubicin use, or from heavy metal or chemical exposure. Echocardiogram will show normal or decreased wall thickness, poor wall thickening in systole, left ventricular dilation in a spherical pattern, and diminished stroke volume. Patients who have this disorder tend to present with fatigue and dyspnea. Suspect this diagnosis in any alcoholic patient presenting with symptoms and signs of congestive heart failure. In this patient, an echocardiogram would be expected to reveal bilateral ventricular dilatation with impaired contraction throughout both chambers.

___________- is a cardiac condition that is primarily caused by vasoconstriction or coronary artery disease. These patients may have *transient ST-segment elevation* during the painful episode. *Cardiac enzymes remain normal*. Treatment consists of _______, which may be used to prevent or treat cardiac vasospasm. Lifestyle modifications should be used in all patients.

Prinzmetal angina, CCB (or Variant Angina)

A 45-year-old woman comes to the emergency department complaining of recurring episodes of chest pain that each last a few minutes since she awoke this morning. She denies shortness of breath, radiation of the pain, or chest pain on exertion. She reports that she had similar episodes within the last month. The patient states that she has been healthy otherwise, takes no medications, and has no family history of coronary artery disease. Her vital signs and physical examination are within normal limits. Electrocardiogram shows transient ST-segment elevation. Serial cardiac enzymes reveal no abnormalities over a 24-hour period. Cardiac catheterization shows no significant areas of plaque or stenosis. Which of the following classes of drugs may be most effective in this patient?

Prinzmetal angina, or variant angina, is classically characterized by angina without the associated precipitating factors of angina pectoris. It is caused by a transient coronary artery spasm and displays a transient ST elevation on electrocardiogram. It usually has no other diagnostic findings. Calcium channel blockers have been shown to treat and prevent episodes of coronary vasospasm by decreasing contractility of cardiac muscle and producing vasodilation.

A 61-year-old woman comes to the health care provider for her first physical examination in more than 10 years. She reports that she has been in excellent health, does not smoke or drink, and runs 3 miles daily. She is a retired accountant and has 3 healthy grown children. She has been taking 81 mg of aspirin daily after reading about its cardioprotective effects in the newspaper. On physical examination she appears well. Blood pressure is 122/76 mm Hg, pulse 70/min, and respirations 14/min. She is afebrile. Head and neck examination is normal. There is no jugulovenous distention. Lungs are clear. On cardiac examination she has a regular S1 and S2, and II/VI crescendo blowing diastolic murmur is heard at the aortic area. Abdominal examination is normal. Rectal examination shows no masses and brown, guaiac-negative stool. Which of the following most likely explains the cardiac findings on physical examination?

Prior Strep Infection (findings of asymptomatic aortic insufficiency on physical examination. This lesion may result from a number of causes, several of which are infectious in etiology. Aortic insufficiency may result as a sequela of rheumatic heart disease, which occurs as an immunologic response to a streptococcal infection. Acute rheumatic fever is typically characterized by cardiac involvement that may cause pericarditis, myocarditis, or endocarditis. Often the initial cardiac manifestations are asymptomatic and become apparent only years later with the development of cardiac valvular disease)

__________- is defined as a fall in systolic blood pressure >10 mm Hg on inspiration. It can be associated with cardiac tamponade and chronic obstructive pulmonary disease (COPD).

Pulsus Paradoxus

A woman is brought into the emergency department following an automobile accident in which her chest was hit by the steering wheel. Her blood pressure is 120/90 mm Hg. When she inhales, her systolic blood pressure drops to 100 mm Hg. This finding defines which of the following terms?

Pulsus Paradoxus ( is defined as a fall in systolic blood pressure >10 mm Hg on inspiration. It can be associated with cardiac tamponade and COPD)

___________- is a repeated variation in the amplitude of the pulse pressure. It can be associated with profound left ventricular dysfunction.

Pulsus alternans

__________- is a double pulsation occurring during systole. It can be associated with aortic regurgitation and hypertrophic cardiomyopathy.

Pulsus bisferiens

A woman is brought into the emergency department following an automobile accident in which her chest was hit by the steering wheel. Her blood pressure is 120/90 mm Hg. When she inhales, her systolic blood pressure drops to 100 mm Hg. This finding defines which of the following terms?

Pulsus paradoxus is defined as a fall in systolic blood pressure >10 mm Hg on inspiration. It can be associated with cardiac tamponade and chronic obstructive pulmonary disease (COPD).

__________- is a weak pulse upstroke caused by diminished stroke volume. It can be associated with hypovolemia, aortic stenosis, mitral stenosis, and left ventricular failure.

Pulsus parvus

__________- is a delayed pulse upstroke. It can be associated with aortic stenosis

Pulsus tardus

____________- a phosphodiesterase inhibitor, is used to enhance the effects of nitric acid, which increases cyclic GMP. This effect is useful to treat erectile dysfunction. This class of medication is strictly contraindicated for use with nitrates and protease inhibitors (class of medications used in the management of HIV and AIDS). If used together, significant hypotension and even cardiovascular collapse may occur. These agents cannot be used within 24 hours of the last dose of either of these agents

Sildenafil (Viagra)

A 40-year-old woman is brought to the emergency department following a suicide attempt with imipramine (Tofranil). Her fiancé found her unresponsive, with an empty bottle of the imipramine at her side. The imipramine had been his, and the prescription had been filled that morning. Her past medical history is significant for hypertension, atrial fibrillation, diabetes, and asthma. Her medications include furosemide, procainamide, glyburide, prednisone, and albuterol. She has no known drug allergies. She is afebrile, has a blood pressure of 100/60 mm Hg, pulse of 62/min, and respirations of 22/min. A gastric lavage yields multiple pill fragments. She is confused and somnolent, and has shallow respirations. Her physical examination is otherwise unremarkable. On an electrocardiogram, which of the following abnormalities would most likely reflect possible cardiac toxicity?

QT Prolongation (prolongation of the QT interval is highly predictive of both cardiac and CNS toxicities from tricyclic antidepressant ingestion. This medication has high lethality associated with its overdose because of its cardiac effects)

A 71-year-old man comes to his health care provider for follow-up of a recent emergency department visit. The patient has a 2-year history of mild congestive heart failure in the setting of longstanding hypertension. He reports that yesterday he sought care at the local emergency department for palpitations and shortness of breath. He was told that his heart was "fibrillating," but later the fibrillation had "stopped on its own." His medications include a thiazide diuretic and an ACE inhibitor. On physical examination he appears well and in no distress. His blood pressure is 130/80 mm Hg and pulse 100/min and regular. His lungs have scant bibasilar rales and no gallops are appreciated. He has a grade 2 holosystolic murmur heard best at the apex. His jugular venous pressure (JVP) is 10 cm at 30 degrees. An electrocardiogram taken in the office reveals atrial fibrillation at a rate of 94/min with normal ST segments. Which of the following is the most appropriate next step in management?

Recall that in the treatment of medical conditions, certain medications overlap syndromes and are efficacious in many areas. This "co-treatment" option maximizes the benefits of each drug in a regimen and often addresses 2 or more issues simultaneously. Ischemia-mediated arrhthymia is the most common cause of death in a person who has congestive heart failure (CHF). Beta-blockers will control the rate in atrial fibrillation, as well as provide a mortality benefit for the CHF. Beta-blockers are both anti-ischemic and antiarrhythmic. In this case, ACE inhibitors have been shown to be very beneficial in prolonging the survival of patients who have CHF, as they are effective as vasodilators. They are also useful antihypertensive agents. Given this, discontinuing his ACE inhibitor (choice A) is clearly incorrect. This patient requires rate control for his atrial fibrillation so that acute heart failure does not recur. Short of restoring this patient's atrial contractions, rate control is the best method to ensure adequate management of atrial fibrillation. Digoxin, with or without a nodal agent such as a beta-blocker, has been shown to be reasonably effective at rate control, but it is less effective in high adrenergic states. Beta-blockers such as metoprolol (Lopressor) and the combined alpha- and beta-blocker carvedilol (Coreg), however, have been clearly shown to lower mortality with CHF, in addition to just controlling the heart rate.

A 70-year-old hypertensive man arrives at the emergency department complaining of shortness of breath. His history is significant for chronic hypertension, paroxysmal nocturnal dyspnea, and nocturia. Physical examination reveals evidence of pulmonary and peripheral edema. The patient is admitted, and furosemide is administered. A low-sodium diet is ordered. The purpose of this dietary restriction is to -_______ ________ water

Reduce Extracellular

-Acute glomerulonephritis, transplant rejection are examples of ________ causes of oliguria. -Bladder tumor and prostatic hyperplasia are examples of ________ causes of oliguria.

Renal, postrenal

A 12-year-old child is brought to his pediatrician for a routine health maintenance visit. He has been well except for occasional attacks of asthma and has met all development milestones. His immunizations are up to date. He occasionally uses albuterol for his asthma. Physical examination is remarkable for a blood pressure of 150/90 mm Hg in both arms. Which of the following is the most likely cause of his hypertension? Albuterol, Chronic Lung Disease, Coarctation of the Aorta, Congenital Heart Disease, Renal Disease

Renovascular disease is the most frequent cause of secondary hypertension in young children. Ailments such as polycystic kidney disease, congenital vascular disease, tumors, and infections can all lead to hypertension, and a urologic evaluation is imperative. Patients who present with hypertension at the extremes of age or whose hypertension is not responsive to traditional agents should have an evaluation performed to assess for secondary causes of hypertension.

A 5-year-old girl from Connecticut is brought to the family clinic with a 3-day history of fever and intermittent joint pain. She is generally healthy, but according to her mother, she had a "cold" about 1 month ago. On physical examination her temperature is 39.6°C (103.2°F), blood pressure 94/60 mm Hg, pulse 114/min, and respirations 22/min. Knees and elbow joints are swollen, warm, and tender to palpation. There is a grade III/VI diastolic murmur best heard at the apex. Multiple fine, pink macules are noted on her trunk. These macules blanch in the center. Which of the following is the most likely diagnosis?

Rheumatic fever is an inflammatory disease, possibly autoimmune in nature. Immune responses to group A streptococcal antigens during pharyngitis resulting in antibody cross-reactions with myocardial antigens remain central to the pathogenesis. Rheumatic fever involves many tissues, including the heart, joints, skin, and CNS. Preceding infection with group A Streptococcus is a prerequisite to the development of acute rheumatic fever. Initially, fever, dyspnea, chest pain, and cardiac murmur develop. Jones criteria (CANCER). Lyme disease may begin with migratory polyarthralgias and progress to attacks of asymmetric oligoarthritis in large joints. Erythema chronica migrans, the diagnostic annular red lesion at the site of the tick bite, occurs in 50 to 80% of cases during the early phase of illness. Cardiac involvement is atypical. If it does occur, it typically causes heart block.

A 68-year-old man is brought to the emergency department with excruciating back pain that began suddenly 45 minutes ago. The pain is constant and is not exacerbated by sneezing or coughing. He is diaphoretic and has a systolic blood pressure of 90 mm Hg. There is an 8-cm pulsatile mass deep in his epigastrium, above the umbilicus. A chest radiograph is unremarkable. Two years ago, he was diagnosed with prostatic cancer and was treated with orchiectomy and radiation. At that time his blood pressure was high, for which he declined treatment. Which of the following is the most likely diagnosis?

Rupture of Abdominal Aortic Aneurysm (reach or exceed a size of 5.5 cm in males and 5.0 cm in females. Often the first manifestation is excruciating back pain, as the blood leaks into the retroperitoneal space before the aneurysm blows out into the peritoneal cavity. The combination of a big pulsatile mass and sudden severe back pain should always lead to this presumptive diagnosis. Looking for orthopedic or neurologic explanations can be a deadly mistake. In any patient who has a rupturing aneurysm or who is hemodynamically unstable, immediate surgical intervention is required)

This hypertensive patient has symptoms and signs of congestive heart failure (paroxysmal nocturnal dyspnea, nocturia, and pulmonary and peripheral edema), for which dietary ______ restriction is recommended to reduce water retention.

Sodium

A 29-year-old man is brought to the emergency department in a comatose state a few hours after complaining of sudden onset of excruciating headache. His friend does not know if the patient has any underlying medical conditions. Neurologic examination reveals dilated pupils poorly responsive to light. A CT scan of the head without contrast demonstrates hyperdensity within the suprasellar cistern, whereas MRI scan is unremarkable. Lumbar puncture shows hemorrhagic cerebrospinal fluid. Which of the following is the most likely diagnosis?

Ruptured Berry Aneurysm (Rupture of a berry aneurysm is the most common cause of subarachnoid bleeding.Berry aneurysms develop as a result of congenital weakness at branching points of the arteries in the circle of Willis. These outpouchings tend to expand progressively, but in most cases they remain asymptomatic. Hypertension facilitates development and rupture of berry aneurysm. One third of patients recover, one third die, and one third develop re-bleeding. Rapid onset of coma is an ominous sign.)

___________- should be considered in hypertensive patients who present either at extremes of age or who have other clinical symptoms that point toward a certain etiology. It may also be suspected in patients with hypertension that is difficult to control or who have to take multiple medications for hypertension control. In otherwise healthy patients, attempts should be made to control blood pressure with diet and medications before considering other causes.

Secondary HTN

A 52-year-old man is discharged from the hospital after an uncomplicated myocardial infarction. Several weeks later, he visits his primary care provider complaining of insomnia, anorexia, and depressed mood. He appears to be clinically depressed. He denies any current chest pain or shortness of breath. Which of the following would be the most appropriate medication to initiate for this patient?

Sertraline (Zoloft- pt who has cardiac complications SSRI has been demonstrated to be the SAFEST and Most Effective Med for Tx of Clinical Depression)

A patient having _______ will have decreased perfusion to the kidneys. This decreased perfusion will cause the kidneys to compensate for this impaired blood flow by increasing the release of renin, which will then lead to the formation of angiotensin I, which gets converted to angiotensin II, and later conversion to aldosterone occurs. This will result in *vasoconstriction and retention of sodium*, which will retain water and conserve water loss from the body.

Shock (The urine in these patients will have a low fractional excretion of sodium, which will be <1%. This low fractional excretion of sodium will help to preserve fluid in this shock state, which will permit more volume being available for the body to use.)

Which of the following is an example of a prerenal cause of oliguria?

Shock (not all oliguria is caused by intrinsic renal disease. Shock, of any etiology, is an example of a prerenal cause of oliguria in which poor renal perfusion leads to inadequate urine output.)

A 63-year-old retired airline pilot comes to the office for a first time visit. He is switching primary care facilities because he was very unhappy with the care that he previously received. He brings his medical record, which documents longstanding cirrhosis and portal hypertension caused by previous alcohol abuse. He has no history of gastrointestinal bleeding. He has a very long medication list, and he reports that his pharmacist told him that his medications "were wrong" and that his health care provider was prescribing drugs improperly. On review of his medication list, a number of medication incompatibilities are found. Co-administration of which of the following is likely to cause this patient the greatest harm if not corrected?v

Spironolactone and Oral Potassium (Spironolactone is a potassium-sparing diuretic. The mistake of calling this drug a "diuretic" and cueing your brain to think "replenish electrolytes" could cause this patient to die from hyperkalemic cardiac arrest. Spironolactone is increasingly used as an add-on agent for chronic heart failure, as there has been mortality benefit demonstrated with its use. Spironolactone also is a direct aldosterone antagonist and can be beneficial in the setting of cirrhosis with ascites, to lessen fluid accumulation)

A 22-year-old man comes to the emergency department with a 3-day history of fever, chills, a cough, pleuritic chest pain, and low back pain. He says that the symptoms came on "out of the blue." He is the son of a wealthy local businesswoman and still lives at home, which he says "is cool because my parents are never around." His temperature is 39°C (102.2°F), blood pressure 120/80 mm Hg, pulse 70/min, and respirations 16/min. Physical examination shows oval retinal hemorrhages with a clear, pale center and pinpoint lesions between his toes. Blood cultures are drawn. A chest radiograph shows multiple patchy infiltrates. Laboratory studies show hemoglobin 11 g/dL, hematocrit 39%, and erythrocyte sedimentation rate 39 mm/h. Which of the following is the most likely pathogen?

Staph aureus (This patient has acute bacterial endocarditis, most likely caused by Staphylococcus aureus, the most common organism causing endocarditis in intravenous drug abusers. The "pinpoint lesions" between his toes are signs of injection drug abuse. Acute endocarditis in drug abusers typically presents with a high fever, pleuritic chest pain, and a cough. The tricuspid valve is commonly affected in these patients)

A 45-year-old woman has a history of symptomatic ventricular couplets, for which she had been placed on amiodarone (Cordarone). A week after this intervention, she experiences a syncopal episode and is brought to the emergency department. The patient is awake and alert with a blood pressure of 110/70 mm Hg and pulse 90/min. Electrolytes are within normal limits. She is placed on a cardiac monitor and is noted to have a transient wide complex tachycardia with waves of alternating morphology. She appears to be in torsades de pointes. Which of the following is the most appropriate next step?

Stop Amiodarone and Observe (diagnosed with polymorphic ventricular tachycardia called torsades de pointes. This can be a fatal rhythm and is associated with a prolongation of the QT interval, which could have occurred as a result of the amiodarone administration. The appropriate step is thus to discontinue the amiodarone and observe)

Rupture of a berry aneurysm is the most common cause of ___________________. Berry aneurysms develop as a result of congenital weakness at branching points of the arteries in the circle of Willis. These outpouchings tend to expand progressively, but in most cases they remain asymptomatic. Hypertension facilitates development and rupture of berry aneurysm. One third of patients recover, one third die, and one third develop re-bleeding. Rapid onset of coma is an ominous sign.

Subarachnoid Hemorrhage

A 67-year-old woman comes to the clinic for review of her medications. She has had a history of hypertension for 25 years, type 2 diabetes for 20 years, and congestive heart failure for 5 years. She is a former smoker of two packs of cigarettes per day and her lipid status is not known at this time. Her current medications include nifedipine (Adalat, Procardia), hydralazine (Apresoline), isosorbide dinitrate (Isordil), glyburide (DiaBeta, Micronase), a multivitamin, and conjugated estrogens. Today in the clinic her blood pressure is 160/90 mm Hg, her fasting blood glucose is 210 mg/dL, and her hemoglobin A1c is 7.9%. She reports moderate dyspnea on exertion, unchanged from previous visits. Which of the following is the most appropriate intervention at this time?

Stop Hydralazine and Add Captopril (Pt has poorly controlled HTN and DM. She needs improved therapy for both; the issue is how best to do that. The concept underlying this question is the absolute importance of using ACE inhibitor therapy on both type 1 and type 2 diabetic patients. Many clinical trials have shown the beneficial effects of ACE inhibitors on preventing nephropathy and slowing the progression of established nephropathy in diabetics. It is the standard of care that all diabetics be given an ACE inhibitor if they are able to tolerate its blood pressure effects. Given that she has congestive heart failure and hypertension, the ACE inhibitor also will be efficacious in their treatment. In fact, ACE inhibitors have been shown to be superior to hydralazine and isosorbide dinitrate in terms of morbidity and mortality in treatment of CHF)

A 10-year-old girl is brought to the office because of fever and chills for 3 days. She had been complaining of a headache and feeling tired for 10 days before she developed fever. The parents had tried giving her acetaminophen, but the child's condition was not improving. Her past medical history is significant for frequent streptococcal throat infections over the past 2 years and a new onset heart murmur detected at her last well child visit 2 months earlier. She is on no medication currently, except for acetaminophen. Otherwise, she has been wearing dental braces for the past year and had a primary tooth extracted 2 weeks earlier in the attempt of liberating space for permanent teeth. On physical examination, the patient is diaphoretic, in moderate distress with a temperature of 39.0°C (102.2°F), blood pressure of 90/60 mm Hg, pulse of 110/min, and respirations of 24/min. On her fingernails, you note several splinter hemorrhages. Auscultation confirms a grade 2/6 high-pitched, blowing, systolic ejection murmur, best heard at the apex that radiates to the left axilla. A chest radiograph is unremarkable, but the electrocardiogram shows signs of left ventricular strain. An echocardiogram reveals vegetations on the mitral valve. Which of the following microorganisms is the most likely cause of this patient's current condition?

Strep Viridans (developed acute endocarditis most likely caused by Streptococcus viridans that entered the bloodstream during the recent dental procedure. The predisposing factor for her current illness is rheumatic heart disease, a sequela of prior Group A β-hemolytic streptococcal pharyngitis, which was complicated by involvement of the mitral valve.)

A 25-year-old woman involved in an automobile accident is admitted as an emergency patient. A major artery severed in her leg caused an estimated 600 mL blood to be lost. Her blood pressure is 90/60 mm Hg. Which of the following would be expected to increase in response to hemorrhage?

Sympathetic Nerve Activity (decrease in blood pressure caused by hemorrhage activates the baroreceptor reflex, which tends to increase sympathetic nerve activity and decrease parasympathetic vagal nerve activity)

___________- +pulmonary edema, normal neck veins, lack of edema in extremities, or a hepatojugular reflex and a abnormal ejection fraction.

Systolic Heart Failure (Ventricular Fxn Impaired)

A 5-month-old girl is brought to the office by her mother, who states that the girl had an episode following feeding during which she began to breathe deeply, became blue, and then lost consciousness. The mother states that she picked her up and held her, and the infant regained her usual color and became alert. Physical examination reveals a harsh systolic murmur. The remainder of the physical examination is unremarkable. Which of the following is the most likely diagnosis?

Tetralogy of Fallot

___________- can present with cyanotic spells associated with feeding and/or crying. Episodically there can be near-occlusion of the right ventricular outflow tract with profound cyanosis. These episodes are often referred to as "tet spells" or "hypercyanotic spells."

Tetralogy of Fallot

____________- It is the most common type of cyanotic congenital heart disease.

Tetralogy of Fallot

A 15-month-old boy is brought to the emergency department by his parents because he suddenly turned blue and had difficulty breathing while he was playing in the back yard of their home. The parents have noticed several times in the previous 2-3 months that he had developed a bluish discoloration around the lips but did not think much of it, as it was wintertime. Recently, however, the boy had increasing fatigability and would stop in the middle of playing to catch his breath by sitting down or squatting. This morning when he suddenly turned blue and started behaving in a very flustered manner they rushed him to the emergency department. The parents deny any other significant medical conditions or allergies to medication. They have a 4-year-old daughter who is in good health. On physical examination, the patient is in the fifth percentile for height and weight. His lips and fingertips are bluish in color and he frequently stops to catch his breath while playing in the examination room. There is a hint of clubbing of his fingers. Vital signs are within normal limits, but a complete blood count shows a red blood cell count of 6 x 1012/L and a hematocrit of 66%. A chest radiograph shows a boot-shaped heart with an uptilted apex and clear lung fields. On auscultation, there is a harsh systolic ejection murmur and a single S2 is heard. Which of the following is the most likely diagnosis?

Tetralogy of Fallot (defined as pulmonary stenosis, ventricular septal defect, dextroposition of the aorta (overriding), and right ventricular hypertrophy. It is the most common type of cyanotic congenital heart disease)

_____________- Typically, patients present with cyanosis, delayed growth and development, and dyspnea. Paroxysmal hypercyanotic attacks (hypoxic, blue, or tet spells) manifest with episodes of restlessness, cyanosis, and gasping respirations. Clubbing of the fingers and toes occurs secondary to chronic hypoxia. A loud, harsh, systolic ejection murmur is heard. S2 is single or very soft because of the pulmonary stenosis. Chest radiographs reveal a boot-shaped heart (coeur en sabot) with uptilted apex.

Tetralogy of fallot

A 1-week-old baby girl has a coarctation of the aorta just distal to the subclavian arteries. The blood pressure distal to the constriction is 50% lower than normal. Which of the following is increased in this infant?

The aorta is constricted at a point beyond the arterial branches to the head and arms but proximal to the kidneys. Collateral vessels in the body wall carry much of the blood flow to the lower body, and the arterial pressure in the lower body is about 50% lower compared with the pressure in the upper body. The lower than normal pressure at the level of the kidneys causes renin to be secreted with the eventual conversion to angiotensin I and angiotensin II. The renin-aldosterone system results in salt and water retention, so that within a few days to weeks the arterial pressure in the lower body (at the level of the kidneys) increases to normal, but in doing so, the blood pressure in the upper body has increased to hypertensive levels. This is the underlying pathophysiology as to why coarctation of the aorta is a secondary cause of hypertension. Correction of the coarctation will then allow the kidney to be normally perfused, and this will result in a "cure" for the patient's hypertension.

A 75-year-old man with a 40 pack per year history of smoking and hypercholesterolemia is diagnosed with severe atherosclerosis. Atherosclerotic occlusion of which of the following arteries would result in insufficient perfusion of the urinary bladder?

The bladder is supplied by the vesicular branches of the internal iliac arteries. The internal iliacs arise from the common iliac artery.

A 12-year-old African American boy is brought to the office for a well child examination. He has been in good health and only complains of an occasional headache. He has been doing reasonably well in school but has some social problems that his mother attributes to his physical appearance. She has tried persuading her son to eat healthier, but he seems to enjoy eating fast food much more than home cooked meals. His past medical history is unremarkable, and the family history is significant for adult-onset hypertension in his father's family. He takes no medication. On physical examination, the patient is in no acute distress. He is 145 centimeters tall and weighs 92 kilograms. His vital signs are within normal limits, but his blood pressure is 145/90 mm Hg. The health care provider checks that the cuff size is appropriate and remeasures his blood pressure in all four extremities, only to confirm that it is above the ninety-fifth percentile for his age. After discussing the finding with his mother, he is scheduled for several more blood pressure evaluations over the following 6 weeks. All readings yield results mildly above the ninety-fifth percentile for his age. Which of the following recommendations is most appropriate at this time?

The child should initiate a weight reduction diet with limited salt intake and regular exercise. Systemic hypertension is defined as blood pressure above the ninety-fifth percentile for age on repeated measurements over a 6-week period. Primary (essential) hypertension usually has no known underlying cause. Predisposing factors include heredity, salt intake, stress, and obesity. It is more commonly seen in adolescents and adults. Secondary hypertension is caused by an associated disease. It is more common in infants and younger children. The most common cause of secondary hypertension in children is renal disease (75-80%). A prior urinary tract infection is seen in 25 to 50% of cases, often related to an obstructive lesion of the urinary tract. In newborns, a history of umbilical artery catheterization may be elicited, with resultant thrombosis of the renal artery. Hypertension usually presents with no symptoms, especially in adolescents who have essential hypertension, and it is diagnosed on routine examination. Headaches, dizziness, vision changes, and seizures may be present. Blood pressure should be measured over several visits and compared with normal values for age. It should be measured in all extremities to rule out coarctation of the aorta. All children who have secondary hypertension should have a renal evaluation, including culture, ultrasound, renin levels, blood urea nitrogen, and creatinine. Echocardiography should be used to assess ventricular function and size. Therapy of hypertension in children is the same as in adults: diet, exercise, and medication. Pharmacologic management includes angiotensin-converting enzyme inhibitors, calcium-channel blockers, and diuretics.

On a routine physical examination, a midsystolic ejection murmur is detected in the pulmonic area of a 35-year-old woman. The cardiac examination also reveals a prominent right ventricular cardiac impulse and wide and fixed splitting of the second heart sound. EKG shows right axis deviation and chest radiograph shows enlargement of the right ventricle and atrium. Which of the following is the most likely diagnosis?

The classic findings in atrial septal defect are a prominent right ventricular cardiac impulse, a systolic ejection murmur heard in the pulmonic area and along the left sternal border, and fixed splitting of the second heart sound. These findings are caused by an abnormal left-to-right shunt through the defect, creating a volume overload on the right side. The increase in volume on the right side creates the flow murmur, the dilatation of the right-sided chambers, and the delayed closure of the pulmonic valve, all of which are present in this case. The delayed closure of the pulmonic valve occurs because of the increased volume of blood that is delivered to the right ventricle.

An elderly man presents with complaints of dizziness, headaches, diarrhea, nausea and vomiting, weakness, palpitations, and a change in vision with a yellowish to blue tint to his vision. He is taking multiple medications. He has a history of chronic heart failure and hypertension. His wife states that he has had a few episodes of confused, delirious behavior over the past few weeks. Which of the following agents might be responsible for this man's symptoms? Allopurinol, Hydralazine, Niacin, Digoxin, Spironolactone

The collection of symptoms described—dizziness, headaches, diarrhea, nausea and vomiting, weakness, palpitations, and a change in vision with a yellowish to blue tint to the vision—are classic side effects of digoxin (Lanoxin). EKG changes, such as biventricular tachycardia, may also occur. Digoxin is a medication that has been shown to improve symptoms in patients who have heart failure, as it is a positive inotropic and negative chronotropic agent. Patients who are hypokalemic are especially prone to the development of digoxin toxicity.

A 63-year-old woman comes to the emergency department complaining of severe mid-abdominal pain. The patient reports that the pain has increased in intensity over the past few days. There has been no associated nausea or vomiting, no change in bowel habits, and no relief afforded by position changes. The woman is postmenopausal and does not take hormone replacement therapy. She has a 30-year history of hypertension and has been noncompliant with her calcium channel blocker and thiazide diuretic therapy. On examination, her abdomen is obese, but there is a suggestion of a nontender, pulsatile mass in the epigastric region. The remainder of the physical examination is normal. Imaging studies are done, and immediate surgical evaluation is indicated. Which of the following results were most likely obtained from the imaging studies? A. An abdominal aortic segment spanning 7 cm in diameter B. A liver span of more than 9 cm C. Multiple cysts within the liver D. Posterior herniation of a lumbar intervertebral disk E. Stones in the renal pelvis

The correct answer is A. Abdominal aortic aneurysms larger than 5 cm have greater than a 30% chance of rupturing in 3 years. As most ruptures result in death, and as the mortality of surgical repair has fallen sharply, surgical repair of the aneurysm is indicated. Aneurysms that are not repaired expand on average at about 0.4 cm per year. Repair of these aneurysms can be accomplished via either traditional open approaches or new endovascular approaches. Because aneurysms tend to increase in diameter with time, patients who are identified with having aortic aneurysms are serially followed up with ultrasound. Patients who have abdominal aortic aneurysms (AAA) are typically asymptomatic and these aneurysms are typically identified as incidental findings. If a patient is symptomatic, the most common symptoms are back and abdominal pain. If rupture occurs, patients are at high risk for death (about 90% mortality with rupture) and patients are immediately taken to the operating room for surgical repair. Elective repair is indicated for symptomatic patients or asymptomatic patients if the diameter reaches 5.5 cm (men) and 5.0 cm (women). The larger the diameter of the abdominal aorta, the more likely the aneurysm will rupture, which is why elective repair is warranted.

A 63-year-old woman comes to the emergency department complaining of severe mid-abdominal pain. The patient reports that the pain has increased in intensity over the past few days. There has been no associated nausea or vomiting, no change in bowel habits, and no relief afforded by position changes. The woman is postmenopausal and does not take hormone replacement therapy. She has a 30-year history of hypertension and has been noncompliant with her calcium channel blocker and thiazide diuretic therapy. On examination, her abdomen is obese, but there is a suggestion of a nontender, pulsatile mass in the epigastric region. The remainder of the physical examination is normal. Imaging studies are done, and immediate surgical evaluation is indicated. Which of the following results were most likely obtained from the imaging studies? A. An abdominal aortic segment spanning 7 cm in diameter B. A liver span of more than 9 cm C. Multiple cysts within the liver D. Posterior herniation of a lumbar intervertebral disk E. Stones in the renal pelvis

The correct answer is A. Abdominal aortic aneurysms larger than 5 cm have greater than a 30% chance of rupturing in 3 years. As most ruptures result in death, and as the mortality of surgical repair has fallen sharply, surgical repair of the aneurysm is indicated. Aneurysms that are not repaired expand on average at about 0.4 cm per year. Repair of these aneurysms can be accomplished via either traditional open approaches or new endovascular approaches. A liver span of 9 cm (choice B) is within normal limits for most of the population. Liver cysts (choice C) are common incidental findings on imaging of the abdomen. Although there is a large differential diagnosis for liver cysts, most etiologies have a benign course and can be followed conservatively. Most disk herniations (choice D) respond to medical treatment. The percentage of disk herniations that require surgical intervention is less than 10%. In addition, the relationship between disk herniations and actual symptomatic disease is not linear, and many people with one fail to have the other. Urinary stone disease (choice E) represents one of the most common problems of the urinary tract. Calcium, uric acid, struvite, or cystine stones within the renal pelvis that are nonobstructive can be followed medically without surgical intervention. Patients with obstructing ureteral stones would be symptomatic with renal colic pain, which radiates into the anterior abdomen along with having flank pain during percussion. Reviewed on 4/17/14 Because aneurysms tend to increase in diameter with time, patients who are identified with having aortic aneurysms are serially followed up with ultrasound. Patients who have abdominal aortic aneurysms (AAA) are typically asymptomatic and these aneurysms are typically identified as incidental findings. If a patient is symptomatic, the most common symptoms are back and abdominal pain. If rupture occurs, patients are at high risk for death (about 90% mortality with rupture) and patients are immediately taken to the operating room for surgical repair. Elective repair is indicated for symptomatic patients or asymptomatic patients if the diameter reaches 5.5 cm (men) and 5.0 cm (women). The larger the diameter of the abdominal aorta, the more likely the aneurysm will rupture, which is why elective repair is warranted.

A 73-year-old man comes to the emergency department complaining of abdominal pain. He describes a dull, aching, constant pain in his mid-umbilical region. The pain has persisted over the past few days with increasing intensity, and it is not relieved by changes in position or eating. The patient has a past medical history significant for hypertension and coronary artery disease. He had a myocardial infarction 3 years ago. The man has moderate peripheral vascular disease with a prior femoral-popliteal bypass graft on the left. On physical examination, his blood pressure is 180/100 mm Hg and his pulse is 86/min. He has a loud S4 on chest examination. Abdominal examination shows a pulsatile mass in his abdomen and venous stasis changes bilaterally on his lower extremities. Which of the following is the most appropriate diagnostic test at this time? A. Abdominal ultrasound (U/S) B. CT scan of the abdomen C. CT scan of the spine D. Lumbosacral (L/S) spine films E. Spinal MRI

The correct answer is A. Abdominal ultrasound (U/S) is the most cost-effective screening test for a suspected abdominal aortic aneurysm (AAA). Ultrasound of the abdomen is considered to be the definitive test to perform when AAA is suspected, as it has a sensitivity and specificity of almost 100%. The screening test of choice for abdominal aortic aneurysm is an abdominal ultrasound. A CT scan with contrast can provide a greater level of detail of the aneurysm and more accurate dimensions for the evaluation of possible surgical intervention.

A 73-year-old man comes to the emergency department complaining of abdominal pain. He describes a dull, aching, constant pain in his mid-umbilical region. The pain has persisted over the past few days with increasing intensity, and it is not relieved by changes in position or eating. The patient has a past medical history significant for hypertension and coronary artery disease. He had a myocardial infarction 3 years ago. The man has moderate peripheral vascular disease with a prior femoral-popliteal bypass graft on the left. On physical examination, his blood pressure is 180/100 mm Hg and his pulse is 86/min. He has a loud S4 on chest examination. Abdominal examination shows a pulsatile mass in his abdomen and venous stasis changes bilaterally on his lower extremities. Which of the following is the most appropriate diagnostic test at this time? A. Abdominal ultrasound (U/S) B. CT scan of the abdomen C. CT scan of the spine D. Lumbosacral (L/S) spine films E. Spinal MRI

The correct answer is A. Abdominal ultrasound (U/S) is the most cost-effective screening test for a suspected abdominal aortic aneurysm (AAA). Ultrasound of the abdomen is considered to be the definitive test to perform when AAA is suspected, as it has a sensitivity and specificity of almost 100%. CT scan of the abdomen (choice B) with IV contrast is about twice as expensive as U/S, adds little if any benefit, and exposes the patient to unnecessary radiation. CT scan of the abdomen is typically performed preoperatively so that the surgeon can develop the surgical plan based on the detail that this study provides. CT scan of the spine (choice C), lumbosacral (L/S) spine film (choice D), and spinal MRI scan (choice E) are imaging studies directed at evaluating spinal pathology, which is not suspected here. MRI scan provides the highest resolution and is useful for detecting abscesses or cord compression. CT scan is excellent for disk pathology and even bone pathology. Plain radiographs are useful for very gross visualization of bone density and integrity. Reviewed on 4/17/14 The screening test of choice for abdominal aortic aneurysm is an abdominal ultrasound. A CT scan with contrast can provide a greater level of detail of the aneurysm and more accurate dimensions for the evaluation of possible surgical intervention.

After an accident at work resulting in severe hemorrhage, a machinist is rushed to the emergency department. Which of the following sets of autonomic responses would be predicted in this patient? A. Decrease in blood pressure and increased pulse B. Increase in the jugular veins (jugular venous distention) C. Hypotension and bradycardia D. Decrease in respirations E. Widening of pulse pressure

The correct answer is A. After a decrease in blood pressure (e.g., after a hemorrhage), one would expect an increase in sympathetic outflow and decrease in parasympathetic outflow. As a result of the hemorrhage, there would be less blood in the body to circulate, with resultant increase in heart rate, decrease in both blood pressure and gastrointestinal motility, and dilation of the pupils.

After an accident at work resulting in severe hemorrhage, a machinist is rushed to the emergency department. Which of the following sets of autonomic responses would be predicted in this patient? A. Decrease in blood pressure and increased pulse B. Increase in the jugular veins (jugular venous distention) C. Hypotension and bradycardia D. Decrease in respirations E. Widening of pulse pressure

The correct answer is A. After a decrease in blood pressure (e.g., after a hemorrhage), one would expect an increase in sympathetic outflow and decrease in parasympathetic outflow. As a result of the hemorrhage, there would be less blood in the body to circulate, with resultant increase in heart rate, decrease in both blood pressure and gastrointestinal motility, and dilation of the pupils. Choice B describes cardiogenic shock with an increase in the jugular venous pressure secondary to poor emptying of the heart and systolic failure. Choice C is seen with neurogenic shock, in which there is a disconnection between the brain and the cardiovascular system. Hypotension should normally result in reflex tachycardia, but this does not occur in a patient who has neurgenic shock. Choice D, hypotension with slowing of respirations, does not occur as a normal response. If the patient becomes hypotensive, the normal physiologic response is tachypnea, which occurs in order for the body to circulate the fewer red blood cells faster. Choice E, widened pulse pressure, does not occur in response to hypotension, because hypotension itself decreases systolic blood pressure (less blood for the heart to pump out and increased rate of contraction allows less diastolic filling time); this fall in systolic blood pressure narrows rather than widens pulse pressure. Reviewed on 4/17/14 Shock is a clinical condition in which there is inadequate organ perfusion and tissue oxygenation. It is classically associated with hypotension and tachycardia and tachypnea. The normal physiologic response to shock is to have sympathetic release of hormones, which will stimulate the heart to beat faster and stronger so that the volume of blood can be circulated faster and with more force to overcome the deficiency of the blood supply to the end organs and to improve perfusion. First-line management of shock is fluid resuscitation with crystalloids. If the patient does not respond to crystalloids, blood transfusions (primarily to allow better release of oxygen to the tissues) and colloids such as albumin can be used along with vasopressors, which can help to maintain the vascular integrity and can be used to continue to stimulate the heart.

A man is an unrestrained front-seat passenger in a car when it crashes; he sustains closed comminuted fractures of both femoral shafts. Shortly after admission, he develops a blood pressure of 80/50 mm Hg, pulse 110/min, and venous pressure of 0. He becomes pale, cold, and clammy. The rest of the physical examination and x-rays of the chest and pelvis are unremarkable. Sonogram of the abdomen performed in the emergency department is likewise negative. Which of the following most likely explains the patient's low blood pressure? A. Blood loss at the fracture sites B. Fat embolism C. Neurogenic shock from pain D. Unrecognized intracranial bleeding E. Unrecognized pericardial tamponade

The correct answer is A. After extensive trauma to certain areas of the body, enough blood may accumulate to send the patient into hypovolemic shock. One of these areas is the femur (as seen in the present patient); the others are the thorax, abdomen, and pelvis. These patients should be treated with massive resuscitation and transfusion to overcome their hypovolemia until reduction and fixation can be performed. After extensive trauma, the thorax, abdomen, pelvis, or femurs have the potential to accumulate enough blood to produce hypovolemic shock. Intracranial bleeding does not lead to hypovolemic shock; there is not enough space inside the head for sizable blood loss to accumulate.

A 65-year-old patient has experienced several transient ischemic attacks over the past few months. Because his general health is poor, he is not considered an appropriate candidate for carotid endarterectomy. Examination reveals a regular rate on heart examination with a blood pressure of 130/86 mm Hg. The decision is made to treat him medically. Which of the following agents would be most appropriate for his therapy? A. Aspirin B. Coumadin (Warfarin) C. Dipyridamole (Persantine) D. Heparin E. Clopidogrel (Plavix) plus aspirin

The correct answer is A. Although treatment of transient ischemic attacks (actually prophylaxis against stroke) remains a controversial area, you should be aware of current recommendations. Daily aspirin therapy has been shown in prospective, randomized studies to reduce the incidence of stroke and death in patients who have transient ischemic attacks. If alternative therapy is needed, either because the patient cannot tolerate aspirin or because aspirin therapy has failed, the antiplatelet agent clopidogrel (Plavix) can be used.

A 65-year-old patient has experienced several transient ischemic attacks over the past few months. Because his general health is poor, he is not considered an appropriate candidate for carotid endarterectomy. Examination reveals a regular rate on heart examination with a blood pressure of 130/86 mm Hg. The decision is made to treat him medically. Which of the following agents would be most appropriate for his therapy? A. Aspirin B. Coumadin (Warfarin) C. Dipyridamole (Persantine) D. Heparin E. Clopidogrel (Plavix) plus aspirin

The correct answer is A. Although treatment of transient ischemic attacks (actually prophylaxis against stroke) remains a controversial area, you should be aware of current recommendations. Daily aspirin therapy has been shown in prospective, randomized studies to reduce the incidence of stroke and death in patients who have transient ischemic attacks. If alternative therapy is needed, either because the patient cannot tolerate aspirin or because aspirin therapy has failed, the antiplatelet agent clopidogrel (Plavix) can be used. Anticoagulation with coumadin (choice B) does not decrease the risk for stroke and death any better than aspirin for patients who do not have atrial fibrillation, but it does increase the risk for intracerebral hemorrhage. Despite the antiplatelet activity of dipyridamole (choice C), therapeutic trials have not shown efficacy in the prophylaxis of stroke unless it is combined with aspirin (a product known as Aggrenox, aspirin and dipyridamole). Anticoagulation with heparin (choice D) does not decrease the risk for stroke and death but does increase the risk for intracerebral hemorrhage. Its use is limited because there is no oral heparin formulation that is available. There is no added benefit for using aspirin in combination with clopidogrel in stroke prevention (choice E). Clopidogrel alone may be used or it can be a back-up to aspirin for stroke prevention for those who cannot take aspirin. Reviewed on 4/17/14 When a patient develops a TIA, he or she is at extremely high risk for acute stroke situation. These patients need to have evaluation as to why they developed the TIA to address the underlying cause so that stroke can be prevented. If significant carotid stenosis is found, carotid endarterectomy is recommended. For patients who are not surgical candidates, stenting or angioplasty is recommended. Antiplatelet agents are preferred first-line agents in patients who have TIAs who do not have underlying atrial fibrillation or who do not have significant stenosis (<60%) post-TIA. For patients who have atrial fibrillation, warfarin or dabigatran (Pradaxa) are preferred nonsurgical therapies.

A 57-year-old woman comes to the emergency department complaining of a painful left foot. The patient describes the acute onset of foot tingling and numbness 3 days ago. Since that time the symptoms have progressed to constant pain and inability to bear weight on that foot. The patient's other medical history is significant for atrial fibrillation secondary to amyloidosis. She has been in atrial fibrillation intermittently for the past 3 years and has had attempted chemical and electrical cardioversion, but without long-term success. She currently takes no medications. On examination, she has an irregularly irregular pulse at 94/min. An electrocardiogram shows rhythm compatible with atrial fibrillation. Which of the following additional findings would most likely support the suspected diagnosis? A. Absent dorsalis pedis pulse on the left foot B. Lactate dehydrogenase level of 500 U/L C. Loss of light touch sensation in her affected limb D. Prothrombin time of 16 seconds (normal 11-13 seconds) E. Regular heart rhythm

The correct answer is A. An embolization as a result of atrial fibrillation that would cause the symptoms described by the patient would most likely lead to an absence of distal pulses in the affected leg, unless the embolism underwent autolysis. Patients who have atrial fibrillation are at increased risk for the development of thrombi in their left atrium. If the patient regains normal sinus rhythm (from the patient having paroxysmal atrial fibrillation), the organized atrial contraction may cause the clot to leave the left atrium and go into the systemic circulation. These emboli may migrate to the brain and cause a stroke, to the intestines and cause bowel ischemia, or to the peripheral vascular system and cause limb ischemia. Signs of acute limb ischemia include pain, pallor, paresthesias, poikilothermia (coolness), and paralysis. Limb-threatening ischemia may be treated with surgical embolectomy, TPA therapy, or catheter-directed urokinase therapy.

A 64-year-old man comes to the office for a routine checkup. He has been taking metformin (Glucophage) for his diabetes. The patient's vital signs show an elevated blood pressure, which is confirmed by his daily blood-pressure measurements at home. Urine studies show evidence of mild proteinuria. The health care provider decides to add a new medication to his regimen to treat his hypertension. What is the best class of medication to add at this time? A. ACE inhibitors B. Beta-blockers C. Calcium channel blockers D. Diuretics E. Alpha-blockers

The correct answer is A. Angiotensin-converting enzyme (ACE) inhibitors are the drug class of choice for treating hypertension in patients who have diabetic nephropathy, because they improve renal function and delay end-stage renal disease. ACE inhibitors block the angiotensin-converting enzyme in the renin-angiotensin system, causing vasodilation and increasing blood flow to the kidneys. Patients who have diabetes mellitus are at high risk for complications related to this endocrine disorder. These patients have increased risk for coronary artery disease, stroke, peripheral vascular disease, and renal disease. Blood pressure goals for these patients are lower (130/80 mm Hg) and these patients commonly need pharmacologic therapy to meet these goals. ACE inhibitors (or ARBs if not tolerated) are preferential agents to use because of their efficacy and ability to further protect the heart and kidneys. They are vasodilators that decrease the systemic peripheral vascular resistance, which helps to avoid cardiac remodeling, and they decrease the glomerular filtration pressure, which lessens protein loss and helps to preserve renal function.

A 28-year-old man presents to his primary care provider for his yearly physical examination. He currently smokes approximately one pack of cigarettes a day and is interested in quitting. He has no significant past medical history and has no allergies. He is interested in smoking cessation classes but would like additional help. Which of the following medications would be the most appropriate agent for treating nicotine dependence in this patient? A. Bupropion (Wellbutrin, Zyban) B. Buspirone (BuSpar) C. Clonazepam (Klonopin) D. Fluoxetine (Prozac) E. Nefazodone (Serzone)

The correct answer is A. Bupropion was approved by the U.S. Food and Drug Administration in 1996 as a treatment for nicotine dependence. The specific mechanism of action is unclear but is thought to relate to reducing craving for nicotine as well as reducing withdrawal symptoms after cessation. When used as a smoking cessation therapy, the medication is known as Zyban. This agent is an antidepressant that inhibits neuronal uptake of norepinephrine and dopamine. Tobacco abuse is a major promoter of cardiovascular disease. Smoking is the most common cause of preventable death and disease. Health care providers play an important role in assisting patients in stopping smoking. In addition to counseling, several pharmacologic methods are available to assist smokers in stopping: nicotine replacement therapy, bupropion, or varenicline (Chantix) are all first-line agents. Clonidine (Catapres) and nortriptyline (Pamelor) are considered second-line agents. Motivation of the smoker to stop plays an important role in determining the success of these methods.

Which of the following best describes why carotid sinus massage will result in the slowing of PSVT along with return of the patient to normal sinus rhythm? A. PSVT is caused by a reentry circuit that involves the AV node, and vagal stimulation from this maneuver will stimulate the vagus nerve to break the reentry cycle. B. Carotid massage will cause hypoventilation, which will stimulate the vagus nerve slowing the heart. C. Carotid massage will interfere with venous return of the heart, causing less blood back into the right atria, sufficiently slowing the heart and breaking the reentry circuit. D. PSVT is one arrhythmia that will not respond to carotid massage, so there is no effect in breaking the reentry circuit that occurs with this condition. E. Carotid sinus massage inhibits the activity of the vagus nerve, which permits the PSVT cycle to be broken as a result of this maneuver.

The correct answer is A. Carotid massage of the baroreceptors will cause vagus nerve stimulation of the heart to increase, resulting in an increased parasympathetic action on the heart. This stimulation will slow the heart's conduction through the AV node. When the speed of the impulse is slowed through the antegrade pathway, it will break the reentry cycle of PSVT by slowing the antegrade pathway to a speed that approaches the speed of the retrograde pathway, thus terminating the essential requirement that the reentry dysrhythmia needs to work.

A 59-year-old woman comes to the health care provider because of recurrent episodes of lightheadedness upon getting up in the morning and occasionally upon standing up from a chair. She reports that on two occasions she has "passed out" soon after getting up from bed. Her temperature is 37°C (98.6°F), blood pressure is 130/80 mm Hg, pulse is 70/min and regular, and respirations are 14/min. She takes a beta-blocker and a thiazide diuretic for moderate hypertension diagnosed 6 months ago. She had rheumatic fever as a child. A thorough physical examination, including chest auscultation, is unremarkable. Which of the following is the most likely cause of this patient's symptoms? A. Antihypertensive treatment B. Aortic stenosis C. Drop attacks D. Reflex syncope E. Transient ischemic attacks F. Vasovagal syncope

The correct answer is A. Clinical history is crucial in the diagnostic approach to syncope. Furthermore, measurement of orthostatic blood pressures and pulses should be performed first with the patient in a supine position, and then checked again sitting and standing. This patient's episodes of lightheadedness and syncope can be best explained as an effect of antihypertensive therapy. Orthostatic hypotension is one of the most frequent side effects of antihypertensive drugs and should always be considered in the differential diagnosis of syncope of unexplained origin. Differential diagnosis of syncope or near syncope is an important condition to understand. The history provides the most important clues to the diagnosis. Always suspect orthostatic hypotension in patients: Who have symptoms of dizziness precipitated by standing (or any head-up position) Whose symptoms are relieved by lying flat Who have history of new medications (especially blood pressure medications) Who are taking antihypertensive medications with lightheadedness/syncope, these are usually the precipitating cause unless proven otherwise Patients should undergo a posture test as the initial test in this evaluation. If the results are still questionable, the patient should undergo tilt table testing.

A 67-year-old man comes to the clinic complaining of steady, dull back pain over the past 3 weeks. He states that he has recently moved after retiring from a career in banking and is searching for a new health care provider. His past medical history is significant for diverticulosis, prior smoking, and hypertension. He says that he has run out of his blood pressure medication. He denies trauma to his back and otherwise feels well. On physical examination, his blood pressure is 170/93 mm Hg with a pulse of 88/min. He has no tenderness over the spinal processes or paraspinal areas. His abdomen is obese, but there is a suggestion of a non-tender, pulsatile mass in the epigastric region. The remainder of the physical examination is normal. Which of the following diagnoses should be considered at this time? A. Abdominal aortic aneurysm (AAA) B. Acute aortic dissection C. Cauda equina syndrome D. Lumbosacral disk herniation E. Pancreatitis

The correct answer is A. It is imperative to recognize the potential presence of an abdominal aortic aneurysm (AAA). The combination of the history of hypertension and smoking, the new back pain, and a pulsatile mass on examination is highly suggestive for abdominal aneurysm. Additionally, he is a male older than 65 years of age, which also puts him at higher risk for AAA development. The back pain occurs as the expanding mass compresses structures in the retroperitoneum. It is particularly important to make the diagnosis, because large aneurysms (greater than 5 cm in diameter) are associated with a very high risk for rupture and subsequent mortality. Abdominal aortic aneurysm should be considered in older males, smokers, and patients who have atherosclerosis who have a palpable mass in the abdomen. AAA is often found incidentally when the patient has an imaging procedure on the abdomen as part of evaluation for another abdominal condition. Risk for rupture is highly dependent on the diameter of the aorta. Elective repair is recommended for males who have a 5.5-cm and females who have a 5.0-cm diameter abdominal aorta. Elective surgery has a much better prognosis than repair of the abdominal aorta following rupture.

A 67-year-old man comes to the clinic complaining of steady, dull back pain over the past 3 weeks. He states that he has recently moved after retiring from a career in banking and is searching for a new health care provider. His past medical history is significant for diverticulosis, prior smoking, and hypertension. He says that he has run out of his blood pressure medication. He denies trauma to his back and otherwise feels well. On physical examination, his blood pressure is 170/93 mm Hg with a pulse of 88/min. He has no tenderness over the spinal processes or paraspinal areas. His abdomen is obese, but there is a suggestion of a non-tender, pulsatile mass in the epigastric region. The remainder of the physical examination is normal. Which of the following diagnoses should be considered at this time? A. Abdominal aortic aneurysm (AAA) B. Acute aortic dissection C. Cauda equina syndrome D. Lumbosacral disk herniation E. Pancreatitis

The correct answer is A. It is imperative to recognize the potential presence of an abdominal aortic aneurysm (AAA). The combination of the history of hypertension and smoking, the new back pain, and a pulsatile mass on examination is highly suggestive for abdominal aneurysm. Additionally, he is a male older than 65 years of age, which also puts him at higher risk for AAA development. The back pain occurs as the expanding mass compresses structures in the retroperitoneum. It is particularly important to make the diagnosis, because large aneurysms (greater than 5 cm in diameter) are associated with a very high risk for rupture and subsequent mortality. Acute aortic dissection (choice B) is a common aortic catastrophe requiring admission to the hospital. This condition results from an extension of an intimal tear in the wall of the artery. The hallmark of acute dissection pain is sudden onset, severe chest and abdominal pain that often radiates from an anterior to posterior direction. The pain is classically described as tearing in quality. Hypertension and trauma are common precipitants for this condition. The cauda equina syndrome (choice C) is also important to recognize, as it is a neurologic emergency. It is caused by compression of the lower lumbar and sacral nerve roots, however, and produces sensory loss in a saddle distribution, decreased reflexes, urinary incontinence, and flaccid and weak legs, none of which are present in this case. Although lumbosacral disk herniation (choice D) is one of the most common etiologies of low back pain, the pain is usually described as sharp or lancinating, with radiation from the back down to the legs. Approximately 90% of disk herniations respond to conservative medical management. Pancreatitis (choice E) may also cause dull pain radiating to the back, but the pain is usually excruciating. The findings in this case that make pancreatitis less likely are the non-tender abdomen and lack of any constitutional symptoms. Patients who have pancreatitis appear acutely ill. Reviewed on 4/17/14 Abdominal aortic aneurysm should be considered in older males, smokers, and patients who have atherosclerosis who have a palpable mass in the abdomen. AAA is often found incidentally when the patient has an imaging procedure on the abdomen as part of evaluation for another abdominal condition. Risk for rupture is highly dependent on the diameter of the aorta. Elective repair is recommended for males who have a 5.5-cm and females who have a 5.0-cm diameter abdominal aorta. Elective surgery has a much better prognosis than repair of the abdominal aorta following rupture.

A 28-year-old man seeks help for progressive edema affecting his right lower extremity. It first started about 2 years ago but has become much worse in the last 6 months. Physical examination shows pitting edema from the groin to the toes, as well as very prominent varicose veins and an area of chronic cellulitis above the medial malleolus. These findings are confined to the right side and are not present at all on the left. He also is noted to have a palpable thrill and an audible bruit over the right groin. His resting pulse is 115/min. He gives a history of having suffered a gunshot wound to his right groin 3 years ago, for which he had no specific therapy. Which of the following is the most likely diagnosis? A. Arteriovenous fistula B. Chronic lymphedema C. Deep thrombophlebitis D. Postphlebitic syndrome E. Superficial saphenous venous insufficiency

The correct answer is A. The bruit and the resting pulse rate are virtually pathognomonic for an arteriovenous fistula, particularly in view of the history of untreated penetrating trauma to the groin. Significant venous hypertension can develop in that setting, leading to the other presenting symptoms. Patients who have trauma (especially penetrating trauma) affecting an artery or vein may develop arteriovenous fistula when this traumatized area subsequently heals. Classic findings associated with this condition are bruits and thrills in a vein that becomes arterialized as a result of this healing.

A 28-year-old man seeks help for progressive edema affecting his right lower extremity. It first started about 2 years ago but has become much worse in the last 6 months. Physical examination shows pitting edema from the groin to the toes, as well as very prominent varicose veins and an area of chronic cellulitis above the medial malleolus. These findings are confined to the right side and are not present at all on the left. He also is noted to have a palpable thrill and an audible bruit over the right groin. His resting pulse is 115/min. He gives a history of having suffered a gunshot wound to his right groin 3 years ago, for which he had no specific therapy. Which of the following is the most likely diagnosis? A. Arteriovenous fistula B. Chronic lymphedema C. Deep thrombophlebitis D. Postphlebitic syndrome E. Superficial saphenous venous insufficiency

The correct answer is A. The bruit and the resting pulse rate are virtually pathognomonic for an arteriovenous fistula, particularly in view of the history of untreated penetrating trauma to the groin. Significant venous hypertension can develop in that setting, leading to the other presenting symptoms. Chronic lymphedema (choice B) could follow surgery or radiation to the groin, but not penetrating trauma. Furthermore, lymphatic disease does not produce bruits or tachycardia. Lymphedema classically causes continued swelling in the affected extremity. Lymphedema is the progressive swelling of a body part; it can be inherited or an acquired disruption of the lymphatic system that results in the accumulation of lymph in the interstitial space. Worldwide, lymphedema may be caused by nematode infection known as filariasis. Deep thrombophlebitis (choice C) and postphlebitic syndrome (choice D) can indeed produce edema and varicosities, but not a bruit or tachycardia. Superficial saphenous venous insufficiency (choice E) can lead to varicose veins, but not to all the other findings exhibited by this patient. It typically will result in swollen veins that occur after the venous valves become incompetent. Reviewed on 9/2/14 Patients who have trauma (especially penetrating trauma) affecting an artery or vein may develop arteriovenous fistula when this traumatized area subsequently heals. Classic findings associated with this condition are bruits and thrills in a vein that becomes arterialized as a result of this healing.

The mortality rate per year is greatest with disease of which coronary anatomy? A. Left main coronary B. Right coronary C. Left circumflex D. Left anterior descending

The correct answer is A. The left main coronary artery is the primary arterial supply to the left ventricle. Blockage of this artery leads to anterior wall and lateral wall myocardial infarction. Mortality for patients who have left main coronary artery disease is more than 10 times greater than in patients who have one- or two-vessel disease involving the other coronaries. Left main coronary artery is the proximal artery for both the left anterior descending and left circumflex arteries. Blockage of the left main artery will affect blood supply to the left ventricle, which is considered to be the workhorse for the entire heart. Blockage in this area affects more muscle mass in the heart than any other artery. Because of the high mortality associated with blockage in this artery, this artery is notoriously known as the "widow maker."

A 50-year-old man is admitted to the hospital after sustaining an acute myocardial infarction. Eight hours after this event, his blood pressure is 70/50 mm Hg and his pulse is 45/min. An electrocardiogram reveals sinus bradycardia rhythm. Which of the following is the most appropriate intervention? A. Administer atropine intravenously B. Administer dobutamine C. Administer a beta-blocker D. Insert a transvenous pacemaker E. Perform cardiac catheterization

The correct answer is A. The patient is hypotensive and bradycardic. This suggests a vagal response, and administering an agent that is vagolytic, such as atropine, is the correct treatment. Atropine is given for symptomatic bradycardia post-MI at a dosage of 0.5 mg every 5 minutes up to a total of 3 mg. As the vagus nerve is blocked, the patient's heart rate will increase. Immediate pharmacotherapy for a hypotensive bradycardic patient post-myocardial infarction is atropine; if atropine fails to increase the heart rate, pacing may be required. Increasing the heart rate with atropine by blocking vagal activity (parasympatholytic) has the potential to increase the blood pressure as the pulse increases.

A 50-year-old man is admitted to the hospital after sustaining an acute myocardial infarction. Eight hours after this event, his blood pressure is 70/50 mm Hg and his pulse is 45/min. An electrocardiogram reveals sinus bradycardia rhythm. Which of the following is the most appropriate intervention? A. Administer atropine intravenously B. Administer dobutamine C. Administer a beta-blocker D. Insert a transvenous pacemaker E. Perform cardiac catheterization

The correct answer is A. The patient is hypotensive and bradycardic. This suggests a vagal response, and administering an agent that is vagolytic, such as atropine, is the correct treatment. Atropine is given for symptomatic bradycardia post-MI at a dosage of 0.5 mg every 5 minutes up to a total of 3 mg. As the vagus nerve is blocked, the patient's heart rate will increase. Inotropic agents, such as dobutamine (choice B), are not first-line treatment for post-MI bradycardia. Inotropic agents will cause an increase in the myocardial oxygen demand and they have the potential to lead to increasing the size of the myocardial infarction. They should be used in the setting of hypotension that does not respond to fluid resuscitation. If other methods of resuscitation, such as IV hydration, fail, then a pressor is indicated. A beta-blocker, such as metoprolol (choice C), is indicated in the setting of a myocardial infarction, given its cardioprotective effect. In this acute setting, however, its effect on the pulse and blood pressure will be counterproductive in a patient who is hypotensive and bradycardic. If the bradyarrhythmia and hypotension persist after the administration of atropine, the insertion of a temporary pacemaker is indicated (choice D). If the patient develops a sick sinus syndrome as a result of the infarct and is symptomatic in terms of hypotension and syncope, then a pacemaker may be needed. Cardiac catheterization is indicated in the acute setting (choice E). In this patient, 8 hours have elapsed, and his ST segments have resolved. If he develops another ST-segment elevation myocardial infarction, then he will need an emergent catheterization. Reviewed on 4/17/14 Immediate pharmacotherapy for a hypotensive bradycardic patient post-myocardial infarction is atropine; if atropine fails to increase the heart rate, pacing may be required. Increasing the heart rate with atropine by blocking vagal activity (parasympatholytic) has the potential to increase the blood pressure as the pulse increases.

An arteriogram is performed on a patient who has atherosclerosis. Luminal narrowing of which of the following vessels would compromise blood flow through the renal arteries? A. Abdominal aorta B. Celiac trunk C. Common iliac artery D. Inferior mesenteric artery E. Superior mesenteric artery

The correct answer is A. The renal arteries emerge from the abdominal aorta at about the level of the L1/L2 intervertebral disk and travel at nearly right angles to it (on the right, passing posterior to the inferior vena cava) to enter the hilum of the kidney. Occlusion to a blood vessel results in impaired blood supply distal to the site of occlusion. The renal arteries are a direct tributary off the abdominal aorta. Abdominal aortic dissection or rupture of an abdominal aortic aneurysm may contribute to acute renal failure because of impaired blood supply to the kidneys. Additionally, when the abdominal aorta is clamped as part of surgical repair, it is important to clamp the abdominal aorta distal to the takeoff from the renal arteries to try to maintain blood supply to the kidneys. It is common for acute kidney damage to occur following surgery on the abdominal aorta because of this clamping and also because of hypotension that may occur as a result of this repair.

An arteriogram is performed on a patient who has atherosclerosis. Luminal narrowing of which of the following vessels would compromise blood flow through the renal arteries? A. Abdominal aorta B. Celiac trunk C. Common iliac artery D. Inferior mesenteric artery E. Superior mesenteric artery

The correct answer is A. The renal arteries emerge from the abdominal aorta at about the level of the L1/L2 intervertebral disk and travel at nearly right angles to it (on the right, passing posterior to the inferior vena cava) to enter the hilum of the kidney. The celiac trunk (choice B) gives off the common hepatic, splenic, and left gastric arteries. This artery primarily supplies blood to the colon. The common iliac artery (choice C) gives off the internal and external iliac arteries. These arteries supply blood to the lower extremities. Occasionally a persistent, supernumerary renal artery may be seen arising from the junction of the abdominal aorta and the common iliac artery. In addition, an unascended pelvic kidney may be supplied by the common iliac artery. The inferior mesenteric artery (choice D) gives off the superior rectal, sigmoid, and left colic arteries. Occlusion to this artery causes acute bowel ischemia. The superior mesenteric artery (choice E) gives off the inferior pancreaticoduodenal, intestinal (ileal and jejunal), right colic, middle colic, and ileocolic arteries. Occlusion of this artery causes acute bowel ischemia. Reviewed on 4/17/14 Occlusion to a blood vessel results in impaired blood supply distal to the site of occlusion. The renal arteries are a direct tributary off the abdominal aorta. Abdominal aortic dissection or rupture of an abdominal aortic aneurysm may contribute to acute renal failure because of impaired blood supply to the kidneys. Additionally, when the abdominal aorta is clamped as part of surgical repair, it is important to clamp the abdominal aorta distal to the takeoff from the renal arteries to try to maintain blood supply to the kidneys. It is common for acute kidney damage to occur following surgery on the abdominal aorta because of this clamping and also because of hypotension that may occur as a result of this repair.

A 55-year-old woman with a long-standing history of atrial fibrillation secondary to mitral regurgitation comes to the emergency department with a painful right foot. The patient reports that over the past few hours her foot has become more painful and now is nearly insensate. She describes the pain as burning and states that it is not relieved by any intervention. She takes warfarin (Coumadin), atenolol (Tenormin), digoxin (Lanoxin), and aspirin. On physical examination, her pulse is irregularly irregular. Her lungs are clear, and she has a loud holosystolic murmur heard best at the apex. Her right foot is gray and cool to the touch and has poor capillary refill. Dorsalis pedis and posterior tibial pulses are absent on the right. Her prothrombin time is 14.4 seconds (normal 11-13 seconds) and her INR is 1.4 with goal of anticoagulation being 2-3. Which of the following is the most appropriate course of action? A. Arrange for her to be seen urgently by a vascular surgeon in the emergency department now B. Arrange for her to be seen by a neurologist within the next few days C. Arrange for her to undergo an MRI of the head emergently D. Ask her to make an appointment to be seen in your office within 1 week E. Instruct her to soak her leg in warm water and to place a fitted stocking on her affected leg

The correct answer is A. The symptoms and signs that she is describing, particularly in the context of atrial fibrillation (AF), suggest peripheral embolization, which is a surgical emergency. The treatment of choice involves immediate embolectomy performed by a vascular surgeon, TPA infusion therapy, or insertion of a catheter with urokinase-directed therapy. After successful embolism removal or dissolution, anticoagulation is performed along with administration of heparin, which is given to prevent new clot formation. Her subtherapeutic prothrombin time and persistent AF on examination are supportive of this diagnosis. Atrial fibrillation is a risk for formation of a thrombus in the left atrium. This thrombus can be flushed into the systemic circulation and result in peripheral embolism that puts a patient's limb at risk for ischemia and infarction. When peripheral embolism is identified, emergent embolectomy or methods to dissolve the embolism needs to be performed to salvage the limb from necrosis.

A 55-year-old man with a recent syncopal episode is admitted to the hospital with congestive heart failure. His blood pressure is 160/100 mm Hg and pulse 90/min. He has a grade 2/6 harsh systolic ejection murmur. An echocardiogram reveals a thickened ventricular septum and systolic anterior motion of the mitral valve. Which of the following will most likely be found in this patient? A. Decreased murmur with hand grip B. Decreased murmur with Valsalva C. Delayed carotid upstroke D. Increased murmur with squatting E. Murmur radiating to carotid arteries

The correct answer is A. The thickened ventricular septum and the systolic anterior motion of the mitral valve suggest hypertrophic cardiomyopathy. The murmur is harsh and systolic and decreases when afterload increases as a result of hand grip exercise. Hand grip increases systemic vascular resistance, which allows the heart to fill with blood against increased afterload; this filled ventricle will lessen the obstruction that occurs with hypertrophic cardiomyopathy.

A 60-year-old man comes to the emergency department complaining of tearing chest pain that began abruptly and radiates to the back. His electrocardiogram is normal. Chest radiograph shows a widening of the mediastinum. What is the most likely diagnosis? A. Aortic dissection B. Pericardial tamponade C. Pulmonary embolism D. Myocardial infarction E. Abdominal aortic aneurysm

The correct answer is A. This is a classic presentation of aortic dissection. The patient may be in shock, and his pulses may be unequal. Usually there is a history of hypertension or Marfan syndrome in the affected patient. Aortic dissection is a potentially lethal cardiac emergency and requires immediate medical or surgical intervention. If it is misdiagnosed or left untreated, it may lead to mortality.

A 4-year-old, apparently healthy child is examined by a health care provider. The health care provider hears a loud systolic ejection murmur with a prominent systolic ejection click. He also hears a soft, early diastolic murmur. Both murmurs are heard best at the upper right sternal border. An electrocardiogram shows left ventricular hypertrophy. The child is active and participates in normal activities without problems. Which of the following is the most likely diagnosis? A. Aortic valve stenosis B. Atrial septal defect C. Tetralogy of Fallot D. Transposition of great arteries E. Ventricular septal defect

The correct answer is A. This is aortic valve stenosis, which accounts for 5% of diagnosed cardiac defects but may actually be the most common congenital anomaly of the heart, because many minor cases are never diagnosed. Most cases are caused by bicuspid aortic valves and characteristically produce a systolic ejection murmur. An accompanying aortic insufficiency may produce an early diastolic murmur. The timing of surgical correction depends on the severity of the individual case.

A 54-year-old man comes to his health care provider complaining of intermittent palpitations. The patient reports that a few times over the past few months he has had episodes of "pounding in his chest" that are associated with shortness of breath and occasional chest pain. He is forced to sit down if he is standing, because of weakness and vertigo. The patient has a history of hypertension and mitral valve prolapse. He takes nifedipine (Procardia, Adalat) and hydrochlorothiazide daily. While sitting in the office, the patient begins to complain of increasing shortness of breath and palpitations. His blood pressure is 85/50 mm Hg and his pulse is 110-130/min and irregularly irregular. Which of the following is the most appropriate management at this time? A. Call 911 for assistance B. Give the patient an oral dose of a beta-blocker C. Give the patient an oral dose of digoxin D. Give the patient an oral dose of a calcium-channel blocker E. Make arrangements to have the patient brought to the local emergency department for electrical cardioversion

The correct answer is A. This patient has acute atrial fibrillation (AF) with rapid ventricular response (RVR) and is consequently hypotensive. This is a medical emergency. Even the health care provider caring for this patient is ill equipped to deal with a potentially life-threatening episode of AF with RVR. Activating the emergency medical response system is always appropriate and ensures that trained persons who have additional equipment and medications appropriate to an emergency situation will be on hand as soon as possible. The patient will need to have IV access, intravenous fluid support, and medication or cardioversion as part of the treatment regimen. According to the Advanced Cardiac Life Support protocol, a patient who has atrial fibrillation with a rapid ventricular response who is hypotensive should be cardioverted, something that emergency medical personnel will have available to them. Acute onset of atrial fibrillation with a rapid ventricular response is associated with hypotension. The patient may have acute deterioration that would necessitate immediate cardioversion/defibrillation. Patients need to be given volume and airway support, and management of patients in an emergency department setting is preferred. Because of the patient rapidly deteriorating, patients should be transferred to the emergency setting via emergency medical support personnel, who can intervene in case the patient becomes unstable.

A 54-year-old man comes to his health care provider complaining of intermittent palpitations. The patient reports that a few times over the past few months he has had episodes of "pounding in his chest" that are associated with shortness of breath and occasional chest pain. He is forced to sit down if he is standing, because of weakness and vertigo. The patient has a history of hypertension and mitral valve prolapse. He takes nifedipine (Procardia, Adalat) and hydrochlorothiazide daily. While sitting in the office, the patient begins to complain of increasing shortness of breath and palpitations. His blood pressure is 85/50 mm Hg and his pulse is 110-130/min and irregularly irregular. Which of the following is the most appropriate management at this time? A. Call 911 for assistance B. Give the patient an oral dose of a beta-blocker C. Give the patient an oral dose of digoxin D. Give the patient an oral dose of a calcium-channel blocker E. Make arrangements to have the patient brought to the local emergency department for electrical cardioversion

The correct answer is A. This patient has acute atrial fibrillation (AF) with rapid ventricular response (RVR) and is consequently hypotensive. This is a medical emergency. Even the health care provider caring for this patient is ill equipped to deal with a potentially life-threatening episode of AF with RVR. Activating the emergency medical response system is always appropriate and ensures that trained persons who have additional equipment and medications appropriate to an emergency situation will be on hand as soon as possible. The patient will need to have IV access, intravenous fluid support, and medication or cardioversion as part of the treatment regimen. According to the Advanced Cardiac Life Support protocol, a patient who has atrial fibrillation with a rapid ventricular response who is hypotensive should be cardioverted, something that emergency medical personnel will have available to them. Giving the patient an oral dose of a beta-blocker (choice B), digoxin (choice C), or a calcium-channel blocker (choice D) is not appropriate for an emergent setting. These medications will take too long to be effective. These agents are not typically used to cardiovert patients who are in acute atrial fibrillation. Beta-blockers and calcium-channel blockers should not be given to patients who are already hypotensive, because this will further lower blood pressure and make the patient worse. Digoxin may be used in rate control by acting as a negative chronotropic and positive inotropic agent, but it is not as effective in situations with high adrenergic activity, such as acute atrial fibrillation. Making arrangements to have the patient brought to the local emergency department for electrical cardioversion (choice E) is appropriate only after 911 has been called in the case that the patient needs immediate assistance in the office. Without calling 911, the patient may have a seriously adverse event en route to the hospital. Reviewed on 9/4/14 Acute onset of atrial fibrillation with a rapid ventricular response is associated with hypotension. The patient may have acute deterioration that would necessitate immediate cardioversion/defibrillation. Patients need to be given volume and airway support, and management of patients in an emergency department setting is preferred. Because of the patient rapidly deteriorating, patients should be transferred to the emergency setting via emergency medical support personnel, who can intervene in case the patient becomes unstable.

A 64-year-old woman comes to her primary health care provider for management of her hypertension, which has been treated unsuccessfully for several years. She was recently hospitalized for pulmonary edema, and an echocardiogram at that time showed a moderately depressed ejection fraction. She was diagnosed with congestive heart failure. Her medications include a thiazide diuretic and a calcium channel blocker. She has an allergy to furosemide. Her review of systems is positive for two-pillow orthopnea and occasional paroxysmal nocturnal dyspnea. On physical examination, her blood pressure is 150/80 mm Hg and her pulse is 80/min and regular. Her lungs are clear, and there are no extra heart sounds. Her extremities are without edema. Which of the following is the most appropriate management at this time? A. Add an ACE inhibitor to her regimen B. Add an angiotensin II receptor blocking agent to her regimen C. Add hydralazine (Apresoline) to her regimen D. Increase the dose of her calcium channel blocker E. Increase the dose of her thiazide diuretic

The correct answer is A. This patient has both hypertension and congestive heart failure (CHF). An important concept to recognize in the treatment of medical conditions is that certain medications overlap syndromes and are efficacious in many areas. This "co-treatment" option maximizes each drug in a regimen and often addresses two or more issues simultaneously. In this case, ACE inhibitors have been shown to be very beneficial in prolonging the survival of CHF patients.

A 16-year-old boy is brought to the emergency department by his parents because of an episode of severe chest pain several hours earlier. Review of symptoms reveals that the patient is recovering from a flu-like illness that started a week earlier. He had been complaining of fever, chills, abdominal discomfort, and feeling tired during the prior week. He has had a low-grade fever for 5 to 6 days and a sore throat, but pharyngeal swab cultures done at the primary care physician's office had been negative and he was taking over-the-counter cold medication only. Physical examination reveals a well developed and well nourished young man in moderate distress. His temperature is 37.8°C (100.0°F), pulse is 120/min, and respirations are 28/min. A differential white blood cell count shows a normal number of neutrophils and marginally elevated lymphocytes. An electrocardiogram shows low voltage QRS complexes throughout the limb leads. Chest radiography is remarkable for increased pulmonary markings and an enlarged heart silhouette. Which of the following is the most appropriate next step in the management? A. Admit to monitored floor for further evaluation and management B. Discharge from emergency department after stabilization of vital signs C. Overnight observation in the emergency department D. Reassurance and discharge E. Recommend symptomatic antipyretic therapy, bed rest, a low salt diet, and discharge from emergency department

The correct answer is A. This patient has signs of myocarditis and should be admitted to a monitored hospital bed for further evaluation and management. Myocarditis is an inflammation of the myocardium. The etiology may be multiple, but it is most commonly viral, caused by adenovirus and Coxsackie B. Bacteria (diphtheria), Rickettsia, fungi, and parasites are also infectious causes of myocarditis. Connective tissue diseases, granulomatous diseases, and toxins may cause noninfectious myocarditis. The most common clinical presentation is heart failure. Older children may present with chest pain secondary to myocardial ischemia or concurrent pericarditis. Arrhythmias and sudden death are less common. The age of the patient also influences the clinical presentation, and it is more acute or fulminant in infancy. Viral myocarditis is usually preceded by a viral illness. In these cases, the patient may present with fever, heart failure, respiratory distress, and cyanosis. The erythrocyte sedimentation rate, creatine kinase, and lactate dehydrogenase all may be elevated. The presence of lymphocytosis or neutropenia on a differential white blood cell count supports the diagnosis of viral myocarditis. Serum viral titers are helpful when they are positive, and polymerase chain reaction may help identify the specific virus. Chest radiography shows an enlarged heart and pulmonary edema. Electrocardiography shows sinus tachycardia, reduced QRS complex, and abnormal S and ST waves. Echocardiography shows poor ventricular function and possible pericardial effusions, and absence of congenital heart disease and coronary artery involvement. The diagnosis is confirmed by endomyocardial biopsy. Treatment includes management of heart failure and arrhythmias. In the acute phase, the patient should be admitted to the hospital even if only mild signs of respiratory distress or congestive heart failure are present. Rapid progression to overt heart failure or hemodynamic collapse may occur. Pericardiocentesis is performed to alleviate the tamponade if present. The role of systemic steroids is controversial. If refractory to medical management, heart failure is ultimately treated with heart transplantation. Although spontaneous resolution may occur, most patients do very poorly without treatment.

A 77-year-old man comes to the health care provider because of decreasing exercise tolerance. Just 1 year earlier he was able to play doubles tennis for 2 hours. Over the past few months, however, he has had progressive dyspnea on exertion and now can walk only 2 blocks on level ground before becoming short-winded. He has also been awaking from sleep with shortness of breath and requires 3 pillows to sleep comfortably. He has a history of rheumatic fever as a teenager. On physical examination his blood pressure is 168/60 mm Hg, pulse 92/min, and respirations 18/min. He is afebrile. He has jugulovenous distention lying supine. He has bibasilar rales extending 1/4 up both posterior lung fields. He has a regular S1 and S2, with a blowing diastolic murmur heard at the aortic area, which is grade II/VI. An S3 is audible. The liver edge is mildly tender, and there is moderate lower extremity edema extending to both knees. Which of the following medications will most likely be effective in the management of his cardiac disorder? A. Captopril (Capoten) B. Warfarin (Coumadin) C. Digoxin (Lanoxin) D. Furosemide (Lasix) E. Isosorbide dinitrate (Isordil)

The correct answer is A. This patient has the physical findings of aortic insufficiency and the development of congestive heart failure with biventricular failure. Left ventricular symptoms described here are the findings of pulmonary congestion, and right-sided heart failure is demonstrated by the jugulovenous distention, congested liver, and peripheral edema. Because the "backward" failure symptoms are caused by regurgitant flow across the incompetent aortic valve, the most useful therapy would be afterload reduction with an ACE inhibitor such as captopril, which will also help to prevent cardiac remodeling as the left ventricle becomes overloaded from the increased amount (volume) of blood that occurs because of aortic regurgitation. Aortic regurgitation may be helped by vasodilator therapy, which will act as afterload reducers. In addition to ACE inhibitors, pure vasodilators such as hydralazine and nifedipine can also be used. For patients who have ongoing symptoms who are surgical candidates, aortic valve replacement is performed when the disease decompensates.

A 67-year-old man comes to the health care provider complaining of increasing dyspnea on exertion. Over the past 3 weeks, he has noted increasing shortness of breath while walking to the bus stop three blocks from his home. He has also found it difficult to sleep comfortably while lying flat and now sleeps on three pillows. He has awoken several times in the past month with shortness of breath. He denies any history of chest pain. He does not smoke, and his cholesterol tests were normal 1 year earlier. On physical examination, he appears comfortable at rest. His blood pressure is 158/56 mm Hg, pulse is 86/min and regular, and respirations are 16/min. He has jugulovenous distention while being examined at 30 degrees. His lungs have bibasilar rales. On cardiac examination, there is a regular S1 and S2, with a blowing diastolic murmur heard loudest at the left sternal border. A I/VI systolic murmur is heard throughout the precordium. An extra heart sound immediately following the S2 is heard. There is mild lower extremity pedal edema bilaterally. Which of the following is the most likely cause of his symptoms? A. Aortic insufficiency B. Aortic stenosis C. Mitral regurgitation D. Mitral stenosis E. Tricuspid regurgitation

The correct answer is A. This patient has the typical symptoms of congestive heart failure with left ventricular failure. The physical examination has the characteristic findings for aortic insufficiency, including wide pulse pressure, blowing diastolic murmur, and an S3 consistent with left ventricular dilatation. The soft systolic murmur is caused by the volume overload and increased flow in the left ventricle and does not imply an additional valvular disorder. Aortic regurgitation presents with signs of heart failure caused by progressive flow of blood back into the left ventricle rather than the forward progression of blood into the systemic circulation. This causes a volume overload into the left ventricle and dilation of the left ventricle. In addition to the diastolic murmur, patients will go on to develop dyspnea, fatigue, weakness, orthopnea, and paroxysmal nocturnal dyspnea. This may progress to heart failure with altered mental status, basal lung crepitations, and poor urine output.

A 49-year-old man with a history of hypertension and hyperlipidemia has had a severe headache, blurry vision, and red urine for the past 2 days. He was in his usual state of health until 2 days ago. He has been compliant with his medications, including atenolol 50 mg/day and hydrochlorothiazide 12.5 mg/day. His temperature is 37.0°C (98.6°F), blood pressure 196/140 mm Hg, pulse 83/min, and respirations 10/min. The remainder of his physical examination is unremarkable. Which of the following is the most appropriate intervention at this time? A. Admit him to the hospital for evaluation and treatment B. Begin therapy with clonidine (Catapres) and recheck blood pressure in 30 to 60 minutes C. Begin therapy with felodipine (Plendil) and recheck blood pressure in 24 to 48 hours D. Increase the doses of his current medications and recheck blood pressure in 1 week E. Schedule an outpatient CT scan of the head and recheck blood pressure in 1 week

The correct answer is A. This patient is having a hypertensive emergency, which is defined as severe hypertension with end-organ damage. It happens for unknown reasons in approximately 1% of hypertensive patients. Our patient has hematuria, headache, and blurred vision. He needs to be admitted to the hospital and placed on intravenous blood pressure medications such as labetolol as soon as possible. Remember not to lower the blood pressure too far, too rapidly, because doing so might compromise cerebral, renal, or coronary perfusion. The goal for decreasing blood pressure is no more than 25% of the mean arterial blood pressure within the first minutes to hours. Hypertensive emergency includes an elevated blood pressure (systolic >210 and/or diastolic >130 mm Hg) along with rapid decompensation of vital organ function. Parenteral medications are used to more rapidly lower blood pressure over a period of minutes to hours. This should be administered in an intensive care unit because of the need for close blood pressure monitoring. The initial goal is to reduce mean arterial blood pressure by no more than 25% within minutes to 1 hour, with further lowering over the next 2 to 6 hours.

A 34-year-old woman with HIV and active Pneumocystis pneumonia is admitted to the hospital. Her last CD4 count was 44 cells/mm3 (normal 400-1,400 cells/mm3). She has been doing reasonably well since admission with a stable course on appropriate antibiotics. On day 2 of admission, vital signs reveal blood pressure 80/40 mm Hg and pulse 110/min. Temperature is 38.3°C (101°F) orally. Her extremities are cool and damp. Mental status examination is normal. The remainder of her physical examination is unchanged. What is the next best step in management? A. Intravenous fluids B. Intravenous pressor support C. Central venous pressor support D. Add additional antibiotics to treat empiric sepsis E. Blood transfusion

The correct answer is A. This patient is hypotensive but has minimal effects from the hypotension, such as altered mental status or signs of shock. This blood pressure, however, is still not acceptable, and the cause for it must be determined. In the interim, the symptom (low blood pressure) must be treated. The standard therapy in all such situations is volume. Because she has no signs of altered mental status, there is no urgent indication for pharmacologic blood pressure support, and IV fluids are the first-line initial management tools for this patient.

A 34-year-old woman with HIV and active Pneumocystis pneumonia is admitted to the hospital. Her last CD4 count was 44 cells/mm3 (normal 400-1,400 cells/mm3). She has been doing reasonably well since admission with a stable course on appropriate antibiotics. On day 2 of admission, vital signs reveal blood pressure 80/40 mm Hg and pulse 110/min. Temperature is 38.3°C (101°F) orally. Her extremities are cool and damp. Mental status examination is normal. The remainder of her physical examination is unchanged. What is the next best step in management? A. Intravenous fluids B. Intravenous pressor support C. Central venous pressor support D. Add additional antibiotics to treat empiric sepsis E. Blood transfusion

The correct answer is A. This patient is hypotensive but has minimal effects from the hypotension, such as altered mental status or signs of shock. This blood pressure, however, is still not acceptable, and the cause for it must be determined. In the interim, the symptom (low blood pressure) must be treated. The standard therapy in all such situations is volume. Because she has no signs of altered mental status, there is no urgent indication for pharmacologic blood pressure support, and IV fluids are the first-line initial management tools for this patient. Intravenous pressor support (choice B) is not first-line treatment and should be reserved for situations that do not resolve after fluid resuscitation is performed. Central venous pressor support (choice C) is not indicated before an attempt at volume resuscitation has been made. If this patient were hemodynamically unstable, it might be appropriate to initiate this therapy at the same time as massive volume resuscitation is begun. Adding additional antibiotics to treat empiric sepsis (choice D) is not appropriate until hemodynamic stability has been achieved. Remember the ABCs of emergency care. Exercise caution for circulation before caution for antibiotic therapy. In addition, there is no evidence that this patient is suffering from sepsis syndrome. It may be simply that she is hypovolemic, given her tachycardia and clinical presentation. Blood transfusion (choice E) is inappropriate unless the cause for the hypotension is blood loss or the patient is severely anemic and would benefit from blood component therapy. Fluid resuscitation would still be first-line therapy in this situation, until blood transfusions were available. Reviewed on 4/17/14 Initial management of a hypotensive, tachycardic patient is an intravenous fluid bolus followed by consideration of pharmacotherapy for blood pressure support.

A 37-year-old African American man with a history of mild hypertension comes in for an annual examination. He has no complaints. He reports compliance with his low-salt diet. His only medication is hydrochlorothiazide, 25 mg each day. A thorough review of systems is negative. His temperature is 37.0°C (98.6°F), blood pressure 160/90 mm Hg, pulse 83/min, and respirations 10/min. Physical examination is within normal limits. Laboratory studies show: Na+ 142 mEq/L K+ 3.9 mEq/L Cl− 109 mEq/L HCO3− 22 mEq/L Glucose 96 mg/dL BUN 13 mg/dL Creatinine 0.6 mg/dL Which of the following is the most appropriate intervention at this time? A. Add a second antihypertensive medication B. Schedule for a captopril renal scan C. Send a 24-hour urine collection for cortisol D. Send a 24-hour urine collection for vanillylmandelic acid, metanephrine, and catecholamine E. Send urine and serum aldosterone levels

The correct answer is A. This patient most likely has essential hypertension. Secondary forms of hypertension should only be suspected in those patients who develop hypertension at extremes of age (less than 25 or more than 55 years of age), who have suspicious physical examination findings, or who are resistant to multiple blood pressure medications. This patient is a healthy man who is only on one medication and does not have any laboratory or physical examination stigmata of secondary hypertension. Simply adding another medication to his current regimen is appropriate at this time.

A 22-year-old man is evaluated for mitral regurgitation due to mitral valve prolapse. Examination reveals a tall, slender young man with long extremities and long tapering fingers. Pupillary dilation followed by slit-lamp examination reveals bilateral dislocation of the lenses of the eyes. This patient is potentially at increased risk for development of which of the following? A. Aortic dissection B. Lisch nodules C. Noncaseating granulomata D. Progressive dementia E. Rapidly progressive renal failure

The correct answer is A. This young man is displaying features of Marfan syndrome, a genetic (often autosomal-dominant) disease of connective tissue that affects the skeleton (tall stature, long fingers and toes, hyperextensible joints), eyes (often subluxation of the lenses bilaterally), and the cardiovascular system (cystic medial necrosis predisposing to aortic dissection or aortic valve incompetence). Marfan syndrome is caused by mutations in the fibrillin gene, leading to defects in the structure of elastic tissue. Marfan syndrome is an autosomal-dominant inherited disorder of connective tissue characterized by loss of elastic tissue. Features of Marfan syndrome include musculoskeletal deformities such as increased arm span compared to height, highly arched palate, arachnodactyly and scoliosis, eye complications such as lens subluxation and dislocation, glaucoma, cataracts, retinal detachment, and aortic root dilation leading to increased risk for aortic dissection. Risk for aortic dissection is an aortic root dilation of at least 4.5 cm and also pregnancy, which has an increased volume of fluid in the circulation.

A 22-year-old man is evaluated for mitral regurgitation due to mitral valve prolapse. Examination reveals a tall, slender young man with long extremities and long tapering fingers. Pupillary dilation followed by slit-lamp examination reveals bilateral dislocation of the lenses of the eyes. This patient is potentially at increased risk for development of which of the following? A. Aortic dissection B. Lisch nodules C. Noncaseating granulomata D. Progressive dementia E. Rapidly progressive renal failure

The correct answer is A. This young man is displaying features of Marfan syndrome, a genetic (often autosomal-dominant) disease of connective tissue that affects the skeleton (tall stature, long fingers and toes, hyperextensible joints), eyes (often subluxation of the lenses bilaterally), and the cardiovascular system (cystic medial necrosis predisposing to aortic dissection or aortic valve incompetence). Marfan syndrome is caused by mutations in the fibrillin gene, leading to defects in the structure of elastic tissue. Lisch nodules (choice B) are pigmented nodules found on the iris of the eye. They are associated with type I neurofibromatosis. Noncaseating granulomata (choice C) can be seen in a variety of disorders, including sarcoidosis and berylliosis. Progressive dementia (choice D) is a feature of a number of disorders, including Alzheimer disease, Huntington disease, and Pick disease. Rapidly progressive renal failure (choice E) is not a feature of Marfan disease. It occurs in a number of other disorders, such as Goodpasture disease. Reviewed on 4/17/14 Marfan syndrome is an autosomal-dominant inherited disorder of connective tissue characterized by loss of elastic tissue. Features of Marfan syndrome include musculoskeletal deformities such as increased arm span compared to height, highly arched palate, arachnodactyly and scoliosis, eye complications such as lens subluxation and dislocation, glaucoma, cataracts, retinal detachment, and aortic root dilation leading to increased risk for aortic dissection. Risk for aortic dissection is an aortic root dilation of at least 4.5 cm and also pregnancy, which has an increased volume of fluid in the circulation.

A 40-year-old woman is brought to the emergency department following a suicide attempt with imipramine (Tofranil). Her fiancé found her unresponsive, with an empty bottle of the imipramine at her side. The imipramine had been his, and the prescription had been filled that morning. Her past medical history is significant for hypertension, atrial fibrillation, diabetes, and asthma. Her medications include furosemide, procainamide, glyburide, prednisone, and albuterol. She has no known drug allergies. She is afebrile, has a blood pressure of 100/60 mm Hg, pulse of 62/min, and respirations of 22/min. A gastric lavage yields multiple pill fragments. She is confused and somnolent, and has shallow respirations. Her physical examination is otherwise unremarkable. On an electrocardiogram, which of the following abnormalities would most likely reflect possible cardiac toxicity? A. Left deviation of the QRS axis B. Prolongation of the QT interval C. Shortening of the PR interval D. ST-segment depression E. T-wave inversion

The correct answer is B. A prolongation of the QT interval is highly predictive of both cardiac and CNS toxicities from tricyclic antidepressant ingestion. This medication has high lethality associated with its overdose because of its cardiac effects. QT prolongation may be secondary to the use of various medications or from electrolyte abnormalities or it may be an inherited condition (prolonged QT syndrome). Tricyclic antidepressants, certain antiarrhythmics (especially class III Vaughn Williams antiarrhythmics such as amiodarone), and hypocalcemia, hypomagnesemia, and hypokalemia may lead to prolonged QT syndrome. Patients who have this condition have an increased risk for syncope, ventricular arrhythmias (especially torsades des pointes), and sudden cardiac death.

A 40-year-old woman is brought to the emergency department following a suicide attempt with imipramine (Tofranil). Her fiancé found her unresponsive, with an empty bottle of the imipramine at her side. The imipramine had been his, and the prescription had been filled that morning. Her past medical history is significant for hypertension, atrial fibrillation, diabetes, and asthma. Her medications include furosemide, procainamide, glyburide, prednisone, and albuterol. She has no known drug allergies. She is afebrile, has a blood pressure of 100/60 mm Hg, pulse of 62/min, and respirations of 22/min. A gastric lavage yields multiple pill fragments. She is confused and somnolent, and has shallow respirations. Her physical examination is otherwise unremarkable. On an electrocardiogram, which of the following abnormalities would most likely reflect possible cardiac toxicity? A. Left deviation of the QRS axis B. Prolongation of the QT interval C. Shortening of the PR interval D. ST-segment depression E. T-wave inversion

The correct answer is B. A prolongation of the QT interval is highly predictive of both cardiac and CNS toxicities from tricyclic antidepressant ingestion. This medication has high lethality associated with its overdose because of its cardiac effects. Left deviation of the QRS axis (choice A), which can be seen with conditions such as left ventricular hypertrophy, left bundle branch block, and left posterior hemiblock, is not typically associated with tricyclic cardiac toxicity. Pre-excitation syndromes (choice C) are not seen with tricyclic toxicity, but they can be seen Wolff-Parkinson-White syndrome and Lown-Ganong-Levine syndromes, which are caused by the congenital presence of accessory pathways that lead to the ventricles becoming pre-excited. These conditions are not associated with the use of tricyclic antidepressants. Neither ST-segment depression (choice D) nor T-wave inversion (choice E) is directly associated with cardiac toxicity from tricyclic overdose. These changes may be seen, however, in conjunction with the more classic ECG manifestations of tricyclic toxicity (prolonged QT interval, right axis deviation) if the resulting cardiac toxicity leads to diminished coronary perfusion and ischemia. Reviewed on 4/17/14 QT prolongation may be secondary to the use of various medications or from electrolyte abnormalities or it may be an inherited condition (prolonged QT syndrome). Tricyclic antidepressants, certain antiarrhythmics (especially class III Vaughn Williams antiarrhythmics such as amiodarone), and hypocalcemia, hypomagnesemia, and hypokalemia may lead to prolonged QT syndrome. Patients who have this condition have an increased risk for syncope, ventricular arrhythmias (especially torsades des pointes), and sudden cardiac death.

A 60-year-old woman with a long history of hypertension comes to the emergency department with a complaint of increasing and recurrent shortness of breath with minimal exertion. She is noted to have a prominent precordial impulse, and a chest radiograph reveals a prominent left ventricular shadow. A stress test is negative for ischemia. She is found to have left ventricular hypertrophy and a normal ventricular ejection fraction on echocardiogram. An S3 is heard on auscultation of the heart. Which of the following is the most likely underlying diagnosis causing this presentation? A. Chronic obstructive pulmonary disease (COPD) B. Diastolic dysfunction C. Myocardial ischemia D. Reactive airways disease E. Systolic congestive heart failure

The correct answer is B. Increased resistance to filling of one or more cardiac ventricles has been termed diastolic heart failure and can produce increased pulmonary capillary wedge pressures and respiratory complaints. In myocardial hypertrophy, impaired diastolic relaxation occurs. The key to differentiating systolic from diastolic types of heart failure is in the assessment of ejection fraction; it is normal and preserved with diastolic heart failure and impaired with systolic.

During the performance of a supraclavicular node biopsy under local anesthesia, a hissing sound is suddenly heard, and the patient suddenly dies. At the time of the catastrophic event, the target node was under traction, and the final cut was being made blindly behind it to free it up completely. The patient, an otherwise healthy 24-year-old man, was inhaling at that moment. Which of the following most likely caused this patient's death? A. Arterial injury with air embolization B. Major vein injury with air embolism C. Sudden pneumothorax with lung collapse D. Sympathetic discharge E. Tracheal injury

The correct answer is B. Major veins at the base of the neck have negative pressure during inspiration and, if injured at that moment, will suck air rather than bleed. The air embolism then leads to sudden death.

During the performance of a supraclavicular node biopsy under local anesthesia, a hissing sound is suddenly heard, and the patient suddenly dies. At the time of the catastrophic event, the target node was under traction, and the final cut was being made blindly behind it to free it up completely. The patient, an otherwise healthy 24-year-old man, was inhaling at that moment. Which of the following most likely caused this patient's death? A. Arterial injury with air embolization B. Major vein injury with air embolism C. Sudden pneumothorax with lung collapse D. Sympathetic discharge E. Tracheal injury

The correct answer is B. Major veins at the base of the neck have negative pressure during inspiration and, if injured at that moment, will suck air rather than bleed. The air embolism then leads to sudden death. Arterial injury (choice A) would have led to massive bleeding but not to sudden death. Pneumothorax (choice C) can indeed happen when surgery is being done in the supraclavicular area, and a sucking sound might even be heard. Sudden lung collapse in a young, healthy person leads to dyspnea, however, not to sudden death. Sympathetic discharge (choice D) would be hard to produce while pulling and dissecting a node. If it were done, however, there would be vasoconstriction, tachycardia, perspiration, and hypertension, rather than sudden death. Tracheal injury (choice E) would have led to complications later in the hospital but not to sudden death. Reviewed on 4/17/14 Venous air embolism is typically iatrogenically produced and is associated with a high mortality. It was previously associated with neurosurgical procedures that were performed in a sitting position, but is now more commonly encountered when a central line is placed. It can also be associated with major trauma (blunt and penetrating) and with surgical procedures performed at the apices of the lungs. For this condition to occur, there must be contact of the vein with a supply of gas such as air. Complications have been known to occur with as little as 20 mL of air into the venous circulation. Rapid entry of large amounts of air into the venous circulation put a stress on the right ventricle. Pulmonary arterial pressures rise and this may result in right ventricular obstruction and poor filling of the left side of the heart. Large amounts of air in the venous system may be rapidly fatal via this mechanism.

A 63-year-old woman comes to the emergency department complaining of chest pain. The patient states that the pain began during her morning walk. It started as a dull pressure over her breastbone and then radiated to her left arm. Over the next few minutes, it escalated in intensity and was not relieved by rest. She called 911 and was brought to the emergency department. Her past medical history is significant for hypertension and hyperlipidemia. Her medications include atenolol and simvastatin daily. On physical examination, her blood pressure is 190/100 mm Hg, and her pulse is 60/min. Which of the following is the most appropriate agent to lower her blood pressure? A. Intravenous norepinephrine B. Intravenous nitroglycerin C. Oral hydrochlorothiazide D. Oral metoprolol (Lopressor) E. Oral furosemide (Lasix)

The correct answer is B. Nitroglycerin is a potent vasodilator that acts predominantly on venous compliance to reduce preload to the heart. When administered intravenously, it can be rapidly titrated to produce optimized blood pressure control. IV nitroglycerin is effective when given in the setting of unstable angina. Patients who have abrupt hypertension and angina symptoms need to have blood pressure lowered in an expeditious way. Nitroglycerin is the preferred agent in the setting of hypertension and angina because of its ability to drop the preload, vasodilate, and dilate the coronary arteries. Parenteral agents are preferred because of their rapid onset of activity and ability to be titrated. Various parenteral agents are approved in this setting, including sodium nitroprusside (Nipride) and labetolol (Trandate).

A 14-year-old boy is brought to the health care provider with decreased exercise tolerance. He is up to date on all of his childhood immunizations and has been generally healthy until now. He is noted to have a grade III/VI systolic ejection murmur best heard at the left upper sternal border and a grade II/VI mid-diastolic murmur at the lower left sternal border. The first heart sound is normal. The second heart sound is widely split and fixed. A right ventricular impulse is palpated. On a chest roentgenogram, the pulmonary artery segment is enlarged, and pulmonary vascular markings are increased. An electrocardiogram shows right axis deviation. Which of the following congenital heart diseases does this patient most likely have? A. Aortic stenosis B. Atrial septal defect C. Coarctation of the aorta D. Patent ductus arteriosus E. Ventricular septal defect

The correct answer is B. One of the most common types of structural congenital heart disease to present in adolescence is atrial septal defect (ASD), and the most common presentation is a heart murmur. Some patients, however, present with arrhythmias, decreased exercise tolerance, or a paradoxic embolus.

Which of the following EKG abnormalities is treated as an equivalent to an ST-segment elevation acute myocardial infarction in a patient who has acute onset of chest pain? A. Right bundle branch block B. Left bundle branch block C. Left anterior hemiblock D. Chaotic atrial tachycardia E. PR-segment depression

The correct answer is B. Patients who have acute signs and symptoms consistent with myocardial infarction pattern who display a new-onset left bundle branch block pattern are treated equivalently to patients who have ST-segment elevation myocardial infarction until cardiac enzyme patterns are completed. Under appropriate circumstances, these patients are eligible for percutaneous cardiac intervention and thrombolytic therapy (if they arrive within an appropriate timeframe and there are no contraindications). Patients who have acute myocardial infarction often have such damage to the left ventricle that the repolarization pattern is affected, which results in a left bundle branch block pattern on the EKG. Other causes of left bundle branch block include heart failure, aortic stenosis, and left ventricular aneurysm. Patients who have new-onset left bundle branch block pattern may have this as a manifestation of acute myocardial infarction and should be treated as such in the setting of a patient who presents with cardiac signs and symptoms. Because some treatment modalities are based on history and physical examination findings and EKG findings, treatment decisions may be based on those two findings, as it may take several hours for cardiac enzymes to become positive.

A 47-year-old woman with a history of rheumatic fever is examined by her health care provider. Physical examination is significant for a low-pitched, rumbling, diastolic murmur preceded by an opening snap. This murmur radiates into the apex of the heart. The affected valve can be best evaluated by auscultation at which of the following locations?

The correct answer is B. Solid knowledge of cardiac anatomy and its clinical correlations are crucial for performing physical examinations. This question tests two facts. First it requires that you recognize the patient has mitral stenosis. Classic clues to this diagnosis are "low-pitched, rumbling, diastolic murmur," and "opening snap." The mitral valve is the most commonly affected valve in rheumatic fever, followed by the aortic and tricuspid valves. The mitral valve is most audible over the left fifth intercostal space at the midclavicular line.

A 47-year-old woman with a history of rheumatic fever is examined by her health care provider. Physical examination is significant for a low-pitched, rumbling, diastolic murmur preceded by an opening snap. This murmur radiates into the apex of the heart. The affected valve can be best evaluated by auscultation at which of the following locations? A. Left second intercostal space B. Left fifth intercostal space C. Left lower sternal body border D. Right second intercostal space E. Right fifth intercostal space

The correct answer is B. Solid knowledge of cardiac anatomy and its clinical correlations are crucial for performing physical examinations. This question tests two facts. First it requires that you recognize the patient has mitral stenosis. Classic clues to this diagnosis are "low-pitched, rumbling, diastolic murmur," and "opening snap." The mitral valve is the most commonly affected valve in rheumatic fever, followed by the aortic and tricuspid valves. The mitral valve is most audible over the left fifth intercostal space at the midclavicular line. Mitral valve stenosis is typically caused by rheumatic fever. The mitral valve progressively thickens, which interferes with its mobility to fully open during diastole. Diastolic murmurs are classically not innocent murmurs. Mitral stenosis is the only heart valvular abnormality that is associated with an opening snap. This murmur is classically described as lower pitched and rumbling. It radiates into the apex of the heart and is best heard in the left fifth intercostal space.

On a routine physical examination, a midsystolic ejection murmur is detected in the pulmonic area of a 35-year-old woman. The cardiac examination also reveals a prominent right ventricular cardiac impulse and wide and fixed splitting of the second heart sound. EKG shows right axis deviation and chest radiograph shows enlargement of the right ventricle and atrium. Which of the following is the most likely diagnosis? A. Aortic stenosis B. Atrial septal defect C. Mitral regurgitation D. Mitral stenosis E. Pulmonary valve stenosis

The correct answer is B. The classic findings in atrial septal defect are a prominent right ventricular cardiac impulse, a systolic ejection murmur heard in the pulmonic area and along the left sternal border, and fixed splitting of the second heart sound. These findings are caused by an abnormal left-to-right shunt through the defect, creating a volume overload on the right side. The increase in volume on the right side creates the flow murmur, the dilatation of the right-sided chambers, and the delayed closure of the pulmonic valve, all of which are present in this case. The delayed closure of the pulmonic valve occurs because of the increased volume of blood that is delivered to the right ventricle. Atrial septal defect is an abnormal opening between the left and right atrium. Following birth, the left heart pressures are higher than the right heart pressure, which results in left-to-right shunting of blood from the left atrium into the right atrium. This results in right atrial enlargement and possibly right ventricular enlargement from the increased volume of blood on the right side of the heart. There is fixed splitting of the second heart sound; without the atrial septal defect, the second heart sound becomes single with expiration because less blood is flowing to the right side of the heart, which results in the pulmonic valve closing sooner, as there is less blood for the right ventricle to expel.

A 55-year-old man with no known cardiac history comes to the emergency department complaining of crushing substernal chest pressure that began 20 minutes prior to his arrival. He took sildenafil earlier in the evening prior to sexual intercourse. His past medical history is significant for bilateral inguinal hernia repairs 6 weeks ago, remote peptic ulcer disease, and prior cocaine and heroin addiction. On examination, he is diaphoretic and appears anxious. His blood pressure is 150/75 mm Hg with a pulse of 100/min. An electrocardiogram (ECG) obtained while he had severe chest pain revealed tall positive T waves and 1-mm elevation of the ST segments in leads V2 through V5. A serum creatine kinase (CK) drawn on presentation returned at 85 U/L (normal <250 U/L). Which of the following is the most likely diagnosis? A. Aortic dissection B. Myocardial infarction (MI) C. Pulmonary embolus (PE) D. Spontaneous pneumothorax E. Stable angina

The correct answer is B. The combination of substernal chest pressure with ECG findings of hyperacute T waves and ST elevation in this clinical setting suggests early anterior myocardial infarction. The myocardial damage causes time-dependent effects on the electrical properties of the myocardial cells. The earliest electrical evidence of myocardial injury is often development of hyperacute T waves, followed by elevation of the ST segments, inversion of the T wave, return of the ST segments to normal, and finally, the development of Q waves. It does not matter that the CK, a marker of myocardial injury, is not elevated, as it takes several hours for the CK to appreciably rise after myocardial damage has occurred. Myocardial infarction is diagnosed by having positive findings in at least two out of the following three criteria: typical symptoms of chest pain, EKG changes (>1 mm ST-segment elevation or depression in two or more contiguous leads, T-wave inversions), and elevations in cardiac markers. There is no need to wait for the cardiac markers to be elevated to diagnose myocardial infarction. The markers take at least 4 hours to become elevated. Treatment can be based on the classic EKG findings and patient history alone when deciding whether an ST-segment elevation MI is occurring. There are definite time limitations when considering thrombolytic therapy, although that window continues to increase in the setting of continued chest pain on the part of the patient.

A 41-year-old man comes to his health care provider for a routine physical examination. He is new to this office and brings his previous medical record with him. He has no significant past medical history, but he does have a strong family history of cancer and heart disease. His father and his brother both had myocardial infarctions before age 55 years, and his sister, mother, and aunt had breast cancer. He exercises regularly and eats well, with most of his diet being low in saturated fat and cholesterol. He smokes one pack of cigarettes per week. His review of systems is unremarkable. He is very anxious and would like only minimal interventions done because of his good health. Which of the following is the most age-appropriate screening test in this patient? A. Fasting lipid profile B. Non-fasting total cholesterol level C. Oral glucose tolerance test D. Prostate examination E. Sigmoidoscopy

The correct answer is B. The current recommendations for routine, age-appropriate screening are based in some measure on data from clinical trials. Depending on the source of the recommendations, there is considerable variability in these recommendations. One current recommendation is that at least every 5 years a random cholesterol level should be checked. Typically, total cholesterol and HDL cholesterol can be drawn as a baseline in a nonfasting state. If abnormalities are found in these measurements, a fasting blood lipid (ideally a 12-hour fast) can be performed. The most common cause of death in the United States is coronary artery disease. There are several known risk factors for coronary artery disease that have been identified: smoking, diabetes mellitus, obesity, and hyperlipidemia. The recommended initial screening test is a total cholesterol and HDL level, which can be performed in a nonfasting state. If an elevated total cholesterol level is identified, a complete fasting lipid profile is performed and treatment decisions are primarily based on the LDL level and the patient's other cardiac risks. There may also be treatment recommended for elevated triglycerides, depending on the patient's cardiac risks.

A 59-year-old man who is scheduled for an abdominal aortic aneurysm (AAA) repair in 3 weeks presents to the health care provider's office. The patient's AAA was diagnosed last week via ultrasound. Imaging at that time revealed a 5.5-cm aneurysm of his abdominal aorta extending bilaterally into his iliac arteries. The patient also has moderate hypertension, with a mean daily blood pressure of 150/95 mm Hg. On physical examination, the patient appears in no distress. He weighs 274 pounds and is 5 feet 9 inches tall (BMI is 40.5). His lungs are clear, and he has a loud S4. His AAA is palpable as a pulsatile mass in his abdomen. He complains of very mild back pain. Which of the following is the most appropriate intervention to prepare this patient for surgery? A. Arrange long-term physical therapy B. Improve blood pressure control C. Initiate a weight loss program D. Prescribe a nonsteroidal anti-inflammatory drug (NSAID) E. Prescribe a regimen of regular aerobic exercise

The correct answer is B. The most important interventions involve lessening the chance for rupture of an abdominal aortic aneurysm (AAA). Other interventions include counseling for smoking cessation and improving blood pressure control.

A 45-year-old woman has a history of symptomatic ventricular couplets, for which she had been placed on amiodarone (Cordarone). A week after this intervention, she experiences a syncopal episode and is brought to the emergency department. The patient is awake and alert with a blood pressure of 110/70 mm Hg and pulse 90/min. Electrolytes are within normal limits. She is placed on a cardiac monitor and is noted to have a transient wide complex tachycardia with waves of alternating morphology. She appears to be in torsades de pointes. Which of the following is the most appropriate next step? A. Check the level of serum calcium B. Discontinue amiodarone (Cordarone) and observe C. Increase the amiodarone (Cordarone) dose D. Administer IV potassium E. Apply 360 J of defibrillating current

The correct answer is B. The patient is diagnosed with polymorphic ventricular tachycardia called torsades de pointes. This can be a fatal rhythm and is associated with a prolongation of the QT interval, which could have occurred as a result of the amiodarone administration. The appropriate step is thus to discontinue the amiodarone and observe. Amiodarone (Cordarone) is a commonly used antiarrhythmic agent. It has efficacy in treating both ventricular and atrial arrhythmias. It is a class III antiarrhythmic under the Vaughan Williams Classification system, and has prolongation of the QT interval as a complication. Prolonged QT intervals put a patient at risk for torsades des pointes or R on T phenomena leading to ventricular tachycardia. Torsades is a polymorphic ventricular tachycardia with a twisting morphology that can result in a fairly normal cardiac output or a hemodynamically compromised situation. IV magnesium sulfate may play a role in the management of torsades provided that the patient is magnesium deficient. Amiodarone should be withdrawn as part of the treatment regimen and any underlying electrolyte abnormalities should be corrected.

A health care provider examines a 2-month-old infant who had been born at term. There is a continuous murmur at the upper left sternal border. The murmur radiates over the lung fields anteriorly. The peripheral pulses in all extremities are full and show widened pulse pressure. Which of the following is the most likely diagnosis? A. Coarctation of the aorta B. Patent ductus arteriosus C. Peripheral pulmonic stenosis D. Persistent truncus arteriosus E. Ventricular septal defect

The correct answer is B. This is patent ductus arteriosus, which is a failure of closure of the duct between the pulmonary artery and the aorta. As many as 80% of significantly premature (<28-week gestation) infants have patent ductus arteriosus. In term infants, delayed closure is diagnosed if the murmur of the patent ductus (described in the question stem) is still present at 6-8 weeks of age. Indomethacin is given soon after birth in premature infants and is typically started within the first 12 hours of life. This treatment is given only during the first 3 days of life in an attempt to close the patent ductus. Infants should be evaluated for other cardiac disease, as a patent ductus arteriosus may be partially compensating for other cardiac anomalies. Infants who have heart failure require prompt surgical correction. Infants who do not have heart failure or complicating cardiac defects typically undergo elective surgery at 6 months to 3 years of age to reduce the risk for infective endocarditis later involving the patent ductus. If this condition is identified in the adult population, treatment consists of a percutaneous catheter to close or surgical ligation of this structure.

A 25-year-old man comes to the health care provider with chest pain at rest that is not always related to exercise. He reports two prior episodes of fainting during exercise but has otherwise been healthy. He reports a similar history in other family members and notes that his father suddenly collapsed and died at age 50 after playing tennis. He denies use of cocaine or other recreational drugs and does not take any medications. Physical examination reveals a systolic ejection murmur that is loudest along the left sternal border. The rest of the physical examination is unremarkable. Echocardiography shows asymmetric septal hypertrophy without obstruction. Which of the following interventions would be most likely to decrease this patient's systolic murmur? A. Inhaling amyl nitrate B. Lying down C. Standing up D. Use of digoxin (Lanoxin) E. Valsalva maneuver

The correct answer is B. This patient has hypertrophic obstructive cardiomyopathy. This is a cardiac condition in which the interventricular septum hypertrophies excessively and asymmetrically. The hypertrophied septum and the anterior leaflet of the mitral valve produce left ventricular outflow obstruction. Most cases are inherited through an autosomal-dominant mode of transmission. The systolic ejection murmur is diminished when the patient lies down, because this maneuver increases cardiac size by increasing venous return and tends to diminish the intensity of the murmur. Afterload is increased, and venous return is increased to the heart. This increases the ventricular size and diminishes the murmur.

A 38-year-old English professor complains of chest pain with exertion. The symptoms began approximately 4 months ago and have been accompanied by dyspnea. On several occasions, she became lightheaded and felt faint while exercising. She has no prior medical history. She does not smoke or drink. On physical examination, her blood pressure is 154/92 mm Hg, pulse is 82/min, and respirations are 16/min. She is afebrile. A head and neck examination is normal. There is no jugular venous distention. Her lungs are clear. Her carotid upstrokes are delayed and diminished in amplitude. On cardiac examination, there is a forceful apical impulse and a soft S2. There is a harsh, late-peaking, crescendo-decrescendo systolic murmur that is heard best at the right second intercostal space and radiates to the carotids. The remainder of the physical examination is unremarkable. An electrocardiogram reveals a normal sinus rhythm and left ventricular hypertrophy by voltage criteria. Which of the following is the most likely diagnosis? A. Aortic insufficiency B. Aortic stenosis C. Ischemia of the left anterior descending artery D. Ischemia of the left main coronary artery E. Mitral regurgitation

The correct answer is B. This patient's symptoms are suggestive of coronary artery disease. She is premenopausal, however, making this diagnosis less likely in the absence of a very strong family history of early coronary artery disease or of a familial hyperlipidemia. Also, her ECG does not reveal changes consistent with coronary artery ischemia. She has the characteristic findings of aortic stenosis with systolic murmur, with a crescendo-decrescendo pattern that peaks in midsystole and radiates into the carotids. In this patient's age group, aortic stenosis is most likely to result from a congenital bicuspid aortic valve or, less likely, as a sequelae of rheumatic valvular disease. The anginal-type symptoms develop in the presence of normal coronary arteries because there is increased demand by the hypertrophied left ventricle, which exceeds the coronary arteries' ability to meet demands. Aortic stenosis results in obstruction of blood flow across the aortic valve secondary to aortic calcification. Presentation includes dyspnea with exertion, chest pain, or syncope. Murmur is classically systolic ejection, crescendo-decrescendo with radiation into the carotids. Doppler studies will show a pressure gradient across the stenotic aortic valve. In an asymptomatic patient, replacement of the aortic valve should be performed in patients who are having another heart surgery performed. Echocardiogram is performed annually in asymptomatic patients to identify patients for surgery before severe left ventricular dysfunction develops.

Valvuloplasty is indicated for mitral valve stenosis in which of the following scenarios? A. With a valve area of 4 cm2 B. Only after the patient has symptoms C. With a NYHA classification of I D. With a left atrial pressure of 10 mm Hg E. Patient with history of rheumatic fever

The correct answer is B. Valvotomy is indicated in the symptomatic patient who has a mitral orifice less than 1.0 cm2/m2 BSA (body surface area).

Valvuloplasty is indicated for mitral valve stenosis in which of the following scenarios? A. With a valve area of 4 cm2 B. Only after the patient has symptoms C. With a NYHA classification of I D. With a left atrial pressure of 10 mm Hg E. Patient with history of rheumatic fever

The correct answer is B. Valvotomy is indicated in the symptomatic patient who has a mitral orifice less than 1.0 cm2/m2 BSA (body surface area). A normal valve orifice is 4 to 6 cm2 (choice A). Significant obstruction begins when the orifice is less than 2 cm2. Patients who have a NYHA classification of I (choice C) are uncompromised. Ordinary activity does not cause symptoms. NYHA class I patients have risk factors for coronary artery disease but have yet to develop symptoms. Normal left atrial pressure is 2-10 mm Hg (choice D). Severe mitral stenosis with a valve opening of 1 cm2 would require a left atrial pressure of 25 mm Hg to maintain normal cardiac output. Patients with history of rheumatic fever may have damage to the mitral valve, but this history is not a reason to perform valvuloplasty (choice E). Reviewed on 4/17/14 Mitral valve surgery with balloon valvuloplasty or replacement occurs in the setting of symptoms that no longer respond to diuretic therapy. if the pulmonary artery pressure and/or pulmonary artery pressure increase >25 mm Hg with exercise or resting pulmonary pressures of >60 mm Hg. Measurements consistent with need for replacement include a gradient 5-10 mm Hg or valve area of 1-1.5 cm. These measurements are indicative of significant obstruction of the mitral valve.

A 2-month-old baby boy is brought to the clinic by his mother for a routine checkup. On physical examination, the health care provider hears a continuous machinery murmur near the left sternal border at the second interspace. What is the most likely diagnosis? A. Coarctation of the aorta B. Ventricular septal defect C. Patent ductus arteriosus D. Atrial septal defect E. Pulmonary stenosis

The correct answer is C. A continuous machinery murmur is characteristic of patent ductus arteriosus (PDA). PDA is also characterized by bounding pulses with wide pulse pressure. The ductus ateriosus is a structure that is necessary during gestation but becomes unnecessary after birth when the newborn takes a deep breath and the lungs begin to function. It normally closes within a few hours to days after birth. It most commonly fails to close in premature infants or in infants who are born to mothers living in high altitudes. If the ductus allows a large volume of blood to flow through, there is a wide pulse pressure that is identified along with bounding pulses. A continuous, machinery-sounding murmur is classically associated with this disorder. The murmur is continuous because the aortic pressure is higher than the pulmonary pressure during both systole and diastole. EKG may be normal or may show signs of ventricular hypertrophy. Indomethacin can be used to close the ductus and surgical ligation can be used if indomethacin fails. Diuretics may be used to treat extra volume states until more definitive closure can be performed.

A 25-year-old woman is discovered on physical examination to have a midsystolic click and a high-pitched heart murmur. Which of the following additional cardiovascular findings is she most likely to exhibit? A. Atrial fibrillation B. Decreased peripheral pulse pressure C. Premature ventricular contractions D. Slowed carotid upstroke E. Wide pulse pressure

The correct answer is C. A midsystolic click and a high-pitched heart murmur in a young woman is the classic presentation of mitral valve prolapse. Patients are usually asymptomatic but may have dyspnea, tachycardia, chest pain, syncope, eventual congestive heart failure, or, rarely, sudden death. Prolapse may coincide with tricuspid or pulmonary valve disease or with psychiatric conditions such as anxiety or depression. Complications may include atrial thrombosis, calcification, infective endocarditis, emboli to the brain, rupture of chordae, mitral regurgitation, arrhythmias, and premature ventricular contractions (PVCs). Mitral valve prolapse is the most common valvular heart condition in the United States. Most patients are asymptomatic and rarely have cardiac-related symptoms. A conclusion from the Framingham Heart Study found that the only association found with mitral valve prolapse was low body weight. If a patient does have symptoms, palpitations are the most common manifestation.

A 55-year-old man with hypertension and a past medical history of myocardial infarction is prescribed atenolol (Tenormin). This medication will lower his blood pressure by A. centrally acting agents B. blocking the conversion of angiotensin I to angiotensin II C. decreasing cardiac output D. decreasing intravascular volume E. increasing renin release from the kidney

The correct answer is C. Atenolol is a beta-adrenergic receptor blocking agent used in the treatment of hypertension. Medications in this drug class lower blood pressure by reducing both cardiac output and decreasing renin release from the kidney (to a lesser extent). Beta-blockers work as antihypertensive agents by blocking the sympathetic effect of the beta-adrenergic system. Blocking the beta-adrenergic system causes a lowering of the heart rate. The force of contraction of the ventricles is impeded and this results in a lower cardiac output. Because the heart is not contracting as strongly, there is less myocardial oxygen demand in the heart, which is one of the reasons that these agents are cardioprotective in patients who have heart disease. Because the heart is not contracting as strongly, cardiac output is lessened. Beta-blockers also cause an increase in smooth muscle contraction, which is why some patients who have reactive airways disease do not tolerate these agents because of an increased tendency for bronchospasm. Beta-blockers should not be used by patients who have peripheral vascular disease, as the impaired cardiac output will further decrease the delivery of blood into the peripheral circulation.

A 55-year-old man with hypertension and a past medical history of myocardial infarction is prescribed atenolol (Tenormin). This medication will lower his blood pressure by A. centrally acting agents B. blocking the conversion of angiotensin I to angiotensin II C. decreasing cardiac output D. decreasing intravascular volume E. increasing renin release from the kidney

The correct answer is C. Atenolol is a beta-adrenergic receptor blocking agent used in the treatment of hypertension. Medications in this drug class lower blood pressure by reducing both cardiac output and decreasing renin release from the kidney (to a lesser extent). Centrally acting agents (choice A), also known as centrally adrenergic inhibitors, block the brain's ability to release adrenergic agents that would increase the heart rate and blood pressure. Agents in this category include methyldopa (Aldomet) and clonidine (Catapres). Angiotensin-converting enzyme (ACE) inhibitors block the conversion of angiotensin I to angiotensin II (choice B). Diuretics decrease intravascular volume (choice D), which ultimately leads to a reduction in blood pressure. Increasing renin release from the kidney (choice E) would increase, not decrease, blood pressure. Reviewed on 4/17/14 Beta-blockers work as antihypertensive agents by blocking the sympathetic effect of the beta-adrenergic system. Blocking the beta-adrenergic system causes a lowering of the heart rate. The force of contraction of the ventricles is impeded and this results in a lower cardiac output. Because the heart is not contracting as strongly, there is less myocardial oxygen demand in the heart, which is one of the reasons that these agents are cardioprotective in patients who have heart disease. Because the heart is not contracting as strongly, cardiac output is lessened. Beta-blockers also cause an increase in smooth muscle contraction, which is why some patients who have reactive airways disease do not tolerate these agents because of an increased tendency for bronchospasm. Beta-blockers should not be used by patients who have peripheral vascular disease, as the impaired cardiac output will further decrease the delivery of blood into the peripheral circulation.

A 23-year-old man sustains multiple stab wounds to his left chest. At the time of emergency department admission, his blood pressure is 70/45 mm Hg, pulse 110/min, and central venous pressure 4 mm Hg. He is diaphoretic and anxious, and he is notably pale. It is quickly determined that he has a left hemopneumothorax and so a chest tube is inserted. A total of 380 mL of blood is recovered initially, and another 120 mL is suctioned during the next hour. He is given 2 liters of Ringer's lactate, followed by 2 units of blood. Reassessment at the end of the first hour shows that his lung is expanded and his central venous pressure has gone up to 22 mm Hg, but his blood pressure is only 85/70 mm Hg, his pulse has gone up to 115/min, and he remains diaphoretic. Which of the following is the most likely diagnosis at this time? A. Continued bleeding B. Intrinsic cardiogenic shock C. Pericardial tamponade D. Tension pneumothorax E. Vasomotor shock

The correct answer is C. Cardiac/pericardial tamponade is caused by the accumulation of blood within the pericardial space, creating an increase in the intrapericardial pressure. This in turn results in restricted cardiac filling with a resultant decrease in cardiac output. Shock with high central venous pressure in a chest trauma victim suggests either pericardial tamponade or tension pneumothorax. The latter has been ruled out by the re-expanded lung. The problem was not initially obvious because of concomitant bleeding but became clear once blood volume was restored. Patients who have this condition will have hypotension, jugular venous distention, and distant heart sounds (known as Beck triad). Characteristics of pericardial tamponade are low blood pressure, distended neck veins (high CVP), and muffled heart sounds (Beck triad). Cardiac tamponade can be hidden by massive blood loss, because the CVP will fail to rise and venous distention will be absent. Pericardiocentesis and creation of pericardial window is the surgical treatment of a patient who has pericardial tamponade. Echocardiogram is essential for evaluation of a patient suspected of having this condition. Pericardiocentesis may be complicated by puncture or laceration of cardiac blood vessels.

A 23-year-old man sustains multiple stab wounds to his left chest. At the time of emergency department admission, his blood pressure is 70/45 mm Hg, pulse 110/min, and central venous pressure 4 mm Hg. He is diaphoretic and anxious, and he is notably pale. It is quickly determined that he has a left hemopneumothorax and so a chest tube is inserted. A total of 380 mL of blood is recovered initially, and another 120 mL is suctioned during the next hour. He is given 2 liters of Ringer's lactate, followed by 2 units of blood. Reassessment at the end of the first hour shows that his lung is expanded and his central venous pressure has gone up to 22 mm Hg, but his blood pressure is only 85/70 mm Hg, his pulse has gone up to 115/min, and he remains diaphoretic. Which of the following is the most likely diagnosis at this time? A. Continued bleeding B. Intrinsic cardiogenic shock C. Pericardial tamponade D. Tension pneumothorax E. Vasomotor shock

The correct answer is C. Cardiac/pericardial tamponade is caused by the accumulation of blood within the pericardial space, creating an increase in the intrapericardial pressure. This in turn results in restricted cardiac filling with a resultant decrease in cardiac output. Shock with high central venous pressure in a chest trauma victim suggests either pericardial tamponade or tension pneumothorax. The latter has been ruled out by the re-expanded lung. The problem was not initially obvious because of concomitant bleeding but became clear once blood volume was restored. Patients who have this condition will have hypotension, jugular venous distention, and distant heart sounds (known as Beck triad). Continued bleeding (choice A) could be occurring but is not responsible for the hypotension. If that were the case, the central venous pressure should be low. The volume of blood recovered via the chest tube is also not alarming. Intrinsic cardiogenic shock (choice B) would have identical findings but under a different clinical case scenario. Cardiogenic shock scenario typically would occur in a setting post-myocardial infarction (especially anterior wall) after major loss of the pumping action of the heart. Tension pneumothorax (choice D) has been ruled out by the re-expanded lung. Patients classically complain of painful dyspnea, cyanosis, diaphoresis, and tachycardia. Tension pneumothorax can occur in the setting of penetrating or blunt injury to the chest. Chest film classically shows that the trachea is shifted to the contralateral side of the chest. Shifting of the trachea causes compression of the vessels with impaired filling of the heart. Vasomotor shock (choice E) has low central venous pressure and a pink patient who is not sweating. Vasomotor shock occurs as a result of impaired ability of the body to continue to meet metabolic demand in the setting of low blood pressure. Reviewed on 4/17/14 Characteristics of pericardial tamponade are low blood pressure, distended neck veins (high CVP), and muffled heart sounds (Beck triad). Cardiac tamponade can be hidden by massive blood loss, because the CVP will fail to rise and venous distention will be absent. Pericardiocentesis and creation of pericardial window is the surgical treatment of a patient who has pericardial tamponade. Echocardiogram is essential for evaluation of a patient suspected of having this condition. Pericardiocentesis may be complicated by puncture or laceration of cardiac blood vessels.

A 52-year-old man comes to the hospital complaining of palpitations. The patient reports that while cooking breakfast this morning, he felt his heart "racing in his chest" and was unable to catch his breath. He states that sitting down brought no relief. He called for an ambulance and he was brought to the emergency department. The man has no significant past medical history and takes no medications regularly other than ranitidine (Zantac) for occasional heartburn. On examination, the patient is quite thin but well developed and in mild distress. His pulse is 140/min and irregularly irregular. There are no murmurs, and the lung examination is clear. Which of the following findings on his echocardiogram would suggest a diagnosis of longstanding atrial fibrillation? A. Dilated left ventricle B. Dilated right ventricle C. Enlarged left atrium D. Hypertrophied ventricular septum E. Pericardial thickening

The correct answer is C. Echocardiography is an invaluable tool for assessing cardiovascular function in both normal and disease states. In the case of atrial fibrillation (AF), a chronic course versus an acute and self-limited course portends completely different treatment strategies and long-term prognoses. The most common cause of chronic AF is valvular disease, followed by congestive heart failure (CHF). Hyperthyroidism that is not adequately treated is another cause of atrial fibrillation and should be suspected in a patient who has new-onset AF. The most common anatomic correlate seen in patients who have AF (Framingham Heart Study) is an enlarged left atrium. Patients who have an enlarged left atrium are less likely to remain in normal sinus rhythm following cardioversion. Atrial fibrillation is the most common sustained cardiac dysrhythmia. For patients who have hemodynamic instability, the patient should be immediately cardioverted. When cardioversion (either medical or electronic) is performed on an elective basis, the patient should have a screening echocardiogram performed (transesophageal approach is preferred), and anticoagulation should be done for several weeks before and following the elective cardioversion if a left atrial thrombus is found on echo. The most common abnormality on the echocardiogram is enlargement of the left atrium. Other echocardiogram findings may include left ventricular enlargement, global wall motion abnormalities, valvular abnormalities, or pericardial disease.

A 52-year-old man comes to the hospital complaining of palpitations. The patient reports that while cooking breakfast this morning, he felt his heart "racing in his chest" and was unable to catch his breath. He states that sitting down brought no relief. He called for an ambulance and he was brought to the emergency department. The man has no significant past medical history and takes no medications regularly other than ranitidine (Zantac) for occasional heartburn. On examination, the patient is quite thin but well developed and in mild distress. His pulse is 140/min and irregularly irregular. There are no murmurs, and the lung examination is clear. Which of the following findings on his echocardiogram would suggest a diagnosis of longstanding atrial fibrillation? A. Dilated left ventricle B. Dilated right ventricle C. Enlarged left atrium D. Hypertrophied ventricular septum E. Pericardial thickening

The correct answer is C. Echocardiography is an invaluable tool for assessing cardiovascular function in both normal and disease states. In the case of atrial fibrillation (AF), a chronic course versus an acute and self-limited course portends completely different treatment strategies and long-term prognoses. The most common cause of chronic AF is valvular disease, followed by congestive heart failure (CHF). Hyperthyroidism that is not adequately treated is another cause of atrial fibrillation and should be suspected in a patient who has new-onset AF. The most common anatomic correlate seen in patients who have AF (Framingham Heart Study) is an enlarged left atrium. Patients who have an enlarged left atrium are less likely to remain in normal sinus rhythm following cardioversion. A dilated left ventricle (choice A) is commonly seen with both CHF and AF. An enlarged left atrium, however, is more closely correlated with chronic AF than is left ventricular dilation. The causal relationship between an enlarged left atrium and AF is unclear. A dilated right ventricle (choice B) is commonly seen in severe right heart failure (RHF). The most common cause of RHF is left heart failure (LHF). There is no consistent relationship between right heart size and the presence of AF. A hypertrophied ventricular septum (choice D) is commonly seen in hypertensive heart disease or in hypertrophic cardiomyopathy. Chronic AF in these patients, however, is much less common. Pericardial thickening (choice E), as seen with chronic pericardial inflammation, is not routinely associated with chronic AF. Acute AF may be associated with acute pericarditis. Reviewed on 4/17/14 Atrial fibrillation is the most common sustained cardiac dysrhythmia. For patients who have hemodynamic instability, the patient should be immediately cardioverted. When cardioversion (either medical or electronic) is performed on an elective basis, the patient should have a screening echocardiogram performed (transesophageal approach is preferred), and anticoagulation should be done for several weeks before and following the elective cardioversion if a left atrial thrombus is found on echo. The most common abnormality on the echocardiogram is enlargement of the left atrium. Other echocardiogram findings may include left ventricular enlargement, global wall motion abnormalities, valvular abnormalities, or pericardial disease.

A 59-year-old man with a history of myocardial infarction presents to his health care provider complaining of shortness of breath. On examination, his pulse is 110/min and his respiratory rate is 22/min. He has rales in both lung fields, a normal sinus rhythm with an S3 gallop, and 2+ pitting ankle edema. A chest radiograph reveals cardiomegaly, and his ejection fraction on echocardiogram is calculated at 37%. Which of the following medications would alleviate this patient's symptoms by significantly reducing both the preload and afterload on the heart without affecting its inotropic state? A. Digoxin (Lanoxin) B. Diltiazem (Cardizem, Cartia, Dilacor) C. Enalapril (Vasotec) D. Furosemide (Lasix) E. Propranolol (Inderal)

The correct answer is C. Everything in this case points to congestive heart failure (CHF): dyspnea, elevated heart rate, S3, peripheral edema, and reduced ejection fraction. A drug that will alleviate the symptoms by decreasing both preload and afterload is necessary. Enalapril is an angiotensin-converting enzyme (ACE) inhibitor. ACE inhibitors (captopril, enalapril, lisinopril, benazepril, ramipril) work by blocking the conversion of angiotensin I to angiotensin II in the lungs. Angiotensin II is both a potent vasoconstrictor and a stimulator of aldosterone production. Aldosterone acts by promoting sodium (and thus water) reabsorption by the kidney. An ACE inhibitor will therefore promote vasodilatation (reducing afterload), as well as reduce intravascular volume (decreasing aldosterone, as less renin is being released).

A 59-year-old man with a history of myocardial infarction presents to his health care provider complaining of shortness of breath. On examination, his pulse is 110/min and his respiratory rate is 22/min. He has rales in both lung fields, a normal sinus rhythm with an S3 gallop, and 2+ pitting ankle edema. A chest radiograph reveals cardiomegaly, and his ejection fraction on echocardiogram is calculated at 37%. Which of the following medications would alleviate this patient's symptoms by significantly reducing both the preload and afterload on the heart without affecting its inotropic state? A. Digoxin (Lanoxin) B. Diltiazem (Cardizem, Cartia, Dilacor) C. Enalapril (Vasotec) D. Furosemide (Lasix) E. Propranolol (Inderal)

The correct answer is C. Everything in this case points to congestive heart failure (CHF): dyspnea, elevated heart rate, S3, peripheral edema, and reduced ejection fraction. A drug that will alleviate the symptoms by decreasing both preload and afterload is necessary. Enalapril is an angiotensin-converting enzyme (ACE) inhibitor. ACE inhibitors (captopril, enalapril, lisinopril, benazepril, ramipril) work by blocking the conversion of angiotensin I to angiotensin II in the lungs. Angiotensin II is both a potent vasoconstrictor and a stimulator of aldosterone production. Aldosterone acts by promoting sodium (and thus water) reabsorption by the kidney. An ACE inhibitor will therefore promote vasodilatation (reducing afterload), as well as reduce intravascular volume (decreasing aldosterone, as less renin is being released). Although digoxin (choice A) is a mainstay in the therapy of CHF, it has no effect on either preload or afterload. Instead it works by having a positive inotropic and negative chronotropic effect on the heart. The mechanism involves inhibition of Na+/K+ ATPase and a consequent increase in intracellular calcium, which in turn increases the heart's contractile force. Diltiazem (choice B) is a calcium channel blocker. As their name implies, calcium channel blockers block the influx of calcium (via specific calcium channels) into the cell from the extracellular fluid; this interferes with excitation-contraction coupling in cardiac muscle, producing a negative inotropic effect. Although these drugs can be used in the treatment of hypertension and angina (they inhibit the contraction of the vascular smooth muscle and thereby decrease peripheral vascular resistance/afterload), they are not first-line agents in the management of patients who have CHF because of their negative inotropic effect in patients who have impaired systolic function. Furosemide (choice D) is a loop diuretic. It decreases intravascular volume (and hence preload), but at typical doses it has no significant effect on either afterload or inotropic state. Diuretics are often used in the treatment of CHF, and furosemide is used for the immediate relief of pulmonary congestion. Propranolol (choice E) is a beta-blocker. It is a negative inotropic agent. Beta-blockers are indicated for all stages of chronic heart failure, as a mortality benefit has been demonstrated from their use. Reviewed on 4/17/14 ACE inhibitors are first-line agents for patients who have heart failure because they act as vasodilators. ACE inhibitors decrease afterload, which allows the heart to have improvement in its ejection fraction. They do not have an inotropic or chronotropic effect on the heart. There is a mortality benefit demonstrated with the use of these agents. Beta-blockers and the aldosterone antagonist, spironolactone, also have been shown to have an improvement in mortality in patients who have chronic heart failure.

A young man sustains a gunshot wound to the base of his neck. He was shot point blank with a .38-caliber revolver. The entrance wound is above the left clavicle, below the level of the cricoid cartilage, and just lateral to the sternomastoid muscle. The exit wound is just above the spinous process of the right scapula. He has normal breath sounds on both sides, is awake and alert, is talking with a normal tone of voice, is neurologically intact, and is hemodynamically stable. Portable radiographs of the neck and chest taken in the emergency department show some air in the tissues of the lower neck but are otherwise non-diagnostic. Which of the following is the most appropriate next step in management? A. Observation for several hours B. CT scan of the lower neck and upper chest C. Angiogram, esophagogram, esophagoscopy, and bronchoscopy prior to surgical exploration D. Immediate surgical exploration of the lower neck through a collar incision E. Immediate surgical exploration of the upper chest through a median sternotomy

The correct answer is C. Gunshot wounds to the base of the neck need exploratory surgery, but the exact approach and incision are determined by a more accurate knowledge of the location and extent of the injuries. If time permits, diagnostic studies thus should precede surgical intervention. The major vessels, the tracheobronchial tree, and the esophagus are the potential targets that have to be investigated. The esophagram needs to be performed with a water-based solution in case the patient has damage to the mediastinum, which would result in leakage from the esophagus and potential mediastinitis. The neck area is an area with vital structures such as vascular, neurologic, GI, and pulmonary serving important physiologic functions. Penetrating trauma in this area can have devastating consequences. The most damaging complications are profuse bleeding and airway occlusion. Patients who appear to be functioning normally can have rapid deterioration of their clinical picture, so a careful evaluation must be performed to plan the best course of action.

A young man sustains a gunshot wound to the base of his neck. He was shot point blank with a .38-caliber revolver. The entrance wound is above the left clavicle, below the level of the cricoid cartilage, and just lateral to the sternomastoid muscle. The exit wound is just above the spinous process of the right scapula. He has normal breath sounds on both sides, is awake and alert, is talking with a normal tone of voice, is neurologically intact, and is hemodynamically stable. Portable radiographs of the neck and chest taken in the emergency department show some air in the tissues of the lower neck but are otherwise non-diagnostic. Which of the following is the most appropriate next step in management? A. Observation for several hours B. CT scan of the lower neck and upper chest C. Angiogram, esophagogram, esophagoscopy, and bronchoscopy prior to surgical exploration D. Immediate surgical exploration of the lower neck through a collar incision E. Immediate surgical exploration of the upper chest through a median sternotomy

The correct answer is C. Gunshot wounds to the base of the neck need exploratory surgery, but the exact approach and incision are determined by a more accurate knowledge of the location and extent of the injuries. If time permits, diagnostic studies thus should precede surgical intervention. The major vessels, the tracheobronchial tree, and the esophagus are the potential targets that have to be investigated. The esophagram needs to be performed with a water-based solution in case the patient has damage to the mediastinum, which would result in leakage from the esophagus and potential mediastinitis. Observation (choice A) might be appropriate for a stab wound in a completely asymptomatic patient. In gunshot wounds, we have to expect that injuries will exist, and they should not be neglected waiting for overt clinical signs. CT scan (choice B) has done wonders for our assessment of closed head injuries and blunt abdominal trauma, but it is not the study that would tell us what has happened to the major vessels, the esophagus, or the tracheobronchial tree in a gunshot wound. Immediate surgical exploration, either through the neck or the chest, or in combination, might be forced by a rapidly deteriorating situation. In the absence of such imperative, a decision to open the neck (choice D) or the chest (choice E) is premature at this point. This patient is currently hemodynamically stable, so he is deserving of a more deliberate surgical plan developed before undergoing surgical exploration. Reviewed on 4/17/14 The neck area is an area with vital structures such as vascular, neurologic, GI, and pulmonary serving important physiologic functions. Penetrating trauma in this area can have devastating consequences. The most damaging complications are profuse bleeding and airway occlusion. Patients who appear to be functioning normally can have rapid deterioration of their clinical picture, so a careful evaluation must be performed to plan the best course of action.

Patients having dilated cardiomyopathy will have which of the following changes on the EKG? A. Persistent ST-segment elevation B. Increased voltage in the chest leads C. Low amplitude of the QRS complexes D. Widening of the P wave in lead II E. Widening of the QRS complexes in leads V5 and V6

The correct answer is C. In dilated cardiomyopathy the EKG often shows sinus tachycardia, or atrial fibrillation, ventricular arrhythmias, left atrial abnormality, diffuse nonspecific ST-wave abnormalities, intravascular conduction defects, and low voltage. The low voltage is caused by the impaired cardiac function of the heart because of the cardiomyopathy. Because the EKG measures the electrical strength of contraction, dilated cardiomyopathy will show impairment of the strength of contraction as the cardiac myocytes become less functional. The cardiomyopathies are a diffuse group of heart muscle diseases that are associated with mechanical and/or electrical dysfunction. Because of the compromised function of the heart, the EKG will show decreased voltage as a result of the poor function of the cardiac myocytes. Cardiomyopathy often results in heart failure and poor exercise tolerance as a result of the impaired pump function that this condition causes.

Patients having dilated cardiomyopathy will have which of the following changes on the EKG? A. Persistent ST-segment elevation B. Increased voltage in the chest leads C. Low amplitude of the QRS complexes D. Widening of the P wave in lead II E. Widening of the QRS complexes in leads V5 and V6

The correct answer is C. In dilated cardiomyopathy the EKG often shows sinus tachycardia, or atrial fibrillation, ventricular arrhythmias, left atrial abnormality, diffuse nonspecific ST-wave abnormalities, intravascular conduction defects, and low voltage. The low voltage is caused by the impaired cardiac function of the heart because of the cardiomyopathy. Because the EKG measures the electrical strength of contraction, dilated cardiomyopathy will show impairment of the strength of contraction as the cardiac myocytes become less functional. Persistent ST-segment elevation (choice A) is classically associated with left ventricular aneurysm following acute myocardial infarction in a patient whose symptoms have resolved. Increased voltage in the chest leads (choice B) is associated with hypertrophy of the left or right ventricles. Because the EKG measures electrical activity, a hypertrophied ventricle will have increased amplitude of the R wave from the increased force of contraction that the hypertrophy provides. Widening of the P waves in lead II (choice D) is associated with left atrial enlargement. This condition is often referred to as "P mitrale," as it is often associated with mitral stenosis and subsequent increased force of pressure that the left atrium has to exert to overcome this resistance. Widening of the QRS complexes in leads V5 and V6 (choice E) is associated with left bundle branch block and left ventricular hypertrophy. This increased force of contraction is in sharp contrast to a patient who has impaired ventricular dysfunction caused by cardiomyopathy. Reviewed on 4/17/14 The cardiomyopathies are a diffuse group of heart muscle diseases that are associated with mechanical and/or electrical dysfunction. Because of the compromised function of the heart, the EKG will show decreased voltage as a result of the poor function of the cardiac myocytes. Cardiomyopathy often results in heart failure and poor exercise tolerance as a result of the impaired pump function that this condition causes.

A 55-year-old man comes to the emergency department complaining of chest pain that radiates to his left arm and shortness of breath for the past hour. Which of the following cardiac enzymes is the most sensitive for acute muscle damage? A. CK-MB B. LDH C. Myoglobin D. AST E. Troponin T

The correct answer is C. Myoglobin identifies any muscle damage but is so sensitive that it is not routinely used in the evaluation of myocardial infarction. It is heme protein that is mainly found in muscle. It rises in 2-3 hours and is rapidly cleared by the kidneys. It may be used as a "rule out" for patients presenting urgently to a health care facility because of its sensitivity; if myoglobin is normal, there is little chance that any significant muscle damage is present. Any muscle damage in the body will cause myoglobin levels to increase.

A 55-year-old man comes to the emergency department complaining of chest pain that radiates to his left arm and shortness of breath for the past hour. Which of the following cardiac enzymes is the most sensitive for acute muscle damage? A. CK-MB B. LDH C. Myoglobin D. AST E. Troponin T

The correct answer is C. Myoglobin identifies any muscle damage but is so sensitive that it is not routinely used in the evaluation of myocardial infarction. It is heme protein that is mainly found in muscle. It rises in 2-3 hours and is rapidly cleared by the kidneys. It may be used as a "rule out" for patients presenting urgently to a health care facility because of its sensitivity; if myoglobin is normal, there is little chance that any significant muscle damage is present. Any muscle damage in the body will cause myoglobin levels to increase. Creatine kinase MB (CK-MB) (choice A) is an isoenzyme in cardiac muscle that rises 4-6 hours after the onset of chest pain in acute myocardial infarction. It peaks within 24 hours and returns to normal within 2-3 days. Although CK-MB is a sensitive cardiac enzyme used to help diagnose acute myocardial infarction, it does not have the same sensitivity as myoglobin. CK-MB does have more specificity for myocardial damage than troponin, but it does not approach the same sensitivity as myoglobin. Of the choices, troponin T (choice E) has the most specificity. Troponin T is detected as early as 1-3 hours after myocardial injury, remains elevated for 5-7 days post-myocardial infarction, and returns to normal within 2 weeks. Patients who have chronic renal disease may also have elevation of troponin T, whereas tropinin I is the most specific of the cardiac enzymes, because it is only produced by cardiac myocytes. LDH or lactate dehydrogenase enzyme (choice B) is found in most tissues; an elevation indicates cellular injury. It is used to confirm the diagnosis of acute myocardial infarction when other pertinent laboratory values are also positive. It peaks within 2-5 days post-myocardial infarction. It is not commonly used in the evaluation of myocardial infarction, because this test has been replaced by other tests that have much more sensitivity and specificity associated with them. Serum aspartate aminotransferase (AST; choice D), also indicates cellular damage, and it is elevated in acute myocardial infarction. High levels are found in hepatocytes and it is usually used to assess for liver disease and not cardiac damage. It peaks within 24 hours and returns to normal within 4 Reviewed on 11/13/14 Patients suspected of having myocardial infarction typically will have only the CK-MB and troponin levels drawn. Myoglobin test could be used but rarely is in the setting of suspected cardiac damage because it is too sensitive at identifying muscle damage. Troponin I is more specific than troponin T because it is not affected by chronic kidney diseaseMost providers will primarily rely on troponin I to help risk-stratify patients who present to an urgent care setting with complaints related to the chest.

Which of the following drugs antagonizes both the vascular and cardiac actions of norepinephrine (NE)? A. Atenolol (Tenormin) B. Esmolol (Brevibloc) C. Labetalol (Trandate) D. Metaproterenol (Alupent) E. Prazosin (Minipress)

The correct answer is C. Norepinephrine (NE) is an agonist at alpha-1, alpha-2, and beta-1 receptors. NE exerts its vascular actions via alpha (predominantly alpha-1) receptors and its cardiac actions via beta-1 receptors. Labetalol is a selective antagonist at alpha receptors and nonselective antagonist at beta receptors and therefore could prevent all actions of NE. Labetolol is antagonistic to both alpha and beta adrenergics. It has a role in treating hypertensive emergencies and may be used to treat acute sympathomimetic overdose with such agents as cocaine or amphetamines. When treating sympathomimetic overdose, remember not to treat this condition with beta-blockers, since unopposed alpha activity makes vasoconstriction worse. As a general rule, IV beta-blockers are effective and considered first-line therapy for most episodes such as aortic dissection, but they should be avoided in high adrenergic states such as sympathomimetic overdose.

A 45-year-old woman comes to the emergency department complaining of recurring episodes of chest pain that each last a few minutes since she awoke this morning. She denies shortness of breath, radiation of the pain, or chest pain on exertion. She reports that she had similar episodes within the last month. The patient states that she has been healthy otherwise, takes no medications, and has no family history of coronary artery disease. Her vital signs and physical examination are within normal limits. Electrocardiogram shows transient ST-segment elevation. Serial cardiac enzymes reveal no abnormalities over a 24-hour period. Cardiac catheterization shows no significant areas of plaque or stenosis. Which of the following classes of drugs may be most effective in this patient? A. ACE inhibitors B. Beta-blockers C. Calcium channel blockers D. Diuretics E. Alpha-adrenoceptor antagonists

The correct answer is C. Prinzmetal angina, or variant angina, is classically characterized by angina without the associated precipitating factors of angina pectoris. It is caused by a transient coronary artery spasm and displays a transient ST elevation on electrocardiogram. It usually has no other diagnostic findings. Calcium channel blockers have been shown to treat and prevent episodes of coronary vasospasm by decreasing contractility of cardiac muscle and producing vasodilation.

A 45-year-old woman comes to the emergency department complaining of recurring episodes of chest pain that each last a few minutes since she awoke this morning. She denies shortness of breath, radiation of the pain, or chest pain on exertion. She reports that she had similar episodes within the last month. The patient states that she has been healthy otherwise, takes no medications, and has no family history of coronary artery disease. Her vital signs and physical examination are within normal limits. Electrocardiogram shows transient ST-segment elevation. Serial cardiac enzymes reveal no abnormalities over a 24-hour period. Cardiac catheterization shows no significant areas of plaque or stenosis. Which of the following classes of drugs may be most effective in this patient? A. ACE inhibitors B. Beta-blockers C. Calcium channel blockers D. Diuretics E. Alpha-adrenoceptor antagonists

The correct answer is C. Prinzmetal angina, or variant angina, is classically characterized by angina without the associated precipitating factors of angina pectoris. It is caused by a transient coronary artery spasm and displays a transient ST elevation on electrocardiogram. It usually has no other diagnostic findings. Calcium channel blockers have been shown to treat and prevent episodes of coronary vasospasm by decreasing contractility of cardiac muscle and producing vasodilation. Angiotensin-converting enzyme (ACE) inhibitors (choice A) block the angiotensin-converting enzyme in the renin-angiotensin system, causing vasodilation. ACE inhibitors are useful in delaying the progression of renal disease in patients who have diabetes and in treating patients who have congestive heart failure, especially if they have impaired left ventricular function. There is no advantage to the use of these agents as ant-antianginals. Beta-blockers (choice B) are appropriate agents in the management of angina pectoris because they decrease both heart rate and blood pressure and thus the heart's oxygen demand. Beta-blockers also reduce mortality in patients who have coronary artery disease and post-myocardial infarction. These agents do not prevent vasospasm, so they are not clinically indicated in the setting of variant or Prinzmetal angina. Diuretics (choice D) are the recommended first-line treatment for mild to moderate essential hypertension. This class of drugs decreases blood pressure by decreasing sodium, blood volume, and afterload. These drugs also provide symptomatic relief for patients who have congestive heart failure, but they are not indicated in the treatment of variant angina. Alpha-adrenoceptor antagonists (choice E) decrease peripheral vascular resistance and are used in the treatment of hypertension and benign prostatic hyperplasia. They are not used in the treatment of angina pectoris. Reviewed on 4/17/14 Prinzmetal angina or variant angina is a cardiac condition that is primarily caused by vasoconstriction or coronary artery disease. These patients may have transient ST-segment elevation during the painful episode. Cardiac enzymes remain normal, as there is no infarction pattern. Although it is recognized that cardiac plaque may be a precipitant for that spasm, plaques were once thought to be a benign problem. Treatment consists of calcium channel blockers, which may be used to prevent or treat cardiac vasospasm. Lifestyle modifications should be used in all patients, including those who have variant angina.

A 71-year-old retired physician complains of increasing calf pain when walking uphill. The symptoms have gradually increased over the past 2 months. His past medical history is significant for a transient ischemic attack 6 months ago. Over the past month, his blood pressure has worsened despite previous control with diltiazem, hydrochlorothiazide, and propranolol. His other medications are isosorbide dinitrate and aspirin. On physical examination, his blood pressure is 152/90 mm Hg and his pulse is 66/min. There is a right carotid bruit. His lower extremities are mildly cool and have diminished pulses at the dorsalis pedis. An ACE inhibitor is added to help control his blood pressure. Which of the following is the most appropriate in the management of his leg pain? A. Decrease diltiazem (Cardizem) dose B. Decrease hydrochlorothiazide dose C. Decrease propranolol (Inderal) dose D. Increase aspirin dose E. Increase isosorbide dinitrate (Isordil) dose

The correct answer is C. Propranolol, a beta-blocker, is considered to be relatively contraindicated in patients who have peripheral claudication. Although beta-blockers are excellent medications in patients who have hypertension, the beta blockade may predispose to arterial vasoconstriction peripherally. In addition, beta-blockers are negative inotropic agents that decrease the force of contraction of the heart. Because the heart is not contracting as strongly, there is less blood being propelled into the peripheral vascular system. The dose of beta-blockers in this patient should therefore be reduced or gradually eliminated. Peripheral vascular disease is most commonly caused by atherosclerosis. Most patients are asymptomatic, but when they become symptomatic claudication is the most common manifestation of the peripheral vascular disease. Risk factor reduction is essential in treating this condition. Exercise to improve claudication symptoms is indicated. Useful medications include cilostazol (Pletal), a phosphodiesterase (platelet aggregation) inhibitor and pentoxifylline (Trental), an agent used to improve the red blood cell flexibility to deform in order to fit smaller diameter vessels. Beta-blockers should not be used in patients who have peripheral vascular disease, as they interfere with the delivery of blood to the peripheral circulation because of their negative inotropic effects.

A woman is brought into the emergency department following an automobile accident in which her chest was hit by the steering wheel. Her blood pressure is 120/90 mm Hg. When she inhales, her systolic blood pressure drops to 100 mm Hg. This finding defines which of the following terms? A. Pulsus alternans B. Pulsus bisferiens C. Pulsus paradoxus D. Pulsus parvus E. Pulsus tardus

The correct answer is C. Pulsus paradoxus is defined as a fall in systolic blood pressure >10 mm Hg on inspiration. It can be associated with cardiac tamponade and chronic obstructive pulmonary disease (COPD). Pulsus paradoxus can follow trauma that results in cardiac tamponade. With cardiac tamponade, there is an accumulation of fluid in the pericardial space that creates an increase in the intrapericardial pressure and a restriction in the ability of the heart to fill. Echocardiogram is the procedure of choice to identify this condition. Pericardiocentesis and creation of a pericardial window may be lifesaving so that this pericardial pressure is relieved. Careful monitoring of patients who have thoracic trauma is needed to identify this condition, as this may result in a life-threatening emergency. Pulsus paradoxus is a fall in systolic blood pressure of at least 10 mm Hg when the patient takes a deep breath. This is caused by the rise in intrathoracic pressure increasing venous return to the heart. Because there is increased blood return to the heart, there is more extracardiac constriction to filling of the heart itself, and the cardiac output will be impaired; this impairment of cardiac output is simply illustrated by the fall in the systolic blood pressure.

A 71-year-old man comes to his health care provider for follow-up of a recent emergency department visit. The patient has a 2-year history of mild congestive heart failure in the setting of longstanding hypertension. He reports that yesterday he sought care at the local emergency department for palpitations and shortness of breath. He was told that his heart was "fibrillating," but later the fibrillation had "stopped on its own." His medications include a thiazide diuretic and an ACE inhibitor. On physical examination he appears well and in no distress. His blood pressure is 130/80 mm Hg and pulse 100/min and regular. His lungs have scant bibasilar rales and no gallops are appreciated. He has a grade 2 holosystolic murmur heard best at the apex. His jugular venous pressure (JVP) is 10 cm at 30 degrees. An electrocardiogram taken in the office reveals atrial fibrillation at a rate of 94/min with normal ST segments. Which of the following is the most appropriate next step in management? A. Discontinue the ACE inhibitor B. Initiate amiodarone therapy C. Initiate beta-blocker therapy D. Initiate digoxin (Lanoxin) therapy E. Initiate furosemide (Lasix) therapy

The correct answer is C. Recall that in the treatment of medical conditions, certain medications overlap syndromes and are efficacious in many areas. This "co-treatment" option maximizes the benefits of each drug in a regimen and often addresses 2 or more issues simultaneously. Ischemia-mediated arrhthymia is the most common cause of death in a person who has congestive heart failure (CHF). Beta-blockers will control the rate in atrial fibrillation, as well as provide a mortality benefit for the CHF. Beta-blockers are both anti-ischemic and antiarrhythmic. In this case, ACE inhibitors have been shown to be very beneficial in prolonging the survival of patients who have CHF, as they are effective as vasodilators. They are also useful antihypertensive agents. Given this, discontinuing his ACE inhibitor (choice A) is clearly incorrect. This patient requires rate control for his atrial fibrillation so that acute heart failure does not recur. Short of restoring this patient's atrial contractions, rate control is the best method to ensure adequate management of atrial fibrillation. Digoxin, with or without a nodal agent such as a beta-blocker, has been shown to be reasonably effective at rate control, but it is less effective in high adrenergic states. Beta-blockers such as metoprolol (Lopressor) and the combined alpha- and beta-blocker carvedilol (Coreg), however, have been clearly shown to lower mortality with CHF, in addition to just controlling the heart rate.

A 5-year-old girl from Connecticut is brought to the family clinic with a 3-day history of fever and intermittent joint pain. She is generally healthy, but according to her mother, she had a "cold" about 1 month ago. On physical examination her temperature is 39.6°C (103.2°F), blood pressure 94/60 mm Hg, pulse 114/min, and respirations 22/min. Knees and elbow joints are swollen, warm, and tender to palpation. There is a grade III/VI diastolic murmur best heard at the apex. Multiple fine, pink macules are noted on her trunk. These macules blanch in the center. Which of the following is the most likely diagnosis? A. Juvenile rheumatoid arthritis B. Lyme disease C. Rheumatic fever D. Scarlet fever E. Septic arthritis

The correct answer is C. Rheumatic fever is an inflammatory disease, possibly autoimmune in nature. Immune responses to group A streptococcal antigens during pharyngitis resulting in antibody cross-reactions with myocardial antigens remain central to the pathogenesis. Rheumatic fever involves many tissues, including the heart, joints, skin, and CNS. Preceding infection with group A Streptococcus is a prerequisite to the development of acute rheumatic fever. Initially, fever, dyspnea, chest pain, and cardiac murmur develop. The Jones criteria for diagnosis are listed below: Major: Carditis Erythema marginatum and subcutaneous nodules Sydenham's chorea Arthritis *Two major criteria OR 1 major and 2 minor establish the diagnosis. The initial episode of rheumatic fever can be prevented by early treatment of streptococcal pharyngitis. Optimal therapy is intramuscular benzathine penicillin G as a single dose. Penicillin V or amoxicillin for 10 days can also be used. With penicillin allergy, either erythromycin succinate for 10 days or azithromycin for 5 days is effective. Rheumatic fever is an inflammatory disease that develops after Group A beta hemolytic Streptococcus infection (strep throat or scarlet fever) and can involve the heart (murmur), joints (swelling and pain), skin (blanching maculopapular rash), and brain (Sydenham chorea). Mortality from rheumatic fever is directly related to heart involvement. Once rheumatic fever is established, the patient is given ongoing penicillin prophylaxis to prevent reinfection with Streptococcus and to cause the regression of rheumatic heart disease.

A 1-week-old baby girl has a coarctation of the aorta just distal to the subclavian arteries. The blood pressure distal to the constriction is 50% lower than normal. Which of the following is increased in this infant? A. Blood flow in the lower body B. Glomerular filtration rate C. Plasma levels of renin D. Renal excretion of sodium E. Renal excretion of water

The correct answer is C. The aorta is constricted at a point beyond the arterial branches to the head and arms but proximal to the kidneys. Collateral vessels in the body wall carry much of the blood flow to the lower body, and the arterial pressure in the lower body is about 50% lower compared with the pressure in the upper body. The lower than normal pressure at the level of the kidneys causes renin to be secreted with the eventual conversion to angiotensin I and angiotensin II. The renin-aldosterone system results in salt and water retention, so that within a few days to weeks the arterial pressure in the lower body (at the level of the kidneys) increases to normal, but in doing so, the blood pressure in the upper body has increased to hypertensive levels. This is the underlying pathophysiology as to why coarctation of the aorta is a secondary cause of hypertension. Correction of the coarctation will then allow the kidney to be normally perfused, and this will result in a "cure" for the patient's hypertension. Coarctation is a condition in which there is constriction of the aorta resulting in increased blood pressure proximal to the site of constriction and decreased blood pressure distal to the site of constriction. The most common site for this constriction to occur in the aorta is at the site of the ductus arteriosus. It may be associated with a bicuspid aortic valve, a VSD, or subvalvular aortic stenosis. It is more commonly seen in males, but females who have Turner syndrome are also at increased risk. The lower extremities typically have decreased blood pressure and pulses. There may also be a differential in blood pressure between the left and right arms, depending on where the coarctation occurs. This condition may be treated with surgical repair or balloon dilation with or without placement of a stent.

A 1-week-old baby girl has a coarctation of the aorta just distal to the subclavian arteries. The blood pressure distal to the constriction is 50% lower than normal. Which of the following is increased in this infant? A. Blood flow in the lower body B. Glomerular filtration rate C. Plasma levels of renin D. Renal excretion of sodium E. Renal excretion of water

The correct answer is C. The aorta is constricted at a point beyond the arterial branches to the head and arms but proximal to the kidneys. Collateral vessels in the body wall carry much of the blood flow to the lower body, and the arterial pressure in the lower body is about 50% lower compared with the pressure in the upper body. The lower than normal pressure at the level of the kidneys causes renin to be secreted with the eventual conversion to angiotensin I and angiotensin II. The renin-aldosterone system results in salt and water retention, so that within a few days to weeks the arterial pressure in the lower body (at the level of the kidneys) increases to normal, but in doing so, the blood pressure in the upper body has increased to hypertensive levels. This is the underlying pathophysiology as to why coarctation of the aorta is a secondary cause of hypertension. Correction of the coarctation will then allow the kidney to be normally perfused, and this will result in a "cure" for the patient's hypertension. Blood flow in the lower body (choice A) is lower than normal at this early stage of aortic coarctation. This is why the blood pressure in the lower extremities is lower than the blood pressure in the upper extremities. Blood flow, however, can be normal above and below the constriction if the body is able to compensate fully. The decrease in blood pressure at the level of the kidneys causes the glomerular filtration rate (choice B) to decrease. Increased plasma levels of angiotensin II cause salt and water retention; thus, salt and water excretion (choices D and E) are decreased when the kidney is under the influence of aldosterone. Aldosterone acts on the distal tubule to save sodium, and this saving of sodium will result in the body then saving water (water follows salt). Reviewed on 4/17/14 Coarctation is a condition in which there is constriction of the aorta resulting in increased blood pressure proximal to the site of constriction and decreased blood pressure distal to the site of constriction. The most common site for this constriction to occur in the aorta is at the site of the ductus arteriosus. It may be associated with a bicuspid aortic valve, a VSD, or subvalvular aortic stenosis. It is more commonly seen in males, but females who have Turner syndrome are also at increased risk. The lower extremities typically have decreased blood pressure and pulses. There may also be a differential in blood pressure between the left and right arms, depending on where the coarctation occurs. This condition may be treated with surgical repair or balloon dilation with or without placement of a stent.

A 75-year-old man with a 40 pack per year history of smoking and hypercholesterolemia is diagnosed with severe atherosclerosis. Atherosclerotic occlusion of which of the following arteries would result in insufficient perfusion of the urinary bladder? A. External iliac B. Inferior epigastric C. Internal iliac D. Internal pudendal E. Lateral sacral

The correct answer is C. The bladder is supplied by the vesicular branches of the internal iliac arteries. The internal iliacs arise from the common iliac artery.

A 75-year-old man with a 40 pack per year history of smoking and hypercholesterolemia is diagnosed with severe atherosclerosis. Atherosclerotic occlusion of which of the following arteries would result in insufficient perfusion of the urinary bladder? A. External iliac B. Inferior epigastric C. Internal iliac D. Internal pudendal E. Lateral sacral

The correct answer is C. The bladder is supplied by the vesicular branches of the internal iliac arteries. The internal iliacs arise from the common iliac artery. The external iliac (choice A) also arises from the common iliac artery. It makes no contribution to the blood supply of the bladder. The inferior epigastric (choice B) is a branch of the external iliac artery. It serves as a landmark in the inguinal region. Indirect inguinal hernias lie lateral to the inferior epigastric arteries, whereas direct inguinal hernias lie medial to these vessels. The internal pudendal (choice D) is a branch of the anterior division of the internal iliac artery. It gives rise to the inferior rectal artery, perineal artery, artery of the bulb in men, urethral artery, deep artery of the penis or clitoris, and dorsal artery of the penis or clitoris. The lateral sacral (choice E) is a branch of the posterior division of the internal iliac artery. It supplies sacral structures. Reviewed on 4/17/14 Atherosclerosis can affect blood supply to the structures that the vessel supplies. Atherosclerosis is classically a slowly progressive condition; as plaque accumulates in the vessel wall, the distal blood flow pattern is lessened. The initial process of atherosclerosis is a fatty streak that has now been identified in childhood. As the plaque advances, it physically causes obstruction. An acute event such as acute myocardial infarction may cause rupture of this lipid-laden plaque.

A 25-year-old woman involved in an automobile accident is admitted as an emergency patient. A major artery severed in her leg caused an estimated 600 mL blood to be lost. Her blood pressure is 90/60 mm Hg. Which of the following would be expected to increase in response to hemorrhage? A. Arteriolar diameter in skeletal muscle B. Sodium excretion C. Sympathetic nerve activity D. Vagal nerve activity E. Water excretion

The correct answer is C. The decrease in blood pressure caused by hemorrhage activates the baroreceptor reflex, which tends to increase sympathetic nerve activity and decrease parasympathetic (vagal) nerve activity (choice D). The increase in sympathetic nerve activity constricts arterioles in skeletal muscle (choice A) and elsewhere in the body. The fact the patient has lost 600 mL of blood and is not extremely hypotensive (blood pressure 90/60 mm Hg) is a testament to the patient's ability to vasoconstrict to maintain blood pressure and perfusion. Her compensatory responses include the following mechanisms: baroreceptor reflex, chemoreceptor reflex, epinephrine and norepinephrine released from the adrenal medulla, formation of angiotensin II, formation of vasopressin, and the capillary fluid shift mechanism. Activation of the renin-angiotensin system during hemorrhage also plays an important role in maintaining blood pressure. Angiotensin II increases blood pressure acutely by constricting arterioles throughout the body (choice A), and chronically by decreasing the renal excretion of both salt (choice B) and water (choice E). The decrease in salt and water excretion returns blood volume to a normal value.

A 25-year-old woman involved in an automobile accident is admitted as an emergency patient. A major artery severed in her leg caused an estimated 600 mL blood to be lost. Her blood pressure is 90/60 mm Hg. Which of the following would be expected to increase in response to hemorrhage? A. Arteriolar diameter in skeletal muscle B. Sodium excretion C. Sympathetic nerve activity D. Vagal nerve activity E. Water excretion

The correct answer is C. The decrease in blood pressure caused by hemorrhage activates the baroreceptor reflex, which tends to increase sympathetic nerve activity and decrease parasympathetic (vagal) nerve activity (choice D). The increase in sympathetic nerve activity constricts arterioles in skeletal muscle (choice A) and elsewhere in the body. The fact the patient has lost 600 mL of blood and is not extremely hypotensive (blood pressure 90/60 mm Hg) is a testament to the patient's ability to vasoconstrict to maintain blood pressure and perfusion. Her compensatory responses include the following mechanisms: baroreceptor reflex, chemoreceptor reflex, epinephrine and norepinephrine released from the adrenal medulla, formation of angiotensin II, formation of vasopressin, and the capillary fluid shift mechanism. Activation of the renin-angiotensin system during hemorrhage also plays an important role in maintaining blood pressure. Angiotensin II increases blood pressure acutely by constricting arterioles throughout the body (choice A), and chronically by decreasing the renal excretion of both salt (choice B) and water (choice E). The decrease in salt and water excretion returns blood volume to a normal value. Review When a patient is faced with trauma resulting in massive blood loss, there are several compensatory mechanisms that come into play. The body's first response is to stop the bleeding by vasoconstriction of the affected blood vessel. Platelets then migrate to this area to seal the site of injury and further close the site of the damage. The body's natural sympathetic system takes over by increasing the blood pressure and pulse, causing tachypnea, and increasing the vasoconstriction of the blood vessels to maintain volume. Acutely, the renin-angiotensin-aldosterone system then causes the body to save salt and water and to thereby increase intravascular volume.

A 25-year-old woman involved in an automobile accident is admitted as an emergency patient. A major artery severed in her leg caused an estimated 600 mL blood to be lost. Her blood pressure is 90/60 mm Hg. Which of the following would be expected to increase in response to hemorrhage? A. Arteriolar diameter in skeletal muscle B. Sodium excretion C. Sympathetic nerve activity D. Vagal nerve activity E. Water excretion

The correct answer is C. The decrease in blood pressure caused by hemorrhage activates the baroreceptor reflex, which tends to increase sympathetic nerve activity and decrease parasympathetic (vagal) nerve activity (choice D). The increase in sympathetic nerve activity constricts arterioles in skeletal muscle (choice A) and elsewhere in the body. The fact the patient has lost 600 mL of blood and is not extremely hypotensive (blood pressure 90/60 mm Hg) is a testament to the patient's ability to vasoconstrict to maintain blood pressure and perfusion. Her compensatory responses include the following mechanisms: baroreceptor reflex, chemoreceptor reflex, epinephrine and norepinephrine released from the adrenal medulla, formation of angiotensin II, formation of vasopressin, and the capillary fluid shift mechanism. Activation of the renin-angiotensin system during hemorrhage also plays an important role in maintaining blood pressure. Angiotensin II increases blood pressure acutely by constricting arterioles throughout the body (choice A), and chronically by decreasing the renal excretion of both salt (choice B) and water (choice E). The decrease in salt and water excretion returns blood volume to a normal value. Reviewed on 4/17/14 When a patient is faced with trauma resulting in massive blood loss, there are several compensatory mechanisms that come into play. The body's first response is to stop the bleeding by vasoconstriction of the affected blood vessel. Platelets then migrate to this area to seal the site of injury and further close the site of the damage. The body's natural sympathetic system takes over by increasing the blood pressure and pulse, causing tachypnea, and increasing the vasoconstriction of the blood vessels to maintain volume. Acutely, the renin-angiotensin-aldosterone system then causes the body to save salt and water and to thereby increase intravascular volume.

A cyanotic infant is discovered to have a ventricular septal defect, an overriding aorta, right ventricular hypertrophy, and complete pulmonic stenosis. Which of the following accompanying congenital anomalies improves survival in this patient? A. Bicuspid aortic valve B. Ostium secundum defect C. Patent ductus arteriosus D. Patent foramen ovale E. Preductal coarctation of aorta

The correct answer is C. The ductus arteriosus connects the aorta with the pulmonary artery. If it remains patent after birth, it allows oxygenated blood to flow from the aorta to the pulmonary artery. In this patient, who has tetralogy of Fallot with complete right ventricular outflow obstruction, this anastomosis is a crucial source of blood to the pulmonary vasculature. Patients who have a patent ductus arteriosus have an advantage in the setting of tetralogy of Fallot. If tetralogy of Fallot is diagnosed in the newborn period, the infant may benefit from the use of the prostaglandin E1 agent alprostadil (Prostin VR Pediatric), which can be used to maintain the patency of the ductus arteriosus. A patent ductus arteriosus provides an alternative source of pulmonary blood flow while the infant has definitive surgery performed.

A cyanotic infant is discovered to have a ventricular septal defect, an overriding aorta, right ventricular hypertrophy, and complete pulmonic stenosis. Which of the following accompanying congenital anomalies improves survival in this patient? A. Bicuspid aortic valve B. Ostium secundum defect C. Patent ductus arteriosus D. Patent foramen ovale E. Preductal coarctation of aorta

The correct answer is C. The ductus arteriosus connects the aorta with the pulmonary artery. If it remains patent after birth, it allows oxygenated blood to flow from the aorta to the pulmonary artery. In this patient, who has tetralogy of Fallot with complete right ventricular outflow obstruction, this anastomosis is a crucial source of blood to the pulmonary vasculature. A bicuspid aortic valve (choice A) may be asymptomatic but can lead to infective endocarditis, left ventricular overload, and sudden death. It is a common cause of aortic stenosis. It would not benefit a patient who has tetralogy of Fallot in any way. Ostium secundum defect (choice B) is the most common form of atrial septal defect (ASD). ASD is an acyanotic congenital heart disease that would not improve cardiovascular function in a patient who has tetralogy of Fallot. A patent foramen ovale (choice D) is a slit-like remnant of communication between the left and right atria in the fetus. It is usually not of clinical significance. A preductal coarctation of the aorta (choice E) involves narrowing of the aorta proximal to the opening of the ductus arteriosus. This would prevent adequate blood flow through a possible life-preserving patent ductus arteriosus and would result in the patient's death. Reviewed on 4/17/14 Patients who have a patent ductus arteriosus have an advantage in the setting of tetralogy of Fallot. If tetralogy of Fallot is diagnosed in the newborn period, the infant may benefit from the use of the prostaglandin E1 agent alprostadil (Prostin VR Pediatric), which can be used to maintain the patency of the ductus arteriosus. A patent ductus arteriosus provides an alternative source of pulmonary blood flow while the infant has definitive surgery performed.

A 55-year-old woman has been known for years to have mitral valve prolapse. She has now developed exertional dyspnea, orthopnea, and atrial fibrillation. She has an apical, high-pitched, holosystolic heart murmur that radiates to the axilla and back. Because of her deterioration, surgery has been recommended. Which of the following is the most appropriate procedure? A. Aortic valve replacement B. Mitral commissurotomy C. Mitral valve annuloplasty D. Mitral valve replacement E. Both aortic and mitral valve replacement

The correct answer is C. The physical findings are classic for mitral valve insufficiency. Whenever possible, repair of the native mitral valve is preferable to replacement. The way to repair an insufficient valve is to tighten the annulus, bringing the leaflets closer to one another, allowing better coaptation of the valve. Repair of the mitral valve, rather than replacement, allows better preservation of the left ventricle and no need for long-term anticoagulation.

A 55-year-old woman has been known for years to have mitral valve prolapse. She has now developed exertional dyspnea, orthopnea, and atrial fibrillation. She has an apical, high-pitched, holosystolic heart murmur that radiates to the axilla and back. Because of her deterioration, surgery has been recommended. Which of the following is the most appropriate procedure? A. Aortic valve replacement B. Mitral commissurotomy C. Mitral valve annuloplasty D. Mitral valve replacement E. Both aortic and mitral valve replacement

The correct answer is C. The physical findings are classic for mitral valve insufficiency. Whenever possible, repair of the native mitral valve is preferable to replacement. The way to repair an insufficient valve is to tighten the annulus, bringing the leaflets closer to one another, allowing better coaptation of the valve. Repair of the mitral valve, rather than replacement, allows better preservation of the left ventricle and no need for long-term anticoagulation. There are no physical findings indicating involvement of the aortic valve; therefore, choices A and E are wrong. Mitral commissurotomy (choice B) is the preferred operation for mitral stenosis. This patient has no signs of stenosis and no history of rheumatic heart disease to suggest that she might have it. Mitral valve replacement (choice D) is the choice when repair of the native valve cannot be done due to severe valve deformity or degeneration. Reviewed on 4/17/14 Mitral insufficiency (regurgitation) is associated with a holosytolic blowing murmur at the apex of the heart that radiates into the axilla. Mild to moderate disease can be asymptomatic for years, but as it progresses there is eccentric cardiac hypertrophy with increased left ventricular end diastolic volume. This volume overload leads to left ventricular dysfunction. Mitral valve prolapse is a risk factor for the development of mitral regurgitation. Treatment of mitral regurgitation will depend on whether the condition is acute or chronic. For chronic cases, treatment will depend on the left ventricular function; with an adequate left ventricular ejection fraction, acceptable forms of treatment are ACE inhibitors, beta-blockers, diuretics, and surgery, which would include valvuloplasty or annuloplasty (preferentially) or mechanical valve replacement with or without anticoagulation (depending on whether it is a mechanical valve or bioprosthetic valve). Mitral valve repair is preferred over replacement because it is able to better support left ventricular function.

Which of the following diuretics acts at the distal tubule of the nephron? A. Ethacrynic acid (Edecrin) B. Furosemide (Lasix) C. Hydrochlorothiazide (Microzide) D. Mannitol (Osmitrol) E. Spironolactone (Aldactone)

The correct answer is C. The thiazide diuretics (e.g., hydrochlorothiazide, chlorothiazide, benzthiazide) promote diuresis by inhibiting reabsorption of NaCl, primarily in the early distal tubule.

A 57-year-old man comes to his health care provider for a preoperative evaluation. He has been a long-time patient in this office and has been treated for hypertension and gastritis. He has been scheduled for an elective cholecystectomy in 2 days because of ongoing gallbladder symptoms. He currently takes omeprazole (Prilosec) for his gastritis and thiazide for his hypertension daily. He smokes two packs of cigarettes per day. His home blood pressure log shows that his systolic pressures range from 150 to 190 mm Hg, and his diastolic pressures range from 80 to 105 mm Hg, indicating that his blood pressure may be not adequately controlled for the surgical procedure. Which of the following medications is most appropriate in the perioperative period for added blood pressure control? A. Captopril (Capoten) B. Clonidine (Catapres) C. Metoprolol (Lopressor) D. Nifedipine (Adalat, Procardia) E. Prazosin (Minipress)

The correct answer is C. There is an extensive body of literature indicating that beta-blockers given to non-cardiac surgical patients who are at risk for cardiac events are associated with a more favorable outcome in terms of postoperative cardiovascular morbidity and mortality. This patient has somewhat poorly controlled hypertension, as well as at least three cardiovascular risk factors (hypertension, tobacco, age). Ideally one would like to have better control of the blood pressure and to reduce any risk for adverse perioperative events before the patient has elective surgery. Beta-blockers can achieve both of these endpoints. These agents are available parenterally and can be given on a dose titration basis. Although the most recent blood pressure guidelines do not list beta-blockers as first-line agents for blood pressure control, these agents may have utility in the perioperative setting because of their cardiac morbidity benefit. Beta-blockers should be used in patents who have multiple risk factors for cardiovascular disease who are undergoing non-cardiac surgery, as beta-blockers have been shown to decrease mortality in these patients. These agents, particularly the combined alpha- and beta-blocker, labetolol (Trandate), are helpful at treating high levels of blood pressure intraoperatively and are available via the parenteral route.

On physical examination of a patient, the health care provider hears a rumbling mid-diastolic murmur with an opening snap heard best at the apex of the heart. What is the most likely explanation for this finding? A. Mitral regurgitation B. Aortic regurgitation C. Mitral stenosis D. Aortic stenosis E. Tricuspid regurgitation

The correct answer is C. This is a characteristic description of mitral stenosis. Mitral regurgitation (choice A) causes a holosystolic murmur that is heard best at the apex of the heart. Mitral stenosis is typically seen as a complication of rheumatic fever in the United States. During acute rheumatic fever, the mitral valve becomes thickened and retracted, leading to mitral regurgitation. Years later, the fusion of the mitral valve leaflets leads to mitral stenosis. The stenotic mitral valve will have difficulty with motion, with a resultant opening snap, and the poor movement of the valve results in a decreased S1 intensity followed by loud P2 when pulmonary hypertension ensues. The murmur is classically described as a diastolic blowing murmur that may be associated with an opening snap.

On physical examination of a patient, the health care provider hears a rumbling mid-diastolic murmur with an opening snap heard best at the apex of the heart. What is the most likely explanation for this finding? A. Mitral regurgitation B. Aortic regurgitation C. Mitral stenosis D. Aortic stenosis E. Tricuspid regurgitation

The correct answer is C. This is a characteristic description of mitral stenosis. Mitral regurgitation (choice A) causes a holosystolic murmur that is heard best at the apex of the heart. Aortic regurgitation (choice B) causes a blowing diastolic murmur that radiates to the apex and produces a wide pulse pressure. Aortic stenosis (choice D) causes a systolic ejection murmur at the second right interspace that radiates to the neck. Tricuspid regurgitation (choice E) causes a holosystolic murmur that is louder on inspiration. Reviewed on 4/17/14 Mitral stenosis is typically seen as a complication of rheumatic fever in the United States. During acute rheumatic fever, the mitral valve becomes thickened and retracted, leading to mitral regurgitation. Years later, the fusion of the mitral valve leaflets leads to mitral stenosis. The stenotic mitral valve will have difficulty with motion, with a resultant opening snap, and the poor movement of the valve results in a decreased S1 intensity followed by loud P2 when pulmonary hypertension ensues. The murmur is classically described as a diastolic blowing murmur that may be associated with an opening snap.

A 71-year-old white man comes to the emergency department because of blurry vision and blood-tinged urine. He states that he has a long history of hypertension treated with a beta-blocker, an ACE inhibitor, and a calcium channel blocker. He reports that he has had 3 days of blurry vision and urine that is "cola" colored. The symptoms began when he ran out of his medications 3 days ago and he has "not had the time" to get the prescriptions refilled. His physical examination is remarkable for a blood pressure of 200/110 mm Hg in both arms and funduscopic examination showing flame hemorrhages and papilledema. Urinalysis shows red blood cells and some dysmorphic red blood cell casts. He has an abdominal bruit. Which of the following is the most appropriate next step in management? A. Order an ECG and observe the patient B. Give the patient his usual daily doses of BP medications and observe him C. Give the patient intravenous medication to reduce his BP D. Order an abdominal CT scan to rule out aortic aneurysm E. Perform a bedside cystoscopy to evaluate the hematuria

The correct answer is C. This patient has a hypertensive emergency, as evidenced by both the presence of elevated systolic and diastolic blood pressure and evidence of end-organ damage. The blood pressure numbers themselves are a good reason for urgency, but the presence of increased intracranial pressure and renal failure require that this patient be triaged as an emergency. Elevated blood pressure alone, in the absence of symptoms or end-organ damage, rarely requires emergency therapy. The therapy for a hypertensive emergency requires IMMEDIATE LOWERING of the blood pressure by approximately 20-30 mm Hg by intravenous administration of medication. The blood pressure should not be reduced to normotensive levels because of the risk for watershed cerebral infarcts with such a dramatic reduction.

A 71-year-old white man comes to the emergency department because of blurry vision and blood-tinged urine. He states that he has a long history of hypertension treated with a beta-blocker, an ACE inhibitor, and a calcium channel blocker. He reports that he has had 3 days of blurry vision and urine that is "cola" colored. The symptoms began when he ran out of his medications 3 days ago and he has "not had the time" to get the prescriptions refilled. His physical examination is remarkable for a blood pressure of 200/110 mm Hg in both arms and funduscopic examination showing flame hemorrhages and papilledema. Urinalysis shows red blood cells and some dysmorphic red blood cell casts. He has an abdominal bruit. Which of the following is the most appropriate next step in management? A. Order an ECG and observe the patient B. Give the patient his usual daily doses of BP medications and observe him C. Give the patient intravenous medication to reduce his BP D. Order an abdominal CT scan to rule out aortic aneurysm E. Perform a bedside cystoscopy to evaluate the hematuria

The correct answer is C. This patient has a hypertensive emergency, as evidenced by both the presence of elevated systolic and diastolic blood pressure and evidence of end-organ damage. The blood pressure numbers themselves are a good reason for urgency, but the presence of increased intracranial pressure and renal failure require that this patient be triaged as an emergency. Elevated blood pressure alone, in the absence of symptoms or end-organ damage, rarely requires emergency therapy. The therapy for a hypertensive emergency requires IMMEDIATE LOWERING of the blood pressure by approximately 20-30 mm Hg by intravenous administration of medication. The blood pressure should not be reduced to normotensive levels because of the risk for watershed cerebral infarcts with such a dramatic reduction. Ordering an ECG and observing the patient (choice A) is incorrect. Observation is certainly warranted, but not without intervention. Giving the patient his usual daily doses of blood pressure medications and observing him (choice B) would not achieve a rapid enough reduction of the blood pressure, given the presence of end-organ damage on physical examination. Ordering an abdominal CT scan to rule out aortic aneurysm (choice D) is not an appropriate intervention in the case of an active hypertensive emergency. Performing a bedside cystoscopy to evaluate the hematuria (choice E) is not indicated, as the almost certain cause for his hematuria is his elevated blood pressure. Reviewed on 4/17/14 Patients who have hypertensive emergencies should be urgently treated with parenteral agents, because these agents have the fastest onset of activity compared to the oral or transdermal agents. Another advantage with the use of parenteral agents for blood pressure lowering is the ability to titrate these agents to carefully regulate the patient's blood pressure.

A 7-year-old girl is brought to the office because of fever, headache, and malaise that has been increasing over the previous 7 days. She had been away at her grandparents' house in the countryside for vacation and had come down with a sore throat and fever. Her grandmother had given her some over-the-counter cold medication and kept her in bed for a few days, but when she had not been getting better, they had brought her back to the city. Her mother noted a faint red rash that has been waxing and waning on her chest, back, and shoulders, that would become very red after a hot bath. The girl's past medical history is unremarkable and she has not been on any medications recently. On physical examination, the patient is in moderate distress with a temperature of 39.4°C (102.9°F), pulse of 120/min, and respirations of 30/min. She complains of chest discomfort and repeatedly coughs during the examination. Deep breaths cause moderate chest pain, so her breathing is shallow. She also has difficulty swallowing. The tonsils are edematous, erythematous, and covered with purulent exudate. Her ankles are swollen, erythematous, and with limited range of motion. The rash her mother described is present only on the right shoulder in the form of an annular erythematous plaque, 5 cm in diameter. A rapid strep test done in the office is positive. Which of the following complications is the most likely to develop in this patient? A. Aortic valve disease B. Complications are unlikely to happen in this patient C. Mitral valve disease D. Pulmonary valve disease E. Tricuspid valve disease

The correct answer is C. This patient has acute rheumatic fever with signs of evolving carditis. Mitral valve disease is the most common complication, occurring in up to 70% of cases. Acute rheumatic fever is caused by infection with group A β-hemolytic streptococci. It is most commonly seen in children aged 5-15 years because they are most susceptible to infection with group A β-hemolytic streptococci. Skin infections with group A β-hemolytic streptococci do not predispose to rheumatic fever, but upper respiratory infections do. Acute rheumatic fever usually presents 1-5 weeks after a preceding streptococcal pharyngitis. Diagnosis is based on the Jones criteria that include clinical signs and symptoms.

A 66-year-old woman with newly diagnosed type 2 diabetes comes to the health care provider for a blood pressure check. At a previous visit 3 months ago, her blood pressure was 140/95 mm Hg and she was advised to attempt a moderate weight loss with a low-impact exercise program and diet modification. Unfortunately her weight has increased 2 kg (4.4 lb) since her last visit. Her blood pressure is now 144/95 mm Hg. She is mildly resistant to starting medication at this time. Laboratory studies show: Na+ 140 mEq/L K+ 3.9 mEq/L Cl− 109 mEq/L HCO3− 22 mEq/L Glucose 176 mg/dl BUN 23 mg/dl Creatinine 1.2 mg/dl Urinalysis shows: Color Clear Leukocyte esterase Negative Nitrite Negative Blood Negative Microscopic No cells, no casts Protein 3+ Which of the following is the most appropriate intervention at this time? A. Begin therapy with atenolol (Tenormin) B. Begin therapy with labetalol (Trandate) C. Begin therapy with lisinopril (Zestril, Prinivil) D. Begin therapy with verapamil (Calan, Isoptin, Verelan) E. Continue lifestyle modification

The correct answer is C. This patient has diabetes, hypertension, and proteinuria. Her blood pressure must be well controlled to prevent progression of nephropathy. ACE inhibitors have been shown to prevent progression of diabetic nephropathy in type 1 diabetics and likely have similar preventive effects for type 2 diabetics. ACE inhibitors should be first-line therapy for diabetic patients who have hypertension. This patient's potassium is not elevated, so this class of medications is not contraindicated. ACE inhibitors are used in the management of hypertension for patients who have the following comorbidities: diabetes, post-MI, or heart failure with systolic dysfunction or chronic renal disease. They are effective in preventing progression of albuminuria and proteinuria in diabetic patients and patients who have chronic renal disease, and they also prevent cardiac remodeling in patients who have systolic cardiac dysfunction.

A 57-year-old man with hypertension and hyperlipidemia comes to the emergency department complaining of crushing substernal chest pain, diaphoresis, and nausea for 3 hours. He denies palpitations or shortness of breath. He has no previous history of angina or myocardial infarction. His temperature is 37.0°C (98.6°F), blood pressure is 82/60 mm Hg, pulse is 103/min, and respirations are 20/min. Physical examination shows jugular venous distention, a tachycardic heart that is without murmurs, and clear lungs. An electrocardiogram is shown. A right-sided electrocardiogram is also performed that shows ST elevation in V3 and V4. Which of the following is the most appropriate medical intervention at this time? A. Dopamine B. Furosemide (Lasix) C. Intravenous fluids D. Metoprolol (Lopressor, Toprol XL) E. Nitroglycerin

The correct answer is C. This patient is having a right ventricular infarct. The triad of jugular venous distention, hypotension, and clear lungs should make you suspect the diagnosis. In addition, the electrocardiogram shows inferior ST-segment elevation (II, III, and aVF), which is consistent with RV infarct. Right-sided electrocardiogram can be used to support this diagnosis. The finding of elevations in lead V4 on a right-sided electrocardiogram is 100% specific for a right-sided infarct. The management of right ventricular infarct requires aggressive fluid resuscitation, as this type of myocardial infarction is fluid-dependent. The infarcted patient is now preload-dependent. They require aggressive intervention to keep the preload high. Aggressive intravenous fluids therefore should be started until the patient stabilizes or goes for cardiac catheterization. Intravenous fluids should be first-line in maintaining adequate blood pressure. Right ventricular infarct should be considered in all patients who present with ST-segment elevation in the inferior leads, hypotension, and clear lungs. These patients should not receive any medications that will decrease preload (e.g., nitrates, diuretics). Right-sided EKGs can be helpful when making this diagnosis. The treatment of choice is fluids to increase the preload.

In preparation for an inguinal hernia repair, a 22-year-old man has a spinal anesthetic placed. The level of sensory block turns out to be much higher than had been planned, and shortly thereafter his blood pressure drops to 75/20 mm Hg. He looks warm and flushed, and his central venous pressure is near 0 mm Hg. He has no history of blood pressure abnormalities. Which of the following should be included in his therapy? A. Diuretics and fluid restriction B. Whole blood and clotting factors C. Inotropic agents and cardiac assist pump D. Vasoconstrictors and intravenous fluids E. Vasodilators and intravenous fluids

The correct answer is D. A high spinal anesthetic can produce vasomotor shock by inducing widespread vasodilation. Vasoconstrictors are the appropriate therapy, but because the capacity of the vascular tree is also increased under these circumstances, filling it up with additional volume is also helpful. Narcotic-induced and spinal anesthesia-induced hypotension should be treated by volume support and vasopressors such as dopamine or dobutamine. This treatment will help to reverse the vasodilation that the spinal anesthesia induces.

A 12-year-old girl presents to her health care provider with an erythematous macular skin rash in a bathing suit distribution. She has a history of streptococcal sore throat a few weeks ago. Which of the following signs would be necessary to make the diagnosis of rheumatic fever? A. Elevated erythrocyte sedimentation rate B. Leukocytosis C. Migratory polyarthritis D. Prolonged PR interval on EKG E. Temperature >38°C (100.4°F)

The correct answer is C. This patient's macular skin rash in a bathing suit distribution is also known as erythema marginatum, one of the five major Jones criteria for diagnosing rheumatic fever. The other four major criteria are migratory polyarthritis, Sydenham chorea, subcutaneous nodules, and pancarditis. In context of prior streptococcal infection, the presence of two of five major criteria or one major plus two minor criteria (choices A, B, D, or E) is sufficient to establish the diagnosis. Other minor criteria include previous rheumatic fever and elevated C-reactive protein. A mnemonic for the major criteria is CANCER (not pleasant, but easy to remember): Chorea, Arthritis, Nodules, Carditis, Erythema marginatum, which is consistent with Rheumatic fever diagnosis. Rheumatic fever is most commonly seen in developing countries. It is a condition that results from previous streptococcal infection and occurs as a result of an autoimmune process. Jones criteria are used collectively to confirm the diagnosis. The most serious long-term complication is related to the heart. The mitral valve is most commonly affected by this disease, with the aortic valve second most commonly affected. Patients who have rheumatic fever will have secondary prophylaxis against Streptococcal infection with chronic penicillin therapy to prevent recurrent disease and also to improve the prognosis of established rheumatic heart disease.

A 58-year-old man with history of atrial fibrillation is prescribed warfarin to prevent clot and embolism formation. His prothrombin time (PT) is regularly monitored. Administration of which of the following drugs would result in an increase in his PT and require readjustment of his warfarin dosage? A. Rifampin B. Carbamazepine (Tegretol) C. Ketoconazole D. Phenytoin (Dilantin)

The correct answer is C. This question is testing a favorite exam concept—the relationship between the hepatic cytochrome P-450 system and the metabolism of drugs. Certain drugs can affect the metabolism of other drugs by either inducing or inhibiting hepatic microsomal enzyme activity. This means that the amount of circulating warfarin increased and that you are looking for a drug that inhibits the P-450 system (thereby decreasing warfarin metabolism). Of the drugs listed, only ketoconazole inhibits the P-450 system. Other important inhibitors include cimetidine (Tagamet) and isoniazid (INH). Warfarin (Coumadin) has multiple medication and food interactions. When prescribing this medication, a careful medication history should be taken along with patient education with regard to dietary instructions. For warfarin to be effective at preventing complications from atrial fibrillation, a therapeutic INR is necessary and frequent monitoring needs to be performed on an ongoing basis. Genetic testing is available for predicting how the patient will metabolize warfarin, which can help the health care provider to determine proper dosing of this medication (slow versus fast metabolizers).

A 2-year-old boy is shot in the arm in a drive-by shooting. His brachial artery is partially transected and there is copious bleeding. Emergency medical technicians are able to control the site of bleeding by local pressure and the child stops losing blood, though he is hypotensive and tachycardic. Intravenous fluid resuscitation is urgently needed, but several attempts at starting peripheral intravenous lines are unsuccessful. Which of the following is the best alternative route in this situation? A. Central line via subclavian puncture B. Hypodermoclysis C. Intraosseous cannulation in the proximal tibia D. Percutaneous femoral vein cannulation E. Saphenous vein cutdown

The correct answer is C. When intravenous lines cannot be established, the preferred alternate route is intraosseous (IO) cannulation. This is done by placing a trocar in the bone marrow of a long bone. The site of choice in children is the proximal tibia; alternative sites are the distal tibia and proximal femur. Careful attention should be made in regard to IO location in order to avoid injury to the growth plate which could stunt bony growth as the child grows older.

A patient with chronic, well-compensated congestive heart failure presents with increased dyspnea and peripheral edema. Which of the following precipitating factors least likely contributes to this clinical outcome? A. Discontinuation of medications due to cost and/or side effects B. Development of a systemic infection C. Onset of cardiac dysrhythmias D. Initiation of a low sodium diet E. Hyperthyroidism

The correct answer is D. A low sodium diet is indicated in patients who have mild heart failure and may provide symptomatic improvement, especially if accompanied by periods of physical rest. Indiscriminate use of salt is one of the major precipitants for deterioration in the clinical status of a patient who has chronic heart failure. Increased sodium in the diet leads to increased water retention with an increase in the amount of intravascular volume that the patient must circulate. When a patient who has stable, well controlled heart failure has sudden deterioration of his or her symptoms, a search for the etiology for this deterioration should be undertaken. A patient who ingests a high salt load (highly processed foods and soups are notorious for causing this) can have exacerbation of symptoms caused by the increased workload on the heart because of this increased volume that must be circulated. There may also be more serious causes for deterioration of symptoms, including myocardial infarction, so a careful assessment should be done. Keep in mind that heart failure is a constellation of symptoms and a careful search for the underlying etiology for this change in symptoms should be performed rather than just treating the symptoms of heart failure.

A 15-year-old girl is brought to a pediatric cardiology clinic with a complaint of chest pain. She states the pain has come and gone over the past year but has increased in frequency over the past few weeks. She describes it as a sharp pain over her left chest. Physical examination reveals a healthy appearing 15-year-old girl. Her temperature is 37.2°C (99°F), pulse 90/min, and respirations 20/min. Lung examination is normal. Cardiac examination reveals a late systolic murmur preceded by a click at the apex. No heave or rub is present. Electrocardiogram and chest radiograph are unremarkable. Which of the following is the most likely diagnosis? A. Atrial septal defect B. Mitral regurgitation C. Mitral stenosis D. Mitral valve prolapse E. Tricuspid regurgitation

The correct answer is D. An apical click followed by a late systolic murmur is classic for mitral valve prolapse. It can be a source of subjective chest pain in children. Mitral valve prolapse is more common in females. Antibiotic prophylaxis is no longer routinely recommended by the AHA prior to dental procedures unless prolapse is accompanied by severe regurgitation.

A 15-year-old girl is brought to a pediatric cardiology clinic with a complaint of chest pain. She states the pain has come and gone over the past year but has increased in frequency over the past few weeks. She describes it as a sharp pain over her left chest. Physical examination reveals a healthy appearing 15-year-old girl. Her temperature is 37.2°C (99°F), pulse 90/min, and respirations 20/min. Lung examination is normal. Cardiac examination reveals a late systolic murmur preceded by a click at the apex. No heave or rub is present. Electrocardiogram and chest radiograph are unremarkable. Which of the following is the most likely diagnosis? A. Atrial septal defect B. Mitral regurgitation C. Mitral stenosis D. Mitral valve prolapse E. Tricuspid regurgitation

The correct answer is D. An apical click followed by a late systolic murmur is classic for mitral valve prolapse. It can be a source of subjective chest pain in children. Mitral valve prolapse is more common in females. Antibiotic prophylaxis is no longer routinely recommended by the AHA prior to dental procedures unless prolapse is accompanied by severe regurgitation. An atrial septal defect (choice A) is characterized by a fixed and widely split second heart sound. Mitral (choice B) and tricuspid regurgitation (choice E) produce holosystolic murmurs with relatively uniform intensity. Mitral regurgitation is heard at the apex, whereas tricuspid regurgitation is best heard along the lower left sternal border. Mitral stenosis (choice C) is characterized by a mid-diastolic murmur heard after an opening snap. Reviewed on 4/17/14 Classically, mitral valve prolapse (MVP) presents with an apical click followed by a systolic murmur. This condition can produce subjective chest pain and palpitations or can be an incidental finding picked up in an asymptomatic patient. Antimicrobial prophylaxis is no longer recommended by the American Heart Association for MVP unless there is severe regurgitation.

An elderly homeless man who regularly resides outside the emergency department is acting strangely. He is often seen drinking alcohol and asking for money, but today he seems disoriented, confused, and weak. It is bitterly cold and there is snow on the ground, so one of the health care providers is concerned that the combination of cold weather and alcohol would predispose the patient to hypothermia. Two emergency medical technicians help bring the man inside so he can be fully evaluated. His temperature is 32.0°C (89.6°F), blood pressure 80/40 mm Hg, pulse 53/min, and respirations 10/min. Electrocardiogram would most likely show which of the following findings? A. Delta wave B. Depressed ST segments C. Diffuse broadening of T waves D. Osborn waves E. U waves

The correct answer is D. An upward deflection following the R wave (classically in lead II) describes an Osborn wave (or J wave of Osborn), which is pathognomonic of hypothermia. The Osborn wave appears on the EKG following ST-segment elevation and is seen when the patient's body temperature is 32-33°C (89-91°F). Although these findings are pathognomonic for hypothermia, their absence does not rule out heart involvement in a patient who has hypothermia.

A patient admitted to the emergency department with chest pain is diagnosed with myocardial infarction. On discharge, the patient is prescribed aspirin but develops an allergic hypersensitivity reaction. Clopidogel (Plavix) is then prescribed. Which of the following is the mechanism of action of this drug? A. It binds to the active site of cyclo-oxygenase via acetylation B. It blocks the binding of plasmin to fibrin C. It hinders the production of thromboxane A2 D. It inhibits adenosine diphosphate-induced platelet aggregation E. It stimulates platelet adenylyl cyclase

The correct answer is D. Clopidogrel (Plavix) is an antiplatelet drug that inhibits adenosine diphosphate-induced platelet aggregation and interacts with platelet glycoprotein IIb/IIIa, a fibrinogen receptor that links platelets together and leads to platelet plug formation. The drug thus prevents fibrinogen from binding to platelets. The drug is useful both for patients who have coronary disease and for those who have cerebrovascular disease such as stroke. Of course, hemorrhagic stroke should first be ruled out before it is deemed safe to use any anticoagulant. Clopidogrel is used in both acute coronary syndrome (for patients not having an open bypass) and for thrombotic event prevention. For stroke prevention, it is used when aspirin or warfarin (Coumadin) cannot be used.

A patient admitted to the emergency department with chest pain is diagnosed with myocardial infarction. On discharge, the patient is prescribed aspirin but develops an allergic hypersensitivity reaction. Clopidogel (Plavix) is then prescribed. Which of the following is the mechanism of action of this drug? A. It binds to the active site of cyclo-oxygenase via acetylation B. It blocks the binding of plasmin to fibrin C. It hinders the production of thromboxane A2 D. It inhibits adenosine diphosphate-induced platelet aggregation E. It stimulates platelet adenylyl cyclase

The correct answer is D. Clopidogrel (Plavix) is an antiplatelet drug that inhibits adenosine diphosphate-induced platelet aggregation and interacts with platelet glycoprotein IIb/IIIa, a fibrinogen receptor that links platelets together and leads to platelet plug formation. The drug thus prevents fibrinogen from binding to platelets. The drug is useful both for patients who have coronary disease and for those who have cerebrovascular disease such as stroke. Of course, hemorrhagic stroke should first be ruled out before it is deemed safe to use any anticoagulant. Clopidogrel is used in both acute coronary syndrome (for patients not having an open bypass) and for thrombotic event prevention. For stroke prevention, it is used when aspirin or warfarin (Coumadin) cannot be used. Aspirin acetylates cyclo-oxygenase at or near the active site (choice A) and thus hinders the production of thromboxane A2 (choice C). Aminocaproic acid strongly inhibits fibrinolysis. It blocks the binding of plasmin to fibrin (choice B) by attaching to lysine binding sites on plasmin and plasminogen. Dipyridamole (Persantine) is a phosphodiesterase inhibitor that also blocks the reuptake of adenosine, which acts on the platelets to stimulate adenylyl cyclase (choice E). Reviewed on 4/17/14 Antiplatelet agents have been shown to be effective in the setting of acute coronary syndrome and for prevention of thromboembolic events. Aspirin is commonly used in these settings, but for patients who have a contraindication for aspirin use, clopidogrel (Plavix) is a useful alternative. The primary side effect with these antiplatelet agents is bleeding. There is a Black Box warning for clopidogrel for patients who are poor metabolizers of this medication; if they cannot metabolize clopidogrel appropriately, they wil have diminished efficacy with its use. Genotype testing is available to identify these patients.

A 22-year-old construction worker is brought to the emergency department after being pinned by a steel beam that fell on his legs and mid-torso. The patient was brought to the hospital by the EMS team who had extricated him. At the scene, his Glasgow score was 15/15, and he was alert and oriented. On arrival the patient appears pale and bloody and in a significant amount of pain, but still alert. Examination reveals bilaterally crushed lower extremities with a visibly pulsating bleed. Emergency laboratory data are as follows: Sodium 143 mEq/L (135-145 mEq/L) Potassium 5.2 mEq/L (3.5-5.2 mEq/L) Bicarbonate 20 mEq/L (20-29 mEq/L) Chloride 98 mEq/L (96-106 mEq/L) Urea nitrogen (BUN) 37 mg/dL (7-20 mg/dL) Creatinine 1.1 mg/dL (0.8-1.4 mg/dL) Creatine phosphokinase 15,300 U/L (52-200 U/L) Aspartate aminotransferase 112 IU/L (<40 IU/L) Alanine aminotransferase 99 IU/L (<40 IU/L) Hematocrit 38% (41-50%) Arterial blood gas on 40% oxygen pO2 50 mm Hg (75-100 mm Hg) PCO2 37 mm Hg (35-45 mm Hg) pH 7.33 (7.35-7.45) Which of the following is the most appropriate intervention at this time? A. 12-lead electrocardiogram B. Echocardiogram C. Fractionated creatine kinase levels D. Intravenous crystalloid and bicarbonate E. Right upper quadrant ultrasound

The correct answer is D. Crush injuries such as the one sustained by this patient often result in a massive release of muscle contents. Creatine kinase (CK) is an enzyme released by dead or damaged muscle into the blood. By itself it is harmless, but it is a marker for myoglobin, which is directly nephrotoxic. Myoglobin, which is not measured by conventional assay, is released after damage to muscle. Aldolase levels can also be used to assess the degree of rhabdomyolysis. Much data exist showing early intervention with copious alkalinized IV crystalloid can prevent renal damage. The management of myoglobinuria includes: Fluid challenge to maintain urine output 1-2 mL/kg/h Alkalinization of the urine: add sodium bicarbonate to the IV fluid Forced diuresis: add mannitol to the IV fluid

Which of the following factors is recognized as contributing to atherosclerotic risk? A. Decreased lipoprotein A B. Hypohomocysteinemia C. Decreased fibrinogen levels D. Decreased fibrinolytic activity E. Hypoinsulinemia

The correct answer is D. Decreased fibrinolytic activity or an elevated plasminogen activator inhibitor helps to modulate the balance between fibrinogen factors. Fibrinolytic activity helps to clear areas of plaque, which contributes to atherosclerosis. Atherosclerotic risk factors include abnormalities in blood lipids, genetic inherited patterns, poor diet high in cholesterol and fatty acids, and sedentary lifestyle. Other risk factors for atherosclerosis include impaired fibrinolysis and inflammatory conditions. Lipid abnormalities that contribute to atherosclerosis include elevated LDL cholesterol, low HDL cholesterol, and an increase in the triglyceride levels. Other laboratory associations with increased lipid risks include an elevated concentration of apolipoprotein B (apoB) and increase in lipoprotein (a). Atherosclerosis is a slowly progressive process with few symptoms until the disease is manifested clinically by an acute cardiovascular event.

Which of the following factors is recognized as contributing to atherosclerotic risk? A. Decreased lipoprotein A B. Hypohomocysteinemia C. Decreased fibrinogen levels D. Decreased fibrinolytic activity E. Hypoinsulinemia

The correct answer is D. Decreased fibrinolytic activity or an elevated plasminogen activator inhibitor helps to modulate the balance between fibrinogen factors. Fibrinolytic activity helps to clear areas of plaque, which contributes to atherosclerosis. Lipoprotein A (choice A) is thought to be a modulator of fibrinolysis. Elevated levels are found to be a risk for atherosclerotic development. Elevated levels of homocysteine, not lower levels of homocysteine (choice B) have been shown to correlate with thrombosis and coronary risk. Folate and vitamin B12 supplementation may help to lower homocysteine levels, but there is no evidence that proves that lowering elevated homocysteine levels affects coronary artery disease risk. Fibrinogen (choice C) is an acute phase reactant that serves as a marker for inflammation, in addition to contributing to the pathogenesis of coronary artery disease. Hyperinsulinemia, not hypoinsulinemia (choice E), is associated with an increased incidence of CAD because of alterations in lipid metabolism and increased platelet aggregation. Reviewed on 4/17/14 Atherosclerotic risk factors include abnormalities in blood lipids, genetic inherited patterns, poor diet high in cholesterol and fatty acids, and sedentary lifestyle. Other risk factors for atherosclerosis include impaired fibrinolysis and inflammatory conditions. Lipid abnormalities that contribute to atherosclerosis include elevated LDL cholesterol, low HDL cholesterol, and an increase in the triglyceride levels. Other laboratory associations with increased lipid risks include an elevated concentration of apolipoprotein B (apoB) and increase in lipoprotein (a). Atherosclerosis is a slowly progressive process with few symptoms until the disease is manifested clinically by an acute cardiovascular event.

A 57-year-old woman comes to her health care provider for follow-up of a fasting serum total cholesterol level of 236 mg/dL. She is post-menopausal since age 52 and has not been on hormone replacement therapy. She has a positive family history for coronary artery disease and she has smoked one half pack of cigarettes per day for the past 20 years. During her last physical examination a lipid profile was ordered, and she presents today for evaluation of those results. Which of the following lipid panels would most strongly suggest the need for pharmacologic therapy in this patient? A. Total cholesterol 180 mg/dL, LDL cholesterol 140 mg/dL B. Total cholesterol 184 mg/dL, LDL cholesterol 100 mg/dL C. Total cholesterol 230 mg/dL, LDL cholesterol 100 mg/dL D. Total cholesterol 245 mg/dL, LDL cholesterol 165 mg/dL E. Total cholesterol 285 mg/dL, LDL cholesterol 100 mg/dL

The correct answer is D. For those patients in whom a fasting panel has been obtained, a stepwise approach to intervention based on the patient's LDL and risk factors may be used. A patient who has 2+ risk factors (this patient, because of smoking history and positive family history) and an LDL of greater than 160 mg/dL warrants medical therapy.

A 1-day-old infant girl appears dusky in the newborn nursery during feeding. Oxygen is immediately administered by nasal cannula. Shortly afterward, she develops tachypnea. On physical examination her blood pressure from the right upper arm is 50/30 mm Hg, pulse 180/min, and respirations 60/min. Echocardiogram is consistent with hypoplastic left heart syndrome. Which of the following would likely be found on auscultation? A. Continuous ductal murmur, bounding pulses B. Continuous ductal murmur, poor peripheral pulses C. Holosystolic murmur, poor peripheral pulses, quiet second heart sound D. No murmur, precordial hyperactivity, loud second heart sound E. No murmur, precordial hyperactivity, quiet second heart sound

The correct answer is D. Hypoplastic left heart (HLH) syndrome is a group of closely related cardiac anomalies characterized by underdevelopment of the left cardiac chambers, atresia or stenosis of the aortic and/or the mitral orifices, and hypoplasia of the aorta. These anomalies are an especially common cause of heart failure in the first week of life. The left atrium and ventricle often exhibit endocardial fibroelastosis. Pulmonary venous blood traverses a patent foramen ovale, and a dilated and hypertrophied right ventricle acts as the systemic as well as pulmonary ventricle; the systemic circulation receives blood by way of a patent ductus arteriosus. Infants who have HLH syndrome develop poor perfusion, and metabolic acidosis when systemic blood flow decreases. When the ductus closes, inadequate blood flow to the body occurs because the ductus is the only path for blood to flow from the right ventricle to the body. Even if the ductus remains open, when the infant is given oxygen, the oxygen will dilate the vasculature of the pulmonary circulation, and blood will preferentially flow to the lower pressure pulmonary system, depriving the systemic circulation of adequate perfusion. ECG usually shows right axis deviation, right atrial and ventricular enlargement, and nonspecific ST- and T-wave abnormalities in the left precordial leads. Chest radiography may show only slight enlargement shortly after birth, but with clinical deterioration there is marked cardiomegaly with increased pulmonary vascular markings. Echocardiography is diagnostic and will show a diminutive aortic root and left ventricular cavity and absence or poor visualization of aortic and mitral valves. An infant who has HLH syndrome has a hyperdynamic precordium because the enlarged right ventricle is contracting against systemic pressure. The infant also has a loud second heart sound (S2) because the pulmonary artery acts as the aorta by pumping blood to the systemic circulation through the ductus arteriosus. The high end-systolic pressure markedly enhances S2. The flow from the right ventricle to the pulmonary artery is not turbulent; therefore, there is usually no significant murmur heard on auscultation. When the ductus closes, or when the pulmonary vasculature resistance falls, the flow to the systemic circulation will decrease, causing greatly diminished peripheral pulses. Management of the hypoplastic left heart syndrome patient includes infusion of prostaglandin E1 and administration of room air while on a ventilator (instead of oxygen, which will dilate the vasculature of the pulmonary circulation, causing blood to flow into the pulmonary system rather than the systemic circulation). Prostaglandin E1 may open the ductus arteriosus and restore systemic blood flow. Administration of room air or even hypobaric oxygen (FiO2 less than 21%) and the use of muscle relaxants can prevent hyperventilation and subsequent pulmonary vasodilation, thus reversing systemic hypoperfusion and metabolic acidosis. Hypoplastic left heart (HLH) syndrome is a group of cardiac anomalies characterized by underdevelopment of the left atrium and ventricle, atresia or stenosis of the aortic and/or the mitral orifices, and hypoplasia of the aorta. These anomalies are an especially common cause of heart failure in the first week of life. Echocardiography is diagnostic and will show a diminutive aortic root and left ventricular cavity, and an absence or poor visualization of aortic and mitral valves.

A 52-year-old man is discharged from the hospital after an uncomplicated myocardial infarction. Several weeks later, he visits his primary care provider complaining of insomnia, anorexia, and depressed mood. He appears to be clinically depressed. He denies any current chest pain or shortness of breath. Which of the following would be the most appropriate medication to initiate for this patient? A. Methylphenidate (Ritalin, Concerta, Metadate) B. Nortriptyline (Pamelor) C. Phenelzine (Nardil) D. Sertraline (Zoloft) E. Thioridazine (Mellaril)

The correct answer is D. In a patient who has cardiac complications, a selective serotonin reuptake inhibitor (SSRI) such as sertraline has been demonstrated to be the safest and most effective medication for the treatment of clinical depression. It has overtaken tricyclics in the management of depression in patients who have pre-existing heart disease, because it is not associated with prolongation of the QT interval that the tricyclics are known to cause. When compared to tricyclics, SSRIs also have a safer profile if a patient overdoses. Major depression disorder is a commonly encountered condition in many clinical conditions. Patients who have life-threatening events such as myocardial infarction or who undergo coronary artery bypass grafting are at increased risk for major depression disorder. SSRIs are first-line therapy used in the management of depression in this setting because of a lower risk for cardiac side effects and fewer drug interactions with its use when SSRIs are compared to tricyclic antidepressants.

Which of the following is an example of a prerenal cause of oliguria? A. Acute glomerulonephritis B. Bladder tumor C. Prostatic hyperplasia D. Shock E. Transplant rejection

The correct answer is D. It is important to remember that not all oliguria is caused by intrinsic renal disease. Shock, of any etiology, is an example of a prerenal cause of oliguria in which poor renal perfusion leads to inadequate urine output. Other examples of prerenal causes include hepatorenal syndrome, postoperative oliguria, and dehydration. Postrenal causes of oliguria usually involve bilateral obstruction and may be caused by disease of the ureters, bladder neck, or urethra. A patient having shock will have decreased perfusion to the kidneys. This decreased perfusion will cause the kidneys to compensate for this impaired blood flow by increasing the release of renin, which will then lead to the formation of angiotensin I, which gets converted to angiotensin II, and later conversion to aldosterone occurs. This will result in vasoconstriction and retention of sodium, which will retain water and conserve water loss from the body. The urine in these patients will have a low fractional excretion of sodium, which will be <1%. This low fractional excretion of sodium will help to preserve fluid in this shock state, which will permit more volume being available for the body to use.

A 72-year-old man is scheduled to have elective sigmoid resection for diverticular disease. He has a history of heart disease and had a documented myocardial infarction 2 years ago. He currently does not have angina, but he lives a sedentary life because "he gets out of breath" if he exerts himself. Physical examination reveals jugular venous distention. His hemoglobin level is 9 g/dL (normal 14-17 g/dL in men). If surgery is indeed needed, which of the following should most likely be done prior to the operation? A. Evaluate the patient as a candidate for coronary revascularization B. Place the patient on intensive respiratory therapy C. Order a transfusion to increase the patient's hemoglobin D. Treat the patient for congestive heart failure E. If at all possible, wait 6 months before performing surgery

The correct answer is D. Jugular venous distention in this setting is indicative of congestive heart failure, a condition that would make elective surgery very risky. Medical treatment for congestive heart failure can reduce the risk. This patient has right-sided heart failure, which is most commonly caused by left-sided heart failure.

A 50-year-old white female police officer experiences an episode of substernal chest pain while chasing a suspect on foot. She arrives in your clinic 4 hours later, describing the pain as a pressure sensation and nonradiating, lasting 5 to 10 minutes and relieved by stopping. It was associated with shortness of breath, but no nausea, vomiting, diaphoresis, or palpitations. Her physical examination and resting ECG are within normal limits. She currently smokes cigarettes. Which of the following is the most appropriate initial course of action? A. Empirically place her on an ACE inhibitor B. Cardiac catheterization C. Upper gastrointestinal series D. Exercise stress test E. CT scanning of the coronaries

The correct answer is D. Patients who have a new onset of chest pain or a change in the previously established pattern of angina are classified as having unstable angina. Anginal symptoms are classically described as prolonged or new chest pain; EKG in these patients can be normal or show signs of ST-segment depression. Cardiac enzymes are normal. Exercise stress testing is the most widely used diagnostic test initially, even though the overall sensitivity of the test is only about 75%. Exercise stress testing interpretation should include patient symptoms during and following exercise, exercise capacity, hemodynamic response, and EKG response to exercise. If the patient has a pre-exercise EKG showing left bundle branch block, the ensuing exercise-induced EKG will be difficult to interpret, which might result in the patient having stress imaging as part of the evaluation strategy.

A 63-year-old retired airline pilot comes to the office for a first time visit. He is switching primary care facilities because he was very unhappy with the care that he previously received. He brings his medical record, which documents longstanding cirrhosis and portal hypertension caused by previous alcohol abuse. He has no history of gastrointestinal bleeding. He has a very long medication list, and he reports that his pharmacist told him that his medications "were wrong" and that his health care provider was prescribing drugs improperly. On review of his medication list, a number of medication incompatibilities are found. Co-administration of which of the following is likely to cause this patient the greatest harm if not corrected? A. Lactulose and oral potassium B. Prednisone and inhaled albuterol (Ventolin, ProAir) C. Propranolol (Inderal) and isosorbide dinitrate (Isordil) D. Spironolactone (Aldactone) and oral potassium E. Spironolactone (Aldactone) and hydrochlorothiazide

The correct answer is D. Pharmacology is one of the most important applied basic science topics in clinical medicine. Possible drug interactions are so important a concern that entire organizations and computer cross-check systems have been created to address this issue. A health care provider must still know, however, the consequences and potential interactions of every medicine prescribed. Spironolactone is a potassium-sparing diuretic. The mistake of calling this drug a "diuretic" and cueing your brain to think "replenish electrolytes" could cause this patient to die from hyperkalemic cardiac arrest. Spironolactone is increasingly used as an add-on agent for chronic heart failure, as there has been mortality benefit demonstrated with its use. Spironolactone also is a direct aldosterone antagonist and can be beneficial in the setting of cirrhosis with ascites, to lessen fluid accumulation. Spironolactone is an aldosterone antagonist that also causes potassium retention (also known as a potassium-sparing diuretic). Spironolactone should be used cautiously with potassium replacement or with agents that are known to cause hyperkalemia such as the ACE inhibitors and ARBs.

A 63-year-old retired airline pilot comes to the office for a first time visit. He is switching primary care facilities because he was very unhappy with the care that he previously received. He brings his medical record, which documents longstanding cirrhosis and portal hypertension caused by previous alcohol abuse. He has no history of gastrointestinal bleeding. He has a very long medication list, and he reports that his pharmacist told him that his medications "were wrong" and that his health care provider was prescribing drugs improperly. On review of his medication list, a number of medication incompatibilities are found. Co-administration of which of the following is likely to cause this patient the greatest harm if not corrected? A. Lactulose and oral potassium B. Prednisone and inhaled albuterol (Ventolin, ProAir) C. Propranolol (Inderal) and isosorbide dinitrate (Isordil) D. Spironolactone (Aldactone) and oral potassium E. Spironolactone (Aldactone) and hydrochlorothiazide

The correct answer is D. Pharmacology is one of the most important applied basic science topics in clinical medicine. Possible drug interactions are so important a concern that entire organizations and computer cross-check systems have been created to address this issue. A health care provider must still know, however, the consequences and potential interactions of every medicine prescribed. Spironolactone is a potassium-sparing diuretic. The mistake of calling this drug a "diuretic" and cueing your brain to think "replenish electrolytes" could cause this patient to die from hyperkalemic cardiac arrest. Spironolactone is increasingly used as an add-on agent for chronic heart failure, as there has been mortality benefit demonstrated with its use. Spironolactone also is a direct aldosterone antagonist and can be beneficial in the setting of cirrhosis with ascites, to lessen fluid accumulation. Lactulose and oral potassium (choice A) is a useful combination for the treatment of hepatic encephalopathy. Lactulose, a nonabsorbable sugar, is a non-metabolizable sugar that acidifies the stool and thus traps ammonia in the GI lumen for excretion. It may cause an osmotic diarrhea, and potassium replacement may become necessary. Prednisone and inhaled albuterol (choice B) is the combination of a glucocorticoid and an inhaled beta-adrenergic agonist for asthma. These agents may work in concert with each other: prednisone to decrease inflammation and albuterol to lessen bronchospasm. Propranolol and isosorbide dinitrate (choice C) is a combination of a nonspecific beta-blocker and a nonspecific venodilator. This combination is useful in the treatment of angina, as the beta-blocker prevents reflex tachycardia secondary to the isosorbide dinitrate-induced drop in preload. Spironolactone and hydrochlorothiazide (choice E) is a combination of a K-sparing diuretic and a thiazide diuretic that is effective in the treatment of cirrhotic edema and ascites. Spironolactone is an aldosterone antagonist and this is helpful in the setting of cirrhosis and ascites. Reviewed on 4/17/14 Spironolactone is an aldosterone antagonist that also causes potassium retention (also known as a potassium-sparing diuretic). Spironolactone should be used cautiously with potassium replacement or with agents that are known to cause hyperkalemia such as the ACE inhibitors and ARBs.

A 60-year-old alcoholic female who appears malnourished presents to her health care provider complaining of shortness of breath and gasping for air on awakening. Cardiac examination reveals an S3 heart sound, a diastolic murmur, and jugular venous distention. Pulmonary rales and peripheral edema are evident. An echocardiogram would be expected to reveal which of the following? A. Carotid pulse tracing with spike and dome configuration B. Bilateral atrial enlargement and ventricular thickening C. Depressed left ventricular function with pericardial effusion D. Left and right ventricular dilatation with poor contraction throughout E. Left ventricular hypertrophy with asymmetric septal hypertrophy

The correct answer is D. Primary acquired cardiomyopathy can occur in the peripartum period or from tachycardia-induced dilated cardiomyopathy. Secondary, toxicity-related cardiomyopathy can occur as a result of alcohol, doxorubicin use, or from heavy metal or chemical exposure. Peripartum dilated cardiomyopathy typically occurs in obese, multiparous women older than age 30. Patients who have pre-eclampsia are at increased risk. When a patient is peripartum, this condition has to occur in the third trimester or within 5 months postpartum. Echocardiogram will show normal or decreased wall thickness, poor wall thickening in systole, left ventricular dilation in a spherical pattern, and diminished stroke volume. Patients who have this disorder tend to present with fatigue and dyspnea. The patient in this case has dilated cardiomyopathy caused by ethanol abuse. Malnourishment often accompanies severe alcoholism and implies thiamine deficiency, which can lead to heart disease (wet beriberi). Suspect this diagnosis in any alcoholic patient presenting with symptoms and signs of congestive heart failure. In this patient, an echocardiogram would be expected to reveal bilateral ventricular dilatation with impaired contraction throughout both chambers.

An elderly man presents with complaints of dizziness, headaches, diarrhea, nausea and vomiting, weakness, palpitations, and a change in vision with a yellowish to blue tint to his vision. He is taking multiple medications. He has a history of chronic heart failure and hypertension. His wife states that he has had a few episodes of confused, delirious behavior over the past few weeks. Which of the following agents might be responsible for this man's symptoms? A. Allopurinol (Zyloprim) B. Hydralazine (Apresoline) C. Niacin D. Digoxin (Lanoxin) E. Spironolactone (Aldactone)

The correct answer is D. The collection of symptoms described—dizziness, headaches, diarrhea, nausea and vomiting, weakness, palpitations, and a change in vision with a yellowish to blue tint to the vision—are classic side effects of digoxin (Lanoxin). EKG changes, such as biventricular tachycardia, may also occur. Digoxin is a medication that has been shown to improve symptoms in patients who have heart failure, as it is a positive inotropic and negative chronotropic agent. Patients who are hypokalemic are especially prone to the development of digoxin toxicity. Patients who have digoxin toxicity experience GI side effects along with dizziness, headache, and visual changes. Patients who have renal failure are at increased risk for the development of digoxin toxicity due to their inability to clear the digoxin. Patients who are hypokalemic are also at high risk for developing digoxin toxicity, and this can occur even in patients who have high normal levels of digoxin measured in the serum.

A 72-year-old woman with no prior medical history comes to the emergency department because of a 3-hour episode of crushing substernal chest pain. The pain radiates to her arm and neck. An electrocardiogram reveals ST-segment elevation in leads II, III, and aVF. The patient has no obvious contraindication to anticoagulation. Which of the following is the treatment that should be currently pursued? A. Avoidance of thrombolytic treatment, given the patient's age B. Administration of IV fluids C. Administration of aspirin and heparin only D. Administration of thrombolytic therapy, heparin, and aspirin E. Cardiac surgery to bypass the occluded vessel

The correct answer is D. The patient is having an acute ST-segment elevation myocardial infarction. The infarct occurs as a result of an atherosclerotic plaque with thrombus formation, leading to coronary artery obstruction. Lysis with a thrombolytic agent has been shown to decrease mortality from early post-myocardial infarction. Aspirin prevents both platelet aggregation and reocclusion of the reperfused vessels. When aspirin and lytic therapy are given in the setting of ST-segment elevation MI, heparin is added to the regimen to stop new clots from forming. Treatment of ST-segment elevation myocardial infarction is with thrombolytic/fibrinolytic agents, aspirin, and heparin or with PCI and angioplasty. This type of myocardial infarction occurs when the blood supply to the myocardium is insufficient to meet the demands of the cardiac myocytes, which results in cell death. Reperfusion therapy is then necessary to restore as much blood flow to the affected tissues as possible. This reperfusion can be performed via medications or PCI with stenting. Intervention needs to be performed as quickly as possible to salvage as much tissue as possible. If given the choice, it appears that PCI has more favorable outcomes than thrombolytic/fibrinolytic therapy when compared head to head.

A 72-year-old woman with no prior medical history comes to the emergency department because of a 3-hour episode of crushing substernal chest pain. The pain radiates to her arm and neck. An electrocardiogram reveals ST-segment elevation in leads II, III, and aVF. The patient has no obvious contraindication to anticoagulation. Which of the following is the treatment that should be currently pursued? A. Avoidance of thrombolytic treatment, given the patient's age B. Administration of IV fluids C. Administration of aspirin and heparin only D. Administration of thrombolytic therapy, heparin, and aspirin E. Cardiac surgery to bypass the occluded vessel

The correct answer is D. The patient is having an acute ST-segment elevation myocardial infarction. The infarct occurs as a result of an atherosclerotic plaque with thrombus formation, leading to coronary artery obstruction. Lysis with a thrombolytic agent has been shown to decrease mortality from early post-myocardial infarction. Aspirin prevents both platelet aggregation and reocclusion of the reperfused vessels. When aspirin and lytic therapy are given in the setting of ST-segment elevation MI, heparin is added to the regimen to stop new clots from forming. Thrombolytic therapy is indicated in patients up to 75 years of age (choice A). Thrombolytic therapy should be used with caution in the elderly population because of the potential for clearance of the medication and increased risk for bleeding as a result of this decreased clearance. Absolute contraindications to thrombolytic therapy include a bleeding diathesis, previous intracranial bleeding, major surgery or trauma within 6 months, gastrointestinal bleeding, or the presence of aortic dissection or a known intracranial tumor. If the patient is hypotensive, IV fluids may be needed (choice B). The hypotension may result from cardiogenic shock as a result of heart failure from the acute myocardial infarction. At this time, the patient appears to be hemodynamically stable, and the treatment in choice D is more appropriate. If the patient was having a right ventricular infarction, IV fluids would need to be given, because this type of myocardial infarction is volume dependent. Aspirin is a platelet aggregation inhibitor and has been shown to reduce mortality in myocardial infarction and ischemia (choice C). Given alone, however, it will not be as effective in preventing mortality in this patient unless the thrombus is immediately lysed with a thrombolytic or a fibrinolytic agent. Heparin forms a complex with antithrombin III and prevents action by thrombin. It is more effective when given in association with a lytic agent, and it is used to prevent new clot formation after the initial clot is lysed. If medical treatment fails or if the patient develops cardiogenic shock, cardiac surgery for coronary artery bypass graft (choice E) may be required. In the immediate period, the thrombus must be lysed and the patient stabilized. Stent placement and balloon angioplasty may also be explored as options via percutaneous coronary intervention (PCI) and angioplasty. Coronary artery bypass grafting could be performed if PCI fails or the test identifies a surgically amenable lesion. Reviewed on 4/17/14 Treatment of ST-segment elevation myocardial infarction is with thrombolytic/fibrinolytic agents, aspirin, and heparin or with PCI and angioplasty. This type of myocardial infarction occurs when the blood supply to the myocardium is insufficient to meet the demands of the cardiac myocytes, which results in cell death. Reperfusion therapy is then necessary to restore as much blood flow to the affected tissues as possible. This reperfusion can be performed via medications or PCI with stenting. Intervention needs to be performed as quickly as possible to salvage as much tissue as possible. If given the choice, it appears that PCI has more favorable outcomes than thrombolytic/fibrinolytic therapy when compared head to head.

A 45-year-old man suddenly loses consciousness and falls to the ground. His pulse is lost. He has been previously healthy and has been on no medication. There is no obvious evidence of trauma. Electrocardiogram reveals wide complex tachycardia at a rate of 200/min. CK-MB levels are elevated. Which of the following is the most appropriate intervention? A. Obtain vital signs B. Administer a bolus of intravenous lidocaine C. Administer a thrombolytic agent D. Perform defibrillation E. Perform synchronous cardioversion

The correct answer is D. The patient is in ventricular tachycardia and is hemodynamically unstable as illustrated by the loss of consciousness. He should be emergently defibrillated with 200 joules of energy initially. In addition, ventricular tachycardia without a pulse should be treated the same way as ventricular fibrillation. Wide complex ventricular tachycardia of uncertain etiology in a patient who loses consciousness is considered to be unstable ventricular tachycardia. Unstable ventricular tachycardia is treated the same way as ventricular fibrillation, namely treatment with immediate defibrillation. The improved mortality seen in patients who have sudden cardiac death is directly related to the improved access to automatic cardiac defibrillators permitting early defibrillation. Although early institution of cardiopulmonary resuscitation has improved mortality, early defibrillation has been proven to save lives.

A 45-year-old man suddenly loses consciousness and falls to the ground. His pulse is lost. He has been previously healthy and has been on no medication. There is no obvious evidence of trauma. Electrocardiogram reveals wide complex tachycardia at a rate of 200/min. CK-MB levels are elevated. Which of the following is the most appropriate intervention? A. Obtain vital signs B. Administer a bolus of intravenous lidocaine C. Administer a thrombolytic agent D. Perform defibrillation E. Perform synchronous cardioversion

The correct answer is D. The patient is in ventricular tachycardia and is hemodynamically unstable as illustrated by the loss of consciousness. He should be emergently defibrillated with 200 joules of energy initially. In addition, ventricular tachycardia without a pulse should be treated the same way as ventricular fibrillation. He is hemodynamically unstable, as apparent from his loss of consciousness. Precious time will be lost by obtaining vital signs (choice A) if defibrillation can be done immediately. A lidocaine or amiodarone bolus should be given once a pulse and sinus rhythm is obtained to raise the ventricular tachycardia threshold. This is especially beneficial if the event is ischemic in origin (choice B). In this scenario, the patient may be having an infarct, and this may be the cause of his ventricular tachycardia. The health care provider urgently needs to restore sinus rhythm, and then the man will be treated for myocardial infarction if indicated (choice C). If the patient were hemodynamically stable and still had a pulse, synchronous cardioversion could be attempted (choice E). Reviewed on 4/17/14 Wide complex ventricular tachycardia of uncertain etiology in a patient who loses consciousness is considered to be unstable ventricular tachycardia. Unstable ventricular tachycardia is treated the same way as ventricular fibrillation, namely treatment with immediate defibrillation. The improved mortality seen in patients who have sudden cardiac death is directly related to the improved access to automatic cardiac defibrillators permitting early defibrillation. Although early institution of cardiopulmonary resuscitation has improved mortality, early defibrillation has been proven to save lives.

A 56-year-old smoker with no previous medical history comes to the emergency department complaining of chest pain and shortness of breath with exertion. He is admitted to the hospital for further evaluation. An exercise stress test supports the diagnosis of coronary artery disease, and the patient undergoes a heart catheterization. There is diffuse coronary artery disease but no clearly stentable lesions. The cardiologist decides that medical management of this patient's coronary disease is appropriate at this time. The patient has no allergies and no other medical conditions. His blood pressure and pulse over 3 days are as follows: Day 1 blood pressure 146/96 mm Hg, pulse 80/min Day 2 blood pressure 150/90 mm Hg, pulse 86/min Day 3 blood pressure 140/96 mm Hg, pulse 73/min Which is the most appropriate blood pressure medication for this patient? A. Diltiazem (Cardizem, Dilacor, Cartia) B. Hydrochlorothiazide (Microzide) C. Lisinopril (Prinivil, Zestril) D. Metoprolol (Lopressor, Toprol-XL) E. Nifedipine (Adalat, Procardia)

The correct answer is D. There is clear and convincing evidence that beta-blockers are indicated in the treatment of patients who have coronary artery disease. This is partly because in addition to lowering blood pressure, beta-blockers keep the heart rate slow, which decreases strain on the heart by decreasing myocardial oxygen demand and increases myocardial perfusion. Because this patient has coronary artery disease and hypertension, the best choice for this patient is metoprolol. Beta-blockers have been shown to decrease mortality in patients who have acute coronary syndrome (in this case, the patient has unstable angina) and in postmyocardial infarction patients. Its effect is to decrease the heart rate and allow increased perfusion of the cardiac tissue. It also decreases the incidence of arrhythmias.

A 15-month-old boy is brought to the emergency department by his parents because he suddenly turned blue and had difficulty breathing while he was playing in the back yard of their home. The parents have noticed several times in the previous 2-3 months that he had developed a bluish discoloration around the lips but did not think much of it, as it was wintertime. Recently, however, the boy had increasing fatigability and would stop in the middle of playing to catch his breath by sitting down or squatting. This morning when he suddenly turned blue and started behaving in a very flustered manner they rushed him to the emergency department. The parents deny any other significant medical conditions or allergies to medication. They have a 4-year-old daughter who is in good health. On physical examination, the patient is in the fifth percentile for height and weight. His lips and fingertips are bluish in color and he frequently stops to catch his breath while playing in the examination room. There is a hint of clubbing of his fingers. Vital signs are within normal limits, but a complete blood count shows a red blood cell count of 6 x 1012/L and a hematocrit of 66%. A chest radiograph shows a boot-shaped heart with an uptilted apex and clear lung fields. On auscultation, there is a harsh systolic ejection murmur and a single S2 is heard. Which of the following is the most likely diagnosis? A. Endocardial cushion defect B. Myocarditis C. Patent ductus arteriosus D. Tetralogy of Fallot E. Transposition of the great vessels

The correct answer is D. This boy has a congenital heart defect known as tetralogy of Fallot. Tetralogy of Fallot is defined as pulmonary stenosis, ventricular septal defect, dextroposition of the aorta (overriding), and right ventricular hypertrophy. It is the most common type of cyanotic congenital heart disease. Symptoms depend on the size of the ventricular septal defect and the degree of the right ventricular outflow tract obstruction. Acyanotic (pink) tetralogy occurs when there is sufficient pulmonary blood flow caused by mild obstruction and the shunting across the ventricular septal defect is balanced. Typically, however, patients present with cyanosis, delayed growth and development, and dyspnea. Paroxysmal hypercyanotic attacks (hypoxic, blue, or tet spells) manifest with episodes of restlessness, cyanosis, and gasping respirations. Clubbing of the fingers and toes occurs secondary to chronic hypoxia. A loud, harsh, systolic ejection murmur is heard. S2 is single or very soft because of the pulmonary stenosis. Chest radiographs reveal a boot-shaped heart (coeur en sabot) with uptilted apex. Lung fields are clear, reflecting decreased pulmonary blood flow. Electrocardiography shows right ventricular hypertrophy and right-axis deviation. An echocardiogram reveals the anatomic abnormalities. Medical management includes maintaining the ductus open in severe right-sided obstructive lesions, but surgical correction is the definitive treatment. Blue spells are treated by placing the child in knee-chest position, sedation, oxygen, and avoiding acidosis. Beta blockade with propranolol, appropriate fluid status, and maintaining the hematocrit at 55 to 65% are also helpful. Complications have become less frequent as the surgical correction is typically performed early. Cerebral thrombosis occurs with extreme polycythemia and dehydration. Patients are usually younger than 2 years of age. Brain abscess, although less common than thrombosis, is more often seen in patients older than 2 years of age. Tet patients are at higher risk for bacterial endocarditis.

A 15-month-old boy is brought to the emergency department by his parents because he suddenly turned blue and had difficulty breathing while he was playing in the back yard of their home. The parents have noticed several times in the previous 2-3 months that he had developed a bluish discoloration around the lips but did not think much of it, as it was wintertime. Recently, however, the boy had increasing fatigability and would stop in the middle of playing to catch his breath by sitting down or squatting. This morning when he suddenly turned blue and started behaving in a very flustered manner they rushed him to the emergency department. The parents deny any other significant medical conditions or allergies to medication. They have a 4-year-old daughter who is in good health. On physical examination, the patient is in the fifth percentile for height and weight. His lips and fingertips are bluish in color and he frequently stops to catch his breath while playing in the examination room. There is a hint of clubbing of his fingers. Vital signs are within normal limits, but a complete blood count shows a red blood cell count of 6 x 1012/L and a hematocrit of 66%. A chest radiograph shows a boot-shaped heart with an uptilted apex and clear lung fields. On auscultation, there is a harsh systolic ejection murmur and a single S2 is heard. Which of the following is the most likely diagnosis? A. Endocardial cushion defect B. Myocarditis C. Patent ductus arteriosus D. Tetralogy of Fallot E. Transposition of the great vessels

The correct answer is D. This boy has a congenital heart defect known as tetralogy of Fallot. Tetralogy of Fallot is defined as pulmonary stenosis, ventricular septal defect, dextroposition of the aorta (overriding), and right ventricular hypertrophy. It is the most common type of cyanotic congenital heart disease. Symptoms depend on the size of the ventricular septal defect and the degree of the right ventricular outflow tract obstruction. Acyanotic (pink) tetralogy occurs when there is sufficient pulmonary blood flow caused by mild obstruction and the shunting across the ventricular septal defect is balanced. Typically, however, patients present with cyanosis, delayed growth and development, and dyspnea. Paroxysmal hypercyanotic attacks (hypoxic, blue, or tet spells) manifest with episodes of restlessness, cyanosis, and gasping respirations. Clubbing of the fingers and toes occurs secondary to chronic hypoxia. A loud, harsh, systolic ejection murmur is heard. S2 is single or very soft because of the pulmonary stenosis. Chest radiographs reveal a boot-shaped heart (coeur en sabot) with uptilted apex. Lung fields are clear, reflecting decreased pulmonary blood flow. Electrocardiography shows right ventricular hypertrophy and right-axis deviation. An echocardiogram reveals the anatomic abnormalities. Medical management includes maintaining the ductus open in severe right-sided obstructive lesions, but surgical correction is the definitive treatment. Blue spells are treated by placing the child in knee-chest position, sedation, oxygen, and avoiding acidosis. Beta blockade with propranolol, appropriate fluid status, and maintaining the hematocrit at 55 to 65% are also helpful. Complications have become less frequent as the surgical correction is typically performed early. Cerebral thrombosis occurs with extreme polycythemia and dehydration. Patients are usually younger than 2 years of age. Brain abscess, although less common than thrombosis, is more often seen in patients older than 2 years of age. Tet patients are at higher risk for bacterial endocarditis. Endocardial cushion defect (choice A) occurs when an atrial septal defect and ventricular septal defect are present and contiguous, and the atrioventricular valves are also abnormal. Patients who have a large atrial septal defect tend to have heart failure early in infancy, with hepatomegaly and failure to thrive. Increased pulmonary blood flow over time leads to pulmonary vascular disease and Eisenmenger physiology. A low-pitched diastolic murmur may be present and the first heart sound is accentuated. Chest radiographs show a markedly enlarged heart, reflecting enlargement of all chambers. Myocarditis (choice B) is an inflammation of the myocardium most commonly caused by adenovirus and Coxsackie B. Connective tissue diseases, granulomatous diseases, and toxins may cause noninfectious myocarditis. The most common clinical presentation is heart failure. Arrhythmias and sudden death are less common. Viral myocarditis is usually preceded by a viral illness. In these cases, the patient may present with fever, heart failure, respiratory distress, and cyanosis. The erythrocyte sedimentation rate, creatine kinase, and lactate dehydrogenase may all be elevated. Chest radiography shows an enlarged heart and pulmonary edema. Patent ductus arteriosus (choice C) results from failure of closure of the ductus arteriosus postnatally. This results in blood flowing from the aorta to the pulmonary artery. A small patent ductus arteriosus presents with no symptoms. Large patent ductus arteriosus can cause heart failure similar to large ventricular septal defects. An apical heave may be observed, in addition to a trill at the second left intercostal space. The characteristic murmur is described as machinery or to-and-fro murmur heard in systole and diastole. Chest radiographs show a prominent pulmonary artery and increased pulmonary vascular markings. Transposition of the great vessels (choice E) occurs when the aorta arises from the right ventricle and the pulmonary artery from the left ventricle, resulting in a parallel circulation. It is the most common congenital heart disease to present with cyanosis in the first 24 hours of life. Cyanosis and symptoms of heart failure begin within hours to days after birth. The second heart sound may be single and loud. A murmur may or may not be present. Chest roentgenograms demonstrate increased pulmonary blood flow as the pulmonary vascular resistance decreases.

A 70-year-old hypertensive man arrives at the emergency department complaining of shortness of breath. His history is significant for chronic hypertension, paroxysmal nocturnal dyspnea, and nocturia. Physical examination reveals evidence of pulmonary and peripheral edema. The patient is admitted, and furosemide is administered. A low-sodium diet is ordered. The purpose of this dietary restriction is to A. decrease tubular reabsorption of sodium. B. increase extracellular water. C. increase intracellular water. D. reduce extracellular water. E. reduce intracellular water.

The correct answer is D. This hypertensive patient has symptoms and signs of congestive heart failure (paroxysmal nocturnal dyspnea, nocturia, and pulmonary and peripheral edema), for which dietary sodium restriction is recommended to reduce water retention. Most of the sodium in the body exists in locations outside cells, such as plasma, interstitial fluid, cerebrospinal fluid, intraocular fluid, semen, peritoneal fluid, and pleural fluid. The concentration of sodium in these fluids is tightly regulated by the kidney's action on blood plasma. When dietary sodium is severely restricted, the losses in the kidney (and small losses in sweat) exceed the intake, and the total body sodium is decreased. The body then tries to maintain serum sodium concentration by excreting more water via the kidneys. Most of the water comes from the extracellular fluid rather than the intracellular fluid (choice E), because sodium ion is not a quantitatively important component of intracellular fluid. Decreased dietary sodium will increase tubular reabsorption of sodium rather than decreasing it (choice A).

A 5-month-old girl is brought to the office by her mother, who states that the girl had an episode following feeding during which she began to breathe deeply, became blue, and then lost consciousness. The mother states that she picked her up and held her, and the infant regained her usual color and became alert. Physical examination reveals a harsh systolic murmur. The remainder of the physical examination is unremarkable. Which of the following is the most likely diagnosis? A. Aortic stenosis B. Coarctation of the aorta C. Patent ductus arteriosus D. Tetralogy of Fallot E. Ventricular septal defect

The correct answer is D. This infant is experiencing a hypoxemic spell, as seen in patients who have tetralogy of Fallot. These hypercyanotic spells, or "Tet spells," usually are self-limited and last less than 10-15 minutes. The spells often occur immediately after feeding or when the child is crying vigorously. Tetralogy of Fallot has the following components: (1) ventricular septal defect, (2) overriding aorta, (3) right ventricular hypertrophy, and (4) pulmonic stenosis. Tetralogy of Fallot can present with cyanotic spells associated with feeding and/or crying. Episodically there can be near-occlusion of the right ventricular outflow tract with profound cyanosis. These episodes are often referred to as "tet spells" or "hypercyanotic spells."

A 5-month-old girl is brought to the office by her mother, who states that the girl had an episode following feeding during which she began to breathe deeply, became blue, and then lost consciousness. The mother states that she picked her up and held her, and the infant regained her usual color and became alert. Physical examination reveals a harsh systolic murmur. The remainder of the physical examination is unremarkable. Which of the following is the most likely diagnosis? A. Aortic stenosis B. Coarctation of the aorta C. Patent ductus arteriosus D. Tetralogy of Fallot E. Ventricular septal defect

The correct answer is D. This infant is experiencing a hypoxemic spell, as seen in patients who have tetralogy of Fallot. These hypercyanotic spells, or "Tet spells," usually are self-limited and last less than 10-15 minutes. The spells often occur immediately after feeding or when the child is crying vigorously. Tetralogy of Fallot has the following components: (1) ventricular septal defect, (2) overriding aorta, (3) right ventricular hypertrophy, and (4) pulmonic stenosis. Aortic stenosis (choice A) is typically not a cyanotic lesion. Children who have aortic stenosis may develop symptoms of congestive heart failure, but they would not have cyanotic spells. Coarctation of the aorta (choice B) is not a cyanotic heart lesion. Typically there are symptoms of congestive heart failure with this condition. Findings include a systolic murmur that radiates to the left axilla, pulse lag in the lower extremities, blood pressure that is greater in the upper than in the lower extremities, and differential blood pressure differences between the arms. Patent ductus arteriosus (choice C) is not a cyanotic heart lesion. It presents with an active precordium and continuous "machinery" murmur. The pulses are bounding, and pulse pressure is widened. Ventricular septal defect (VSD) (choice E) is not a cyanotic heart lesion. A VSD may or may not present with any problems. The most common finding is decreased feeding and poor weight gain in patients who have large left-to-right shunts. Reviewed on 4/17/14 Tetralogy of Fallot can present with cyanotic spells associated with feeding and/or crying. Episodically there can be near-occlusion of the right ventricular outflow tract with profound cyanosis. These episodes are often referred to as "tet spells" or "hypercyanotic spells."

A 2-month-old infant is evaluated by a pediatric specialist health care provider. The infant was noted at birth to have an upper left sternal border ejection murmur. The infant at that time was not cyanotic but slowly developed cyanosis over the next 2 months. At the time of the examination, an electrocardiogram showed right axis deviation and right ventricular hypertrophy. A chest radiograph showed a small heart with a concave main pulmonary artery segment and diminished pulmonary blood flow. Surgical repair is recommended. Which of the following is the most likely diagnosis? A. Complete atrioventricular canal defect B. Hypoplastic left ventricle C. Isolated atrial septal defect D. Tetralogy of Fallot E. Transposition of the great arteries

The correct answer is D. This is tetralogy of Fallot, in which severe obstruction of right ventricular outflow and a ventricular septal defect allow unoxygenated blood to pass from the right side of the heart to the left. In severe cases, cyanosis presents at birth; in milder cases (such as this infant has), it develops more slowly. The upper left sternal border ejection murmur is caused by right ventricle outflow obstruction. The ECG and chest radiographic findings described in the question stem are typical for older infants who have tetralogy of Fallot. Early surgical repair is now recommended for tetralogy of Fallot. Tetralogy of Fallot is one of the most common forms of cyanotic congenital heart disease. There are four abnormalities associated with this condition: VSD, right ventricular outflow obstruction, overriding aorta, and right ventricular hypertrophy. Most patients will have progressive cyanosis after birth, with dyspnea on exertion in children if not identified and treated. Failure to identify and treat these patients will result in profound hypoxemia and secondary erythrocytosis, seizures, and death.

A 2-month-old infant is evaluated by a pediatric specialist health care provider. The infant was noted at birth to have an upper left sternal border ejection murmur. The infant at that time was not cyanotic but slowly developed cyanosis over the next 2 months. At the time of the examination, an electrocardiogram showed right axis deviation and right ventricular hypertrophy. A chest radiograph showed a small heart with a concave main pulmonary artery segment and diminished pulmonary blood flow. Surgical repair is recommended. Which of the following is the most likely diagnosis? A. Complete atrioventricular canal defect B. Hypoplastic left ventricle C. Isolated atrial septal defect D. Tetralogy of Fallot E. Transposition of the great arteries

The correct answer is D. This is tetralogy of Fallot, in which severe obstruction of right ventricular outflow and a ventricular septal defect allow unoxygenated blood to pass from the right side of the heart to the left. In severe cases, cyanosis presents at birth; in milder cases (such as this infant has), it develops more slowly. The upper left sternal border ejection murmur is caused by right ventricle outflow obstruction. The ECG and chest radiographic findings described in the question stem are typical for older infants who have tetralogy of Fallot. Early surgical repair is now recommended for tetralogy of Fallot. Complete atrioventricular canal defect (choice A) characteristically has ECG findings including superior left axis deviation and a counterclockwise loop of electrical impulses. Hypoplastic left ventricle (choice B) causes severe heart failure with loss of peripheral pulses at 2-3 days of life. It does not typically result in cyanosis. Isolated atrial septal defect (choice C) could cause a murmur heard at the upper left border of the sternum, but it would not show the characteristic radiographic findings illustrated in the question stem. This condition does not cause cyanosis. Transposition of the great arteries (choice E) is characterized by severe cyanosis from birth and a normal ECG. Patients who have this disorder present within a few hours of birth with significant cyanosis, which is considered to be a medical emergency. Reviewed on 4/17/14 Tetralogy of Fallot is one of the most common forms of cyanotic congenital heart disease. There are four abnormalities associated with this condition: VSD, right ventricular outflow obstruction, overriding aorta, and right ventricular hypertrophy. Most patients will have progressive cyanosis after birth, with dyspnea on exertion in children if not identified and treated. Failure to identify and treat these patients will result in profound hypoxemia and secondary erythrocytosis, seizures, and death.

A 4,000-g male neonate develops severe cyanosis that begins within minutes of birth. Blood drawn 1 hour after birth shows metabolic acidosis with respiratory acidosis. A chest radiograph shows a narrow mediastinum, narrow heart base, and absence of the pulmonary artery. An electrocardiogram is normal. An echocardiogram is ordered and the report is pending. Which of the following is the most likely diagnosis? A. Aortic valve stenosis B. Complete atrioventricular canal defect C. Tetralogy of Fallot D. Transposition of the great arteries E. Underdeveloped (hypoplastic) left ventricle syndrome

The correct answer is D. This is transposition of the great arteries, in which the aorta arises from the right ventricle and the pulmonary artery arises from the left ventricle. Approximately 5% of congenital cardiac anomalies have transposition of the great arteries. Affected infants present within minutes of birth with severe cyanosis and metabolic acidosis secondary to inability to oxygenate tissues. The only exchange of blood between pulmonic and circulatory systems is typically occurring through a patent ductus arteriosus. The chest radiograph changes include a narrow mediastinum, narrow heart base, and absence of the pulmonary artery; these are caused by superposition of the great vessels (rather than the normal side-to-side position). Surgical repair is usually performed within 7 to 10 days of life. In transposition of great vessels, because the aorta is coming out of the right ventricle it will completely cover the pulmonary trunk. This will be seen as absence of the pulmonary artery with a narrow mediastinum on chest radiograph ("egg on a string" appearance). With this transposition, the pulmonary artery comes out of the left ventricle. Because of these changes, deoxygenated systemic venous blood travels from the right ventricle to the aorta without passing through the lungs, and pulmonary venous blood travels from the left ventricle to the lungs via the pulmonary artery. The systemic and pulmonary circulations parallel each other rather than occur sequentially. This is not compatible with life unless the two chambers connect via VSD, ASD, or patent ductus arteriosus. Prostaglandin E1 is used to maintain patency of the ductus arteriosus. An arterial switch operation is performed in the newborn period most commonly.

A 15-year-old boy is concerned about multiple stretch marks that have developed on his chest, shoulders, and thighs over the previous 3 to 4 years. He plays center on his high school basketball team and is annoyed by the appearance of the skin lesions. His medical history is remarkable for myopia, upward lens displacement, and mitral valve prolapse. He is quite tall for his age and has long, thin extremities and fingers. He is accompanied by his father, who has the same tall stature and thin extremities. The skin lesions this young man is worried about are multiple striae in the pectoral, deltoid, and thigh regions. His chest is flattened on either side with forward projection of the sternum, and he has long, tapering fingers and toes. Which of the following is the most likely cause of his condition? A. Cushing disease B. Exogenous androgen administration C. Hyperhomocysteinemia D. A genetic defect in the fibrillin gene E. Ehlers-Danlos syndrome

The correct answer is D. This patient has Marfan syndrome, an autosomal dominant inherited genetic defect of fibrillin. Abnormalities appear in 3 organ systems: the eye, the skeletal system, and the cardiovascular system. Skin manifestations may consist of striae (stretch marks). The skeletal features include long, narrow extremities and tapering fingers and toes. The patients are typically unusually tall. They may also have pectus excavatum, pectus carinatum, flat feet, and hyperextensibility of the knees and other joints. The ocular findings include long orbits causing myopia, and ectopia lentis (with upward displacement in 70%) sometimes causing acute glaucoma because of drainage obstruction secondary to lens displacement. Cardiovascular defects consist of weakness of the aortal intima leading to aneurysms and mitral valve prolapse. Acute dissection of the aorta is a common cause of death in these patients. Marfan syndrome is an autosomal-dominant genetic disorder characterized by a defect in the fibrillin gene. The most common clinical manifestations include loose skin, hyperextensible joints, tall stature, ectopia lentis, and long, thin extremities.

A 15-year-old boy is concerned about multiple stretch marks that have developed on his chest, shoulders, and thighs over the previous 3 to 4 years. He plays center on his high school basketball team and is annoyed by the appearance of the skin lesions. His medical history is remarkable for myopia, upward lens displacement, and mitral valve prolapse. He is quite tall for his age and has long, thin extremities and fingers. He is accompanied by his father, who has the same tall stature and thin extremities. The skin lesions this young man is worried about are multiple striae in the pectoral, deltoid, and thigh regions. His chest is flattened on either side with forward projection of the sternum, and he has long, tapering fingers and toes. Which of the following is the most likely cause of his condition? A. Cushing disease B. Exogenous androgen administration C. Hyperhomocysteinemia D. A genetic defect in the fibrillin gene E. Ehlers-Danlos syndrome

The correct answer is D. This patient has Marfan syndrome, an autosomal dominant inherited genetic defect of fibrillin. Abnormalities appear in 3 organ systems: the eye, the skeletal system, and the cardiovascular system. Skin manifestations may consist of striae (stretch marks). The skeletal features include long, narrow extremities and tapering fingers and toes. The patients are typically unusually tall. They may also have pectus excavatum, pectus carinatum, flat feet, and hyperextensibility of the knees and other joints. The ocular findings include long orbits causing myopia, and ectopia lentis (with upward displacement in 70%) sometimes causing acute glaucoma because of drainage obstruction secondary to lens displacement. Cardiovascular defects consist of weakness of the aortal intima leading to aneurysms and mitral valve prolapse. Acute dissection of the aorta is a common cause of death in these patients. Cushing disease (choice A) will produce atrophic skin, vascular fragility leading to ecchymoses and hematomas after minor trauma, striae, and plethora with telangiectases, as well as hypertrichosis and acne. These patients are not unusually tall nor do they have wide arm diameter measurements. Ocular lens displacement is not seen, but premature formation of cataracts is characteristic. Buffalo hump and moon facies develop secondary to fat redistribution (thin extremities with a large waist; buffalo hump and moon facies are typical). Exogenous androgen administration (choice B) may lead to coarse skin, acne, and hypertrichosis of the face, axillae, and pubic areas, along with hyperpigmentation of the perineum, axillae, areolae, and nipples. Patients using these agents may also have small testes and gynecomastia. Hyperhomocysteinemia (choice C) is classically associated with deficiency of folate; this condition may be a risk factor for cardiovascular disease. Ehlers-Danlos syndrome (choice E) is an inherited disease of connective tissue. This is a genetically determined disease of collagen that occurs because of reduced activity of lysyl hydroxylase. It is characterized by joint laxity and hyperextensibility of the skin. These patients are not typically unusually tall and do not normally have arachnodactyly. Reviewed on 4/17/14 Marfan syndrome is an autosomal-dominant genetic disorder characterized by a defect in the fibrillin gene. The most common clinical manifestations include loose skin, hyperextensible joints, tall stature, ectopia lentis, and long, thin extremities.

A 22-year-old man comes to the emergency department with a 3-day history of fever, chills, a cough, pleuritic chest pain, and low back pain. He says that the symptoms came on "out of the blue." He is the son of a wealthy local businesswoman and still lives at home, which he says "is cool because my parents are never around." His temperature is 39°C (102.2°F), blood pressure 120/80 mm Hg, pulse 70/min, and respirations 16/min. Physical examination shows oval retinal hemorrhages with a clear, pale center and pinpoint lesions between his toes. Blood cultures are drawn. A chest radiograph shows multiple patchy infiltrates. Laboratory studies show hemoglobin 11 g/dL, hematocrit 39%, and erythrocyte sedimentation rate 39 mm/h. Which of the following is the most likely pathogen? A. Candida albicans B. Pseudomonas aeruginosa C. Serratia marcescens D. Staphylococcus aureus E. Streptococcus viridans

The correct answer is D. This patient has acute bacterial endocarditis, most likely caused by Staphylococcus aureus, the most common organism causing endocarditis in intravenous drug abusers. The "pinpoint lesions" between his toes are signs of injection drug abuse. Acute endocarditis in drug abusers typically presents with a high fever, pleuritic chest pain, and a cough. The tricuspid valve is commonly affected in these patients. A murmur may not be present in early acute endocarditis or in injection drug abusers who have tricuspid valve disease. Roth spots are oval, pale, retinal lesions that are surrounded by hemorrhages. Other physical manifestations of endocarditis include Janeway lesions, which are hemorrhagic, painless, macular plaques typically located on the palms and soles, and Osler nodes, which are small, painful nodular lesions typically found on the pads of the fingers or toes. Other findings include anemia and an elevated erythrocyte sedimentation rate. Diagnosis is with blood cultures, which are typically positive for S. aureus, and with echocardiography. Treatment is with antibiotics.

A 22-year-old man comes to the emergency department with a 3-day history of fever, chills, a cough, pleuritic chest pain, and low back pain. He says that the symptoms came on "out of the blue." He is the son of a wealthy local businesswoman and still lives at home, which he says "is cool because my parents are never around." His temperature is 39°C (102.2°F), blood pressure 120/80 mm Hg, pulse 70/min, and respirations 16/min. Physical examination shows oval retinal hemorrhages with a clear, pale center and pinpoint lesions between his toes. Blood cultures are drawn. A chest radiograph shows multiple patchy infiltrates. Laboratory studies show hemoglobin 11 g/dL, hematocrit 39%, and erythrocyte sedimentation rate 39 mm/h. Which of the following is the most likely pathogen? A. Candida albicans B. Pseudomonas aeruginosa C. Serratia marcescens D. Staphylococcus aureus E. Streptococcus viridans

The correct answer is D. This patient has acute bacterial endocarditis, most likely caused by Staphylococcus aureus, the most common organism causing endocarditis in intravenous drug abusers. The "pinpoint lesions" between his toes are signs of injection drug abuse. Acute endocarditis in drug abusers typically presents with a high fever, pleuritic chest pain, and a cough. The tricuspid valve is commonly affected in these patients. A murmur may not be present in early acute endocarditis or in injection drug abusers who have tricuspid valve disease. Roth spots are oval, pale, retinal lesions that are surrounded by hemorrhages. Other physical manifestations of endocarditis include Janeway lesions, which are hemorrhagic, painless, macular plaques typically located on the palms and soles, and Osler nodes, which are small, painful nodular lesions typically found on the pads of the fingers or toes. Other findings include anemia and an elevated erythrocyte sedimentation rate. Diagnosis is with blood cultures, which are typically positive for S. aureus, and with echocardiography. Treatment is with antibiotics. Candida albicans (choice A), Pseudomonas aeruginosa (choice B), and Serratia marcescens (choice C) are infrequent causes of endocarditis. Streptococcus viridans (choice E) is a common cause of endocarditis in individuals who are not injection drug abusers. The onset of symptoms is usually more gradual, patients present with a low-grade fever, new cardiac murmur, splenomegaly, hematuria, proteinuria, and an elevated erythrocyte sedimentation rate. Reviewed on 4/17/14 Infective endocarditis is an infection that involves the endocardial surface of the heart and includes the valves. If the patient is symptomatic, presentation includes fever, tachycardia, and fatigue. The patient may have a new-onset heart murmur or a change in the previously heard heart murmur. Echocardiogram should be performed in all patients suspected of having endocarditis. Three sets of blood cultures should be performed to identify the pathogen. Injection drug users are at high risk for Staphylococcus aureus infection leading to infective endocarditis.

A 16-year-old boy is brought to the urgent care clinic with a temperature of 38.4°C (101°F) and low back, wrist, and knee pain. He had a sore throat 1 month earlier. His arthritis is diffuse. Pea-sized swellings are noted over the skin on his knees. He has a serpiginous erythematous area on his anterior trunk. His blood and throat cultures are negative, and his CBC is unremarkable. His antistreptolysin-O (ASO) titer is high. Which of the following is the most appropriate therapy? A. Acetaminophen B. Aspirin C. Penicillin D. Penicillin and aspirin E. Supportive care

The correct answer is D. This patient has acute rheumatic fever from group A streptococci. He has migratory polyarthritis, erythema marginatum, and subcutaneous nodules. Other features absent in this patient are chorea and carditis. His ASO titer indicates recent infection with Streptococcus. It is advisable to administer penicillin for the infection. The arthritis can be managed with salicylates. The patient will need long-term treatment with penicillin as a result of his rheumatic fever. Rheumatic fever is treated with penicillin (for the bacterial infection) and aspirin (for the arthralgias). Remember, aspirin is not indicated in viral infections because of the potential to cause Reye syndrome. Penicillin that is given within 9 days of infection is most likely to prevent rheumatic fever.

The EKG of a 60-year-old man reveals widened QRS intervals of 0.14 seconds with distinctly abnormal configurations. Physical examination is significant for paradoxic splitting of the second heart sound. Which of the following valvular defects is likely in this patient? A. Mitral valve prolapse B. Mitral stenosis C. Pulmonic stenosis D. Aortic stenosis E. Aortic regurgitation

The correct answer is D. This patient has bundle branch block, as implied by the QRS interval greater than 0.12 seconds and by paradoxic splitting of the second heart sound. The typical sequence of valve closure is mitral, tricuspid, aortic, and pulmonic. If there is a paradoxic splitting of the second heart sound, the aortic valve closes after the pulmonic valve. This situation can occur with anything that slows the conduction in the left ventricle, such as complete heart block, left bundle branch block, or aortic stenosis. Left bundle branch block results in conduction occurring in a cell-to-cell depolarization pattern that slows the conduction through the left ventricle, which causes the aortic valve to close after the pulmonic valve. Aortic stenosis results in prolonged left ventricular contraction pattern, which results in delayed aortic valve closure and a resultant paradoxic splitting of the S2 heart sound. This increased left ventricular force of contraction (needed to overcome the outflow resistance caused by the stenotic aortic valve) results in widening of the QRS complex. A coexisting left bundle branch block in the setting of aortic stenosis accentuates the paradoxic splitting of the second heart sound.

A 57-year-old woman with a history of rheumatic fever as a child comes to the her health care provider complaining of a 6-month history of slowly progressive dyspnea on exertion and orthopnea. Her temperature is 37°C (98.6°F), blood pressure is 110/60 mm Hg, pulse is 93/min and irregular, and respirations are 18/min. Cardiac examination reveals a localized mid-diastolic murmur near the apex. There is a loud opening snap heard after S2. The rhythm appears irregular. Which of the following additional findings will most likely be present on physical examination? A. Delayed carotid upstroke B. De Musset sign C. Corrigan pulse D. Decreased S1 intensity E. Large A-wave

The correct answer is D. This patient has mitral stenosis. Most adults who have mitral stenosis have had rheumatic fever as a child, although not all patients are aware of having had this infection. Mitral stenosis decreases left ventricular filling and elevates left-sided atrial pressures. This causes pulmonary congestion and results in symptoms of left-sided heart failure, such as shortness of breath and dyspnea on exertion. Hemoptysis sometimes occurs as a result of rupture of small pulmonary blood vessels. Later in the course of the disease, patients may develop pulmonary hypertension and cor pulmonale because the right ventricle has to work against increased pressures secondary to chronic pulmonary congestion. There may be a loud P2 as a sign of pulmonary hypertension and/or right ventricular failure. Patients may have an opening snap caused by the calcification of the stenotic mitral valve. Mitral stenosis is a harsh mid-diastolic murmur heard best at the apex of the heart. Shortening of the interval before the opening snap indicates a higher degree of stenosis (the pressures in the left atrium force the valve open earlier). Atrial fibrillation is common with mitral stenosis. Early in the course of the disease, the intensity of S1 is increased. This is because the stenosis prevents the valve from closing spontaneously after diastole. Late in the course of the disease, the valve becomes too stenotic to open or close, and therefore the intensity of S1 decreases.

A 75-year-old man with angina pectoris has recurrent episodes of atrial tachycardia (240/min). A rapid sequence of normal QRS waves is seen on EKG. The episodes are controllable by the patient's performance of vagal maneuvers. Which of the following is the most likely etiology of this arrhythmia? A. Atrial reentry B. Automatic atrial conduction C. AV dissociation D. AV nodal reentry E. Wandering atrial pacemaker

The correct answer is D. This patient has paroxysmal supraventricular tachycardia (PSVT), which is a regular, rapid (150-250/min) arrhythmia originating in the atria or AV node. AV nodal reentry is the most common cause of this arrhythmia (about 70% of patients). In this condition, the AV node is pathologically divided into two functional pathways. The electrical impulse usually proceeds anterograde down the slow pathway and retrograde up the fast pathway. The P waves are recorded nearly simultaneously with the QRS complexes (which occur in rapid sequence) and are therefore obscured on EKG. This arrhythmia is commonly seen in older patients, about half of whom have underlying heart disease. Reentry PSVTs can be reverted to normal sinus rhythm by interrupting the reentry pathway. For example, the performance of vagal maneuvers often improves the condition by increasing AV nodal refractoriness.

A 50-year-old man is brought to the emergency department complaining of light-headedness. He has a history of lung cancer, which was diagnosed a month ago and found to be widely metastatic to the bone and pericardium. On physical examination his blood pressure is 70/40 mm Hg and pulse 100/min. Heart sounds are distant and soft. ECG demonstrates low voltage, and electrical alternans is present. Chest radiograph shows that the cardiac silhouette has a "water bottle" appearance. Which of the following is the most appropriate intervention? A. Beta-blockers B. Nonsteroidal anti-inflammatory drugs C. Steroids D. Pericardiocentesis E. Cardiac catheterization

The correct answer is D. This patient has pericardial/cardiac tamponade, most likely as a result of his malignancy. Lung cancer is particularly likely to cause pericardial effusions. Furthermore, because this patient has metastases to the pericardium, he might be bleeding into the pericardial space. This tamponade may be the cause of his significant hypotension and the soft cardiac sounds. Electrical alternans, a phenomenon in which the QRS changes axis, is indicative of pericardial effusion, because the heart is moving freely in the fluid, causing this change in axis noted on the ECG. Emergently, this patient needs decompression of the pericardial space with the aid of pericardiocentesis, in which a catheter directly drains the fluid in the pericardial sac.

A 50-year-old man is brought to the emergency department complaining of light-headedness. He has a history of lung cancer, which was diagnosed a month ago and found to be widely metastatic to the bone and pericardium. On physical examination his blood pressure is 70/40 mm Hg and pulse 100/min. Heart sounds are distant and soft. ECG demonstrates low voltage, and electrical alternans is present. Chest radiograph shows that the cardiac silhouette has a "water bottle" appearance. Which of the following is the most appropriate intervention? A. Beta-blockers B. Nonsteroidal anti-inflammatory drugs C. Steroids D. Pericardiocentesis E. Cardiac catheterization

The correct answer is D. This patient has pericardial/cardiac tamponade, most likely as a result of his malignancy. Lung cancer is particularly likely to cause pericardial effusions. Furthermore, because this patient has metastases to the pericardium, he might be bleeding into the pericardial space. This tamponade may be the cause of his significant hypotension and the soft cardiac sounds. Electrical alternans, a phenomenon in which the QRS changes axis, is indicative of pericardial effusion, because the heart is moving freely in the fluid, causing this change in axis noted on the ECG. Emergently, this patient needs decompression of the pericardial space with the aid of pericardiocentesis, in which a catheter directly drains the fluid in the pericardial sac. Beta-blockers (choice A) would be of no benefit in treating cardiac tamponade. Nonsteroidal anti-inflammatory drugs (NSAIDs) (choice B) can be useful in treating pericarditis, which may lead to pericardial effusions. This is a longer term option, however, and will have little utility emergently. Steroids (choice C) may similarly be used in pericarditis, after NSAIDs have failed. This is an option to be explored after the pericardial fluid has been drained, however. Cardiac catheterization is often used to confirm the diagnosis of tamponade (choice E). Typically the pressure equalizes across the right atrium and ventricle. Emergently, however, this patient should have pericardiocentesis. Reviewed on 4/17/14 Cardiac tamponade results from the accumulation of pericardial fluid, blood, pus, or air within the pericardial space that creates an increase in the intrapericardial pressure, which restricts cardiac filling and cardiac output. Cardiac/pericardial tamponade is a medical emergency that needs immediate treatment. Initial treatment can be with needle pericardiocentesis or creation of a pericardial window with ongoing drainage of the effusion with a catheter. Cancer is a condition that is associated with pericardial effusion and pericardial constriction, which may be complicated by cardiac tamponade. Clinical signs include tachycardia, hypotension, distant heart sounds, elevated jugular pressure, and pulsus paradoxus. Echocardiogram readily confirms this diagnosis.

A 67-year-old woman comes to the clinic for review of her medications. She has had a history of hypertension for 25 years, type 2 diabetes for 20 years, and congestive heart failure for 5 years. She is a former smoker of two packs of cigarettes per day and her lipid status is not known at this time. Her current medications include nifedipine (Adalat, Procardia), hydralazine (Apresoline), isosorbide dinitrate (Isordil), glyburide (DiaBeta, Micronase), a multivitamin, and conjugated estrogens. Today in the clinic her blood pressure is 160/90 mm Hg, her fasting blood glucose is 210 mg/dL, and her hemoglobin A1c is 7.9%. She reports moderate dyspnea on exertion, unchanged from previous visits. Which of the following is the most appropriate intervention at this time? A. Increase her glyburide dosage B. Increase her nifedipine dosage C. Discontinue her hydralazine therapy D. Discontinue the hydralazine and add captopril E. Begin simvastatin therapy

The correct answer is D. This patient has poorly controlled hypertension and poorly controlled diabetes. She needs improved therapy for both; the issue is how best to do that. The concept underlying this question is the absolute importance of using ACE inhibitor therapy on both type 1 and type 2 diabetic patients. Many clinical trials have shown the beneficial effects of ACE inhibitors on preventing nephropathy and slowing the progression of established nephropathy in diabetics. It is the standard of care that all diabetics be given an ACE inhibitor if they are able to tolerate its blood pressure effects. Given that she has congestive heart failure and hypertension, the ACE inhibitor also will be efficacious in their treatment. In fact, ACE inhibitors have been shown to be superior to hydralazine and isosorbide dinitrate in terms of morbidity and mortality in treatment of CHF; all patients who have symptomatic CHF, regardless of ejection fraction, should be placed on one. Diabetic patients and patients who have chronic kidney disease, coronary artery disease, peripheral vascular disease, carotid artery disease, or abdominal aortic aneurysm should have a lower blood pressure goal of 130/80 mm Hg according to the American Heart Association. ACE inhibitors (ARBs if ACEs are not tolerated) are preferentially given as first-line agents because of their efficacy in limiting renal disease in this high-risk population. ACE inhibitors are also first-line agents for patients who have chronic heart failure or left ventricular failure.

A 55-year-old man is brought to the emergency department because of increasing incoherence over the past 24 hours. He has a history of hypertension and diabetes. He is disoriented. On physical examination, he is afebrile. His blood pressure is 230/130 mm Hg, pulse is 120/min, and respirations are 24/min. He has an S4 on cardiac examination. The patient is placed on a cardiac monitor, and IV and intra-arterial lines are placed. A head CT scan shows no mass or bleed. Which of the following is the most appropriate next step in management? A. Observe in a quiet room B. Check chemistry for an anion gap C. Administer a diuretic D. Administer labetolol (Trandate) E. Perform a lumbar puncture

The correct answer is D. This patient is having a hypertensive emergency, given the presence of end-organ damage (mental status changes) in the setting of hypertension. Hypertensive encephalopathy allows the definition of hypertensive emergency rather than hypertensive urgency to be used. Immediate therapy is needed, before the laboratory results are known. This requires immediate but not precipitous lowering of the blood pressure over a period of minutes to hours. Labetolol acts as a combined alpha- and beta-blocker and will decrease systemic vascular resistance, mean arterial pressure, and heart rate with minimal change in cardiac output. It is given intravenously with an onset of action in 5 to 10 minutes with a duration of action of 3 to 8 hours. Hypertensive emergency is defined as a systolic blood pressure >210 mm Hg and diastolic blood pressure >130 mm Hg along with changes consistent with end-organ damage. When this condition is suspected, treatment should be performed without delay. Blood pressure should be lowered within minutes to hours with parenteral agents used in a monitored setting. The initial goal of therapy is to lower mean arterial blood pressure by no more than 25% within the first hour. Further blood pressure lowering may be needed in patients who have intracranial hemorrhage or aortic dissection. If the patient's blood pressure is lowered too rapidly there may be an increase in renal, cerebral, or coronary ischemia.

A 55-year-old man is brought to the emergency department because of increasing incoherence over the past 24 hours. He has a history of hypertension and diabetes. He is disoriented. On physical examination, he is afebrile. His blood pressure is 230/130 mm Hg, pulse is 120/min, and respirations are 24/min. He has an S4 on cardiac examination. The patient is placed on a cardiac monitor, and IV and intra-arterial lines are placed. A head CT scan shows no mass or bleed. Which of the following is the most appropriate next step in management? A. Observe in a quiet room B. Check chemistry for an anion gap C. Administer a diuretic D. Administer labetolol (Trandate) E. Perform a lumbar puncture

The correct answer is D. This patient is having a hypertensive emergency, given the presence of end-organ damage (mental status changes) in the setting of hypertension. Hypertensive encephalopathy allows the definition of hypertensive emergency rather than hypertensive urgency to be used. Immediate therapy is needed, before the laboratory results are known. This requires immediate but not precipitous lowering of the blood pressure over a period of minutes to hours. Labetolol acts as a combined alpha- and beta-blocker and will decrease systemic vascular resistance, mean arterial pressure, and heart rate with minimal change in cardiac output. It is given intravenously with an onset of action in 5 to 10 minutes with a duration of action of 3 to 8 hours. Observation (choice A) is not appropriate for this patient, because he has an emergent medical condition that needs to be addressed. This would be considered a harmful response. Diabetic ketoacidosis or sepsis (choice B) may present with mental status changes and an anion gap. The patient most likely has hypertensive encephalopathy, however, and this needs to be addressed first. A diuretic (choice C) may reduce the intravascular volume and act as a minor vasodilator but will not cause enough blood pressure lowering compared to labetolol. It may be used as an adjunct therapy, especially if signs of left ventricular overload are seen. The patient has mental status changes, and meningitis is on the differential. Management of the blood pressure should not be delayed, however, to perform a lumbar puncture (choice E). Furthermore, he is severely hypertensive with signs of hypertensive emergency, placing meningitis lower on the differential. Reviewed on 4/17/14 Hypertensive emergency is defined as a systolic blood pressure >210 mm Hg and diastolic blood pressure >130 mm Hg along with changes consistent with end-organ damage. When this condition is suspected, treatment should be performed without delay. Blood pressure should be lowered within minutes to hours with parenteral agents used in a monitored setting. The initial goal of therapy is to lower mean arterial blood pressure by no more than 25% within the first hour. Further blood pressure lowering may be needed in patients who have intracranial hemorrhage or aortic dissection. If the patient's blood pressure is lowered too rapidly there may be an increase in renal, cerebral, or coronary ischemia.

Which of the following is an advantage of tissue plasminogen activator (tPA) over streptokinase for fibrinolytic therapy? A. It can be used in the setting of acute myocardial infarction B. It cannot cause hemorrhage C. It is less expensive D. It is not likely to produce an allergic reaction E. It results in the activation of plasminogen

The correct answer is D. Tissue plasminogen activator is produced by and secreted from endothelial cells. Because it is not a foreign protein like streptokinase (derived from hemolytic streptococci), tPA is not allergenic. Both drugs can be used in the setting of acute myocardial infarction (choice A). Both drugs can result in hemorrhage (choice B), especially during prolonged therapy for treatment of pulmonary embolism or venous thrombosis. tPA is as much as 10 times more expensive (choice C) than streptokinase. Both drugs result in the activation of plasminogen (choice E). Thrombolytic therapy such as tPA is used in the setting of acute ST-segment elevation myocardial infarction if percutaneous intervention cannot be done and provided the patient has no contraindications for its use. Ideally drug-to-door therapy should be given within 30 minutes of patient presentation to the emergency department setting. This medication is most effective at lysing the clot if it is given within 12 hours of onset of chest pain, although studies are showing some benefit up to 24 hours after the initial onset of chest pain if the patient is having ongoing pain. The most commonly used agents in practice are reteplace (Retavase), altevase (Activase), and tenecteplase (TNKase), which act to convert tissue plasminogen to plasmin, which promotes fibrinolysis.

Which of the following is an advantage of tissue plasminogen activator (tPA) over streptokinase for fibrinolytic therapy? A. It can be used in the setting of acute myocardial infarction B. It cannot cause hemorrhage C. It is less expensive D. It is not likely to produce an allergic reaction E. It results in the activation of plasminogen

The correct answer is D. Tissue plasminogen activator is produced by and secreted from endothelial cells. Because it is not a foreign protein like streptokinase (derived from hemolytic streptococci), tPA is not allergenic. Both drugs can be used in the setting of acute myocardial infarction (choice A). Both drugs can result in hemorrhage (choice B), especially during prolonged therapy for treatment of pulmonary embolism or venous thrombosis. tPA is as much as 10 times more expensive (choice C) than streptokinase. Both drugs result in the activation of plasminogen (choice E). Reviewed on 4/17/14 Thrombolytic therapy such as tPA is used in the setting of acute ST-segment elevation myocardial infarction if percutaneous intervention cannot be done and provided the patient has no contraindications for its use. Ideally drug-to-door therapy should be given within 30 minutes of patient presentation to the emergency department setting. This medication is most effective at lysing the clot if it is given within 12 hours of onset of chest pain, although studies are showing some benefit up to 24 hours after the initial onset of chest pain if the patient is having ongoing pain. The most commonly used agents in practice are reteplace (Retavase), altevase (Activase), and tenecteplase (TNKase), which act to convert tissue plasminogen to plasmin, which promotes fibrinolysis.

A 68-year-old man is brought to the emergency department with excruciating back pain that began suddenly 45 minutes ago. The pain is constant and is not exacerbated by sneezing or coughing. He is diaphoretic and has a systolic blood pressure of 90 mm Hg. There is an 8-cm pulsatile mass deep in his epigastrium, above the umbilicus. A chest radiograph is unremarkable. Two years ago, he was diagnosed with prostatic cancer and was treated with orchiectomy and radiation. At that time his blood pressure was high, for which he declined treatment. Which of the following is the most likely diagnosis? A. Dissecting thoracic aortic aneurysm B. Fracture of lumbar pedicles with cord compression C. Herniated disc D. Metastatic tumor to the lumbar spine E. Rupturing abdominal aortic aneurysm

The correct answer is E. Abdominal aortic aneurysms have a high incidence of rupture once they reach or exceed a size of 5.5 cm in males and 5.0 cm in females. Often the first manifestation is excruciating back pain, as the blood leaks into the retroperitoneal space before the aneurysm blows out into the peritoneal cavity. The combination of a big pulsatile mass and sudden severe back pain should always lead to this presumptive diagnosis. Looking for orthopedic or neurologic explanations can be a deadly mistake. In any patient who has a rupturing aneurysm or who is hemodynamically unstable, immediate surgical intervention is required. If the patient was hemodynamically stable, abdominal ultrasound is the next best step. An abdominal ultrasound is a good choice for a preliminary determination of aneurysm presence, size, and extent, and it is cost-effective for this purpose. CT scan can be done later to assess a more detailed anatomy of the lesion and any other intra-abdominal pathology. Treatment will depend on aneurysm size; most health care providers agree to operate after size surpasses 5.5 cm in males and 5.0 cm in females. Angiography and CT scan remain the gold standard for planning the surgery prior to elective surgical resection.

A 71-year-old man comes to the hospital with an episode of bright red blood per rectum. The patient reports that, a few hours ago, he passed a grossly bloody bowel movement. The passage was associated with some cramping lower abdominal pain. The patient's past medical history is significant for coronary artery disease and a myocardial infarction 3 years ago. He is poorly compliant with his beta-blocker and diuretic therapy, and his blood pressures have run around 140/85 mm Hg. While the patient is in the hospital, he has another episode of large volume, bright red blood per rectum. His blood pressure is 100/60 mm Hg while supine, and his pulse is 120/min. His hematocrit is 28%. He then begins to complain of substernal chest tightness radiating to his left shoulder. An electrocardiogram shows new T-wave inversions in the anterior leads. Which of the following is the most appropriate intervention at this juncture? A. Administration of aspirin by mouth B. Administration of a beta-blocking agent C. Administration of nitroglycerin sublingually D. Administration of nitroglycerin topically E. Blood transfusion

The correct answer is E. According to the available data, the patient is presenting with myocardial ischemia (as manifested by his angina) in the presence of anemia, which is caused by his acute bleeding secondary to diverticulosis. The appropriate treatment, therefore, is a blood transfusion, because he is not tolerating the acute blood loss.

A 68-year-old Latin woman is admitted to the medical service for evaluation of her difficult to control hypertension. She has been treated for hypertension for 3 years by her primary care health care provider. She has a documented intolerance to ACE inhibitors manifested by a rapid decline in her renal function. She also has had two episodes of acute pulmonary edema in the past. She has ceased smoking, has altered her diet, and claims she is compliant with her medications. At her last office visit, a mildly elevated creatinine of 1.2 mg/dL was noted, as was microscopic hematuria. Physical examination is remarkable for a blood pressure of 180/100 mm Hg, a prominent apical impulse, and an abdominal examination that reveals audible bruits. Laboratory studies are as follows: Hematocrit 38% (41-50%) Leukocyte count 5,800/mm3 (5,000-10,000/mm3) Sodium 144 mEq/L (135-145 mEq/L) Potassium 3.6 mEq/L (3.5-5.2 mEq/L) Bicarbonate 28 mEq/L (20-29 mEq/L) Urea nitrogen (BUN) 22 mg/dL (7-20 mg/dL) Creatinine 1.3 mg/dL (0.6-1.2 mg/dL in females) Which of the following is the most likely cause of this patient's hypertension? A. Aldosterone-secreting tumor B. Coarctation of the aorta C. Essential hypertension D. Pheochromocytoma E. Renal artery stenosis

The correct answer is E. Although essential hypertension is the most common cause of hypertension, accounting for approximately 91% of all hypertensive diagnoses, in this case there are signs and symptoms suggestive of another diagnosis. For this patient, her sensitivity to ACE inhibitors, her elevated creatinine, abdominal bruits, and congestive heart failure all strongly suggest some degree of renal artery stenosis. Renal artery stenosis is typically caused by atherosclerosis or fibromuscular dysplasia (prominent cause in young women). This condition can present with accelerated or difficult to control hypertension. This condition should be readily suspected in patients whose renal function rapidly worsens after initiation of the use of ACE inhibitors or ARBs. Patients who have renal artery stenosis depend on vasoconstriction of the efferent arteriole to maintain GFR. ACE inhibitors abolish the vasomotor tone in the efferent arteriole, however, resulting in worsening renal function. The renal function improves with the removal of ACE inhibitors. This effect is seen only in bilateral renal artery stenosis or unilateral stenosis in a single functioning kidney. Atherosclerosis is a generalized disease that may result in bilateral renal artery stenosis.

A 61-year-old man is brought to the emergency department for chest pain. The patient has a long history of coronary artery disease and is status post-coronary bypass procedure 6 years ago. The patient has chronic stable angina that is usually precipitated by activity and relieved by rest. About 3 weeks ago, his health care provider prescribed sildenafil (Viagra), and he has been using the drug with success. This morning he developed acute onset of substernal chest pain radiating to his left arm. This pain is not relieved by rest. The patient last took a sildenafil (Viagra) the night before. Which of the following treatments is absolutely contraindicated in this situation? A. Aspirin (ASA) B. Captopril (Capoten) C. Metoprolol (Lopressor) D. Morphine E. Nitroglycerin F. Tissue plasminogen activator (tPA)

The correct answer is E. Although nitrate therapy is typically one of the cornerstones of treatment for cardiac ischemia, the co-administration of nitrates within 24 hours after taking sildenafil is absolutely contraindicated. The vasodilatory effects of nitrates are profoundly amplified when administered in the presence of sildenafil, which can lead to refractory and life-threatening hypotension and cardiovascular collapse because of a synergistic effect. Patients using sildenafil therefore should be instructed to report their use on presentation to any emergency department and to never take nitrates while using the drug.

A 61-year-old man is brought to the emergency department for chest pain. The patient has a long history of coronary artery disease and is status post-coronary bypass procedure 6 years ago. The patient has chronic stable angina that is usually precipitated by activity and relieved by rest. About 3 weeks ago, his health care provider prescribed sildenafil (Viagra), and he has been using the drug with success. This morning he developed acute onset of substernal chest pain radiating to his left arm. This pain is not relieved by rest. The patient last took a sildenafil (Viagra) the night before. Which of the following treatments is absolutely contraindicated in this situation? A. Aspirin (ASA) B. Captopril (Capoten) C. Metoprolol (Lopressor) D. Morphine E. Nitroglycerin F. Tissue plasminogen activator (tPA)

The correct answer is E. Although nitrate therapy is typically one of the cornerstones of treatment for cardiac ischemia, the co-administration of nitrates within 24 hours after taking sildenafil is absolutely contraindicated. The vasodilatory effects of nitrates are profoundly amplified when administered in the presence of sildenafil, which can lead to refractory and life-threatening hypotension and cardiovascular collapse because of a synergistic effect. Patients using sildenafil therefore should be instructed to report their use on presentation to any emergency department and to never take nitrates while using the drug. Of all of the adjunctive treatments for myocardial infarction, aspirin (ASA) (choice A) likely has the largest mortality benefit. All patients who have suspected cardiac ischemia who do not have a history of anaphylaxis to ASA should be given aspirin promptly. A history of peptic ulcer disease or upper gastrointestinal bleeding is not an absolute contraindication in this setting, as the potential benefits would likely outweigh the risks. Angiotensin-converting enzyme inhibitors such as captopril (choice B) may play a role in myocardial remodeling during myocardial infarctions and may actually have a mortality benefit if given in the peri-infarction period. Beta-blockers such as metoprolol (choice C) reduce cardiac demand and decrease the frequency of ventricular arrhythmias after myocardial infarction, with well-established survival benefit. Treating pain and anxiety during a myocardial infarction with narcotics such as morphine (choice D) may reduce cardiac demand. These drugs have classically played an important role in the treatment of myocardial infarction. The use of thrombolytics such as tissue plasminogen activator (tPA) (choice F) has evolved as one of the most important developments in the treatment of acute myocardial infarction, as it can reduce in-hospital mortality by as much as 50% when administered within the first hour of the onset of symptoms. Clear contraindications to the use of tPA include a history of hemorrhagic stroke, a history of an invasive surgical procedure within the preceding 2 weeks, marked hypertension (or hypertension that does not respond to appropriate treatment), or active peptic ulcer disease. This patient thus has no contraindications to receiving tPA or other thrombolytics. Reviewed on 4/17/14 Sildenafil, a phosphodiesterase inhibitor, is used to enhance the effects of nitric acid, which increases cyclic GMP. This effect is useful to treat erectile dysfunction. This class of medication is strictly contraindicated for use with nitrates and protease inhibitors (class of medications used in the management of HIV and AIDS). If used together, significant hypotension and even cardiovascular collapse may occur. These agents cannot be used within 24 hours of the last dose of either of these agents.

A 29-year-old man is brought to the emergency department in a comatose state a few hours after complaining of sudden onset of excruciating headache. His friend does not know if the patient has any underlying medical conditions. Neurologic examination reveals dilated pupils poorly responsive to light. A CT scan of the head without contrast demonstrates hyperdensity within the suprasellar cistern, whereas MRI scan is unremarkable. Lumbar puncture shows hemorrhagic cerebrospinal fluid. Which of the following is the most likely diagnosis? A. Amyloid angiopathy-related hemorrhage B. Cavernous sinus thrombosis C. Hemorrhagic infarction D. Pituitary apoplexy E. Ruptured berry aneurysm

The correct answer is E. Headache of sudden onset ("thunderclap" headache), rapid deterioration of mental status, and blood in the CSF are virtually diagnostic of ruptured berry aneurysms. Note the characteristic hyperdensity on CT scan of the suprasellar cistern, indicating blood in the subarachnoid space. Rupture of a berry aneurysm is the most common cause of subarachnoid bleeding. Berry aneurysms develop as a result of congenital weakness at branching points of the arteries in the circle of Willis. These outpouchings tend to expand progressively, but in most cases they remain asymptomatic. Hypertension facilitates development and rupture of berry aneurysm. One third of patients recover, one third die, and one third develop re-bleeding. Rapid onset of coma is an ominous sign.

A 63-year-old white man who has recently retired from work as a plumber for over 30 years returns to his health care provider saying that he has been feeling very down lately, and has been having decreased appetite and a loss of interest in activities that used to give him pleasure. He is a smoker, drinks no alcohol, and is being treated by his health care provider for moderate essential hypertension. Physical examination is unchanged from his previous visits. Which of the following is the most appropriate next step in management? A. Discuss activities that will help him enjoy his retirement B. Order a thyroid-stimulating hormone level (TSH) C. Order electroconvulsive therapy (ECT) D. Prescribe an antidepressant E. Review the patient's medication history

The correct answer is E. Many medications used to control hypertension, such as propranolol, and in the past, reserpine, are known to occasionally lead to depressive symptoms. By evaluation of the patient's medication record, the physician can evaluate which medications were started at what time and can make adjustments to dosage or switch to alternative medications to control hypertension. Changing the antihypertensive medication will possibly improve the depressive symptoms without the need to start an antidepressant (choice D). Patients presenting with major depression symptoms should have a thorough history and physical examination performed to assess for secondary and treatable causes. These secondary causes may be related to medication side effects, metabolic disorders such as hypothyroidism, or as a consequence of medical conditions such as the high incidence of depression following coronary artery bypass procedures.

A 63-year-old white man who has recently retired from work as a plumber for over 30 years returns to his health care provider saying that he has been feeling very down lately, and has been having decreased appetite and a loss of interest in activities that used to give him pleasure. He is a smoker, drinks no alcohol, and is being treated by his health care provider for moderate essential hypertension. Physical examination is unchanged from his previous visits. Which of the following is the most appropriate next step in management? A. Discuss activities that will help him enjoy his retirement B. Order a thyroid-stimulating hormone level (TSH) C. Order electroconvulsive therapy (ECT) D. Prescribe an antidepressant E. Review the patient's medication history

The correct answer is E. Many medications used to control hypertension, such as propranolol, and in the past, reserpine, are known to occasionally lead to depressive symptoms. By evaluation of the patient's medication record, the physician can evaluate which medications were started at what time and can make adjustments to dosage or switch to alternative medications to control hypertension. Changing the antihypertensive medication will possibly improve the depressive symptoms without the need to start an antidepressant (choice D). Choice A is an appropriate intervention, but it is not the most appropriate next step, as the etiology of the patient's depression may be overlooked. Ordering a TSH level (choice B) is also appropriate, as hypothyroidism can be an organic cause of depression. Given the patient's medication history, however, evaluating possible pharmacologic causes of depression takes precedence in management of patient depression. Electroconvulsive therapy (choice C) is indicated in severe intractable depression when not contraindicated by seizure disorder or other factors, but it is not indicated in this state. Reviewed on 4/17/14 Patients presenting with major depression symptoms should have a thorough history and physical examination performed to assess for secondary and treatable causes. These secondary causes may be related to medication side effects, metabolic disorders such as hypothyroidism, or as a consequence of medical conditions such as the high incidence of depression following coronary artery bypass procedures.

Which of the following medications is the most appropriate choice for the treatment of hypertension during pregnancy? A. Lisinopril (Prinivil, Zestril) B. Atenolol (Tenormin) C. Verapamil (Calan, Isoptin, Verelan) D. Furosemide (Lasix) E. Methyldopa (Aldomet)The correct answer is E.

The correct answer is E. Methyldopa, an alpha-2 adrenergic agonist, decreases blood pressure by decreasing peripheral vascular resistance. It is the drug of choice for treating hypertension during pregnancy that may lead to pre-eclampsia, which is the most common cause of maternal and fetal morbidity and mortality. It acts on the central circulation as an alpha-agonist. It is listed as category B for use in pregnancy, which maintains that it has been proven safe for use in the first trimester of pregnancy. Although the central alpha-agonist methyldopa (Aldomet) is not used commonly as an antihypertensive agent, pregnancy is one of the times that this agent is used. Because methyldopa is a category B agent in pregnancy, providers are often comfortable prescribing this agent during pregnancy.

A 32-year-old woman comes to the hospital for an elective repeat cesarean delivery. Four years ago she had a primary cesarean delivery for a nonreassuring fetal heart rate tracing. Two years ago she chose to have an elective repeat cesarean delivery rather than attempt a vaginal birth after cesarean (VBAC). Her prenatal course was uncomplicated except that she has mitral valve prolapse. An echocardiograph demonstrated the mitral valve prolapse but no other structural cardiac disease. Which of the following is the correct management of this patient specifically as related to her mitral valve disease? A. Administer intravenous antibiotics 30 minutes prior to the procedure B. Administer intravenous antibiotics for 24 hours after the procedure C. Administer intravenous antibiotics immediately after the procedure D. Administer oral antibiotics 6 hours after the procedure E. No antibiotics are needed

The correct answer is E. Mitral valve prolapse affects approximately 5% of women of childbearing age. Consequently the issue of mitral valve prolapse and the need for antibiotics is frequently debated. Bacterial endocarditis is a life-threatening infection that can develop in patients who have structural cardiac disease who are exposed to bacteremia. The risk for any given procedure depends on the nature of the procedure itself and on the nature of the cardiac lesion. Periodically the American Heart Association publishes guidelines for the prevention of bacterial endocarditis. According to the American Heart Association guidelines, antibiotic prophylaxis is not necessary for cesarean delivery or normal vaginal delivery. The possible exception to this is patients who have "high risk" cardiac conditions, which include women who have a history of endocarditis or who have prosthetic heart valves, complex cyanotic congenital heart disease, or surgically corrected systemic pulmonary shunts. Mitral valve prolapse, even if associated with mitral regurgitation (demonstrated by Doppler or a murmur), is considered a moderate risk condition and, therefore, antibiotic prophylaxis is not necessary for elective cesarean delivery. This patient therefore does not require antibiotics prior to, during, or after her cesarean delivery. Mitral valve prolapse without regurgitation requires no antibiotic prophylaxis, nor do American Heart Association guidelines recommend routine antibiotic prophylaxis for cesarean or normal vaginal delivery unless patient is very high risk. Mitral valve prolapse with mitral valve regurgitation is considered a moderate risk condition and antibiotics are not necessary for patients having elective cesarean or spontaneous vaginal delivery.

A patient who is being treated for hypertension related to a myocardial infarction that occurred 2 hours ago is medicated with IV nitroprusside (Nipride). Which of the following is the expected action of this drug? A. Constriction of arterioles alone B. Constriction of both arterioles and venules C. Constriction of venules alone D. Dilatation of arterioles alone E. Dilatation of arterioles and venules

The correct answer is E. Nitroprusside is a very useful IV agent that causes dilatation of both arterioles and venules. It has a very rapid onset of action and is typically used in an emergency department or intensive care unit situation. It can be titrated to effect on a drip-to-drip basis. It is used in settings in which the blood pressure needs to be carefully controlled. It is listed on ACLS protocols as one of the treatments (along with the combined alpha- and beta-blocker labetolol) for hypertensive emergency/hypertensive urgency. Nitroprusside can improve perfusion of vital organs and reduce the workload of the heart because it is both an arteriolar and venodilator. Problems sometimes encountered with this drug include hypotension (best avoided by starting with a low dose and continuously monitoring systemic arterial and pulmonary capillary wedge pressures) and accumulation of toxic metabolite (thiocyanate) of cyanide in patients who have liver or renal failure. Many physicians prefer to use IV nitrate rather than nitroprusside because of its lesser toxicity. Sodium nitroprusside and labetolol are parenteral agents indicated for treating hypertensive urgency and hypertensive emergency. They may also be used to treat hypertension that is related to sympathomimetic drug use (such as cocaine, amphetamines, phenylpropanolamine, MAO-related hypertensive crisis). As a potent arterial and venous dilator, it is able to reduce both preload and afterload. Nitroprusside is typically administered with a beta-blocker, because its potent nitroprusside-induced vasodilation induces a strong compensatory tachycardia that might increase stress to the system.

A patient who is being treated for hypertension related to a myocardial infarction that occurred 2 hours ago is medicated with IV nitroprusside (Nipride). Which of the following is the expected action of this drug? A. Constriction of arterioles alone B. Constriction of both arterioles and venules C. Constriction of venules alone D. Dilatation of arterioles alone E. Dilatation of arterioles and venules

The correct answer is E. Nitroprusside is a very useful IV agent that causes dilatation of both arterioles and venules. It has a very rapid onset of action and is typically used in an emergency department or intensive care unit situation. It can be titrated to effect on a drip-to-drip basis. It is used in settings in which the blood pressure needs to be carefully controlled. It is listed on ACLS protocols as one of the treatments (along with the combined alpha- and beta-blocker labetolol) for hypertensive emergency/hypertensive urgency. Nitroprusside can improve perfusion of vital organs and reduce the workload of the heart because it is both an arteriolar and venodilator. Problems sometimes encountered with this drug include hypotension (best avoided by starting with a low dose and continuously monitoring systemic arterial and pulmonary capillary wedge pressures) and accumulation of toxic metabolite (thiocyanate) of cyanide in patients who have liver or renal failure. Many physicians prefer to use IV nitrate rather than nitroprusside because of its lesser toxicity. Reviewed on 4/17/14 Sodium nitroprusside and labetolol are parenteral agents indicated for treating hypertensive urgency and hypertensive emergency. They may also be used to treat hypertension that is related to sympathomimetic drug use (such as cocaine, amphetamines, phenylpropanolamine, MAO-related hypertensive crisis). As a potent arterial and venous dilator, it is able to reduce both preload and afterload. Nitroprusside is typically administered with a beta-blocker, because its potent nitroprusside-induced vasodilation induces a strong compensatory tachycardia that might increase stress to the system.

A 59-year-old man chooses a health care provider. The health care provider discovers that the patient was recently admitted to the hospital for palpitations and was found to have newly diagnosed atrial fibrillation (AF). He was placed on digoxin (Lanoxin) and verapamil (Calan, Isoptin, Verelan) for rate control and was loaded with warfarin (Coumadin) for anticoagulation. After several trips to the anticoagulation clinic, he decided to stop taking his warfarin, as it was difficult for him to leave work during the day. He now calls the office complaining of a cold, pale, and painful right leg starting an hour ago. Which of the following is the most likely explanation for his symptoms? A. Cerebrovascular accident (CVA) B. Deep vein thrombosis (DVT) C. Multiple sclerosis (MS) D. Raynaud syndrome E. Thromboembolism

The correct answer is E. Overall, atrial fibrillation confers about a 1% annual risk for a thromboembolic event, which, among other things, may cause stroke, intestinal ischemia, renal infarcts, or a threatened limb. Specific etiologies of AF such as valvular disease are associated with higher rates of embolic events. One complication of atrial fibrillation is a thromboembolic event that occurs as a result of a clot being expelled from the left atrium. This arterial embolism may be manifested by signs of poor perfusion to the peripheral vascular system. The signs of acute arterial embolism include the "6 Ps": pain, pallor, paresthesias, paralysis, pulselessness, and poikilothermia. The patient's embolism needs to be urgently addressed to salvage the tissue distal to the site of arterial obstruction.

A 12-year-old child is brought to his pediatrician for a routine health maintenance visit. He has been well except for occasional attacks of asthma and has met all development milestones. His immunizations are up to date. He occasionally uses albuterol for his asthma. Physical examination is remarkable for a blood pressure of 150/90 mm Hg in both arms. Which of the following is the most likely cause of his hypertension? A. Albuterol usage B. Chronic lung disease C. Coarctation of the aorta D. Congenital heart disease E. Renal disease

The correct answer is E. Renovascular disease is the most frequent cause of secondary hypertension in young children. Ailments such as polycystic kidney disease, congenital vascular disease, tumors, and infections can all lead to hypertension, and a urologic evaluation is imperative. Patients who present with hypertension at the extremes of age or whose hypertension is not responsive to traditional agents should have an evaluation performed to assess for secondary causes of hypertension. Renovascular disease is the most common cause of hypertension (65%) in children. It should be considered in the evaluation of a pediatric patient who presents with hypertension, as it is commonly a secondary cause of hypertension. Although the vast majority of patients who have hypertension have essential hypertension, there is a lot of value in identifying anyone who has secondary hypertension because treatment of the underlying cause in these patients will "cure" the patient of their hypertension.

A 12-year-old child is brought to his pediatrician for a routine health maintenance visit. He has been well except for occasional attacks of asthma and has met all development milestones. His immunizations are up to date. He occasionally uses albuterol for his asthma. Physical examination is remarkable for a blood pressure of 150/90 mm Hg in both arms. Which of the following is the most likely cause of his hypertension? A. Albuterol usage B. Chronic lung disease C. Coarctation of the aorta D. Congenital heart disease E. Renal disease

The correct answer is E. Renovascular disease is the most frequent cause of secondary hypertension in young children. Ailments such as polycystic kidney disease, congenital vascular disease, tumors, and infections can all lead to hypertension, and a urologic evaluation is imperative. Patients who present with hypertension at the extremes of age or whose hypertension is not responsive to traditional agents should have an evaluation performed to assess for secondary causes of hypertension. Albuterol usage (choice A) would be associated with jitteriness or tachycardia but not hypertension. Chronic lung disease (choice B) would not elevate the blood pressure. Associated symptoms would include dyspnea, cyanosis, and symptoms of right heart failure. Because this patient has asthma with only occasional attacks, chronic lung disease is not a consideration in this scenario. Coarctation of the aorta (choice C) can cause hypertension but is a less common cause of the disease in this age group. Because this patient's blood pressure is equal in both arms, coarctation is less likely, as there is typically a blood pressure differential between the arms as a result of the coarctation. Congenital heart disease (choice D) such as an atrial septal defect or ventricular septal defect is rare and would be associated with dyspnea, cyanosis, murmurs, and general ill health. These conditions should have been identified earlier in this pediatric patient and there are no supporting facts in this scenario. Reviewed on 4/17/14 Renovascular disease is the most common cause of hypertension (65%) in children. It should be considered in the evaluation of a pediatric patient who presents with hypertension, as it is commonly a secondary cause of hypertension. Although the vast majority of patients who have hypertension have essential hypertension, there is a lot of value in identifying anyone who has secondary hypertension because treatment of the underlying cause in these patients will "cure" the patient of their hypertension.

A man complains to his health care provider that he feels lightheaded and has even fainted during defecation. This is most probably an example of syncope due to which of the following mechanisms? A. Anoxia B. Hyperventilation C. Hypovolemia D. Sinus node disease E. Valsalva mechanism

The correct answer is E. Syncope has a broad differential diagnosis, because fainting can be produced by a wide variety of mechanisms. All of the mechanisms listed in the answers can produce syncope, but only the Valsalva mechanism (in which high intra-abdominal pressures trigger a reflex fall in cardiac output from vagal stimulation) is specifically associated with defecation. This mechanism can also produce fainting during weightlifting and with the use of wind instruments.

A 12-year-old African American boy is brought to the office for a well child examination. He has been in good health and only complains of an occasional headache. He has been doing reasonably well in school but has some social problems that his mother attributes to his physical appearance. She has tried persuading her son to eat healthier, but he seems to enjoy eating fast food much more than home cooked meals. His past medical history is unremarkable, and the family history is significant for adult-onset hypertension in his father's family. He takes no medication. On physical examination, the patient is in no acute distress. He is 145 centimeters tall and weighs 92 kilograms. His vital signs are within normal limits, but his blood pressure is 145/90 mm Hg. The health care provider checks that the cuff size is appropriate and remeasures his blood pressure in all four extremities, only to confirm that it is above the ninety-fifth percentile for his age. After discussing the finding with his mother, he is scheduled for several more blood pressure evaluations over the following 6 weeks. All readings yield results mildly above the ninety-fifth percentile for his age. Which of the following recommendations is most appropriate at this time? A. A low salt and low potassium diet should be started B. No intervention is necessary at this time C. The child should be enrolled in a sports program that involves strenuous physical activity D. The child should immediately be started on a diuretic and angiotensin-converting enzyme inhibitor E. The child should initiate a weight reduction diet with limited salt intake and regular exercise

The correct answer is E. Systemic hypertension is defined as blood pressure above the ninety-fifth percentile for age on repeated measurements over a 6-week period. Primary (essential) hypertension usually has no known underlying cause. Predisposing factors include heredity, salt intake, stress, and obesity. It is more commonly seen in adolescents and adults. Secondary hypertension is caused by an associated disease. It is more common in infants and younger children. The most common cause of secondary hypertension in children is renal disease (75-80%). A prior urinary tract infection is seen in 25 to 50% of cases, often related to an obstructive lesion of the urinary tract. In newborns, a history of umbilical artery catheterization may be elicited, with resultant thrombosis of the renal artery. Hypertension usually presents with no symptoms, especially in adolescents who have essential hypertension, and it is diagnosed on routine examination. Headaches, dizziness, vision changes, and seizures may be present. Blood pressure should be measured over several visits and compared with normal values for age. It should be measured in all extremities to rule out coarctation of the aorta. All children who have secondary hypertension should have a renal evaluation, including culture, ultrasound, renin levels, blood urea nitrogen, and creatinine. Echocardiography should be used to assess ventricular function and size. Therapy of hypertension in children is the same as in adults: diet, exercise, and medication. Pharmacologic management includes angiotensin-converting enzyme inhibitors, calcium-channel blockers, and diuretics.

A 12-year-old African American boy is brought to the office for a well child examination. He has been in good health and only complains of an occasional headache. He has been doing reasonably well in school but has some social problems that his mother attributes to his physical appearance. She has tried persuading her son to eat healthier, but he seems to enjoy eating fast food much more than home cooked meals. His past medical history is unremarkable, and the family history is significant for adult-onset hypertension in his father's family. He takes no medication. On physical examination, the patient is in no acute distress. He is 145 centimeters tall and weighs 92 kilograms. His vital signs are within normal limits, but his blood pressure is 145/90 mm Hg. The health care provider checks that the cuff size is appropriate and remeasures his blood pressure in all four extremities, only to confirm that it is above the ninety-fifth percentile for his age. After discussing the finding with his mother, he is scheduled for several more blood pressure evaluations over the following 6 weeks. All readings yield results mildly above the ninety-fifth percentile for his age. Which of the following recommendations is most appropriate at this time? A. A low salt and low potassium diet should be started B. No intervention is necessary at this time C. The child should be enrolled in a sports program that involves strenuous physical activity D. The child should immediately be started on a diuretic and angiotensin-converting enzyme inhibitor E. The child should initiate a weight reduction diet with limited salt intake and regular exercise

The correct answer is E. Systemic hypertension is defined as blood pressure above the ninety-fifth percentile for age on repeated measurements over a 6-week period. Primary (essential) hypertension usually has no known underlying cause. Predisposing factors include heredity, salt intake, stress, and obesity. It is more commonly seen in adolescents and adults. Secondary hypertension is caused by an associated disease. It is more common in infants and younger children. The most common cause of secondary hypertension in children is renal disease (75-80%). A prior urinary tract infection is seen in 25 to 50% of cases, often related to an obstructive lesion of the urinary tract. In newborns, a history of umbilical artery catheterization may be elicited, with resultant thrombosis of the renal artery. Hypertension usually presents with no symptoms, especially in adolescents who have essential hypertension, and it is diagnosed on routine examination. Headaches, dizziness, vision changes, and seizures may be present. Blood pressure should be measured over several visits and compared with normal values for age. It should be measured in all extremities to rule out coarctation of the aorta. All children who have secondary hypertension should have a renal evaluation, including culture, ultrasound, renin levels, blood urea nitrogen, and creatinine. Echocardiography should be used to assess ventricular function and size. Therapy of hypertension in children is the same as in adults: diet, exercise, and medication. Pharmacologic management includes angiotensin-converting enzyme inhibitors, calcium-channel blockers, and diuretics. Potassium supplementation decreases blood pressure and reduces ventricular hypertrophy in adults. It is not known how potassium supplementation affects children who have hypertension. Prescribing a potassium-rich diet seems reasonable if the patient is on a potassium-depleting diuretic and has normal renal function. This answer is incorrect because it recommends a low salt and low potassium diet (choice A). It is incorrect to say that no intervention is necessary at this time (choice B). The child has confirmed hypertension that is clinically manifesting with headache, and appropriate treatment should be initiated. Aerobic and isotonic exercises should be encouraged in patients who have hypertension. It is not recommended to start with a sports program that involves strenuous physical activity, however (choice C), until the blood pressure is well controlled. Uncontrolled hypertension may result in complications when the patient is exposed to extreme exertion. Initial therapy of mild to moderate essential hypertension in a child or adolescent should be attempted with diet and exercise. If these fail, a diuretic should be added. If the blood pressure is still not well controlled, an angiotensin-converting enzyme inhibitor is indicated. It is therefore incorrect to say that the child should be started immediately on a diuretic and angiotensin-converting enzyme inhibitor (choice D) before allowing the child to adopt a therapeutic lifestyle.

A 35-year-old woman presents to her health care provider with paresthesias of the left shoulder and arm. She has also noticed a hard, bony structure on the left side of her neck above the clavicle. Which of the following diagnoses best accounts for her symptoms? A. Horner syndrome B. Osteopetrosis C. Pancoast tumor D. Shoulder dislocation E. Thoracic outlet obstruction

The correct answer is E. The patient has a left cervical rib. This anatomic variant, which is typically bilateral but can occur on only one side, is caused by formation of an extra rib at the C7 level. Cervical ribs, while often asymptomatic, can cause thoracic outlet obstruction. This results in pain caused by distortion of blood vessels; pain or paresthesias related to brachial plexus impingement (notably sensory disturbances in the distribution of the ulnar nerve), and palpable abnormalities in the greater supraclavicular fossa. Thoracic outlet syndrome occurs as a result of compression of one or more neurovascular structures in the superior portion of the chest. Presence of a cervical rib is a known risk factor for this condition. If the nerve is compressed, the patient will have paresthesias and motor weakness along with pain in the affected dermatome. Physical stress will worsen these symptoms, and the Adson maneuver can be used as part of the assessment for this condition. Treatment may consist of physical therapy with core strengthening. Surgical correction of the anatomic abnormality is used if conservative therapy fails.

A 35-year-old woman presents to her health care provider with paresthesias of the left shoulder and arm. She has also noticed a hard, bony structure on the left side of her neck above the clavicle. Which of the following diagnoses best accounts for her symptoms? A. Horner syndrome B. Osteopetrosis C. Pancoast tumor D. Shoulder dislocation E. Thoracic outlet obstruction

The correct answer is E. The patient has a left cervical rib. This anatomic variant, which is typically bilateral but can occur on only one side, is caused by formation of an extra rib at the C7 level. Cervical ribs, while often asymptomatic, can cause thoracic outlet obstruction. This results in pain caused by distortion of blood vessels; pain or paresthesias related to brachial plexus impingement (notably sensory disturbances in the distribution of the ulnar nerve), and palpable abnormalities in the greater supraclavicular fossa. Horner syndrome (ptosis, anhidrosis, and miosis; choice A) occurs with Pancoast tumors (choice C) at the apex of the lung, as well as with certain brainstem or spinal cord lesions. Osteopetrosis (choice B) is a hereditary disease characterized by increased density and thickening of bone cortex with narrowing of medullary cavities. Bones are brittle and fracture easily. Shoulder dislocation (choice D) does not present with a supraclavicular bony structure, although patients may have paresthesias associated with this condition. Reviewed on 4/17/14 Thoracic outlet syndrome occurs as a result of compression of one or more neurovascular structures in the superior portion of the chest. Presence of a cervical rib is a known risk factor for this condition. If the nerve is compressed, the patient will have paresthesias and motor weakness along with pain in the affected dermatome. Physical stress will worsen these symptoms, and the Adson maneuver can be used as part of the assessment for this condition. Treatment may consist of physical therapy with core strengthening. Surgical correction of the anatomic abnormality is used if conservative therapy fails.

A 68-year-old man presents with complaints of chronic fatigue, exertional and nocturnal dyspnea, orthopnea, and a chronic nonproductive cough. On examination, respiratory wheezing and rhonchi are noted. Cardiac examination reveals a diminished first heart sound and an S3 gallop. The patient indicates that he was recently treated for hypertension and vasospastic angina. On the basis of his initial presentation, which of the following agents was most likely prescribed? A. Amlodipine (Norvasc) B. Captopril (Capoten) C. Furosemide (Lasix) D. Hydralazine (Apresoline) E. Verapamil (Isoptin, Calan)

The correct answer is E. The patient is presenting with classic signs and symptoms of congestive heart failure: chronic fatigue, exertional and nocturnal dyspnea, orthopnea, a chronic nonproductive cough, respiratory wheeze and rhonchi, as well as a diminished first heart sound and an S3 gallop. Verapamil is a non-dihydropyridine calcium channel blocker used to treat both hypertension and vasospastic angina. It has, however, a strong negative inotropic effect on the heart that can cause signs and symptoms of heart failure. Furthermore, some clinical studies have shown that congestive heart failure can develop in a small percentage of individuals taking verapamil. Non-dihydropyridine calcium channel blockers have both negative inotropic and chronotropic activity. Because of their effect on suppression of cardiac activity, there is an increased risk for induction of heart failure. The newer class of calcium channel blockers, the dihydropyridine agents, are effective agents used in the management of hypertension, but they do not have the same level of effect on the heart so they are rarely a cause of heart failure.

A 68-year-old man presents with complaints of chronic fatigue, exertional and nocturnal dyspnea, orthopnea, and a chronic nonproductive cough. On examination, respiratory wheezing and rhonchi are noted. Cardiac examination reveals a diminished first heart sound and an S3 gallop. The patient indicates that he was recently treated for hypertension and vasospastic angina. On the basis of his initial presentation, which of the following agents was most likely prescribed? A. Amlodipine (Norvasc) B. Captopril (Capoten) C. Furosemide (Lasix) D. Hydralazine (Apresoline) E. Verapamil (Isoptin, Calan)

The correct answer is E. The patient is presenting with classic signs and symptoms of congestive heart failure: chronic fatigue, exertional and nocturnal dyspnea, orthopnea, a chronic nonproductive cough, respiratory wheeze and rhonchi, as well as a diminished first heart sound and an S3 gallop. Verapamil is a non-dihydropyridine calcium channel blocker used to treat both hypertension and vasospastic angina. It has, however, a strong negative inotropic effect on the heart that can cause signs and symptoms of heart failure. Furthermore, some clinical studies have shown that congestive heart failure can develop in a small percentage of individuals taking verapamil. Amlodipine (choice A) is a dihydropyridine calcium channel blocker used in the treatment of both hypertension and vasospastic angina. It does not have a negative inotropic effect on the heart, however, and does not cause signs and symptoms of heart failure. Furthermore, amlodipine is generally well tolerated in heart failure patients. Captopril (choice B) is an ACE inhibitor used in the treatment of both hypertension and congestive heart failure. This agent therefore would not potentiate signs and symptoms of congestive heart failure. Furosemide (choice C) is a diuretic commonly used to treat the congestion and edema associated with heart failure. It can also be used to treat hypertension. Hydralazine (choice D) is a vasodilator used primarily to treat signs and symptoms associated with heart failure. The use of this agent in patients who have angina is not recommended because it can potentiate the angina. Reviewed on 4/17/14 Non-dihydropyridine calcium channel blockers have both negative inotropic and chronotropic activity. Because of their effect on suppression of cardiac activity, there is an increased risk for induction of heart failure. The newer class of calcium channel blockers, the dihydropyridine agents, are effective agents used in the management of hypertension, but they do not have the same level of effect on the heart so they are rarely a cause of heart failure.

A 25-year-old man is shot with a .22-caliber revolver. The entrance wound is in the anteromedial aspect of the upper thigh, while the exit wound is approximately 3 inches lower, in the posterolateral aspect of the thigh. He has a large, expanding hematoma in the upper inner thigh. There are no palpable pulses in the foot. The bone is intact by physical examination and x-ray film. Which of the following is the most appropriate next step in management?

There is no question that this patient's femoral vessels are injured, and the expanding hematoma plus absent pulses indicate that the femoral artery is involved (the vein may or may not be injured). Surgical exploration starts with proximal and distal control. Once the hematoma is safely entered, the extent of the injuries can be ascertained and the proper repair performed.

A 25-year-old man is shot with a .22-caliber revolver. The entrance wound is in the anteromedial aspect of the upper thigh, while the exit wound is approximately 3 inches lower, in the posterolateral aspect of the thigh. He has a large, expanding hematoma in the upper inner thigh. There are no palpable pulses in the foot. The bone is intact by physical examination and x-ray film. Which of the following is the most appropriate next step in management? A. Arteriogram B. CT angiography C. Doppler studies D. Embolectomy E. Surgical exploration F. Venogram

The correct answer is E. There is no question that this patient's femoral vessels are injured, and the expanding hematoma plus absent pulses indicate that the femoral artery is involved (the vein may or may not be injured). Surgical exploration starts with proximal and distal control. Once the hematoma is safely entered, the extent of the injuries can be ascertained and the proper repair performed. If there is an obvious vascular injury (absent distal pulses, expanding hematoma), surgical exploration and repair are required. If the penetration is near major vessels and the patient is asymptomatic, Doppler studies or CT angiogram is done.

A previously healthy 45-year-old woman is in a motor vehicle accident. She suffers multiple internal and external injuries from which she is still actively bleeding when reached by an ambulance. Her blood pressure is 50/20 mm Hg. Which of the following would be the most likely result of a complete blood count performed in the emergency department 15 minutes later, if the normal range for this test is 4.2 to 5.4 million red blood cells/uL in the female population and 5,000-10,000 white blood cells/uL? A. Red cell count 1.5 million per microliter; white cell count 2,000 per microliter B. Red cell count 1.5 million per microliter; white cell count 6,000 per microliter C. Red cell count 2.5 million per microliter; white cell count 3,000 per microliter D. Red cell count 2.5 million per microliter; white cell count 6,000 per microliter E. Red cell count 4.5 million per microliter; white cell count 6,000 per microliter

The correct answer is E. These are the only values within normal limits and are the correct choice. The complete blood count (or alternatively the hematocrit or hemoglobin) cannot be used to assess the size of an acute blood loss, because intravascular volume and blood are both lost as a result of trauma. The reason is that the hematocrit or red blood cell count will drop only as fluid from other body sources (mostly extracellular fluid, with a smaller shift of intracellular fluid to extracellular) enters the capillary beds in response to a decreased intracapillary blood pressure. This takes at least a few hours to happen. When fluid resuscitation is given, the hemoglobin and hematocrit and red blood cell counts will dramatically fall, because they will be hemodiluted out. Low or dropping blood pressure is consequently a much better initial indicator of significant hemorrhage.

A 50-year-old surgical nurse consults a health care provider because of a rash above both her ankles. She says that her ankles have been swollen for a few months but she only recently noticed the rash. Physical examination demonstrates marked ankle edema with erythema, mild scaling, and brown discoloration of the overlying skin of the distal lower legs. Varicose veins are also noted. Which of the following is the most likely diagnosis? A. Atopic dermatitis B. Cellulitis C. Lichen simplex chronicus D. Nummular dermatitis E. Stasis dermatitis

The correct answer is E. This is stasis dermatitis, which is a persistent inflammation of the skin of the lower legs. The condition is often related to varicose veins, although it has been postulated that the true cause may instead be perivascular fibrin deposition and abnormal small vessel vasoconstrictive reflexes. This presentation is typical. Most patients are relatively asymptomatic and may not seek medical attention until the edema becomes severe or the lesions become complicated by secondary bacterial infection or ulceration. It is important to increase the venous return to the heart by elevating the ankles while resting and using properly fitted support hose. Unna boot application can be helpful. Purulent lesions can be treated with hydrocolloid dressings. Ulcers are treated with compresses and bland dressings, such as zinc oxide paste. Stasis dermatitis is associated with venous insufficiency and chronic lower extremity edema. It most commonly affects the bilateral medial ankles proximal to the medial malleoli. It classically manifests with hyperpigmented scaling patches; ulceration may occur.

A 50-year-old surgical nurse consults a health care provider because of a rash above both her ankles. She says that her ankles have been swollen for a few months but she only recently noticed the rash. Physical examination demonstrates marked ankle edema with erythema, mild scaling, and brown discoloration of the overlying skin of the distal lower legs. Varicose veins are also noted. Which of the following is the most likely diagnosis? A. Atopic dermatitis B. Cellulitis C. Lichen simplex chronicus D. Nummular dermatitis E. Stasis dermatitis

The correct answer is E. This is stasis dermatitis, which is a persistent inflammation of the skin of the lower legs. The condition is often related to varicose veins, although it has been postulated that the true cause may instead be perivascular fibrin deposition and abnormal small vessel vasoconstrictive reflexes. This presentation is typical. Most patients are relatively asymptomatic and may not seek medical attention until the edema becomes severe or the lesions become complicated by secondary bacterial infection or ulceration. It is important to increase the venous return to the heart by elevating the ankles while resting and using properly fitted support hose. Unna boot application can be helpful. Purulent lesions can be treated with hydrocolloid dressings. Ulcers are treated with compresses and bland dressings, such as zinc oxide paste. Atopic dermatitis (choice A) typically involves the antecubital and popliteal fossas, eyelids, neck, and wrists and is characterized by a pruritic rash and/or papular eruptions. Cellulitis (choice B) is a bacterial infection of the subcutaneous tissues that causes local erythema, tenderness, and often lymphangitis. It is asymmetric and often accompanied by fever. Lichen simplex chronicus (choice C) is a secondary skin lesion caused by chronic scratching and characterized by dry, scaling, well demarcated, hyperpigmented plaques. This condition is seen in patients with poorly controlled atopic dermatitis as a complication of the chronic scratching due to itching. Nummular dermatitis (choice D) causes widespread coin-shaped, crusted skin lesions.

A 68-year-old man is brought to the emergency department following a high-speed automobile accident. He is alert and complains of chest pain and mild back pain. His blood pressure is 80/60 mm Hg. Chest radiograph shows a widened mediastinum, tracheal deviation, bronchial displacement, and loss of the aortic knob. Which of the following is the most likely diagnosis? A. Cardiac tamponade B. Myocardial contusion C. Pulmonary contusion D. Tension pneumothorax E. Traumatic aortic rupture

The correct answer is E. This patient has a traumatic aortic rupture, which is the most common cause of immediate death following a motor vehicle accident. Deceleration causes twisting of the aorta and may lead to rupture. Up to 90% of individuals die at the scene, however a high index of suspicion and early intervention may save the remaining few. The symptoms include chest pain, back pain, and hypotension. Angiography is the gold standard and immediate surgery is mandatory. Radiographic findings include widening of the mediastinum, alteration of the aortic knob, pleural cap, tracheal deviation, bronchial displacement, and esophageal deviation.

A 68-year-old man is brought to the emergency department following a high-speed automobile accident. He is alert and complains of chest pain and mild back pain. His blood pressure is 80/60 mm Hg. Chest radiograph shows a widened mediastinum, tracheal deviation, bronchial displacement, and loss of the aortic knob. Which of the following is the most likely diagnosis? A. Cardiac tamponade B. Myocardial contusion C. Pulmonary contusion D. Tension pneumothorax E. Traumatic aortic rupture

The correct answer is E. This patient has a traumatic aortic rupture, which is the most common cause of immediate death following a motor vehicle accident. Deceleration causes twisting of the aorta and may lead to rupture. Up to 90% of individuals die at the scene, however a high index of suspicion and early intervention may save the remaining few. The symptoms include chest pain, back pain, and hypotension. Angiography is the gold standard and immediate surgery is mandatory. Radiographic findings include widening of the mediastinum, alteration of the aortic knob, pleural cap, tracheal deviation, bronchial displacement, and esophageal deviation. Cardiac tamponade (choice A) often results from penetrating trauma near the heart and is caused by a fluid collection within the pericardium. The clinical features include hypotension, distended neck veins, pulsus paradoxus, and distant heart sounds. Pericardiocentesis is the treatment of cardiac tamponade. Myocardial contusion (choice B) is most commonly seen when there is a direct blow to the chest wall. Findings include right ventricular dysfunction, arrhythmias, and an elevation of creatine kinase. An echocardiogram may document ventricular wall motion abnormalities. Pulmonary contusion (choice C) is a hemorrhage into the pulmonary parenchyma. It is characterized by dyspnea and hypoxia. Tension pneumothorax (choice D) occurs when air is trapped within the pleural space and leads to an increase in the intrathoracic pressure. Findings include jugular venous distention, hypotension, tracheal deviation, and a mediastinal shift. Tension pneumothorax often occurs following blunt trauma. Treatment is with immediate needle thoracocentesis followed by chest tube placement.

A 10-year-old girl is brought to the office because of fever and chills for 3 days. She had been complaining of a headache and feeling tired for 10 days before she developed fever. The parents had tried giving her acetaminophen, but the child's condition was not improving. Her past medical history is significant for frequent streptococcal throat infections over the past 2 years and a new onset heart murmur detected at her last well child visit 2 months earlier. She is on no medication currently, except for acetaminophen. Otherwise, she has been wearing dental braces for the past year and had a primary tooth extracted 2 weeks earlier in the attempt of liberating space for permanent teeth. On physical examination, the patient is diaphoretic, in moderate distress with a temperature of 39.0°C (102.2°F), blood pressure of 90/60 mm Hg, pulse of 110/min, and respirations of 24/min. On her fingernails, you note several splinter hemorrhages. Auscultation confirms a grade 2/6 high-pitched, blowing, systolic ejection murmur, best heard at the apex that radiates to the left axilla. A chest radiograph is unremarkable, but the electrocardiogram shows signs of left ventricular strain. An echocardiogram reveals vegetations on the mitral valve. Which of the following microorganisms is the most likely cause of this patient's current condition? A. Candida species B. Pseudomonas aeruginosa C. Serratia marcescens D. Staphylococcus aureus E. Streptococcus viridans

The correct answer is E. This patient has developed acute endocarditis most likely caused by Streptococcus viridans that entered the bloodstream during the recent dental procedure. The predisposing factor for her current illness is rheumatic heart disease, a sequela of prior Group A β-hemolytic streptococcal pharyngitis, which was complicated by involvement of the mitral valve.

A 47-year-old man comes to his primary care provider for follow-up care of his hypertension. The patient has a 10-year history of hypertension that has been treated with a variety of medications. It is now well controlled with a beta-blocker and thiazide diuretic combination. The patient's mean blood pressure over the past 6 months has been 145/85 mm Hg. His other medical history is remarkable only for type 2 diabetes mellitus. He currently takes a low-dose oral sulfonylurea agent and has had blood glucose values ranging from 110-220 mg/dL. His most recent hemoglobin A1c value was 7.9%. He has no allergies to any medications and he quit smoking cigarettes 2 years ago. Which of the following is the most appropriate management at this time? A. Add insulin and an ACE inhibitor to his regimen B. Discontinue the sulfonylurea agent and begin insulin therapy C. Increase the doses of sulfonylurea and beta-blocker D. Increase the doses of sulfonylurea and thiazide E. Increase the dose of sulfonylurea and add an ACE inhibitor to his regimen

The correct answer is E. This patient has diabetes and hypertension. The management strategy for such patients includes addressing the primary blood sugar problem, the primary blood pressure problem, and the secondary consequences of both the diabetes and the blood pressure. This man has not reached the blood pressure goals for patients who have diabetes, which is 130/80 mm Hg. To further lower his blood pressure, an appropriate intervention would be to add an ACE inhibitor to deal with diabetic complications, such as renal disease, as well as to address further lowering of his blood pressure. Increasing his sulfonylurea dose to better control his sugar is also appropriate, because the patient's hemoglobin A1c is not at the goal of <7%. Patients with diabetes have an increased risk for cardiac and kidney disease. Goals for blood pressure control are lower for patients who are diabetic hypertensive patients. If ACE inhibitors are not initially chosen for blood pressure control in diabetic hypertensive patients, they need to be given as add-on medication. ARBs would be a reasonable alternative to ACE inhibitors, if ACE inhibitors are not tolerated, but there is no advantage of using these 2 agents together. The goal for diabetes long-term control (over the last 2-3 months) is hemoglobin A1c <7%. Oral agents that can be used to lower blood glucose levels include biguanides such as metformin (Glucophage), TZDs such as pioglitazone (Actos), and incretics such as sitagliptin (Januvia), or a combination of these medications.

A 61-year-old woman comes to the health care provider for her first physical examination in more than 10 years. She reports that she has been in excellent health, does not smoke or drink, and runs 3 miles daily. She is a retired accountant and has 3 healthy grown children. She has been taking 81 mg of aspirin daily after reading about its cardioprotective effects in the newspaper. On physical examination she appears well. Blood pressure is 122/76 mm Hg, pulse 70/min, and respirations 14/min. She is afebrile. Head and neck examination is normal. There is no jugulovenous distention. Lungs are clear. On cardiac examination she has a regular S1 and S2, and II/VI crescendo blowing diastolic murmur is heard at the aortic area. Abdominal examination is normal. Rectal examination shows no masses and brown, guaiac-negative stool. Which of the following most likely explains the cardiac findings on physical examination? A. Prior Bacteroides infection B. Prior Escherichia coli infection C. Prior gonococcal infection D. Prior Haemophilus ducreyi infection E. Prior streptococcal infection

The correct answer is E. This patient has findings of asymptomatic aortic insufficiency on physical examination. This lesion may result from a number of causes, several of which are infectious in etiology. Aortic insufficiency may result as a sequela of rheumatic heart disease, which occurs as an immunologic response to a streptococcal infection. Acute rheumatic fever is typically characterized by cardiac involvement that may cause pericarditis, myocarditis, or endocarditis. Often the initial cardiac manifestations are asymptomatic and become apparent only years later with the development of cardiac valvular disease. Aortic regurgitation causes a diastolic murmur from the leakage of blood from the aorta into the left ventricle. Chronic aortic regurgitation remains asymptomatic until the clinical manifestations of heart failure appear. Echocardiography provides the best noninvasive way to track and grade the severity of the aortic regurgitation. Vasodilator therapy improves symptoms and delays the need for replacement/repair of the aortic valve in asymptomatic patients who have chronic, severe aortic regurgitation. Replacement of the valve should occur in symptomatic patients or in patients who have low ejection fractions from the left ventricle <50% or who have left ventricle diameter greatly increased. Aortic regurgitation can be caused by primary disease of the aortic valve leaflets or dilation of the aortic root. It can occur as a complication from rheumatic fever or can be caused by congenital bicuspid aortic valve. Aortic root dilation may result from Marfan syndrome, connective tissue disease, and aortitis secondary to syphilis.

A 61-year-old woman comes to the health care provider for her first physical examination in more than 10 years. She reports that she has been in excellent health, does not smoke or drink, and runs 3 miles daily. She is a retired accountant and has 3 healthy grown children. She has been taking 81 mg of aspirin daily after reading about its cardioprotective effects in the newspaper. On physical examination she appears well. Blood pressure is 122/76 mm Hg, pulse 70/min, and respirations 14/min. She is afebrile. Head and neck examination is normal. There is no jugulovenous distention. Lungs are clear. On cardiac examination she has a regular S1 and S2, and II/VI crescendo blowing diastolic murmur is heard at the aortic area. Abdominal examination is normal. Rectal examination shows no masses and brown, guaiac-negative stool. Which of the following most likely explains the cardiac findings on physical examination? A. Prior Bacteroides infection B. Prior Escherichia coli infection C. Prior gonococcal infection D. Prior Haemophilus ducreyi infection E. Prior streptococcal infection

The correct answer is E. This patient has findings of asymptomatic aortic insufficiency on physical examination. This lesion may result from a number of causes, several of which are infectious in etiology. Aortic insufficiency may result as a sequela of rheumatic heart disease, which occurs as an immunologic response to a streptococcal infection. Acute rheumatic fever is typically characterized by cardiac involvement that may cause pericarditis, myocarditis, or endocarditis. Often the initial cardiac manifestations are asymptomatic and become apparent only years later with the development of cardiac valvular disease. Bacteroides (choice A) is a gram-negative anaerobe of intestinal origin, which, likewise, does not cause cardiac disease. Escherichia coli(choice B), an enteric gram-negative organism, very rarely causes any form of cardiac involvement. Treponema pallidum, the etiology of syphilis, may cause aortic insufficiency in its tertiary stages. Sexually transmitted disease organisms such as Neisseria gonorrhoeae (choice C) and Haemophilus ducreyi (choice D) have no cardiac manifestations, however. Reviewed on 4/17/14 Aortic regurgitation causes a diastolic murmur from the leakage of blood from the aorta into the left ventricle. Chronic aortic regurgitation remains asymptomatic until the clinical manifestations of heart failure appear. Echocardiography provides the best noninvasive way to track and grade the severity of the aortic regurgitation. Vasodilator therapy improves symptoms and delays the need for replacement/repair of the aortic valve in asymptomatic patients who have chronic, severe aortic regurgitation. Replacement of the valve should occur in symptomatic patients or in patients who have low ejection fractions from the left ventricle <50% or who have left ventricle diameter greatly increased. Aortic regurgitation can be caused by primary disease of the aortic valve leaflets or dilation of the aortic root. It can occur as a complication from rheumatic fever or can be caused by congenital bicuspid aortic valve. Aortic root dilation may result from Marfan syndrome, connective tissue disease, and aortitis secondary to syphilis.

A 62-year-old man comes to the health care provider with symptoms of worsening congestive heart failure. He has a history of rheumatic heart disease as a child. Over the past 3 years he has had progressive symptoms of dyspnea on exertion, paroxysmal nocturnal dyspnea, and orthopnea. He has been maintained on digoxin, furosemide, and enalapril for symptoms of his congestive heart failure. Cardiac examination reveals a loud blowing decrescendo diastolic murmur. He has bounding peripheral pulses. Which of the following additional findings would most likely be found on physical examination? A. Bradycardia B. Isolated systolic hypertension C. Pulsus paradoxus D. Tachycardia E. Wide pulse pressure

The correct answer is E. This patient has the characteristic physical findings of an aortic insufficiency, including the loud blowing decrescendo murmur and the bounding peripheral pulses. These patients typically will have a wide pulse pressure (i.e., elevated systolic blood pressure) related to a large left ventricular volume being ejected (until left ventricular failure supervenes). The low diastolic pressure is a result of the rapid run-off from the aorta caused by the regurgitant flow across the aortic valve into the left ventricle as well as the forward flow to the aorta.

A 21-year-old professional dancer complains of several episodes of near loss of consciousness during a performance. She has been in excellent health and is a principal dancer in the New York City Ballet Corps. She has no family history of coronary artery disease. She does not smoke, and a recent cholesterol profile was normal. On physical examination, her blood pressure is 142/88 mm Hg and her pulse is 84/min and regular. She has a brisk carotid upstroke with a double impulse palpable. She has a loud S4 and a harsh systolic murmur heard along the left sternal border. The murmur is accentuated during the Valsalva maneuver and when she stands from a squatting position. An electrocardiogram reveals severe left ventricular hypertrophy. Which of the following is the most appropriate medication in the management of this patient? A. Captopril (Capoten) B. Digoxin (Lanoxin) C. Diltiazem (Cardizem) D. Furosemide (Lasix) E. Metoprolol (Lopressor)

The correct answer is E. This patient has the presentation of hypertrophic cardiomyopathy, a frequent cause of syncope or near syncope in young patients. She has a systolic ejection murmur that is heard at the lower left edge of the sternum, which is exacerbated by exercise and standing and lessened by lying supine or squatting. This murmur can be distinguished from other systolic murmurs by its increase with the Valsalva maneuver or standing from a squatting position. Any maneuver that acts to decrease venous return of blood to the heart and left ventricular size will increase the murmur of hypertrophic cardiomyopathy, because the obstructive component increases as the left ventricular cavity shrinks. Beta-blockers such as metoprolol help to slow ventricular rate, thereby increasing ventricular filling time. When there is increased time for ventricular filling, there will be more blood in the left ventricle and the patient's obstruction will be lessened. This patient is at risk for sudden cardiac death and should be kept from exercising, which may decrease blood volume and which can have accentuated venous blood pooling when the patient rests from exercise. An implantable cardiac defibrillator may also be used to treat the patient's sudden cardiac death should it occur. All affected patients need to refrain from high intensity exercise. The best medical management of hypertrophic cardiomyopathy are beta-blockers, which slow the ventricular rate; thereby, increasing ventricular filling time. A heart filled with blood lessens the obstruction. Beta-blockers also decrease the force of cardiac contraction via their negative inotropic effects, which also lessens the left ventricular outflow obstruction. The patient must refrain from high intensity exercise, because the patient remains at risk for sudden cardiac death. The patient should be evaluated as a candidate for possible placement of an implantable cardiac defibrillator or treated with a sclerosing agent to the left ventricular septum to decrease the outflow obstruction.

Which of the following diuretic agents is most likely to cause hyperkalemia? A. Acetazolamide (Diamox) B. Furosemide (Lasix) C. Hydrochlorothiazide D. Metolazone (Zaroxolyn) E. Triamterene (Dyrenium)

The correct answer is E. Triamterene is grouped in the category of potassium-sparing diuretics. Along with spironolactone (Aldactone) and amiloride (Midamor), these diuretic agents may cause excess renal retention of potassium. In this manner, use of these drugs may increase potassium levels and cause hyperkalemia. The majority of diuretics used in clinical practice result in hypokalemia and hyponatremia because of the loss of these electrolytes as part of their clinical utility in causing diuresis. The potassium-sparing diuretics inhibit distal convoluted tubule aldosterone-induced sodium resorption, which permits sodium loss and potassium retention. Although potassium-sparing diuretics cause weak diuresis, there is evidence showing that there is mortality benefit when these agents are used as add-on agents for chronic heart failure. These agents should be used with caution in patients who have impaired renal function with resultant hyperkalemia. These agents should also be used with caution in patients who are on medications that are known to lower glomerular filtration pressure (such as ACE inhibitors and ARBs), as these agents are known to cause hyperkalemia.

A 19-year-old college student with a history of panic disorder and anxiety comes to the emergency department complaining of shortness of breath and anxiousness before finals. She denies any chest pain, palpitations, diaphoresis, or light-headedness. These episodes are similar to "panic attacks" that she had in the past. She has recently stopped all of her outpatient medications because she believed that she no longer needed them. Her temperature is 37.0°C (98.6°F), blood pressure is 120/70 mm Hg, pulse is 113/min, and respirations are 18/min. Her oxygen saturation is 99% on room air. She appears anxious but in no acute distress. Her heart is tachycardic without any murmurs. Her lungs are clear. The remainder of her examination is unremarkable. She is treated appropriately and her symptoms resolve. Just as she is about to be discharged with a diagnosis of anxiety and panic disorder, a nurse points out the following rhythm strip: Which of the following is the most appropriate management? A. Administer amiodarone B. Anticoagulation C. Cardioversion D. Electrophysiologic study E. Pacemaker placement F. Reassurance

The correct answer is F. This patient has a second-degree atrioventricular (AV) block of the Mobitz type I or Wenckebach type. In this condition, there is a progressive prolongation of the PR interval until there is eventually a blocked beat. This rhythm can be seen in normal patients or can be seen in patients on drugs such as digoxin, beta-blockers, or calcium channel antagonists. Rarely this rhythm can be secondary to ischemia, infarction, or inflammation. The patient should be reassured that the prognosis is good, and treatment is not routinely needed.

A 41-year-old man comes to his health care provider for a routine physical examination. He is new to this office and brings his previous medical record with him. He has no significant past medical history, but he does have a strong family history of cancer and heart disease. His father and his brother both had myocardial infarctions before age 55 years, and his sister, mother, and aunt had breast cancer. He exercises regularly and eats well, with most of his diet being low in saturated fat and cholesterol. He smokes one pack of cigarettes per week. His review of systems is unremarkable. He is very anxious and would like only minimal interventions done because of his good health. Which of the following is the most age-appropriate screening test in this patient?

The current recommendations for routine, age-appropriate screening are based in some measure on data from clinical trials. Depending on the source of the recommendations, there is considerable variability in these recommendations. One current recommendation is that at least every 5 years a random cholesterol level should be checked. Typically, total cholesterol and HDL cholesterol can be drawn as a baseline in a nonfasting state. If abnormalities are found in these measurements, a fasting blood lipid (ideally a 12-hour fast) can be performed.

Which of the following diuretics acts at the distal tubule of the nephron?

Thiazide Diuretics (Hydrochlorothiazide, Chlorothiazide, Benzthiazide- promote Diuresis by inhibiting Reabsorption of NaCl, primarily in the Early Distal Tubule)

_________- promote diuresis by inhibiting reabsorption of NaCl, primarily in the early distal tubule

Thiazide Diuretics (e.g., hydrochlorothiazide, chlorothiazide, benzthiazide)

A 25-year-old woman involved in an automobile accident is admitted as an emergency patient. A major artery severed in her leg caused an estimated 600 mL blood to be lost. Her blood pressure is 90/60 mm Hg. Which of the following would be expected to increase in response to hemorrhage?

The decrease in blood pressure caused by hemorrhage activates the baroreceptor reflex, which tends to increase sympathetic nerve activity and decrease parasympathetic (vagal) nerve activity. The increase in sympathetic nerve activity constricts arterioles in skeletal muscle and elsewhere in the body. Her compensatory responses include the following mechanisms: baroreceptor reflex, chemoreceptor reflex, epinephrine and norepinephrine released from the adrenal medulla, formation of angiotensin II, formation of vasopressin, and the capillary fluid shift mechanism. Activation of the renin-angiotensin system during hemorrhage also plays an important role in maintaining blood pressure. Angiotensin II increases blood pressure acutely by constricting arterioles throughout the body, and chronically by decreasing the renal excretion of both salt and water.

A cyanotic infant is discovered to have a ventricular septal defect, an overriding aorta, right ventricular hypertrophy, and complete pulmonic stenosis. Which of the following accompanying congenital anomalies improves survival in this patient?

The ductus arteriosus connects the aorta with the pulmonary artery. If it remains patent after birth, it allows oxygenated blood to flow from the aorta to the pulmonary artery. In this patient, who has tetralogy of Fallot with complete right ventricular outflow obstruction, this anastomosis is a crucial source of blood to the pulmonary vasculature. Patients who have a patent ductus arteriosus have an advantage in the setting of tetralogy of Fallot. If tetralogy of Fallot is diagnosed in the newborn period, the infant may benefit from the use of the prostaglandin E1 agent alprostadil (Prostin VR Pediatric), which can be used to maintain the patency of the ductus arteriosus. A patent ductus arteriosus provides an alternative source of pulmonary blood flow while the infant has definitive surgery performed.

The mortality rate per year is greatest with disease of which coronary anatomy?

The left main coronary artery is the primary arterial supply to the left ventricle. Blockage of this artery leads to anterior wall and lateral wall myocardial infarction. Mortality for patients who have left main coronary artery disease is more than 10 times greater than in patients who have one- or two-vessel disease involving the other coronaries. Left main coronary artery is the proximal artery for both the left anterior descending and left circumflex arteries. Blockage of the left main artery will affect blood supply to the left ventricle, which is considered to be the workhorse for the entire heart. Blockage in this area affects more muscle mass in the heart than any other artery. Because of the high mortality associated with blockage in this artery, this artery is notoriously known as the "widow maker."

A 52-year-old man comes to the hospital complaining of palpitations. The patient reports that while cooking breakfast this morning, he felt his heart "racing in his chest" and was unable to catch his breath. He states that sitting down brought no relief. He called for an ambulance and he was brought to the emergency department. The man has no significant past medical history and takes no medications regularly other than ranitidine (Zantac) for occasional heartburn. On examination, the patient is quite thin but well developed and in mild distress. His pulse is 140/min and irregularly irregular. There are no murmurs, and the lung examination is clear. Which of the following findings on his echocardiogram would suggest a diagnosis of longstanding atrial fibrillation?

The most common anatomic correlate seen in patients who have AF (Framingham Heart Study) is an enlarged left atrium. Patients who have an enlarged left atrium are less likely to remain in normal sinus rhythm following cardioversion. Echocardiography is an invaluable tool for assessing cardiovascular function in both normal and disease states. In the case of atrial fibrillation (AF), a chronic course versus an acute and self-limited course portends completely different treatment strategies and long-term prognoses. The most common cause of chronic AF is valvular disease, followed by congestive heart failure (CHF). Hyperthyroidism that is not adequately treated is another cause of atrial fibrillation and should be suspected in a patient who has new-onset AF.

A 50-year-old man is admitted to the hospital after sustaining an acute myocardial infarction. Eight hours after this event, his blood pressure is 70/50 mm Hg and his pulse is 45/min. An electrocardiogram reveals sinus bradycardia rhythm. Which of the following is the most appropriate intervention? atropine, dobutamine, beta blocker, pacemaker, cardiac cath

The patient is hypotensive and bradycardic. This suggests a vagal response, and administering an agent that is vagolytic, such as atropine, is the correct treatment. Atropine is given for symptomatic bradycardia post-MI at a dosage of 0.5 mg every 5 minutes up to a total of 3 mg. As the vagus nerve is blocked, the patient's heart rate will increase.

A 45-year-old man suddenly loses consciousness and falls to the ground. His pulse is lost. He has been previously healthy and has been on no medication. There is no obvious evidence of trauma. Electrocardiogram reveals wide complex tachycardia at a rate of 200/min. CK-MB levels are elevated. Which of the following is the most appropriate intervention?

The patient is in ventricular tachycardia and is hemodynamically unstable as illustrated by the loss of consciousness. He should be emergently defibrillated with 200 joules of energy initially. In addition, ventricular tachycardia without a pulse should be treated the same way as ventricular fibrillation.

A 68-year-old man presents with complaints of chronic fatigue, exertional and nocturnal dyspnea, orthopnea, and a chronic nonproductive cough. On examination, respiratory wheezing and rhonchi are noted. Cardiac examination reveals a diminished first heart sound and an S3 gallop. The patient indicates that he was recently treated for hypertension and vasospastic angina. On the basis of his initial presentation, which of the following agents was most likely prescribed? Amlodipine, Captopril, Furosemide, Verapamil, Hydralazine

The patient is presenting with classic signs and symptoms of congestive heart failure: chronic fatigue, exertional and nocturnal dyspnea, orthopnea, a chronic nonproductive cough, respiratory wheeze and rhonchi, as well as a diminished first heart sound and an S3 gallop. Verapamil is a non-dihydropyridine calcium channel blocker used to treat both hypertension and vasospastic angina. It has, however, a strong negative inotropic effect on the heart that can cause signs and symptoms of heart failure. Furthermore, some clinical studies have shown that congestive heart failure can develop in a small percentage of individuals taking verapamil.

A 55-year-old woman with a long-standing history of atrial fibrillation secondary to mitral regurgitation comes to the emergency department with a painful right foot. The patient reports that over the past few hours her foot has become more painful and now is nearly insensate. She describes the pain as burning and states that it is not relieved by any intervention. She takes warfarin (Coumadin), atenolol (Tenormin), digoxin (Lanoxin), and aspirin. On physical examination, her pulse is irregularly irregular. Her lungs are clear, and she has a loud holosystolic murmur heard best at the apex. Her right foot is gray and cool to the touch and has poor capillary refill. Dorsalis pedis and posterior tibial pulses are absent on the right. Her prothrombin time is 14.4 seconds (normal 11-13 seconds) and her INR is 1.4 with goal of anticoagulation being 2-3. Which of the following is the most appropriate course of action?

The symptoms and signs that she is describing, particularly in the context of atrial fibrillation (AF), suggest peripheral embolization, which is a surgical emergency. The treatment of choice involves immediate embolectomy performed by a vascular surgeon, TPA infusion therapy, or insertion of a catheter with urokinase-directed therapy. After successful embolism removal or dissolution, anticoagulation is performed along with administration of heparin, which is given to prevent new clot formation. Her subtherapeutic prothrombin time and persistent AF on examination are supportive of this diagnosis.

Which of the following diuretics acts at the distal tubule of the nephron? Ethacrynic acid, Furosemide, Hydrochlorothiazide, Mannitol, Spironolactone

The thiazide diuretics (e.g., hydrochlorothiazide, chlorothiazide, benzthiazide) promote diuresis by inhibiting reabsorption of NaCl, primarily in the early distal tubule. Thiazide diuretics act to inhibit distal convoluted tubule sodium and chloride reabsorption, causing diuresis and a drop in blood volume and blood pressure. They are very effective antihypertensive agents and are indicated as either first-line treatment for hypertension or preferred second-line agents.

A 55-year-old man with a recent syncopal episode is admitted to the hospital with congestive heart failure. His blood pressure is 160/100 mm Hg and pulse 90/min. He has a grade 2/6 harsh systolic ejection murmur. An echocardiogram reveals a thickened ventricular septum and systolic anterior motion of the mitral valve. Which of the following will most likely be found in this patient?

The thickened ventricular septum and the systolic anterior motion of the mitral valve suggest hypertrophic cardiomyopathy. The murmur is harsh and systolic and decreases when afterload increases as a result of hand grip exercise. Hand grip increases systemic vascular resistance, which allows the heart to fill with blood against increased afterload; this filled ventricle will lessen the obstruction that occurs with hypertrophic cardiomyopathy. The Valsalva maneuver decreases venous return of blood to the heart. This results in less blood in the heart, which decreases the left ventricular cavity size and worsens the obstruction in hypertrophic cardiomyopathy. Squatting increases both venous return and chamber size. It also increases systemic arterial resistance and increases most murmurs, except those caused by hypertrophic cardiomyopathy. The increase in the chamber size from the increased venous return of blood to the heart during squatting causes a reduction in the murmur.

A 56-year-old smoker with no previous medical history comes to the emergency department complaining of chest pain and shortness of breath with exertion. He is admitted to the hospital for further evaluation. An exercise stress test supports the diagnosis of coronary artery disease, and the patient undergoes a heart catheterization. There is diffuse coronary artery disease but no clearly stentable lesions. The cardiologist decides that medical management of this patient's coronary disease is appropriate at this time. The patient has no allergies and no other medical conditions. Diltiazem, HCTZ, Lisinopril, Metoprolol, Nifedipine

There is clear and convincing evidence that beta-blockers are indicated in the treatment of patients who have coronary artery disease. This is partly because in addition to lowering blood pressure, beta-blockers keep the heart rate slow, which decreases strain on the heart by decreasing myocardial oxygen demand and increases myocardial perfusion. Because this patient has coronary artery disease and hypertension, the best choice for this patient is metoprolol. Diltiazem is a non-dihydropyridine CCB that also has blood pressure lowering properties together with heart rate controlling properties. This class of medications has not been shown clearly to decrease mortality in patients who have coronary artery disease, so they are not used as first-line agents. They can be considered in patients who require heart rate control but who cannot tolerate beta-blockers (e.g., asthmatics, peripheral arterial disease). Hydrochlorothiazide together with beta-blockers can be used as a first-line blood pressure medication in many patients. It is probably an underused class of medications. Hydrochlorothiazide has synergistic blood pressure lowering effects with many other classes of blood pressure medications. Because of our patient's proven coronary artery disease, a beta-blocker would be a better choice for this patient. It would be the first add-on agent if beta-blockers failed to reach blood pressure control goals. Lisinopril. ACE inhibitors are indicated as first-line blood pressure medications in patients who have diabetes, because these agents have been shown to slow the progression of diabetic nephropathy. They also have been shown to improve survival and are first-line agents in patients who have known congestive heart failure because they act as an after-load reducer. Finally, in postmyocardial infarction patients who have left ventricular damage, ACE inhibitors seem to have a beneficial effect on survival due to a lessening of the remodeling effect on the ventricle. ACE inhibitors act as vasodilators in patients who have chronic heart failure, and they are most effective in patients who have the lowest ejection fraction from their left ventricle. Nifedipine is a dihydropyridine calcium-channel blocker that acts to inhibit calcium ion influx into vascular smooth muscle and myocardium. It acts as a predominantly peripherally acting antihypertensive. It acts as a peripheral vasodilator and decreases blood pressure. It is known to have the side effect of reflex tachycardia, so it is not first-line therapy for postmyocardial infarction or angina patients.

A 15-month-old boy is brought to the emergency department by his parents because he suddenly turned blue and had difficulty breathing while he was playing in the back yard of their home. The parents have noticed several times in the previous 2-3 months that he had developed a bluish discoloration around the lips but did not think much of it, as it was wintertime. Recently, however, the boy had increasing fatigability and would stop in the middle of playing to catch his breath by sitting down or squatting. This morning when he suddenly turned blue and started behaving in a very flustered manner they rushed him to the emergency department. The parents deny any other significant medical conditions or allergies to medication. They have a 4-year-old daughter who is in good health. On physical examination, the patient is in the fifth percentile for height and weight. His lips and fingertips are bluish in color and he frequently stops to catch his breath while playing in the examination room. There is a hint of clubbing of his fingers. Vital signs are within normal limits, but a complete blood count shows a red blood cell count of 6 x 1012/L and a hematocrit of 66%. A chest radiograph shows a boot-shaped heart with an uptilted apex and clear lung fields. On auscultation, there is a harsh systolic ejection murmur and a single S2 is heard. Which of the following is the most likely diagnosis?

This boy has a congenital heart defect known as tetralogy of Fallot. Tetralogy of Fallot is defined as pulmonary stenosis, ventricular septal defect, dextroposition of the aorta (overriding), and right ventricular hypertrophy. It is the most common type of cyanotic congenital heart disease. Symptoms depend on the size of the ventricular septal defect and the degree of the right ventricular outflow tract obstruction. Acyanotic (pink) tetralogy occurs when there is sufficient pulmonary blood flow caused by mild obstruction and the shunting across the ventricular septal defect is balanced. Typically, however, patients present with cyanosis, delayed growth and development, and dyspnea. Paroxysmal hypercyanotic attacks (hypoxic, blue, or tet spells) manifest with episodes of restlessness, cyanosis, and gasping respirations. Clubbing of the fingers and toes occurs secondary to chronic hypoxia. A loud, harsh, systolic ejection murmur is heard. S2 is single or very soft because of the pulmonary stenosis. Chest radiographs reveal a boot-shaped heart (coeur en sabot) with uptilted apex. Lung fields are clear, reflecting decreased pulmonary blood flow. Electrocardiography shows right ventricular hypertrophy and right-axis deviation. An echocardiogram reveals the anatomic abnormalities. Medical management includes maintaining the ductus open in severe right-sided obstructive lesions, but surgical correction is the definitive treatment. Blue spells are treated by placing the child in knee-chest position, sedation, oxygen, and avoiding acidosis. Beta blockade with propranolol, appropriate fluid status, and maintaining the hematocrit at 55 to 65% are also helpful. Complications have become less frequent as the surgical correction is typically performed early. Cerebral thrombosis occurs with extreme polycythemia and dehydration. Patients are usually younger than 2 years of age. Brain abscess, although less common than thrombosis, is more often seen in patients older than 2 years of age. Tet patients are at higher risk for bacterial endocarditis.

A 5-month-old girl is brought to the office by her mother, who states that the girl had an episode following feeding during which she began to breathe deeply, became blue, and then lost consciousness. The mother states that she picked her up and held her, and the infant regained her usual color and became alert. Physical examination reveals a harsh systolic murmur. The remainder of the physical examination is unremarkable. Which of the following is the most likely diagnosis?

This infant is experiencing a hypoxemic spell, as seen in patients who have tetralogy of Fallot. These hypercyanotic spells, or "Tet spells," usually are self-limited and last less than 10-15 minutes. The spells often occur immediately after feeding or when the child is crying vigorously. Tetralogy of Fallot has the following components: (1) ventricular septal defect, (2) overriding aorta, (3) right ventricular hypertrophy, and (4) pulmonic stenosis.

A 60-year-old man comes to the emergency department complaining of tearing chest pain that began abruptly and radiates to the back. His electrocardiogram is normal. Chest radiograph shows a widening of the mediastinum. What is the most likely diagnosis?

This is a classic presentation of aortic dissection. The patient may be in shock, and his pulses may be unequal. Usually there is a history of hypertension or Marfan syndrome in the affected patient. Aortic dissection is a potentially lethal cardiac emergency and requires immediate medical or surgical intervention. If it is misdiagnosed or left untreated, it may lead to mortality. Aortic dissection is classically described as a tearing sensation in the chest. This condition results from a tear in the intimal wall of the aorta. This results in blood flow being diverted into a false chamber. Chest film will show a widened mediastinum. MRI scan, CT scan, or echocardiogram are used in this evaluation. Treatment of this condition is primarily based on the location of the dissection, with proximal dissections being treated with surgical intervention and distal dissections being treated medically with blood pressure management. Blood pressure control in these patients is an essential part of treatment of this condition.

A 4-year-old, apparently healthy child is examined by a health care provider. The health care provider hears a loud systolic ejection murmur with a prominent systolic ejection click. He also hears a soft, early diastolic murmur. Both murmurs are heard best at the upper right sternal border. An electrocardiogram shows left ventricular hypertrophy. The child is active and participates in normal activities without problems. Which of the following is the most likely diagnosis?

This is aortic valve stenosis, which accounts for 5% of diagnosed cardiac defects but may actually be the most common congenital anomaly of the heart, because many minor cases are never diagnosed. Most cases are caused by bicuspid aortic valves and characteristically produce a systolic ejection murmur. An accompanying aortic insufficiency may produce an early diastolic murmur. The timing of surgical correction depends on the severity of the individual case.

A health care provider examines a 2-month-old infant who had been born at term. There is a continuous murmur at the upper left sternal border. The murmur radiates over the lung fields anteriorly. The peripheral pulses in all extremities are full and show widened pulse pressure. Which of the following is the most likely diagnosis?

This is patent ductus arteriosus, which is a failure of closure of the duct between the pulmonary artery and the aorta. As many as 80% of significantly premature (<28-week gestation) infants have patent ductus arteriosus. In term infants, delayed closure is diagnosed if the murmur of the patent ductus (described in the question stem) is still present at 6-8 weeks of age. Indomethacin is given soon after birth in premature infants and is typically started within the first 12 hours of life. This treatment is given only during the first 3 days of life in an attempt to close the patent ductus. Infants should be evaluated for other cardiac disease, as a patent ductus arteriosus may be partially compensating for other cardiac anomalies. Infants who have heart failure require prompt surgical correction. Infants who do not have heart failure or complicating cardiac defects typically undergo elective surgery at 6 months to 3 years of age to reduce the risk for infective endocarditis later involving the patent ductus. If this condition is identified in the adult population, treatment consists of a percutaneous catheter to close or surgical ligation of this structure. Coarctation of the aorta will cause diminished and delayed pulses in the legs and sometimes the arms. There may be a differential in blood pressure readings between the arms. There is typically higher blood pressure readings in the upper extremities compared to the lower extremities.

A 50-year-old surgical nurse consults a health care provider because of a rash above both her ankles. She says that her ankles have been swollen for a few months but she only recently noticed the rash. Physical examination demonstrates marked ankle edema with erythema, mild scaling, and brown discoloration of the overlying skin of the distal lower legs. Varicose veins are also noted. Which of the following is the most likely diagnosis?

This is stasis dermatitis, which is a persistent inflammation of the skin of the lower legs. The condition is often related to varicose veins, although it has been postulated that the true cause may instead be perivascular fibrin deposition and abnormal small vessel vasoconstrictive reflexes. This presentation is typical. Most patients are relatively asymptomatic and may not seek medical attention until the edema becomes severe or the lesions become complicated by secondary bacterial infection or ulceration. It is important to increase the venous return to the heart by elevating the ankles while resting and using properly fitted support hose. Unna boot application can be helpful. Purulent lesions can be treated with hydrocolloid dressings. Ulcers are treated with compresses and bland dressings, such as zinc oxide paste.

A 32-year-old woman comes to the hospital for an elective repeat cesarean delivery. Four years ago she had a primary cesarean delivery for a nonreassuring fetal heart rate tracing. Two years ago she chose to have an elective repeat cesarean delivery rather than attempt a vaginal birth after cesarean (VBAC). Her prenatal course was uncomplicated except that she has mitral valve prolapse. An echocardiograph demonstrated the mitral valve prolapse but no other structural cardiac disease. Which of the following antibiotic regimen is the correct management of this patient specifically as related to her mitral valve disease?

no antibiotics are needed. mitral prolapse is moderate risk cardiac condition.

When intravenous lines cannot be established, the preferred alternate route is intraosseous (IO) cannulation. This is done by placing a trocar in the bone marrow of a long bone. The site of choice in children is the proximal tibia; alternative sites are the distal tibia and proximal femur. Careful attention should be made in regard to IO location in order to avoid injury to the growth plate which could stunt bony growth as the child grows older.

This is tetralogy of Fallot, in which severe obstruction of right ventricular outflow and a ventricular septal defect allow unoxygenated blood to pass from the right side of the heart to the left. In severe cases, cyanosis presents at birth; in milder cases (such as this infant has), it develops more slowly. The upper left sternal border ejection murmur is caused by right ventricle outflow obstruction. The ECG and chest radiographic findings described in the question stem are typical for older infants who have tetralogy of Fallot. Early surgical repair is now recommended for tetralogy of Fallot.

A 4,000-g male neonate develops severe cyanosis that begins within minutes of birth. Blood drawn 1 hour after birth shows metabolic acidosis with respiratory acidosis. A chest radiograph shows a narrow mediastinum, narrow heart base, and absence of the pulmonary artery. An electrocardiogram is normal. An echocardiogram is ordered and the report is pending. Which of the following is the most likely diagnosis?

This is transposition of the great arteries, in which the aorta arises from the right ventricle and the pulmonary artery arises from the left ventricle. Approximately 5% of congenital cardiac anomalies have transposition of the great arteries. Affected infants present within minutes of birth with severe cyanosis and metabolic acidosis secondary to inability to oxygenate tissues. The only exchange of blood between pulmonic and circulatory systems is typically occurring through a patent ductus arteriosus. The chest radiograph changes include a narrow mediastinum, narrow heart base, and absence of the pulmonary artery; these are caused by superposition of the great vessels (rather than the normal side-to-side position). Surgical repair is usually performed within 7 to 10 days of life. In transposition of great vessels, because the aorta is coming out of the right ventricle it will completely cover the pulmonary trunk. This will be seen as absence of the pulmonary artery with a narrow mediastinum on chest radiograph ("egg on a string" appearance). With this transposition, the pulmonary artery comes out of the left ventricle. Because of these changes, deoxygenated systemic venous blood travels from the right ventricle to the aorta without passing through the lungs, and pulmonary venous blood travels from the left ventricle to the lungs via the pulmonary artery. The systemic and pulmonary circulations parallel each other rather than occur sequentially. This is not compatible with life unless the two chambers connect via VSD, ASD, or patent ductus arteriosus. Prostaglandin E1 is used to maintain patency of the ductus arteriosus. An arterial switch operation is performed in the newborn period most commonly

A 71-year-old white man comes to the emergency department because of blurry vision and blood-tinged urine. He states that he has a long history of hypertension treated with a beta-blocker, an ACE inhibitor, and a calcium channel blocker. He reports that he has had 3 days of blurry vision and urine that is "cola" colored. The symptoms began when he ran out of his medications 3 days ago and he has "not had the time" to get the prescriptions refilled. His physical examination is remarkable for a blood pressure of 200/110 mm Hg in both arms and funduscopic examination showing flame hemorrhages and papilledema. Urinalysis shows red blood cells and some dysmorphic red blood cell casts. He has an abdominal bruit. Which of the following is the most appropriate next step in management?

This patient has a hypertensive emergency, as evidenced by both the presence of elevated systolic and diastolic blood pressure and evidence of end-organ damage. The blood pressure numbers themselves are a good reason for urgency, but the presence of increased intracranial pressure and renal failure require that this patient be triaged as an emergency. Elevated blood pressure alone, in the absence of symptoms or end-organ damage, rarely requires emergency therapy. The therapy for a hypertensive emergency requires IMMEDIATE LOWERING of the blood pressure by approximately 20-30 mm Hg by intravenous administration of medication. The blood pressure should not be reduced to normotensive levels because of the risk for watershed cerebral infarcts with such a dramatic reduction.

A 68-year-old man is brought to the emergency department following a high-speed automobile accident. He is alert and complains of chest pain and mild back pain. His blood pressure is 80/60 mm Hg. Chest radiograph shows a widened mediastinum, tracheal deviation, bronchial displacement, and loss of the aortic knob. Which of the following is the most likely diagnosis?

This patient has a traumatic aortic rupture, which is the most common cause of immediate death following a motor vehicle accident. Deceleration causes twisting of the aorta and may lead to rupture. Up to 90% of individuals die at the scene, however a high index of suspicion and early intervention may save the remaining few. The symptoms include chest pain, back pain, and hypotension. Angiography is the gold standard and immediate surgery is mandatory. Radiographic findings include widening of the mediastinum, alteration of the aortic knob, pleural cap, tracheal deviation, bronchial displacement, and esophageal deviation.

A 22-year-old man comes to the emergency department with a 3-day history of fever, chills, a cough, pleuritic chest pain, and low back pain. He says that the symptoms came on "out of the blue." He is the son of a wealthy local businesswoman and still lives at home, which he says "is cool because my parents are never around." His temperature is 39°C (102.2°F), blood pressure 120/80 mm Hg, pulse 70/min, and respirations 16/min. Physical examination shows oval retinal hemorrhages with a clear, pale center and pinpoint lesions between his toes. Blood cultures are drawn. A chest radiograph shows multiple patchy infiltrates. Laboratory studies show hemoglobin 11 g/dL, hematocrit 39%, and erythrocyte sedimentation rate 39 mm/h. Which of the following is the most likely pathogen?

This patient has acute bacterial endocarditis, most likely caused by Staphylococcus aureus, the most common organism causing endocarditis in intravenous drug abusers. The "pinpoint lesions" between his toes are signs of injection drug abuse. Acute endocarditis in drug abusers typically presents with a high fever, pleuritic chest pain, and a cough. The tricuspid valve is commonly affected in these patients. A murmur may not be present in early acute endocarditis or in injection drug abusers who have tricuspid valve disease. Roth spots are oval, pale, retinal lesions that are surrounded by hemorrhages. Other physical manifestations of endocarditis include Janeway lesions, which are hemorrhagic, painless, macular plaques typically located on the palms and soles, and Osler nodes, which are small, painful nodular lesions typically found on the pads of the fingers or toes. Other findings include anemia and an elevated erythrocyte sedimentation rate. Diagnosis is with blood cultures, which are typically positive for S. aureus, and with echocardiography.

A 47-year-old man comes to his primary care provider for follow-up care of his hypertension. The patient has a 10-year history of hypertension that has been treated with a variety of medications. It is now well controlled with a beta-blocker and thiazide diuretic combination. The patient's mean blood pressure over the past 6 months has been 145/85 mm Hg. His other medical history is remarkable only for type 2 diabetes mellitus. He currently takes a low-dose oral sulfonylurea agent and has had blood glucose values ranging from 110-220 mg/dL. His most recent hemoglobin A1c value was 7.9%. He has no allergies to any medications and he quit smoking cigarettes 2 years ago. Which of the following is the most appropriate management at this time?

This patient has diabetes and hypertension. The management strategy for such patients includes addressing the primary blood sugar problem, the primary blood pressure problem, and the secondary consequences of both the diabetes and the blood pressure. This man has not reached the blood pressure goals for patients who have diabetes, which is 130/80 mm Hg. To further lower his blood pressure, an appropriate intervention would be to add an ACE inhibitor to deal with diabetic complications, such as renal disease, as well as to address further lowering of his blood pressure. Increasing his sulfonylurea dose to better control his sugar is also appropriate, because the patient's hemoglobin A1c is not at the goal of <7%.

The EKG of a 60-year-old man reveals widened QRS intervals of 0.14 seconds with distinctly abnormal configurations. Physical examination is significant for paradoxic splitting of the second heart sound. Which of the following valvular defects is likely in this patient?

This patient has bundle branch block, as implied by the QRS interval greater than 0.12 seconds and by paradoxic splitting of the second heart sound. The typical sequence of valve closure is mitral, tricuspid, aortic, and pulmonic. If there is a paradoxic splitting of the second heart sound, the aortic valve closes after the pulmonic valve. This situation can occur with anything that slows the conduction in the left ventricle, such as complete heart block, left bundle branch block, or aortic stenosis. Left bundle branch block results in conduction occurring in a cell-to-cell depolarization pattern that slows the conduction through the left ventricle, which causes the aortic valve to close after the pulmonic valve.

A 10-year-old girl is brought to the office because of fever and chills for 3 days. She had been complaining of a headache and feeling tired for 10 days before she developed fever. The parents had tried giving her acetaminophen, but the child's condition was not improving. Her past medical history is significant for frequent streptococcal throat infections over the past 2 years and a new onset heart murmur detected at her last well child visit 2 months earlier. She is on no medication currently, except for acetaminophen. Otherwise, she has been wearing dental braces for the past year and had a primary tooth extracted 2 weeks earlier in the attempt of liberating space for permanent teeth. On physical examination, the patient is diaphoretic, in moderate distress with a temperature of 39.0°C (102.2°F), blood pressure of 90/60 mm Hg, pulse of 110/min, and respirations of 24/min. On her fingernails, you note several splinter hemorrhages. Auscultation confirms a grade 2/6 high-pitched, blowing, systolic ejection murmur, best heard at the apex that radiates to the left axilla. A chest radiograph is unremarkable, but the electrocardiogram shows signs of left ventricular strain. An echocardiogram reveals vegetations on the mitral valve. Which of the following microorganisms is the most likely cause of this patient's current condition?

This patient has developed acute endocarditis most likely caused by Streptococcus viridans that entered the bloodstream during the recent dental procedure. The predisposing factor for her current illness is rheumatic heart disease, a sequela of prior Group A β-hemolytic streptococcal pharyngitis, which was complicated by involvement of the mitral valve. Endocarditis is most commonly caused by S. viridans, although in some series S. aureus is more common, especially when there is no underlying heart disease. S. viridans is more common if endocarditis occurs after dental procedures. P. aeruginosa and S. marcescens are seen as common pathogens in intravenous drug abusers. Fungal causes are seen after open-heart surgery. Endocarditis is frequently associated with congenital or rheumatic heart disease. Surgical or dental procedures are commonly the predisposing incident. The highest risk is seen in patients who have high-velocity blood flow, such as in ventricular septal defect and left-sided obstructive lesions. Clinically, the onset of symptoms may be acute or insidious. Fever, chills, arthralgias, myalgias, development of a new murmur, splenomegaly, and petechiae may occur. S. aureus infection is associated with neurologic complications. Skin manifestations are seen late in the disease evolution and most likely represent vasculitis. These include Osler nodes (tender nodules on the finger and toe pads), Janeway lesions (painless hemorrhagic lesions on the palms and soles), and splinter hemorrhages (linear lesions beneath the nails). A positive blood culture is the most important piece of information in establishing the diagnosis of endocarditis. Leukocytosis, increased sedimentation rate, and anemia are helpful but secondary in importance to a positive blood culture. Echocardiography may reveal vegetations on the heart valves. Antibiotics are the mainstay of treatment of endocarditis. Therapy should last 4 to 6 weeks. Congestive heart failure should be treated if present prevention is with antibiotic prophylaxis before and after dental or surgical procedures. Complications of endocarditis include heart failure and systemic embolism. Mortality rate is 20 to 25%.

A 35-year-old woman presents to her health care provider with paresthesias of the left shoulder and arm. She has also noticed a hard, bony structure on the left side of her neck above the clavicle. Which of the following diagnoses best accounts for her symptoms?

Thoracic outlet syndrome (patient has a left cervical rib. This anatomic variant, which is typically bilateral but can occur on only one side, is caused by formation of an extra rib at the C7 level. Cervical ribs, while often asymptomatic, can cause thoracic outlet obstruction. This results in pain caused by distortion of blood vessels; pain or paresthesias related to brachial plexus impingement -notably sensory disturbances in the distribution of the ulnar nerve-, and palpable abnormalities in the greater supraclavicular fossa.)

A 61-year-old woman comes to the health care provider for her first physical examination in more than 10 years. She reports that she has been in excellent health, does not smoke or drink, and runs 3 miles daily. She is a retired accountant and has 3 healthy grown children. She has been taking 81 mg of aspirin daily after reading about its cardioprotective effects in the newspaper. On physical examination she appears well. Blood pressure is 122/76 mm Hg, pulse 70/min, and respirations 14/min. She is afebrile. Head and neck examination is normal. There is no jugulovenous distention. Lungs are clear. On cardiac examination she has a regular S1 and S2, and II/VI crescendo blowing diastolic murmur is heard at the aortic area. Abdominal examination is normal. Rectal examination shows no masses and brown, guaiac-negative stool. Which of the following most likely explains the cardiac findings on physical examination?

This patient has findings of asymptomatic aortic insufficiency on physical examination. This lesion may result from a number of causes, several of which are infectious in etiology. Aortic insufficiency may result as a sequela of rheumatic heart disease, which occurs as an immunologic response to a streptococcal infection. Acute rheumatic fever is typically characterized by cardiac involvement that may cause pericarditis, myocarditis, or endocarditis. Often the initial cardiac manifestations are asymptomatic and become apparent only years later with the development of cardiac valvular disease.

A 25-year-old man comes to the health care provider with chest pain at rest that is not always related to exercise. He reports two prior episodes of fainting during exercise but has otherwise been healthy. He reports a similar history in other family members and notes that his father suddenly collapsed and died at age 50 after playing tennis. He denies use of cocaine or other recreational drugs and does not take any medications. Physical examination reveals a systolic ejection murmur that is loudest along the left sternal border. The rest of the physical examination is unremarkable. Echocardiography shows asymmetric septal hypertrophy without obstruction. Which of the following interventions would be most likely to decrease this patient's systolic murmur?

This patient has hypertrophic obstructive cardiomyopathy. This is a cardiac condition in which the interventricular septum hypertrophies excessively and asymmetrically. The hypertrophied septum and the anterior leaflet of the mitral valve produce left ventricular outflow obstruction. Most cases are inherited through an autosomal-dominant mode of transmission. The systolic ejection murmur is diminished when the patient lies down, because this maneuver increases cardiac size by increasing venous return and tends to diminish the intensity of the murmur. Afterload is increased, and venous return is increased to the heart. This increases the ventricular size and diminishes the murmur. Maneuvers such as Valsalva which diminish left ventricular size bring the anterior leaflet of the mitral valve closer to the septum and increase the amount of obstruction. This increases the intensity of the murmur.

A 57-year-old woman with a history of rheumatic fever as a child comes to the her health care provider complaining of a 6-month history of slowly progressive dyspnea on exertion and orthopnea. Her temperature is 37°C (98.6°F), blood pressure is 110/60 mm Hg, pulse is 93/min and irregular, and respirations are 18/min. Cardiac examination reveals a localized mid-diastolic murmur near the apex. There is a loud opening snap heard after S2. The rhythm appears irregular. Which of the following additional findings will most likely be present on physical examination?

This patient has mitral stenosis. Most adults who have mitral stenosis have had rheumatic fever as a child, although not all patients are aware of having had this infection. Mitral stenosis decreases left ventricular filling and elevates left-sided atrial pressures. This causes pulmonary congestion and results in symptoms of left-sided heart failure, such as shortness of breath and dyspnea on exertion. Hemoptysis sometimes occurs as a result of rupture of small pulmonary blood vessels. Later in the course of the disease, patients may develop pulmonary hypertension and cor pulmonale because the right ventricle has to work against increased pressures secondary to chronic pulmonary congestion. There may be a loud P2 as a sign of pulmonary hypertension and/or right ventricular failure. Patients may have an opening snap caused by the calcification of the stenotic mitral valve. Mitral stenosis is a harsh mid-diastolic murmur heard best at the apex of the heart. Shortening of the interval before the opening snap indicates a higher degree of stenosis (the pressures in the left atrium force the valve open earlier). Atrial fibrillation is common with mitral stenosis. Early in the course of the disease, the intensity of S1 is increased. This is because the stenosis prevents the valve from closing spontaneously after diastole. Late in the course of the disease, the valve becomes too stenotic to open or close, and therefore the intensity of S1 decreases.

A 50-year-old man is brought to the emergency department complaining of light-headedness. He has a history of lung cancer, which was diagnosed a month ago and found to be widely metastatic to the bone and pericardium. On physical examination his blood pressure is 70/40 mm Hg and pulse 100/min. Heart sounds are distant and soft. ECG demonstrates low voltage, and electrical alternans is present. Chest radiograph shows that the cardiac silhouette has a "water bottle" appearance. Which of the following is the most appropriate intervention?

This patient has pericardial/cardiac tamponade, most likely as a result of his malignancy. Lung cancer is particularly likely to cause pericardial effusions. Furthermore, because this patient has metastases to the pericardium, he might be bleeding into the pericardial space. This tamponade may be the cause of his significant hypotension and the soft cardiac sounds. Electrical alternans, a phenomenon in which the QRS changes axis, is indicative of pericardial effusion, because the heart is moving freely in the fluid, causing this change in axis noted on the ECG. Emergently, this patient needs decompression of the pericardial space with the aid of pericardiocentesis, in which a catheter directly drains the fluid in the pericardial sac. Nonsteroidal anti-inflammatory drugs (NSAIDs) can be useful in treating pericarditis, which may lead to pericardial effusions. This is a longer term option, however, and will have little utility emergently. Steroids may similarly be used in pericarditis, after NSAIDs have failed. This is an option to be explored after the pericardial fluid has been drained, however.

A 59-year-old man chooses a health care provider. The health care provider discovers that the patient was recently admitted to the hospital for palpitations and was found to have newly diagnosed atrial fibrillation (AF). He was placed on digoxin (Lanoxin) and verapamil (Calan, Isoptin, Verelan) for rate control and was loaded with warfarin (Coumadin) for anticoagulation. After several trips to the anticoagulation clinic, he decided to stop taking his warfarin, as it was difficult for him to leave work during the day. He now calls the office complaining of a cold, pale, and painful right leg starting an hour ago. Which of the following is the most likely explanation for his symptoms?

Thromboembolism (Overall, atrial fibrillation confers about a 1% annual risk for a thromboembolic event, which, among other things, may cause stroke, intestinal ischemia, renal infarcts, or a threatened limb. Specific etiologies of AF such as valvular disease are associated with higher rates of embolic events.- NO DVT! This is an Arterial problem from thromboembolism)

A 67-year-old woman comes to the clinic for review of her medications. She has had a history of hypertension for 25 years, type 2 diabetes for 20 years, and congestive heart failure for 5 years. She is a former smoker of two packs of cigarettes per day and her lipid status is not known at this time. Her current medications include nifedipine (Adalat, Procardia), hydralazine (Apresoline), isosorbide dinitrate (Isordil), glyburide (DiaBeta, Micronase), a multivitamin, and conjugated estrogens. Today in the clinic her blood pressure is 160/90 mm Hg, her fasting blood glucose is 210 mg/dL, and her hemoglobin A1c is 7.9%. She reports moderate dyspnea on exertion, unchanged from previous visits. Which of the following is the most appropriate intervention at this time?

This patient has poorly controlled hypertension and poorly controlled diabetes. She needs improved therapy for both; the issue is how best to do that. The concept underlying this question is the absolute importance of using ACE inhibitor therapy on both type 1 and type 2 diabetic patients. Many clinical trials have shown the beneficial effects of ACE inhibitors on preventing nephropathy and slowing the progression of established nephropathy in diabetics. It is the standard of care that all diabetics be given an ACE inhibitor if they are able to tolerate its blood pressure effects. Given that she has congestive heart failure and hypertension, the ACE inhibitor also will be efficacious in their treatment. In fact, ACE inhibitors have been shown to be superior to hydralazine and isosorbide dinitrate in terms of morbidity and mortality in treatment of CHF; all patients who have symptomatic CHF, regardless of ejection fraction, should be placed on one.

A 16-year-old boy is brought to the emergency department by his parents because of an episode of severe chest pain several hours earlier. Review of symptoms reveals that the patient is recovering from a flu-like illness that started a week earlier. He had been complaining of fever, chills, abdominal discomfort, and feeling tired during the prior week. He has had a low-grade fever for 5 to 6 days and a sore throat, but pharyngeal swab cultures done at the primary care physician's office had been negative and he was taking over-the-counter cold medication only. Physical examination reveals a well developed and well nourished young man in moderate distress. His temperature is 37.8°C (100.0°F), pulse is 120/min, and respirations are 28/min. A differential white blood cell count shows a normal number of neutrophils and marginally elevated lymphocytes. An electrocardiogram shows low voltage QRS complexes throughout the limb leads. Chest radiography is remarkable for increased pulmonary markings and an enlarged heart silhouette. Which of the following is the most appropriate next step in the management?

This patient has signs of myocarditis and should be admitted to a monitored hospital bed for further evaluation and management. Myocarditis is an inflammation of the myocardium. The etiology may be multiple, but it is most commonly viral, caused by adenovirus and Coxsackie B. Bacteria (diphtheria), Rickettsia, fungi, and parasites are also infectious causes of myocarditis. Connective tissue diseases, granulomatous diseases, and toxins may cause noninfectious myocarditis. The most common clinical presentation is heart failure.

A 77-year-old man comes to the health care provider because of decreasing exercise tolerance. Just 1 year earlier he was able to play doubles tennis for 2 hours. Over the past few months, however, he has had progressive dyspnea on exertion and now can walk only 2 blocks on level ground before becoming short-winded. He has also been awaking from sleep with shortness of breath and requires 3 pillows to sleep comfortably. He has a history of rheumatic fever as a teenager. On physical examination his blood pressure is 168/60 mm Hg, pulse 92/min, and respirations 18/min. He is afebrile. He has jugulovenous distention lying supine. He has bibasilar rales extending 1/4 up both posterior lung fields. He has a regular S1 and S2, with a blowing diastolic murmur heard at the aortic area, which is grade II/VI. An S3 is audible. The liver edge is mildly tender, and there is moderate lower extremity edema extending to both knees. Which of the following medications will most likely be effective in the management of his cardiac disorder?

This patient has the physical findings of aortic insufficiency and the development of congestive heart failure with biventricular failure. Left ventricular symptoms described here are the findings of pulmonary congestion, and right-sided heart failure is demonstrated by the jugulovenous distention, congested liver, and peripheral edema. Because the "backward" failure symptoms are caused by regurgitant flow across the incompetent aortic valve, the most useful therapy would be afterload reduction with an ACE inhibitor such as captopril, which will also help to prevent cardiac remodeling as the left ventricle becomes overloaded from the increased amount (volume) of blood that occurs because of aortic regurgitation. Aortic regurgitation may be helped by vasodilator therapy, which will act as afterload reducers. In addition to ACE inhibitors, pure vasodilators such as hydralazine and nifedipine can also be used. For patients who have ongoing symptoms who are surgical candidates, aortic valve replacement is performed when the disease decompensates.

A 67-year-old man comes to the health care provider complaining of increasing dyspnea on exertion. Over the past 3 weeks, he has noted increasing shortness of breath while walking to the bus stop three blocks from his home. He has also found it difficult to sleep comfortably while lying flat and now sleeps on three pillows. He has awoken several times in the past month with shortness of breath. He denies any history of chest pain. He does not smoke, and his cholesterol tests were normal 1 year earlier. On physical examination, he appears comfortable at rest. His blood pressure is 158/56 mm Hg, pulse is 86/min and regular, and respirations are 16/min. He has jugulovenous distention while being examined at 30 degrees. His lungs have bibasilar rales. On cardiac examination, there is a regular S1 and S2, with a blowing diastolic murmur heard loudest at the left sternal border. A I/VI systolic murmur is heard throughout the precordium. An extra heart sound immediately following the S2 is heard. There is mild lower extremity pedal edema bilaterally. Which of the following is the most likely cause of his symptoms?

This patient has the typical symptoms of congestive heart failure with left ventricular failure. The physical examination has the characteristic findings for aortic insufficiency, including wide pulse pressure, blowing diastolic murmur, and an S3 consistent with left ventricular dilatation. The soft systolic murmur is caused by the volume overload and increased flow in the left ventricle and does not imply an additional valvular disorder.

A 55-year-old man is brought to the emergency department because of increasing incoherence over the past 24 hours. He has a history of hypertension and diabetes. He is disoriented. On physical examination, he is afebrile. His blood pressure is 230/130 mm Hg, pulse is 120/min, and respirations are 24/min. He has an S4 on cardiac examination. The patient is placed on a cardiac monitor, and IV and intra-arterial lines are placed. A head CT scan shows no mass or bleed. Which of the following is the most appropriate next step in management?

This patient is having a hypertensive emergency, given the presence of end-organ damage (mental status changes) in the setting of hypertension. Hypertensive encephalopathy allows the definition of hypertensive emergency rather than hypertensive urgency to be used. Immediate therapy is needed, before the laboratory results are known. This requires immediate but not precipitous lowering of the blood pressure over a period of minutes to hours. Labetolol acts as a combined alpha- and beta-blocker and will decrease systemic vascular resistance, mean arterial pressure, and heart rate with minimal change in cardiac output. It is given intravenously with an onset of action in 5 to 10 minutes with a duration of action of 3 to 8 hours. Hypertensive emergency is defined as a systolic blood pressure >210 mm Hg and diastolic blood pressure >130 mm Hg along with changes consistent with end-organ damage. When this condition is suspected, treatment should be performed without delay. Blood pressure should be lowered within minutes to hours with parenteral agents used in a monitored setting. The initial goal of therapy is to lower mean arterial blood pressure by no more than 25% within the first hour. Further blood pressure lowering may be needed in patients who have intracranial hemorrhage or aortic dissection. If the patient's blood pressure is lowered too rapidly there may be an increase in renal, cerebral, or coronary ischemia.

A 49-year-old man with a history of hypertension and hyperlipidemia has had a severe headache, blurry vision, and red urine for the past 2 days. He was in his usual state of health until 2 days ago. He has been compliant with his medications, including atenolol 50 mg/day and hydrochlorothiazide 12.5 mg/day. His temperature is 37.0°C (98.6°F), blood pressure 196/140 mm Hg, pulse 83/min, and respirations 10/min. The remainder of his physical examination is unremarkable. Which of the following is the most appropriate intervention at this time?

This patient is having a hypertensive emergency, which is defined as severe hypertension with end-organ damage. It happens for unknown reasons in approximately 1% of hypertensive patients. Our patient has hematuria, headache, and blurred vision. He needs to be admitted to the hospital and placed on intravenous blood pressure medications such as labetolol as soon as possible. Remember not to lower the blood pressure too far, too rapidly, because doing so might compromise cerebral, renal, or coronary perfusion. The goal for decreasing blood pressure is no more than 25% of the mean arterial blood pressure within the first minutes to hours.

A 57-year-old man with hypertension and hyperlipidemia comes to the emergency department complaining of crushing substernal chest pain, diaphoresis, and nausea for 3 hours. He denies palpitations or shortness of breath. He has no previous history of angina or myocardial infarction. His temperature is 37.0°C (98.6°F), blood pressure is 82/60 mm Hg, pulse is 103/min, and respirations are 20/min. Physical examination shows jugular venous distention, a tachycardic heart that is without murmurs, and clear lungs. An electrocardiogram is shown. A right-sided electrocardiogram is also performed that shows ST elevation in V3 and V4. Which of the following is the most appropriate medical intervention at this time?

This patient is having a right ventricular infarct. The triad of jugular venous distention, hypotension, and clear lungs should make you suspect the diagnosis. In addition, the electrocardiogram shows inferior ST-segment elevation (II, III, and aVF), which is consistent with RV infarct. Right-sided electrocardiogram can be used to support this diagnosis. The finding of elevations in lead V4 on a right-sided electrocardiogram is 100% specific for a right-sided infarct. The management of right ventricular infarct requires aggressive fluid resuscitation, as this type of myocardial infarction is fluid-dependent. The infarcted patient is now preload-dependent. They require aggressive intervention to keep the preload high. Aggressive intravenous fluids therefore should be started until the patient stabilizes or goes for cardiac catheterization. Intravenous fluids should be first-line in maintaining adequate blood pressure.

A 34-year-old woman with HIV and active Pneumocystis pneumonia is admitted to the hospital. Her last CD4 count was 44 cells/mm3 (normal 400-1,400 cells/mm3). She has been doing reasonably well since admission with a stable course on appropriate antibiotics. On day 2 of admission, vital signs reveal blood pressure 80/40 mm Hg and pulse 110/min. Temperature is 38.3°C (101°F) orally. Her extremities are cool and damp. Mental status examination is normal. The remainder of her physical examination is unchanged. What is the next best step in management?

This patient is hypotensive but has minimal effects from the hypotension, such as altered mental status or signs of shock. This blood pressure, however, is still not acceptable, and the cause for it must be determined. In the interim, the symptom (low blood pressure) must be treated. The standard therapy in all such situations is volume. Because she has no signs of altered mental status, there is no urgent indication for pharmacologic blood pressure support, and IV fluids are the first-line initial management tools for this patient.

A 12-year-old girl presents to her health care provider with an erythematous macular skin rash in a bathing suit distribution. She has a history of streptococcal sore throat a few weeks ago. Which of the following signs would be necessary to make the diagnosis of rheumatic fever?

This patient's macular skin rash in a bathing suit distribution is also known as erythema marginatum, one of the five major Jones criteria for diagnosing rheumatic fever. The other four major criteria are migratory polyarthritis, Sydenham chorea, subcutaneous nodules, and pancarditis. In context of prior streptococcal infection, the presence of two of five major criteria or one major plus two minor criteria is sufficient to establish the diagnosis. A mnemonic for the major criteria is CANCER (not pleasant, but easy to remember): Chorea, Arthritis, Nodules, Carditis, Erythema marginatum, which is consistent with Rheumatic fever diagnosis.

_________________- occurs as a result of compression of one or more neurovascular structures in the superior portion of the chest. Presence of a cervical rib is a known risk factor for this condition. If the nerve is compressed, the patient will have paresthesias and motor weakness along with pain in the affected dermatome. Physical stress will worsen these symptoms, and the Adson maneuver can be used as part of the assessment for this condition. Treatment may consist of physical therapy with core strengthening. Surgical correction of the anatomic abnormality is used if conservative therapy fails.

Thoracic outlet syndrome

Headache of sudden onset ("thunderclap" headache), rapid deterioration of mental status, and blood in the CSF are virtually diagnostic of ruptured ____________.

berry aneurysm

A 59-year-old man chooses a health care provider. The health care provider discovers that the patient was recently admitted to the hospital for palpitations and was found to have newly diagnosed atrial fibrillation (AF). He was placed on digoxin (Lanoxin) and verapamil (Calan, Isoptin, Verelan) for rate control and was loaded with warfarin (Coumadin) for anticoagulation. After several trips to the anticoagulation clinic, he decided to stop taking his warfarin, as it was difficult for him to leave work during the day. He now calls the office complaining of a cold, pale, and painful right leg starting an hour ago. Which of the following is the most likely explanation for his symptoms?

Thromboembolism (not DVT) Overall, atrial fibrillation confers about a 1% annual risk for a thromboembolic event, which, among other things, may cause stroke, intestinal ischemia, renal infarcts, or a threatened limb. Specific etiologies of AF such as valvular disease are associated with higher rates of embolic events.

Which of the following is an advantage of tissue plasminogen activator (tPA) over streptokinase for fibrinolytic therapy?

Tissue plasminogen activator is produced by and secreted from endothelial cells. Because it is not a foreign protein like streptokinase (derived from hemolytic streptococci), tPA is not allergenic. Both drugs can be used in the setting of acute myocardial infarction. Both drugs can result in hemorrhage, especially during prolonged therapy for treatment of pulmonary embolism or venous thrombosis. tPA is as much as 10 times more expensive than streptokinase. Both drugs result in the activation of plasminogen.

____________- is a polymorphic ventricular tachycardia with a twisting morphology that can result in a fairly normal cardiac output or a hemodynamically compromised situation. IV magnesium sulfate may play a role in the management of it provided that the patient is magnesium deficient. If pt is on __________ should be withdrawn as part of the treatment regimen and any underlying electrolyte abnormalities should be corrected.

Torsades, Amiodarone

A 72-year-old man is scheduled to have elective sigmoid resection for diverticular disease. He has a history of heart disease and had a documented myocardial infarction 2 years ago. He currently does not have angina, but he lives a sedentary life because "he gets out of breath" if he exerts himself. Physical examination reveals jugular venous distention. His hemoglobin level is 9 g/dL (normal 14-17 g/dL in men). If surgery is indeed needed, which of the following should most likely be done prior to the operation?

Treat for CHF (Jugular venous distention in this setting is indicative of congestive heart failure, a condition that would make elective surgery very risky. Medical treatment for congestive heart failure can reduce the risk. This patient has right-sided heart failure, which is most commonly caused by left-sided heart failure)

A 72-year-old man is scheduled to have elective sigmoid resection for diverticular disease. He has a history of heart disease and had a documented myocardial infarction 2 years ago. He currently does not have angina, but he lives a sedentary life because "he gets out of breath" if he exerts himself. Physical examination reveals jugular venous distention. His hemoglobin level is 9 g/dL (normal 14-17 g/dL in men). If surgery is indeed needed, which of the following should most likely be done prior to the operation?

Treat the patient for CHF. Jugular venous distention in this setting is indicative of congestive heart failure, a condition that would make elective surgery very risky. Medical treatment for congestive heart failure can reduce the risk. This patient has right-sided heart failure, which is most commonly caused by left-sided heart failure.

Which of the following diuretic agents is most likely to cause hyperkalemia? Acetazolamide, Furosemide, HCTZ, Metolazone, Triamterene

Triamterene is grouped in the category of potassium-sparing diuretics. Along with spironolactone and amiloride, these diuretic agents may cause excess renal retention of potassium. In this manner, use of these drugs may increase potassium levels and cause hyperkalemia. Acetazolamide is a carbonic anhydrase inhibitor that initially causes a moderate amount of potassium loss. In addition to being used as a diuretic, it has a role in the management of glaucoma, heart failure, and altitude sickness (it causes a metabolic acidosis that results in increased respiratory drive as the respiratory system attempts to cause respiratory alkalosis as a compensatory mechanism). Furosemide is a loop diuretic that causes a moderate to severe loss of potassium. Hydrochlorothiazide and metolazone are thiazide diuretics that cause moderate amounts of potassium loss. The majority of diuretics used in clinical practice result in hypokalemia and hyponatremia because of the loss of these electrolytes as part of their clinical utility in causing diuresis. The potassium-sparing diuretics inhibit distal convoluted tubule aldosterone-induced sodium resorption, which permits sodium loss and potassium retention. Although potassium-sparing diuretics cause weak diuresis, there is evidence showing that there is mortality benefit when these agents are used as add-on agents for chronic heart failure. These agents should be used with caution in patients who have impaired renal function with resultant hyperkalemia. These agents should also be used with caution in patients who are on medications that are known to lower glomerular filtration pressure (such as ACE inhibitors and ARBs), as these agents are known to cause hyperkalemia.

___________- causes stimulation of the vagus nerve. One way to perform is by straining down to defecate, carotid massage, eyeball pressure, and putting the face in cold water. -The *Vagus Nerve* is responsible for putting on the breaks in the conduction through the AV Node and decrease SBP by 50mmHg or Sinus Pause for 3 seconds

Valsalva Maneuver

A man complains to his health care provider that he feels lightheaded and has even fainted during defecation. This is most probably an example of syncope due to which of the following mechanisms?

Valsalva Maneuver (Syncope has a broad differential diagnosis, because fainting can be produced by a wide variety of mechanisms. All of the mechanisms listed in the answers can produce syncope, but only the Valsalva mechanism (in which high intra-abdominal pressures trigger a reflex fall in cardiac output from vagal stimulation) is specifically associated with defecation. This mechanism can also produce fainting during weightlifting and with the use of wind instruments.)

Valvuloplasty is indicated for mitral valve stenosis in which of the following scenarios?

Valvotomy is indicated in the symptomatic patient who has a mitral orifice less than 1.0 cm2/m2 BSA. Significant obstruction begins when the orifice is less than 2 cm2. Severe mitral stenosis with a valve opening of 1 cm2 would require a left atrial pressure of 25 mm Hg to maintain normal cardiac output.

Tetralogy of Fallot has the following components: (1) _________ septal defect, (2) overriding aorta, (3) ______ _______ hypertrophy, (4) ___________ stenosis.

Ventricular, Right Ventricular, Pulmonic

A 2-year-old boy is shot in the arm in a drive-by shooting. His brachial artery is partially transected and there is copious bleeding. Emergency medical technicians are able to control the site of bleeding by local pressure and the child stops losing blood, though he is hypotensive and tachycardic. Intravenous fluid resuscitation is urgently needed, but several attempts at starting peripheral intravenous lines are unsuccessful. Which of the following is the best alternative route in this situation?

When intravenous lines cannot be established, the preferred alternate route is intraosseous (IO) cannulation. This is done by placing a trocar in the bone marrow of a long bone. The site of choice in children is the proximal tibia; alternative sites are the distal tibia and proximal femur. Careful attention should be made in regard to IO location in order to avoid injury to the growth plate which could stunt bony growth as the child grows older.

A 62-year-old man comes to the health care provider with symptoms of worsening congestive heart failure. He has a history of rheumatic heart disease as a child. Over the past 3 years he has had progressive symptoms of dyspnea on exertion, paroxysmal nocturnal dyspnea, and orthopnea. He has been maintained on digoxin, furosemide, and enalapril for symptoms of his congestive heart failure. Cardiac examination reveals a loud blowing decrescendo diastolic murmur. He has bounding peripheral pulses. Which of the following additional findings would most likely be found on physical examination?

Wide pulse pressure (aortic insufficiency, including the loud blowing decrescendo murmur and the bounding peripheral pulses. These patients typically will have a wide pulse pressure (i.e., elevated systolic blood pressure) related to a large left ventricular volume being ejected (until left ventricular failure supervenes). The low diastolic pressure is a result of the rapid run-off from the aorta caused by the regurgitant flow across the aortic valve into the left ventricle as well as the forward flow to the aorta.)

Pt diagnosed with polymorphic ventricular tachycardia called torsades de pointes. This can be a fatal rhythm and is associated with a prolongation of the QT interval, which could have occurred as a result of the __________ administration for symptomatic ventricular couplets. The appropriate step is thus to discontinue the Med and observe

amiodarone

______________- correlates well with the duration of the murmur (not the intensity of the murmur). Other physical findings seen with it include: +water hammer pulse +soft or absent A2-caused by inadequate closure of the valve +displaced, hyperdynamic apical impulse +de Musset sign-head bobbing with each heart beat, +Muller sign- pulsation of the uvula, +Traube sign- pistol shot sounds over the femoral artery with compression

aortic regurgitation

The congenital form of _______ aneurysm formation has an association with polycystic kidney disease, Marfan disease, Ehlers-Danlos syndrome, and neurofibromatosis.

berry (congenital weakness at branching points of the arteries in the circle of willis)

A 45-year-old woman has a history of symptomatic ventricular couplets, for which she had been placed on amiodarone (Cordarone). A week after this intervention, she experiences a syncopal episode and is brought to the emergency department. The patient is awake and alert with a blood pressure of 110/70 mm Hg and pulse 90/min. Electrolytes are within normal limits. She is placed on a cardiac monitor and is noted to have a transient wide complex tachycardia with waves of alternating morphology. She appears to be in torsades de pointes. Which of the following is the most appropriate next step?

d/c amiodarone and observe. The patient is diagnosed with polymorphic ventricular tachycardia called torsades de pointes. This can be a fatal rhythm and is associated with a prolongation of the QT interval, which could have occurred as a result of the amiodarone administration. The appropriate step is thus to discontinue the amiodarone and observe.

Increased resistance to filling of one or more cardiac ventricles has been termed ____________ and can produce increased pulmonary capillary wedge pressures and respiratory complaints.

diastolic heart failure

The collection of symptoms described—dizziness, headaches, diarrhea, nausea and vomiting, weakness, palpitations, and a change in vision with a yellowish to blue tint to the vision—are classic side effects of _______. EKG changes, such as biventricular tachycardia, may also occur. It is a medication that has been shown to improve symptoms in patients who have heart failure, as it is a positive inotropic and negative chronotropic agent. Patients who are hypokalemic are especially prone to the development of TOXICITY!

digoxin (Lanoxin)

The key to differentiating Systolic VS Diastolic types of heart failure is in the assessment of _________ -Normal and preserved with _______ heart failure -Impaired with _________.

ejection fraction, Diastolic, Systolic

Increased voltage in the chest leads is associated with _________ -Because the EKG measures electrical activity, it will have increased amplitude of the R wave from the increased force of contraction

hypertrophy of the left or right ventricles

Shock is a clinical condition in which there is inadequate organ perfusion and tissue oxygenation. It is classically associated with -____tension -____cardia -____pnea.

hypo, tachy, tachy (The normal physiologic response to shock is to have sympathetic release of hormones, which will stimulate the heart to beat faster and stronger so that the volume of blood can be circulated faster and with more force to overcome the deficiency of the blood supply to the end organs and to improve perfusion.)

___________ artery- gives off the superior rectal, sigmoid, and left colic arteries. Occlusion to this artery causes acute bowel ischemia

inferior mesenteric

____________-supplies the anterior surface of the left ventricle. Because this artery is a continuation of the left main artery, it will not be as impactful on survival as the more proximal disease that might occur in a patient who has left main disease. Blockage of the artery can lead to anterior wall myocardial infarction

left anterior descending artery

The signs of acute arterial embolism include the "6 Ps": +__________

pain, pallor, paresthesias, paralysis, pulselessness, and poikilothermia

A continuous machinery murmur is characteristic of _________________

patent ductus arteriosus (PDA- also characterized by bounding pulses with wide pulse pressure.)

The majority of diuretics used in clinical practice result in hypokalemia and hyponatremia because of the loss of these electrolytes as part of their clinical utility in causing diuresis. The ____________- inhibit distal convoluted tubule aldosterone-induced sodium resorption, which permits sodium loss and potassium retention. Although they cause weak diuresis, there is evidence showing that there is mortality benefit when these agents are used as add-on agents for chronic heart failure.

potassium-sparing diuretics (Triamterene, Spironolactone, amiloride- *CAUTION w/ Impaired Renal Fxn w/ ^K+)

__________- supplies blood to the right ventricle and posterior surface of the left ventricle. Blockage of this artery may result in right ventricular infarct, inferior wall, or posterior wall myocardial infarction. This myocardial infarction pattern needs to have volume support. Because the majority of patients have the artery supplying the AV node, heart block may occur with blockage of this artery.

right coronary artery

___________ artery- gives off the inferior pancreaticoduodenal, intestinal (ileal and jejunal), right colic, middle colic, and ileocolic arteries. Occlusion of this artery causes acute bowel ischemia

superior mesenteric

One complication of atrial fibrillation is a ___________ event that occurs as a result of a clot being expelled from the left atrium. This arterial clot may be manifested by signs of poor perfusion to the peripheral vascular system. The signs of acute clot include the "6 Ps": pain, pallor, paresthesias, paralysis, pulselessness, and poikilothermia. The patient's clot needs to be urgently addressed to salvage the tissue distal to the site of arterial obstruction.

thromboembolic

In preparation for an inguinal hernia repair, a 22-year-old man has a spinal anesthetic placed. The level of sensory block turns out to be much higher than had been planned, and shortly thereafter his blood pressure drops to 75/20 mm Hg. He looks warm and flushed, and his central venous pressure is near 0 mm Hg. He has no history of blood pressure abnormalities. Which of the following should be included in his therapy?

vasocontrictors and IV fluids. A high spinal anesthetic can produce vasomotor shock by inducing widespread vasodilation. Vasoconstrictors are the appropriate therapy, but because the capacity of the vascular tree is also increased under these circumstances, filling it up with additional volume is also helpful.

A 62-year-old man comes to the health care provider with symptoms of worsening congestive heart failure. He has a history of rheumatic heart disease as a child. Over the past 3 years he has had progressive symptoms of dyspnea on exertion, paroxysmal nocturnal dyspnea, and orthopnea. He has been maintained on digoxin, furosemide, and enalapril for symptoms of his congestive heart failure. Cardiac examination reveals a loud blowing decrescendo diastolic murmur. He has bounding peripheral pulses. Which of the following additional findings would most likely be found on physical examination? bradycardia, isolated systolic htn, pulsus paradoxus, tachycardia, wide pulse pressure

wide pulse pressure. This patient has the characteristic physical findings of an aortic insufficiency, including the loud blowing decrescendo murmur and the bounding peripheral pulses. These patients typically will have a wide pulse pressure (i.e., elevated systolic blood pressure) related to a large left ventricular volume being ejected (until left ventricular failure supervenes). The low diastolic pressure is a result of the rapid run-off from the aorta caused by the regurgitant flow across the aortic valve into the left ventricle as well as the forward flow to the aorta.


Related study sets

Ethics of biotechnologyUnderstand what the IRB is and why it was established. Be able to give examples of research that needs IRB approval vs that which does not.

View Set

O- [ou] - Mots avec le son [ou], au milieu du mot

View Set

Simple, Compound, Complex, and Compound-Complex Sentences

View Set

Information Technology Project Management

View Set

Excel Exam Modules 1-3: ISDS 1100

View Set